Автор: Тарасов Л.В.   Тарасова А.Н.  

Теги: физика   задачи по физике  

ISBN: 5-06-000633-6

Год: 1990

Текст
                    Л В ТАРАСОВ,
А Н ТАРАСОВА
ВОПРОСЫ
И ЗАДАЧИ
ПО ФИЗИКЕ
(Анализ характерных ошибок
поступающих во втузы)
Издание четвертое,
стереотипное
Москва «Высшая школа» 1990

ББК 22.3 Т19 УДК 53 Рецензент канд. физ.-матем. наук А Я. Винокуров (Московский инженерно-физический институт) Тарасов Л. В., Тарасова А. Н. Т 19 Вопросы и задачи по физике (Анализ характерных ошибок поступающих во втузы): Учеб, пособие, —4-е изд., стереотип.—М.: Высш, шк., 1990. — 256 с., ил. ISBN 5-06-000633-6 В пособии в форме диалога между автором и читателем дан ана лиз ошибок, допускаемых поступающими и связанных с недостаточно глубоким пониманием предмета Подробно разбирается большое число вопросов и задач по различным разделам программы Приводятся за- дачи для самостоятельной работы Третье издание вышло в 1984 г 4306021200—448 Т -------------------------103—90 ББК 22.3 001 (01)—90 53 Учебное издание Тарасов Лев Васильевич, Тарасова Альдина Николаевна Вопросы и задачи по физике (Анализ характерных ошибок поступающих во втузы) Зав редакцией Е С Гридасова Редактор Г Н Чернышева Художественный редактор В И Пономаренко Технический редактор С Р Луковенкова Корректор Н Е Жданова ИБ № 8165 Изд № ФМ-26 Сдано в набор 29 06 89 Подп в печать 16 07 90 Формат 60Х90'/16. Бум офс № 2 Гарнитура литературная Печать офсетная Объем 16 усл печ л 16,38 усл кр-отт 17,63 уч изд л Тираж 150 000 экз Заказ № 991 Цена 65 коп Издательство «Высшая школа» 101430 Москва, ГСП-4 Неглинная ул , д 29/14 Текстовые диапозитивы изготовлены на фотонаборном оборудовании издательства с применением ЭВМ Отпечатано в Московской типографии № 6 Госкомпечати СССР 109088, Москва, Ж-88, Южнопортовая ул , 24 ISBN 5-06-000633-6 © Издательство «Высшая школа», 1975 © Издательство «Высшая школа», 1984, с изменениями
СОДЕРЖАНИЕ Предисловие 3 1 Хорошо ли Вы разбираетесь в основных понятиях кинематики? 5 2. Умеете ли Вы рассматривать графически кинематику прямолинейного движения? 14 3 Сможете ли Вы указать, какие силы приложены к телу? 18 4. Умеете ли Вы находить силу трения? 25 5. Хорошо ли Вы знаете законы Ньютона? 28 6 Как Вы решаете кинематические задачи? 35 7. Как Вы решаете задачи на законы Ньютона? 44 8. Насколько усложняет решение задач учет силы трения? 48 9. Существует ли центробежная сила? 53 10. Как Вы объясните невесомость тел? 62 11. Что Вы знаете о работе? 66 12. Чем отличается мгновенная мощность от средней мощности? 72 13. Умеете ли Вы пользоваться законами сохранения энергии и количества движения? 74 14. Знаете ди Вы, что такое гармонические колебания? 87 15. Что буДет с маятником в состоянии невесомости? 94 16. Что Вам известно о волнах? 98 17 Умеете ли Вы пользоваться разложением сил? 104 18. Что Вы знаете о равновесии тел? 108 19. Как Вы отыскиваете положение центра тяжести? 113 20. Знаете ли Вы закон Паскаля? 118 21. Зачем надо знать закон Архимеда? 122 22. Действует ли закон Архимеда в космическом корабле? 128 23 Что Вы знаете о молекулярно-кинетической теории вещества? 130 24. Чем объясняется особенность теплового расширения воды? 137 25. Знаете ли Вы, что такое идеальный газ? 138 26. Какие газовые законы Вы знаете? 141 27. Как Вы решаете задачи на газовые законы? 148 28. Когда выпадает роса? 156 29. Что такое термодинамика? 158 30. Что такое цикл Карно? 163 31. Сколько у газа теплоемкостей? 165 32. Давайте поговорим о поле 166 33. Как описывается электростатическое поле? 170 34. Как ведут себя силовые линии вблизи поверхности проводника? 178 35. Как Вы рассматриваете движение в однородном электростатическом поле? 181 36. Умеете ли Вы обращаться с законом Кулона? 188 37. Знаете ли Вы закон Ома? 192 38. Можно ли включать конденсатор в цепь постоянного тока? 198 39. Умеете ли Вы вычислять сопротивление разветвленного участка цепи? 203 40. Отчего перегорела лампочка? 208 41. Как описывается магнитное поле тока? 212 42. Как взаимодействуют токи? 216 43. Хорошо ли Вы понимаете закон Фарадея и правило Ленца? 220 44. Знакомы ли Вам понятия самоиндукции и индуктивности? 224 45. Знаете ли Вы, как отражаются и преломляются лучи света? 226 46. Как Вы строите изображения в зеркалах и линзах? 231 47 Хорошо ли Вы решаете задачи с зеркалами и линзами? 238 48. Необходима осторожность! 242 Ответы 247
ПРЕДИСЛОВИЕ С удовольствием представляю читателям данную книгу. Она успеш- но выдержала проверку временем. Впервые книга была издана в 1968 г. Это ее четвертое издание. За двадцать лет, прошедших со времени первого издания, она была переведена на многие языки: испанский, английский, немецкий, венгерский, чешский, японский, тамили. Книга была задумана как пособие для поступающих в вузы. Желая помочь абитуриентам лучше подготовиться к вступительным экзаменам по физике, авторы проанализировали характерные ошибки, допускае- мые экзаменующимися, выбрав для удобства анализа форму диалога между автором и читателем. Как известно, анализ ошибок всегда по- учителен, так как при этом заостряется внимание на разных сторо- нах рассматриваемой проблемы, выявляются «тонкости», развивается более глубокое понимание. Как отмечали авторы в своем предисловии к первому изданию, «...нельзя предугадать всех возможных непра- вильных ответов на тот или иной вопрос... Тем не менее можно ука- зать некоторые неправильные ответы на определенные вопросы, с ко- торыми приходится встречаться постоянно. Можно указать многие кон- кретные вопросы, на которые, как правило, даются неправильные от- веты. Именно такой материал положен в основу книги». Надо признать, что авторы прекрасно справились со своей зада- чей. Анализ ошибок выполнен авторами настолько компетентно и сис- тематизированно, что книга окажет значительную помощь не только абитуриентам при их подготовке к вступительным экзаменам, но и учителям школ. Учителя получат хорошо продуманную систему вопро- сов и задач, позволяющую эффективно диагностировать знания учащих- ся по физике, проверять, насколько глубоко понимают они принципи- альные вопросы школьного курса физики, в том числе и достаточно тонкие и трудные вопросы. Конечно, любому абитуриенту полезно по- работать с данной книгой — чтобы не повторять ошибок, которые до него делали другие. Но еще более полезно познакомиться с этой книгой учителям — чтобы предотвратить появление подобных ошибок. Академик АПН СССР В. Г. Разумовский
jfe пренебрегайте кинематикой! Вопрос о |^ож, как именно происходит перемещение тела в пространстве и во времени, представляет тачительный интерес как г физической, так и с Практической точки Прения. и ХОРОШО ЛИ ВЫ РАЗБИРАЕТЕСЬ В ОСНОВНЫХ ПОНЯТИЯХ КИНЕМАТИКИ? t fterop: Начиная изучать механику, Вы знакомитесь с основными поня- тиями кинематики, такими, как перемещение, пройденный путь, ско- рость, ускорение. Обсудим эти понятия. Читатель: Перемещение — это направленный отрезок, соединяющий на- ' чальное и конечное положения тела. Иначе говоря, это вектор, про- веденный из начального положения тела в конечное. Пройденный \ путь — длина этого вектора. $.втор: Вы правильно Понимаете смысл термина «перемещение», однако пройденный путь определили неверно. Пройденный путь есть длина траектории, которую описывает тело, перемещаясь из начального по- ложения в конечное. Длина траектории может быть больше расстоя- ния по прямой между начальным и конечным положениями (рис. 1.1). Читатель: Я понял. Пройденный путь равен модулю перемещения только в случае прямолинейного движения. Автор: И притом происходящего в одну сторону. Хорошим приме- ром, иллюстрирущим различие между перемещением и путем, может служить дорога в горах между двумя соседними селениями. Читатель: Действительно, два таких селения могут находиться друг от друга на очень малом расстоянии по прямой, но чтобы попасть из одного в другое, приходится долго петлять по извивам горной дороги. В этом случае машина проходит длинный путь, тогда как перемеще- ние оказывается относительно малым. 5
Автор: А может ли быть так, чтобы перемещение было равно нулю, а длина пройденного пути -X \ j-g при этом не равна нулю? — \ Читатель: Теперь я уже понимаю. Такое бывает всякий раз, когда мы возвращаемся в исход- ную точку, т. е. когда конечное и начальное рис । । положения тела совпадают. Автор: Предположим, что человек последовательно прошел на север 3 км, на восток 4 км, на север 1 км и на запад 1 км. Пройденный путь равен 9 км. А как найти модуль перемещения? Читатель: Я нарисую траекторию движения человека (рис. 1.2, а). Здесь А — начальное положение, В — конечное. Соединим точки А и В отрезком прямой. Надо найти длину этого отрезка. На рисунке имеются две фигуры: прямоугольный треугольник и трапеция... Автор (перебивая): Не надо рассматривать здесь геометрические фигуры. Введем систему координат с началом в точке А, направив ось у на се- вер, ах — на восток (рис. 1.2, б). Координаты точки В обозначим xt и yt. Легко видеть, что х, = 4 — 1 = 3, У\ ~ 3 4- 1 = 4. Таким обра- зом |АВ| = З2 -|- 42 км = 5 км. Читатель: Я понял: прежде чем вычислять модуль перемещения, сле- дует найти его проекции на оси координат. Автор: Теперь рассмотрим понятие средней скорости. На рцс. 1.3 пред- ставлена зависимость пройденного пути s от времени t. Чему равна средняя скорость за первые две секунды и за первые три секунды? Читатель: А разве это не одна и та же величина? Автор: Речь идет о разных средних скоростях. Напомню, что средняя скорость определяется для промежутка времени, за который произво- дится усреднение. Средняя скорость за промежуток времени А/ есть <v> = As/AZ, (1.1) где As — путь, пройденный за промежуток А/. Читатель: В таком случае средняя скорость за первые две секунды равна 3 м/с, а за первые три секунды 2 м/с. Автор: Правильно. Очень важно помнить, что средняя скорость не имеет смысла, если не указан промежуток времени усреднения. Надо подчерк- нуть, что слово «средняя» означает здесь усреднение по времени. Читатель: Это понятно. Рис 1 2 6
Автор: Тогда решим простую задачу. Первую половину пути тело двига- лось равномерно со скоростью V] = 20 м/с, а вторую — со скоростью v2 = 30 м/с. Чему равна средняя скорость за все время движения? Читатель: По-видимому, 25 м/с. Автор: Неверно. Обозначим весь пройденный путь через s. Первую поло- вину пути тело прошло за время 5/(2^^, а вторую — за время s/ (2v2); полное время движения равно s/ (2vj) + s/ (2v2). Разделив s на это время, получим искомую среднюю скорость: S 2vj v2 ~ s/ ( 2vt) + s/(2v2) ~ и, + V2 ~ 24м/с- Читатель: А я почему-то был уверен, что должно быть 25 м/с. Автор: Так было бы, если бы тело двигалось со скоростью 20 м/с (и со- ответственно 30 м/с) не половину пути, а в течение половины времени движения. Читатель: Надо признать, что даже такое простое понятие, как средняя скорость, не лишено некоторых «тонкостей». Автор: Перейдем к понятию мгновенной скорости. Читатель: Это есть скорость тела в данный момент времени. Или, иначе го- воря, та скорость, какую оно имеет в данной точке траектории. Автор: А как Вы представляете себе скорость в данный момент времени? Читатель: Так и представляю... Если тело движется равномерно, то в разные моменты времени его скорость одна и та же. Если тело движет- ся неравномерно, то в разные моменты времени его скорость различна. Автор: Разве не чувствуете, что слова «скорость в данный момент време- ни» не более чем синоним слов «мгновенная скорость»? Как говорится, «что в лоб, что по лбу». Для измерения скорости надо измерить путь и промежуток времени, за который этот путь пройден. Но о каком прой- денном пути и о каком промежутке времени можно говорить в данный момент времени? Читатель: Можно выбрать какой-нибудь малый промежуток времени А/: от данного момента t до t + АЛ Пусть за этот промежуток тело пройдет путь As. Если At выбран достаточно малым, то отношение As /At и будет представлять собой то, что называют скоростью тела в момент t. Автор: Но что такое «достаточно малый» промежуток времени? С чем именно его следует сравнивать? С годом, часом, секундой? Читатель: В таком случае я затрудняюсь определить понятие мгновенной скорости. Автор: Надо составить последовательность значений средних скоростей за промежутки времени от t до t + At, выбирая все более и более малые значения At. Предел этой последовательности при At, стремя- щемся к нулю, и есть скорость в момент t. Если известна зависимость Пути от времени — зависимость s (/), то скорость в момент t опреде- ляется выражением v(t)= lim (1.2) Читатель: Это есть производная пути по времени: и (/) = ds/d/. Автор: Верно. Предположим, например, htos (/) = at2/1. Используя (1.2), 7
находим .. а (/+ Д02/2--а/72 .. / аМ\ и (/) — hm —------—---------— lim | at Н — I = at. м - > о л/ ^ о \ 2 у Читатель: А нельзя ли определить мгновенную скорость, не прибегая к дифференцированию? Автор: Строго говоря, нельзя. Мгновенная скорость есть предел соот- ветствующей последовательности значений средних скоростей. Иными словами, это — производная пути по времени. И никак иначе. Читатель: Однако, начиная изучать механику, мы все же обходились без математического анализа! Автор: Вы пользовались понятием мгновенной скорости, исходя из накоп- ленного опыта и интуитивных представлений. При этом Вы мысленно оперировали с «очень маленькими» промежутками времени. Читатель: Может быть, этого вполне достаточно? Автор: Нет, недостаточно. Не зная основ дифференциального исчисления, фактически нельзя понять самой сущности непрерывного движения. Древние греки, рассматривая парадокс летящей стрелы, рассуждали так: в данный момент времени летящая стрела находится в строго определенном месте, покоится по отношению к этому месту. Это спра- ведливо для любого момента времени, следовательно, стрела в любой момент покоится, т. е. покоится вообще. Данный парадокс обнаружи- вает непонимание древними греками понятия мгновенной скорости. Читатель: Я припоминаю^ что скорость в данный момент времени может быть определена через тангенс угла наклона касательной к графику пути. Автор: Действительно, мгновенную скорость можно определить геометри- чески, используя график зависимости пути от времени. Однако и в этом случае нам не обойтись без перехода к пределу последовательности средних скоростей. Рассмотрим некоторый график пути $ (/) (рис. 1.4). Данному моменту времени t отвечает на графике точка А. Выберем момент t + A/j; ему отвечает точка В. Средняя скорость за промежуток времени от t до 14-Л/1 есть[ s(/ + A/]) — $(/)] /А/j; она равна tg а1; где а, — угол накло- на секущей АВ (мы полагаем для простоты, что единице пути на s-оси и единице времени на /-оси соответствуют отрезки одинаковой длины). Далее выберем момент времени / + Д/2, причем А/2 < А/]; ему отвечает точка С. Средняя скорость за промежуток времени от / до / + + Л/2 равна tg «2, где а2— угол наклона секущей АС. Затем вы- берем момент / + А/3(А/3 < Д/2); ему отвечает точка D. Соответствую- щая средняя скорость равна tg 03, где а-3 — угол наклона секущей AD. Выбирая все новые и новые моменты времени (при А/ 0), мы полу- чаем последовательность средних скоростей, т. е. последовательность tg ai> tg «2, tg а3, ... Легко видеть, что при At ->0 эта последователь- ность имеет предел tg а, где а — угол наклона касательной, прове- денной к графику пути в точке А. Таким образом, скорость в момент / может быть численно определена через тангенс угла наклона касатель- ной, проведенной к графику пути в точке, отвечающей моменту вре- мени /. Читатель: Такое определение мгновенной скорости весьма наглядно. Чем больше скорость, тем круче идет график пути. Автор: Используя график пути на рис. 1.5, а, найдите те моменты, когда мгновенная скорость равна средней скорости за все время движения. Читатель: Движение прекращается в момент времени, которому на графи- ке отвечает точка А. Средняя скорость за все время движения есть tg р, где р — угол наклона прямой ОА (рис. 1.5, б). Такой же угол наклона имеют касательные в точках В и С, которым соответствуют моменты времени t} и /2. Именно в эти моменты времени мгновенная скорость совпадает со средней скоростью за все время движения. Автор: Совершенно верно. А может ли мгновенная скорость совпадать со средней в течение всего времени движения? Читатель: Может, если движение равномерное. График пути в этом случае прямая линия. Автор: Правильно. А теперь перейдем к понятию ускорения. Читатель: Ускорение есть скорость изменения скорости тела. Оно суще- ствует лишь тогда, когда движение является неравномерным. Если ускорение постоянно, то говорят о равнопеременном движении. Автор: Все это так. Однако следует определить понятие ускорения более строго. Прежде всего различают среднее и мгновенное ускорения. Среднее ускорение за промежуток времени А/ (промежуток усреднения) есть (а) = Ли/А/, (1.3) где Ау — изменение скорости за промежуток А/. Мгновенное уско- рение в момент / есть предел последовательности средних ускорений за промежутки от / до / + А/, вычисленный при А/, стремящемся к нулю: «(')= hm + (U) А/ ->• 0 Лг Читатель: Сравнивая (1.4) с (1.2), можно заметить, что ускорение тела выражается через приращение его скорости точно так же, как выража- ется скорость через приращение пути. Автор: Верно. Скорость — это производная пути по времени, а уско- рение — производная скорости по времени. Как Вы думаете, может ли в некоторый момент времени скорость тела быть равной нулю, а ускоре- ние в этот момент не равным нулю? 8 9
Читатель: По-видимому, это невозможно. Автор: Вы ответили, не подумав. Скорость вертикально брошенного камня равна нулю в самой верхней точке траектории, а ускорение не равно нулю (оно равно ускорению свободного падения). Читатель: Получается, что в этот момент времени величина скорости равна нулю, а изменение скорости отлично от нуля? Автор: Именно так. Предположим, что ускорение лифта направлено вверх. Куда движется лифт — вверх или вниз? Читатель: Наверное, вверх... Автор: Он может двигаться как вверх, так и вниз. В первом случае модуль скорости лифта растет, а во втором уменьшается. Читатель: Значит, направления скорости и ускорения могут не совпадать? Автор: Да. Они могут не совпадать уже в случае прямолинейного движе- ния, как это видно на примере с лифтом. При криволинейном же дви- жении направления скорости и ускорения практически всегда не совпа- дают. Следует заметить, что соотношения (1.3) и (1.4) относятся к частному случаю прямолинейного движения. В общем же случае надо учитывать, что скорость и ускорения — векторы. Пусть, например, в момент tx тело имеет скорость а в момент /2— скорость v2. (Векторы V! и v2 могут различаться как по модулю, так и по направ- лению.) Среднее ускорение (а) за промежуток времени At = t2 — tx \ ~ A? t2 — At ' (1-5) Читатель: Выражение (1.5) очень похоже на (1.3). Автор: Нетрудно, однако, убедиться, что выражение (1.5) качественно отличается от (1.3). Если, например, за промежуток времени At ско- рость не изменяется по модулю, то согласно (1.3) среднее ускорение равно нулю, тогда как согласно (1.5) оно может отличаться от нуля. Иными словами, при криволинейном движении ускорение может суще- ствовать при постоянной по модулю скорости. Читатель: Как-то непривычно говорить об ускорении, когда тело движется равномерно. Автор: Рассмотрим равномерное движение тела по окружности. На рис. 1.6, а изображены два положения тела на окружности: А (в мо- мент tx) и В (в момент /2). Там же даны соответствующие векторы скорости (uj и v2); они одинаковы по модулю, но различны по направ- лению. На рис. 1.6, б показан вектор Аи = v2 — их. Взяв отношение этого вектора к промежутку времени At = t2— tx, получим согласно (1.5) вектор среднего ускорения за промежуток времени At. Читатель: По-видимому, не равно нулю и мгновенное ускорение при рав- номерном движении тела по окружности? Автор: Конечно. С учетом векторной природы скорости и ускорения пре- образуем (1.4) в следующее выражение: lim (16) А/ ->- О Используя это выражение, найдем мгновенное ускорение а (/) тела, равномерно движущегося по окружности. Пусть в момент времени t тело находится в точке А окружности, а в момент t -ф At — в точке В 10
(рис. 1.7). Длина дуги АВ равна As = /?Аа, где R— радиус окруж- ности, а Аа — угловое смещение за промежуток времени А/. Обозначая через у — модуль скорости тела, получим As = v At. Таким образом, /?Аа = v At, откуда At=RAa/v. (1.7) Перенесем вектор и(/-|~ А/) из точки В в точку А и рассмотрим получив- шийся треугольник векторов (на рис. 1.7 заштрихован); он равнобед- ренный. Угол при вершине треугольника равен Аа (поскольку стороны этого угла перпендикулярны сторонам угла АОВ). Длина основания треугольника равна |о (t + At) — v (/)| = 2v sin (Аа/2). (1-8) Используя (1.7) и (1.8), находим выражение для модуля вектора искомого ускорения: |а-(0|= lim = Нга Л^О А/ Ла-0 2у sin (Да/2) RAa/v lim sl" <д°/2>. R Ла—о Аа/2 (1-9) _ sin (Аа/2) , Так как lim —т—75—= 1,то Ла -> 0 Аа/2 (1.10) 'итатель: Получился известный результат для модуля центростремитель- ; ного ускорения! \втор: Совершенно верно. Теперь выясним направление вектора а (/). Как видно из рис. 1.7, вектор [ и (t 4- At) — и (/)] направлен под углом Др = л/2 — Аа/2 к вектору v (t). При А/-+-0 (а следовательно, при Аа->0) угол Ар стремится к прямому углу. Отсюда видно, что век- тор а (/) перпендикулярен вектору v (t). Иначе говоря, вектор центро- стремительного ускорения ориентирован по радиусу к центру окружно- сти. По мере движения тела по окружности направление вектора a(t) соответствующим образом изменяется, модуль же его остается по- стоянным (он равен v2/R). Рис. 1.6 Рис 1 7 11
Читатель: Получается, что при равномерном движении по окружности ни скорость, ни ускорение тела не являются постоянными? Автор: Это так. Оба вектора изменяют направление; вектор v (/) в каждой точке траектории направлен по касательной к окружности, а вектор a(f) все время перпендикулярен вектору v (f), т. е. направлен по ра- диусу к центру окружности. Что же касается модулей указанных векто- ров, то они остаются постоянными. Читатель: Интересно, насколько усложнится рассмотрение неравномер- ного движения по окружности? Автор: В этом случае скорость изменяется не только по направлению, но и по модулю; то же следует сказать и об ускорении. Вектор ускорения а (/) удобно представить в данном случае в виде суммы двух взаимно перпендикулярных векторов: 5(/) = 5„(() + Ht(0. (1.11) Здесь ац (t) — центростремительное ускорение, оно связано с изме- нением скорости тела по направлению; ат (/) — тангенциальное уско- рение, оно связано с изменением модуля скорости тела. Ускорение ац (/) направлено по радиусу к центру, а ускорение ат (/) — по каса- тельной к окружности. Читатель: Значит, тангенциальное ускорение направлено так же, как и скорость тела? Автор: Если модуль скорости со временем возрастает, то так же, а если модуль скорости уменьшается, то в противоположном направлении. Читатель: Я догадываюсь, что модуль центростремительного ускорения описывается, как и прежде, выражением (1.10). Автор: С одним уточнением: теперь и2 не является постоянной, а зави- сит от времени. Соответственно изменяется со временем и модуль центростремительного ускорения. Модуль тангенциального ускорения k«)l = |im |g(< + Ao||i;(/)| (1|2) м о Ш После того как определены модули центростремительного и танген- циального ускорений, полное ускорение тела в рассматриваемый мо- мент времени может быть получено (как по направлению, так и по мо- дулю) на основе соотношения (1.11) с учетом того, что векторы ац (/) и ат (/) взаимно перпендикулярны. Читатель: Наверное, было бы неплохо рассмотреть некоторые задачи на равномерное и неравномерное движения тела по окружности. Автор: Рассмотрим следующую задачу. Тело неравномерно движется по окружности радиусом R. Линейная скорость изменяется во времени по закону v (t) = aQt. Найти ускорение тела в момент t = t0. Определить время т, за которое тело, начав двигаться, пройдет четверть длины окружности. Читатель: Центростремительное ускорение определю по формуле (1.10): ач = (а(Л)2/#- Тангенциальное ускорение можно найти, используя со- отношение (1.12): , ч .. ao(Z + A0-aoZ ,. aoAZ 12
$дтор: Правильно. Если Вы знаете правила отыскания производных, то ’« можете не рассматривать предел, а просто возьмите производную от б. функции скорости по времени. Тем самым Вы найдете тангенциальное |\ ускорение. Теперь определите полное ускорение. Житатель: Я затрудняюсь... М^тор: Воспользуйтесь соотношением (1.11) и тем фактом, что модуль 'суммы двух взаимно перпендикулярных векторов есть модуль гипоте- К нузы прямоугольного треугольника. Емтатель: Я понял: полное ускорение можно определить по теореме Пифа- гора для треугольника, катетами которого являются векторы ац и ах: з = «ц = ao^la^^R2 + 1. Чтобы найти время прохождения четверти длины окружности, надо, наверное, ввести понятие углового ускорения. ikrop-’ Это не обязательно. Заметим, что путь длиной в четверть окруж- 'ности равен As = nR/2. Далее воспользуемся формулой (1.11), где Средняя скорость на этом отрезке пути определяется как средняя ско- рость равнопеременного движения: {v-)=(.vb + v)/1, (1.13) или (у) — aQt/2. Тогда As = <п> t = а//2. (1.14) В нашей задаче л/?/2 = а0т2/2, откуда получаем т = л/ л/?/а0. Шитатель: Соотношение (1.14) есть формула пути при равноускоренном I’;’ движении без начальной скорости. Ивтор: Вы правы. Полезно заметить, что эту формулу мы получили, ис- W пользуя только понятие средней скорости. ВЫДАЧИ Ц,1. Туристы вышли из пункта А. Пройдя по дороге два километра точно на север, М1И повернули на восток и прошли один километр. Дорога повернула на юго-восток, ^туристы через некоторое время пришли в деревню В, находящуюся от пункта А ва расстоянии I = 2,5 км точно в восточном направлении. Какой путь прошли Вуристы? ®.2. Автомобиль ехал из города А на восток в течение t, — 2 ч с постоянной ско- К>стью v, = 60 км/ч. Повернув на юг, он прибыл в город В через t = 3 ч с начала Движения. С какой скоростью v ехал автомобиль на юг, если известно, что кратчай- шее расстояние между городами А и В равно $ = 150 км. $1,3. Автомобиль первые два часа двигался со скоростью и, = 60 км/ч. Определить, t какой скоростью он двигался следующие два часа, если средняя скорость за это ремя равна (v) = 40 км/ч. Какой путь $ прошел автомобиль? *1.4. Автобус третью часть пути шел со скоростью v, = 20 км/ч, половину оставше- гося пути — со скоростью у2=30 км/ч, а остальной путь — со скоростью Ьл = 60 км/ч Определить среднюю скорость (v) навеем пути. ’1.5. Велосипедист одну третью часть пути проехал с постоянной скоростью и, = 12 км/ч, первую половину оставшегося пути проехал с постоянной скоростью и2= 18 км/ч, а затем стал тормозить и через tA = 20 мин остановился. Сколько 13
времени t велосипедист был в пути? Какой путь $ он проехал? Определить, с какой средней скоростью (и) он проехал весь путь. |.в. Тело одну четвертую часть пути двигалось с постоянной скоростью о = 2 м/с, затем третью часть оставшегося пути — с постоянной скоростью «2 = I м/с, а ос- тальной путь — с постоянным ускорением и в конце пути имело скорость и3 = 7 м/с. Определить среднюю скорость (у) за все время движения. Определить также весь пройденный телом путь з, если известно, что на последнем участке ускорения равно а — 1 м/с2. 1.7. В течение времени / = 2 с тело двигалось равноускоренно с ускорением at = 2 м/с2, затем равнозамедленно с ускорением |aj — 0,5 м/с2. Найти полное время движения Т до остановки, пройденный при этом путь з и среднюю ско- рость (у) за время Т. Найти среднее ускорение (а) за промежуток времени от /0 = 0 до/| = Т/2 1.8. Тело движется с постоянным ускорением. Скорость его в момент времени < = 5 с равна = 3 м/с, а в момент времени /2 = 6 с мгновенная скорость тела равна нулю. Определить скорость у0 тела в момент t = 0 и путь з, пройденный телом от t = 0 до t — t2. 1.9. Тело равномерно движется по окружности радиусом R со скоростью v. Найти модуль изменения скорости |Дц| тела при его перемещении по окружности на угол: а) 45°, б) 90°, в) 180°. Найти модуль изменения ускорения |Аа| при этих пере- мещениях. 1.10. Тело неравномерно движется по окружности радиусом R = 1 м. Линейная скорость тела изменяется по закону v (t) = 1 + 3/. Найти ускорение а тела в момент времени ^ = 10. 1.11. Тело движется по окружности с постоянным тангенциальным ускорением а = 2 м/с2. Найти центростремительное ац и полное а ускорения для момента вре- мени, когда вектор полного ускорения составляет угол а = 45° с вектором линейной скорости. 2. УМЕЕТЕ ЛИ ВЫ РАССМАТРИВАТЬ ГРАФИЧЕСКИ КИНЕМАТИКУ ПРЯМОЛИНЕЙНОГО ДВИЖЕНИЯ? Автор: Пусть график скорости имеет вид, изображенный на рис. 2.1. Исходя из этого графика, нарисуйте график зависимости пути от вре- мени. Читатель: Мне никогда не приходилось рисовать такие графики. Автор: Но здесь нет ничего сложного. Давайте рассуждать вместе. Раз- делим весь рассматриваемый промежуток времени на три участка: 1, 2 и 3. Как движется тело на уча- стке /? Какова формула пути для этого участка? Читатель: На участке 1 тело дви- жется равноускоренно без на- чальной скорости. Формула пу- ти для этого участка имеет вид s(/) = a/2/2, (2.1) *’с где а — ускорение тела. Автор: Могли бы Вы, используя график скорости, найти уско- рение? 14
Читатель: Да. Согласно (1.3) ускорение есть отношение изменения ско- рости к промежутку времени, за которое это изменение произошло. Оно , равно отношению отрезков |ЛВ|/|ОВ|. Автор- Хорошо. Необходимо при этом помнить, что отношение |лв|/|ов| берется в соответствующих единицах Это замечание следует иметь в виду и при рассмотрении последующих графиков. А теперь рассмотрите участки 2 и 3. Читатель: На участке 2 тело движется равномерно со скоростью v, при- L обретенной к концу участка 1. Формула пути s — vt. ^[втор- Ваш ответ неточен. Вы не учли, что равномерное движение на- чалось не в начальный момент времени, а в момент t\. К этому моменту тело уже прошло путь at^/2 Зависимость пути от времени имеет для участка 2 следующий вид’ s(t) = a^/2 + v(l-ll). (2.2) Учитывая это замечание, напишите формулу пути для участка 3. Читатель: На участке 3 движение равнозамедленное. Если я правильно понял, формулу пути для этого участка можно записать так: s (/) = at\/2 + v(t2 — /,) + v(t — /2) — at (t — t2)2/2, где ax — ускорение на участке 3. Оно вдвое меньше ускорения а на участке 1, поскольку участок 3 вдвое длиннее участка 1. Автор: Вашу формулу можно немного упростить: S (j) = at\/2 + ,v(t - Q - ax(t - t2)2/2. (2.3) Теперь, чтобы построить график, Вам остается только суммировать результаты (2.1) — (2.3). Читатель: Я понимаю. На участке 1 график пути имеет вид параболы, на участке 2 — прямой линии, на участке 3 — тоже параболы, но перевернутой (с выпуклостью, обращенной вверх). Вот мой рисунок (рис. 2.2). Автор: Ваш рисунок не совсем верен: график пути не должен иметь изломов, он изображается плавной линией, т. е. параболы сопрягаются с прямой линией. Кроме того, вершина перевернутой параболы должна соответствовать моменту времени /3. Вот правильный рисунок (рис.2.3). 15
Читатель: Объясните, пожалуйста. Автор: В § 1 мы говорили, что тангенс угла наклона касательной к оси времени определяет значение скорости в момент времени t (см. рис. 1.4). Таким образом, по наклону касательных к графику пути можно найти скорость тела в тот или иной момент времени. Обратимся к Вашему рисунку (см. рис. 2.2). Из него следует, что в момент (и в момент /2) скорость тела принимает два разных зна- чения: если приближаться к tx слева, то скорость численно равна tga,, а если приближаться к справа, то tg о^. Согласно Вашему графику пути скорость тела в момент (а также в момент t2) должна была бы претерпевать разрыв, чего в действительности, однако, не наблюдается (график скорости на рис. 2.1 непрерывен). Читатель: Я понял. Непрерывность графика скорости приводит к плав- ности графика пути. Автор: Кстати говоря, вершины парабол должны соответствовать момен- там 0 и t3, так как в эти моменты скорость тела равна нулю и касатель- ные к графику пути должны быть для этих точек горизонтальными. А теперь по графику скорости на рис. 2.1 найдите путь, пройденный телом, например, к моменту t2. Читатель: Надо определить по графику скорости ускорение а на участке 1 и скорость v на участке 2, а затем воспользоваться формулой (2.2). Путь, пройденный телом за время /2, равен s О2) = a^/2 + v (/2 /(). Автор: Это правильно. Однако можно поступить проще. Путь, пройденный телом за время /2, численно равен площади фигуры ОАБГ, образуемой графиком скорости на промежутке Ot2. Для закрепления всех этих сведений рассмотрим еще одну задачу. Предположим, что график пути имеет изломы. Этот график изображен на рис. 2.4, где кривая есть парабола с вершиной в точке А. Начертите график скорости. Читатель: Поскольку график пути имеет изломы, то график скорости должен иметь разрывы в соответствующие моменты времени и /2). Вот мой рисунок (рис. 2.5). Автор: Хорошо. А чему равен отрееок |БВ|? Читатель: Он равен tg а, (см. рис. 2.4). Однако угол а, нам все равно неизвестен. Рис 2 4 16
Автор: И тем не менее отрезок |БВ| найти совсем нетрудно. Заметьте, что путь, пройденный телом за время /3, точно такой же, как если бы тело все это время двигалось равномерно (прямая на участке от t2 до t3 на рис. 2.4 представляет собой продол- жение прямой на участке от 0 до / J. Поскольку пройденный путь опреде- ляется площадью фигуры под графи- ком скорости, то, следовательно, площадь прямоугольника АГДВ на рис. 2.5 равна площади треугольника АБВ. |БВ| = 2 |ДВ|, т. е. скорость в момент t2 при приближении к нему слева равна удвоенной скоро- сти равномерного движения на участках от 0 до /, и от /2 Д° h- Читатель: А как нарисовать графики пути и перемещения, если значение скорости после некоторого момента времени становится отрицатель- ным? Рассмотрим, например, график скорости, показанный на рис. 2.6. Автор: Прежде всего отметим, что на рис. 2.6 дана зависимость от вре- мени не скорости, а проекции скорости на направление движения тела на ось х. Отрицательное значение проекции означает, что тело после момента времени tx = 5 с стало двигаться в обратном направлении. Читатель: Получается, что к моменту времени t — 10 с перемещение равно нулю, так как площади треугольников под графиком скорости при дви- жении вдоль оси х и в обратном направлении одинаковы; координата теЛа также равна нулю, если считать, что тело вышло из начала коор- динат. Путь, пройденный за t = 10 с, равен s = 10 м. Для построения графиков зависимости пути и координаты от времени (в данном случае координата выражает перемещение) запишем уравнение движения сначала для равнозамедленного движения — от /0 = 0 до tx = 5 с, а затем для равноускоренного — от tx = 5 с до / = 10 с. Автор: Не надо разбивать движение на два этапа. Ускорение в течение Рис. 2.6 Рис. 2.7 Рис. 2.8 17
х,м' Рис 2 10 всего времени движения постоянно и проекция его на ось координат отрицательна, следовательно, уравнение движения х — vot — at* 2/2 или, в нашем случае, х = 2t — 0,2 t2, справедливое для начального момента времени, верно и для любого момента времени График этой зависимо сти представлен на рис 2 7, а график пути — на рис 2 8 ЗАДАЧИ 2 1. График скорости имеет вид, изображенный на рис 2 5 (/,=2 с, /2=4 с, /3=6 с) Нарисовать график зависимости ускорения от времени Найти среднюю ско рость (и) в случаях 1) за первые четыре секунды, 2) за все время движения (за шесть секунд) В какие моменты времени т мгновенная скорость совпадает со средней скоростью, вычисленной за все время движения? 2.2. Тело брошено вертикально вверх с начальной скоростью у0=4 м/с Нарисовать графики зависимости скорости и координаты от времени 2.3. График зависимости скорости от времени представлен на рис 2 9 Нарисо вать графики зависимости ускорения и координаты от времени 2.4. График зависимости координаты от времени представлен на рис 2 10 Нари совать графики зависимости скорости и ускорения от времени Участок СДА — парабола, АВ — прямая 3. СМОЖЕТЕ ЛИ ВЫ УКАЗАТЬ, КАКИЕ СИЛЫ ПРИЛОЖЕНЫ К ТЕЛУ? Читатель Задачи по механике кажутся мне самыми трудными С чего следует начинать их решение? Автор С рассмотрения сил, приложенных к телу Возьмем в качестве при- мера следующие случаи (рис 3 1) а) тело брошено под углом к гори- зонту, б) тело соскальзывает с наклонной плоскости, в) тело вращается на нити в вертикальной плоскости, г) тело колеблется в вертикальной плоскости Изобразите и поясните силы, приложенные к телу в каждом из этих примеров Читатель Вот мой рисунок (рис 3 2) В первом случае G— сила тяжести, F — сила бросания, во втором — G — сила тяже 18
^Понятие силы является из основных физических понятий. Умеете ли вы достаточно свободно обращаться с этим Понятием? Хорошо ли вы разбираетесь в законах финамики? сти, F — скатывающая сила, Гтр — сила трения, в третьем — G — сила тяжести, — центростремительная сила, Т — сила натяжения нити, в четвертом — G — сила тяжести, F — возвращающая сила, Т — сила натяжения нити 4 втор Во всех четырех случаях Вы допустили ошибки Вот правильный рисунок (рис 3 3) Надо твердо усвоить, что сила возникает в результате взаимодействия тел. Поэтому, чтобы показать силы, приложенные к телу, следует пред- варительно ответить на вопрос, какие тела взаимодействуют с данным телом Так, в первом случае с телом взаимодействует только Земля, она притягивает его (рис 3 3, а) Поэтому к телу приложена единствен- ная сила — сила тяжести G Если бы учитывалось сопротивление воз- духа или, например, воздействие ветра, то тогда следовало бы ввести дополнительные силы «Силы бросания», указанной на Вашем рисунке, в природе нет, поскольку нет взаимодействия, приводящего к появле- нию подобной силы а) Рис 3 1 19
Рис. 3.2 Читатель: Но ведь, чтобы бросить тело, на него обязательно надо по- действовать какой-то силой. Автор: Это так. Когда Вы бросаете тело, Вы действуете на него неко- торой силой. Однако здесь рассматривается движение тела уже после того, как его бросили, т. е. после того, как прекратилось действие Вашей силы. Этой силы больше нет, остался лишь ее результат — начальная скорость тела. Запомните: силу невозможно «накопить» — кончилось взаимодействие тел, и нет силы. Читатель: Но если на тело действует только одна сила тяжести, то поче- му же оно не падает вертикально вниз, а движется по некоторой траек- тории? Автор: Вас удивляет, что направление движения тела не совпадает с направлением действующей на него силы. А между тем это полностью согласуется со вторым законом Ньютона. Ваш вопрос говорит о том, что Вы недостаточно глубоко продумали законы динамики Ньютона. Я предлагаю остановиться на этом позднее (см. § 5), а сейчас вер- немся к нашим примерам. Во втором случае (рис. 3.3, б) тело соскаль- зывает с наклонной плоскости. Какие тела взаимодействуют с ним? Читатель: По-видимому, два тела: Земля и наклонная плоскость. Автор: Правильно. Отсюда выявляем силы, приложенные к телу. Земля обусловливает силу тяжести G, а наклонная плоскость — силу трения скольжения Хтр и силу N, называемую обычно силой реакции опоры. Замечу, что на Вашем рисунке сила $ вообще отсутствовала. Читатель: Но постойте! Получается, что наклонная плоскость действует на тело не одной, а двумя силами? Автор: Сила, конечно, одна. Однако удобно рассматривать ее в виде двух составляющих, одна из которых направлена вдоль плоскости (си- ла трения скольжения), а другая — перпендикулярно ей (сила реакции опоры). Тот факт, что эти силы имеют общее происхождение, т. е. явля- ются составляющими одной и той же силы, отражается в существова- Рис 3 3 20
нии универсальной связи между Ртр и N: Етр = (3.1) где k — коэффициент трения скольжения. Ниже мы остановимся на этой связи подробнее (см. § 4). Читатель: На своем рисунке я изобразил скатывающую силу. Судя по t всему, такой силы нет. Однако, насколько мне известно, термин «скаты- вающая сила» употребляется. Как тут быть? 1йтор: Да, такой термин действительно существует. Однако надо иметь в виду, что скатывающая сила есть просто одна из составляющих силы тяжести, получающаяся при ее разложении на направления вдоль на- клонной плоскости и перпендикулярно ей. Если Вы, перечисляя силы, приложенные к телу, указали силу тяжести, то после этого незачем добавлять скатывающую силу. В третьем случае (рис. 3.3, в) тело вращается в вертикальной пло- скости. Какие тела действуют на него? Читатель: Два тела: Земля и нить. Автор: Правильно. Поэтому к телу приложены две силы: сила тяжести Г и сила реакции нити. Между прочим, о последней из этих двух сил Вы говорили как о силе «натяжения нити», забывая, что сила натяже- L ния нити — это сила, приложенная к нити со стороны тела, тогда как Г, сила, приложенная к телу со стороны нити, есть сила реакции нити. читатель: А как же центростремительная сила? Штор: Терпение! В задачах, в которых рассматривается движение тела по окружности, делают так много ошибок, что я предлагаю остано- виться на этом вопросе подробнее (см. § 9). Здесь же замечу лишь, что центростремительная сила не является какой-то дополнительно прило- женной к телу силой. Это есть равнодействующая сил, приложенных к телу. В данном случае (когда тело находится в низшей точке траекто- рии) центростремительная сила есть разность силы реакции нити и силы тяжести. итатель: Если я правильно понял, то в четвертом примере (рис. 3.3, г) возвращающая сила также есть равнодействующая силы тяжести и силы реакции нити? втор: Совершенно верно. В этом случае, как и в третьем, с телом взаимодействуют нить и Земля и поэтому к телу приложены две силы: сила реакции нити и сила тяжести. Я еще раз хочу подчеркнуть, что сила возникает в результате взаимодействия тел, она не может по- явиться из каких-либо «побочных» соображений. Найдите тела, дейст- вующие на данный объект, и Вы выявите силы, приложенные к этому объекту. Читатель: Наверное, возможны ситуации более сложные, чем те, кото- рые Вы изобразили на рис. 3.1. Автор: Примеров более сложных случаев взаимодействия тел много. До- пустим, что на тело давят с некоторой горизонтально направленной постоянной силой F, в результате чего оно поднимается вверх по наклонной плоскости. Силы, приложенные к телу, изображены на рис. 3.4. Другой пример: колебания заряженного маятника, помещенного 21
внутрь плоского конденсатора. Здесь появляется дополнительная сила Ре — сила, с которой поле конденсатора действует на заряд маятника (рис. 3 5). Разумеется, перечислить все ситуации, которые могут встре- титься при решении задач, практически невозможно. Читатель: А как быть, если в задаче участвует не одно, а несколько тел? Автор: Следует всякий раз четко представлять, движение какого конкрет- но тела (или какой совокупности тел) Вы собираетесь рассматривать. Обратимся, например, к рис. 3.6 Будем рассматривать движение тела 1. С этим телом взаимодействуют Земля, наклонная плоскость и нить АБ. Читатель: А разве тело 2 не взаимодействует с телом 1? Автор: Конечно, тела 1 и 2 взаимодействуют по закону всемирного тяго- тения. Однако эта сила весьма мала, ею можно пренебречь; остаются следующие силы, цриложенные к телу 1: сила тяжести G, сила трения скольжения Гтр, сила реакции опоры $ и сила реакции Т нити АБ (рис. 3.7, а). Читатель: Почему сила трения направлена на Вашем участке влево? Мне кажется, что с таким же основанием ее можно направить в проти- воположную сторону. Автор: Чтобы выяснить направление силы трения, надо знать направление движения тела. Если оно не оговорено в условии задачи, то можно 22
предположить то или иное направление. В данном случае я предполо- жил, что тело 1 движется (вместе со всей системой тел) вправо — блок вращается по часовой стрелке. Конечно, заранее мне это неизве- стно: направление движения становится определенным только после подстановки числовых значений. Если я ошибся, то при вычислении ускорения получится отрицательное число. Тогда следует предполо- жить, что тело движется не вправо, а влево (блок вращается против ча- совой стрелки), направить соответствующим образом силу трения скольжения, получить формулу для вычисления ускорения и опять про- верить его на знак посредством подстановки числовых значений. Читатель: А зачем вторично проверять знак ускорения? Если в пред- положении движения вправо оно получилось отрицательным, то теперь оно, очевидно, должно быть положительным. Автор: Нет, оно и во втором случае может оказаться отрицательным. Читатель: Мне это непонятно. Разве не очевидно, что если тело не дви- жется вправо, то оно движется влево? Автор: Вы забываете, что тело может еще и покоиться. Мы вернемся к < этому вопросу позднее и тогда подробно проанализируем усложнения, связанные с учетом силы трения (см. § 8). А сейчас предположим, что блок вращается по часовой стрелке, и рассмотрим движение тела 2 (рис. 3.6). Читатель: С телом 2 взаимодействуют Земля, наклонная плоскость, нити АБ и ВГ. Силы, приложенные к телу 2, показаны на рис. 3.7, б. Автор: Прекрасно. Теперь рассмотрите тело 3. Читатель: Тело 3 взаимодействует только с Землей и нитью ВГ. Силы, приложенные к нему, показаны на рис. 3.7, в. Автор: После того как Вы выявили силы, приложенные к каждому из тел, можете записать уравнения движения для каждого'из них и затем решить систему получившихся уравнений. Читатель: Вы упоминали о том, что можно рассматривать не обязательно , отдельные тела, но также совокупности тел. Автор: Да, можно рассматривать тела 1, 2 и 3 не по отдельности, как это было проделано тблько что, а как единое целое. В этом случае не следует принимать во внимание силы натяжения нитей, поскольку теперь они являются внутренними силами — силами взаимодействия между отдельными частями рассматриваемого объекта. Система трех , тел как целое взаимодействует только с Землей и наклонной плос- костью. Читатель: Мне хотелось бы кое-что уточнить. Когда я изображал силы на рис. 3.7, б, в, я полагал, что сила натяжения нити ВГ одна и та же по обе стороны от блока. Правильно ли это? Автор: Строго говоря, это неправильно. Если блок вращается по часовой стрелке, то натяжение участка нити ВГ, примыкающего к телу 3, должно быть больше натяжения участка нити ВГ, примыкающего к телу 2. Эта разность сил натяжения и вызывает ускорение враще- ния блока. Однако здесь предполагалось, что массой блока можно пре- небречь, т. е у блока нет массы, которую надо было бы разгонять — блок рассматривался просто как средство изменения направления нити, связывающей тела 2 и 3 Поэтому можно считать, что сила натяже- 23
ния нити ВГ по обе стороны от блока одна и та же. Обычно в школь- ном курсе физики массой блока всегда пренебрегают. Остались ли у Вас еще неясные вопросы? Читатель: Да, у меня есть вопрос относительно точки приложения сил. На всех рисунках Вы прикладывали все силы в одной точке тела. Правильно ли это? Можно ли, в частности, прикладывать в центре тяжести тела силу трения? Автор: Следует иметь в виду, что мы изучаем кинематику и динамику не протяженных тел, а материальных точек, т. е. представляем тело как точечную массу. При этом на рисунке изображается тело, а не точка, исключительно в целях наглядности. Поэтому все силы, при- ложенные к телу, можно прикладывать в одной точке. Кстати говоря, следовало бы уточнить условия, когда можно применять модель мате- риальной точки. Читатель: Нас учили, что модель материальной точки применима тогда, когда размерами тела можно пренебречь по сравнению с другими ха- рактерными размерами в данной задаче. Например, можно пренебречь размерами спутника по сравнению с расстоянием от спутника до по- верхности Земли: поэтому спутник можно рассматривать как мате- риальную точку. Автор: Я не возражаю. Однако вот другая ситуация: санки соскальзы- вают с невысокой горки. Можно ли пренебрегать размерами санок по сравнению с высотой горки? Или, например, те же санки человек тянет за веревку. С чем надо сравнивать размеры санок в этом случае? А меж- ду тем во всех случаях движение санок можно рассматривать как дви-, жение материальной точки. Читатель: Я как-то не задумывался над этим. Автор: Дело в том, что модель материальной точки надо связывать и с характером движения тела. Эта модель применима тогда, когда тело движется поступательно, не вращается, не кувыркается. Если бы санки кувыркались с горки, то модель материальной точки не годилась бы независимо от размеров горки. Подчеркнем: используя модель материальной точки, мы не принимаем во внимание вращательные моменты, которые в реальных условиях мо- гут приводить к повороту тела, к его опрокидыванию. Материальная точка может двигаться только поступательно. Рассмотрим пример. Пусть к телу приложены в разных его точках две силы: в точке А и Р2 в точке Б (рис. 3.8, а). Приложим в точке А силу Р2, равную Рис 3 8 24
силе F2 и параллельную ей, а также силу F2, равную силу Р2, но на- правленную в противоположную сторону (рис. 3.8, б). Так как силы Р2 и Р2 взаимно уравновешивают друг друга, то их добавление физи- чески ничего не меняет. Однако рис. 3.8, б можно трактовать следую- щим образом: в точке А приложены силы Р\ и Р2, обусловливаю- щие поступательное перемещение тела; к телу приложена также пара сил (силы Р2 и Р2), обусловливающая вращение тела. Иными слова- ми, силу Р2 можно перенести в точку А тела, если при этом одновремен- но приложить к телу соответствующий вращательный момент. При, рас- смотрении тела как материальной точки этот вращательный момент, очевидно, не имеет места. 4. УМЕЕТЕ ЛИ ВЫ НАХОДИТЬ СИЛУ ТРЕНИЯ? Автор: Я хочу несколько подробнее остановиться на вычислении силы тре- ния (в разных задачах. Речь идет о сухом трении скольжения (сухим называется трение между поверхностями двух тел при отсутствии меж- ду ними какой-либо прослойки, например смазки). Читатель: Но здесь, кажется, как раз все ясно. Автор: Тем не менее множество ошибок на экзаменах связано с неуме- нием вычислять силу трения. Рассмотрим пример: сани массой m тянут с силой Р, приложенной к веревке, которая составляет с горизонтом угол а; коэффициент трения скольжения равен k (рис. 4.1). Надо найти силу трения скольжения. Как Вы будете это делать? Читатель: А что здесь трудного? Сила трения равна kmg. Автор: Неправильно. Сила трения скольжения равна не kmg, a kN, где N—сила реакции опоры. Вспомните выражение (3.1) из § 3. Читатель: А разве это не одно и то же? Автор: В частном случае сила тяжести и сила реакции опоры могут быть равны друг другу по модулю. Вообще же это совершенно разные силы. Рассмотрим предложенный мною пример. Силы, приложенные к телу: сила тяжести G, сила реакции опоры N, сила трения скольжения Ртр и сила реакции веревки F (см. рис. 4.1). Разложим силу ^на вертикаль- ную (F sin а) и горизонтальную (F cos а) составляющие. Все силы, дей- Рис 4 1 25
ствующие по вертикальному направлению, взаимно уравновешивают- ся. Отсюда находим силу реакции опоры: N = G — F sin а (4.1) Вы видите, что эта сила не равна силе тяжести, а меньше ее на F sin а. Физически это совершенно естественно, так как натянутая кверху ве- ревка как бы «приподнимает» сани, вследствие чего сила, с которой сани давят на поверхность, а следовательно, и сила реакции опоры уменьшаются. Таким образом, в данном случае FTp = k(G - F sin a). (4.2) Если бы, в частности, веревка была горизонтальной (а= 0), то вместо (4.1) будем иметь N — G, откуда следует, что FTp = kG = kmg. Читатель: Понимаю. Просто я никогда не задумывался над этим. Автор: Это довольно распространенная ошибка экзаменующихся, которые пытаются трактовать силу трения скольжения как произведение коэф- фициента трения скольжения на силу тяжести, а не на силу реакции опоры. Постарайтесь не сделать подобной ошибки. Читатель: Я буду пользоваться правилом: чтобы найти силу трения сколь- жения, надо предварительно определить силу реакции опоры. Автор: До сих пор разговор шел о силе трения скольжения. Теперь рассмотрим силу трения покоя. Тут есть своя особенность, на которую учащиеся не всегда обращают внимание. Предположим, что тело по- коится на горизонтальной плоскости, причем на него действует горизон- тальная сила F, стремящаяся сдвинуть тело. Как Вы думаете, чему равна сила трения в этом случае? Читатель: Если тело находится на горизонтальной плоскости и сила F горизонтальна, то в этом случае W — G. Правильно? Автор: Правильно. И что же дальше? Читатель: Отсюда следует, что сила трения равна kG. Автор: Вы сделали характерную ошибку: спутали силу трения покоя с силой трения скольжения. Если бы тело скользило, то тогда Ваш ответ был бы верен. Однако в данном случае тело покоится. Для покоя тела необходимо, чтобы все силы, приложенные к нему, были взаимно уравновешены. На тело действуют четыре силы: сила тяжести б, сила реакции опоры А/, сила F и сила трения покоя FTp (рис. 4.2). Верти- кальные силы G и /V взаимно уравновешены. Должны быть взаимно уравновешены и горизонтальные силы F и FTp. Следовательно, FTP = F. (4.3) Читатель: Получается, что сила трения покоя зависит от внешней силы, стремящейся сдвинуть тело. Автор: Да, это так. По мере увеличения силы F сила трения покоя также возрастает. Однако процесс возрастания силы трения покоя не бесконечен. Существует максимальное значение силы трения покоя: Fm, = J4.4) Коэффициент kQ немного превышает коэффициент k, характеризующий в соответствии с (3.1) силу трения скольжения. Как только внешняя 26
сила F, увеличиваясь, достигает значения k0N, тело начинает дви- гаться. При этом коэффициент k0 становится равным коэффициенту k, так что сила трения несколько уменьшается. При дальнейшем возра- стании силы F сила треиия (теперь это есть сила трения скольжения) не изменяется (вплоть до очень больших скоростей), а тело движется с постепенно увеличивающимся ускорением. Неумение многих экзаменующихся находить силу трения обнаружива- ется на следующем довольно простом вопросе: чему равна сила трения, когда тело массой m покоится на наклонной плоскости с углом а? Можно услышать различные неправильные ответы. Одни говорят, что сила трения равна kmg, другие что она равна kN = kmg cos а. [татель: Я понимаю. Поскольку тело покоится, то речь идет о силе Трения покоя. Она должна быть найдена из условия равновесия сил, действующих вдоль наклонной плоскости. Таких сил в данном случае две: сила трения FTp и скатывающая сила mg sin а. Поэтому правиль- ный ответ есть FTp = mg sin а. тор: Совершенно верно. В заключение рассмотрим следующую задачу. Тело массой m лежит на теле массой М, максимальное значение силы трения покоя между ними характеризуется коэффициентом /г0; между телом М и поверхностью Земли трения нет. Требуется найти мини- мальную силу F (рис. 4.3), при действии которой на тело М происходит И сдвиг верхнего тела относительно нижнего. шртатель: Сначала я буду полагать, что сила F достаточно мала: тело m $Ь.не сдвигается относительно тела М. В этом случае оба тела имеют Ж ускорение W а — F/(М т). пвтор: Правильно. Какая сила сообщает это ускорение телу т? уцтатель: Это ускорение сообщает телу m сила трения покоя Frp—ma, т. е. fTp = Fm/(M 4- m). Отсюда следует, что с увеличением силы F сила трения покоя /7тр тоже должна возрастать. Однако она не может возрастать бесконечно. Ее максимальное значение Л-р макс = k0N = kttmg. Значит, максимальное значение силы F, при котором оба тела еще могут двигаться вместе как единое целое, определяется из условия Рис 4 3 27
Рис 4 5 kQmg = Fm/(M 4- иг). Отсюда находим F = (Af + m) kog. Это и есть искомая минимальная сила, обусловливающая сдвиг тела т по отношению к телу М. Автор: Ваше решение предложенной задачи правильно. ЗАДАЧИ 4.1. Вверх по наклонной плоскости пущена шайба Через некоторое время она останавливается и соскальзывает вниз Определить коэффициент трения k шайбы о плоскость, если время спуска в п раз больше времени подъема 4.2. Брусок массой т находится на наклонной плоскости, угол а наклона к гори- зонту которой можно изменять от 0 до 90° Построить график зависимости силы трения бруска о плоскость от угла а Коэффициент трения скольжения бруска о плоскость равен k 4.3. На тело, покоившееся на горизонтальной плоскости, в момент t = 0 начинает действовать горизонтально направленная сила F, которая со временем увеличива- ется по линейному закону (рис. 4 4) В некоторый момент t = t, тело начнет сколь- зить. Построить график зависимости силы трения от времени 4.4. На рис 4 5 показаны четыре различных случая (а—г) скольжения тела (силу F можно рассматривать как силу реакции нити, за которую тянут тело). Определить силу трения в каждом из указанных случаев, полагая заданными массу т тела, силу F, коэффициент трения скольжения k и угол а. 5. ХОРОШО ЛИ ВЫ ЗНАЕТЕ ЗАКОНЫ НЬЮТОНА? Автор: Сформулируйте, пожалуйста, первый закон Ньютона. Читатель: Тело находится в состоянии покоя или равномерного прямоли- нейного движения до тех пор, пока воздействие со стороны других тел не заставит его изменить это состояние. Автор: Во всех ли системах отсчета выполняется этот закон? Читатель: Я не понимаю Вашего вопроса. Автор: Если Вы говорите, что тело находится в состоянии покоя, то Вы имеете в виду, что оно неподвижно по отношению к какому-то другому телу, которое в данном случае играет роль системы отсчета Без ука- зания системы отсчета говорить о покое или каком-то определенном 28
движении тела не имеет смысла. Характер движения тела зависит от выбора системы отсчета. Например, тело, лежащее на полу движуще- гося вагона, покоится по отношению к системе отсчета, связанной с вагоном, но движется по отношению к системе отсчета, связанной с полотном дороги. После этих пояснений вернемся к поставленному вопросу: вц всех ли системах отсчета выполняется -первый закон Ньютона? Читатель: Ну, наверное, во всех... Автор: Я вижу, что этот вопрос застал Вас врасплох. Опыт показывает, что первый закон Ньютона выполняется не во всех системах отсчета Обратимся к примеру с телом, лежащим на полу движущегося вагона, причем будем пренебрегать трением между телом и полом. Рассмотрим положение тело по отношению к системе отсчета, связанной с вагоном. При этом можно наблюдать такую картину: тело покоится на полу, но затем, несмотря на отсутствие каких-либо воздействий, оно вдруг начи- нает скользить по полу. Налицо нарушение первого закона Ньютона. Нам объяснят этот эффект тем, что вагон, двигавшийся прежде прямо- линейно и равномерно, начал тормозить, а тело из-за отсутствия трения продолжало сохранять состояние равномерного и прямолинейного дви- жения по отношению к полотну дороги. Отсюда можно заключить, что в системе отсчета, связанной с полотном дороги, закон Ньютона выпол- няется, а в системе, связанной с тормозящимся вагоном, он не выпол- няется. Системы отсчета, в которых первый закон Ньютона выполня- ется, называются инерциальными, а в которых не выполняется — не- инерциальными. Для большинства рассматриваемых нами явлений можно считать инерциальной всякую систему отсчета, связанную с поверхностью Земли или с какими-либо телами, которые по отношению к земной поверхности покоятся или движутся прямолинейно и равно- мерно. К неинерциальным системам отсчета относятся системы, движу- щиеся с ускорением, например, вращающиеся системы, замедляющие- ся и ускоряющиеся лифты и т. д. Заметим, что в неинерциальных систе- мах отсчета не выполняется не только первый, но и второй закон Ньютона Читатель: Но если законами Ньютона нельзя пользоваться в системах отсчета, движущихся с ускорением, то как же тогда рассматривать механику в таких системах? ^втор: До сих пор мы имели дело лишь с силами, обусловленными взаимо- действием тел. Однако в механике встречаются также силы еще одного типа: они не обусловлены взаимодействием тел, к ним неприменим тре- тий закон Ньютона. Это силы инерции. Если наряду с «обычными» (обусловленными взаимодействием тел) силами включить в рассмотре- ние силы инерции, то тогда можно пользоваться законами Ньютона и в ускоряющихся системах. Сила инерции равна произведению мас- сы m тела на ускорение а системы и направлена в сторону, проти- воположную ускорению системы: РИ= — та. Пусть неинерциальная система — это лифт, имеющий ускорение а, направленное вертикально вверх, на полу лифта находится тело массой т В системе отсчета, связанной с лифтом, к телу надо приложить две «обычные» силы: силу реакции пола N и силу тяжести G, а кроме того, силу инерции Fa = та, 29
направленную вниз. В данной системе тело покоится, следовательно, А - G - F, = О Читатель: Значит, первым и вторым законами Ньютона все же можно пользоваться в ускоряющихся системах? Автор: Можно, если дополнительно приложить к телу силу инерции. Нельзя, если ограничиться «обычными», т. е. обусловленными взаимо- действием тел, силами. Хотелось бы посоветовать при решении задач пользоваться, как пра- вило, только инерциальными системами отсчета. Тогда все силы, с кото- рыми Вам придется иметь дело, будут «обычными» силами, т. е. будут обусловлены взаимодействием участвующих в задаче тел. Читатель: Но если ограничиваться только инерциальными системами, то тогда нельзя рассмотреть задачу с телом, находящимся в ускоряю- щемся лифте или на вращающемся диске! Автор: Почему нельзя? Выбор системы отсчета зависит от Вас. Если Вы используете в задаче систему, связанную с лифтом или диском (не- инерциальную систему), то тело надо считать покоящимся. Если же Вы используете систему отсчета, связанную с Землей (инерциальную систему), то тело надо рассматривать движущимся либо прямолинейно равноускоренно, либо по окружности. Я советую всякий раз выбирать именно инерциальную систему отсчета. А теперь перейдем ко второму закону Ньютона. Сформулируйте его. Читатель: Этот закон читается так: F = та, где F — сила, действующая на тело, т — его масса, а — ускорение. Автор: Ваш лаконичный ответ весьма характерен. По поводу Вашей фор- мулировки следует сделать три критических замечания — два не очень существенных и одно существенное. Во-первых, не сила есть следствие ускорения, а наоборот, ускорение есть следствие силы. Поэтому логичнее формулу закона записывать так: a = BF/m, (5.1) где В — коэффициент пропорциональности, зависящий от выбора еди- ниц величин, входящих в формулу (5.1). Замечу, что в Вашей формули- ровке не было упоминания о коэффициенте пропорциональности В. Во-вторых, телу сообщают ускорение все силы, приложенные к нему (хотя и не исключено, что некоторые из них могут быть взаимно уравновешены). Поэтому в формулировке закона следовало бы поль- зоваться не термином «сила», а более точным термином «равнодейст- вующая сил». Третье замечание является наиболее важным. Второй закон Ньютона устанавливает связь между силой и ускорением. Но сила и ускорение — величины векторные, они характеризуются не только числовым значе- нием, но и направлением Вы ничего не сказали о направлении. Получа- ется, что закон Ньютона сформулирован не полностью. Правильная формулировка второго закона Ньютона следующая: ускорение тела пропорционально равнодействующей всех сил, приложенных к телу, и направлено в сторону равнодействующей сил Аналитически эта фраза может быть выражена формулой: 30
a=BF/m, (5.2) где m — масса тела. Читатель: Получается, что при действии одной и той же силы тела разной массы приобретут различные ускорения. Автор: Совершенно верно. Чем тяжелее тело, тем меньшее ускорение оно получает (медленнее изменяется его скорость) по сравнению с более легким телом Сила F есть действие, а ускорение а есть «отклик» на это действие. Читатель: Рассматривая в § 3 силы, приложенные к телу, брошенному под углом к горизонту, Вы обещали показать, что направление движе- ния тела не обязательно совпадает с направлением силы, приложенной , к телу. При этрм Вы ссылались на второй закон Ньютона. Автор: Да, сейчас вполне уместно остановиться на этом вопросе. Вспом- ' ните, что в общем случае векторы ускорения и скорости тоже ориенти- рованы различно. Читатель: Это мне понятно. Например, при равномерном движении тела по окружности скорость тела направлена по касательной к окружности, а ускорение — по радиусу к центру. Автор: Ваш пример уместен. Теперь следует обратиться к соотношению (5.2) и твердо уяснить, что по направлению силы ориентировано именно ускорение, а не скорость, что с модулем силы связан модуль именно ускорения, а не скорости. С другой стороны, характер движения тела в данный момент определяется направлением и модулем скорости в данный момент (вектор скорости всегда направлен по касательной к траектории тела). Поскольку ускорение и скорость — разные векто- ры, то в общем случае направления силы и движения тела могут не сов- падать: следовательно, и характер движения тела в данный момент не определяется однозначно силами, действующими на рассматриваемое тело в этот момент. Читатель: Это в общем случае. Но, разумеется, возможно и совпадение направлений силы и скорости? Автор: Конечно, возможно. Поднимите тело и спокойно отпустите его, не придавая ему начальной скорости. В этом случае направление движе- ния будет совпадать с направлением силы тяжести. Если же Вы сооб- щите телу, например, горизонтальную начальную скорость, то направ- ления движения тела и силы тяжести не будут совпадать: тело поле- тит по параболе. В обоих случаях тело движется под действием одной и той же силы — силы тяжести, но характер движения различен. Физик сказал бы, что это обусловлено неодинаковыми начальными условиями: в момент начала движения в одном случае тело не имело скорости, а в другом — обладало определенной горизонтальной скоростью. На рис. 5.1 показаны траектории тел, брошенных с одинаковыми началь- ными скоростями, но под разными углами к горизонту; во всех случаях на тело действует одна и та же сила — сила тяжести. Читатель: Значит, характер движения тела в данный момент определяется не только силами, действующими на тело в этот момент, но и началь- ными условиями? Автор: Совершенно справедливо. Следует подчеркнуть, что начальные условия отражают предысторию движения тела. Они результат дейст- 31
вия сил, которые существовали в прошлом. Этих сил уже нет, но резуль- тат их действия проявляется. С философской точки зрения в этом отра- жается связь прошлого с настоящим, т. е. принцип причинности. Заме- тим, что если бы в формулу второго закона входила скорость, а не ускорение, то указанная связь прошлого с настоящим не проявлялась бы В этом случае скорость тела в данный момент (т. е. характер его движения в данный момент) полностью определялась бы силами, дейст- вующими на тело именно в этот момент,— прошлое нисколько не влияло бы на настоящее. Я приведу еще один пример, иллюстрирую- щий сказанное (рис. 5.2). Шарик подвешен на нити, на него действуют две силы: сила тяжести G и сила реакции 1\ нити. Если отклонить шарик от равновесного положения и затем отпустить, то он будет совершать колебания. Если же отклоненному шарйку сообщить опреде- ленную скорость, направленную перпендикулярно плоскости, в которой произошло отклонение, то шарик будет равномерно двигаться по ок- ружности. Как видите, в зависимости от начальных условий шарик либо совершает колебания (рис. 5.2, а), либо равномерно движется по окружности (рис. 5.2, б), при этом в обоих случаях на него действуют только две силы: сила тяжести и сила реакции нити. Читатель: Я не думал над законами Ньютона в таком плане. Автор: Поэтому неудивительно, что при выяснении вопроса о силах, при- ложенных к телу, иногда начинают исходить не из того, какие тела взаи- модействуют с данным телом, а из того, каков характер движения тела. Вспомните, Вы тоже так поступали. Именно поэтому, когда изобража- ли силы на рис. 3.2, в, г, Вам казалось, что совокупности сил, прило- женных к телу в указанных случаях, должны быть различны. А между тем в обоих случаях к телу приложены две силы: сила тяжести и сила реакции нити. Читатель: Я понял, что силы, приложенные к телу, могут быть одина- ковы, а характер движения тела различен, и поэтому данные о харак- тере движения тела не должны служить отправной точкой при выяв- лении сил, приложенных к этому телу. Автор: Вы весьма точно выразились. .Однако при этом не следует впа- дать в крайность. Хотя разные виды движения могут осуществляться 32
при одних и тех же совокупностях сил (как на рис. 5.2), числовые соотношения между действующими силами при этом различны. Это означает, что при разных видах движения будут различны равнодей- ствующие приложенных сил. Так, например, при равномерном движе- нии тела по окружности равнодействующая сил направлена к центру и обусловливает центростремительное ускорение, тогда как при колеба- нии равнодействующая перпендикулярна нити и является возвращаю- щей силой. Отсюда следует, что хотя данные о характере движения тела и не должны служить отправной точкой при выявлении сил, они отнюдь не излишни. В связи с этим вернемся к примеру, изображен- ному на рис. 5.2. Предположим, что известны угол а между верти- калью и направлением нити и масса m тела; требуется найти силу натяжения Т нити в двух случаях: 1) тело, совершая колебания, попа- дает в крайнее положение; 2) тело равномерно движется по окружности в горизонтальной плоскости. В первом случае равнодействующая силы тяжести и силы реакции нити есть возвращающая сила, она направле- на перпендикулярно нити. Поэтому надо разложить вектор силы тяже- сти G по направлению этой равнодействующей и перпендикулярно ей (т. е. вдоль нити) и приравнять друг другу силы, перпендикулярные равнодействующей, т. е. направленные вдоль нити (рис. 5.3, а). Учиты- вая, что G = mg, получаем ТJ = mg cos а. Во втором случае равнодействующая сил есть центростремительная сила, она направлена по горизонтали. Поэтому надо разложить силу реакции Г2 нити на вертикальное и горизонтальное направления и при- равнять друг другу силы, перпендикулярные равнодействующей, т. е. направленные по вертикали (рис. 5.3, б). Отсюда Т2 cos а = mg, или Т2 == mg/cos а. Мы получили силу реакции нити; искомая сила натяжения равна ей по модулю и направлена в противоположную сторону. Как видите, зна- ние характера движения пригодилось при нахождении силы натяжения нити. Читатель: Если я правильно понял, , то, зная взаимодействие тел, » можно выявить силы, приложен- , ные к телу: зная эти силы и на- чальные условия, можно пред- сказать характер движения тела (модуль и направление скорости тела в любой момент времени). С другой стороны, зная характер движения тела, можно устанав- ливать соотношения между си- лами, приложенными к телу. Правильно ли я рассуждаю? Автор: Да, Вы рассуждаете пра- вильно. Я хочу предложить еще одну несложную задачу на - Л. В. Тарасов, А. Н. Тарасова Рис 5 3 33
второй закон Ньютона. Два тела, массы которых М и т(М > т), под- няты на одинаковую высоту над полом и одновременно отпущены. Одновременно ли они упадут на пол, если сила сопротивления воздуха для обоих тел одна и та же? Для простоты будем полагать, что эта сила постоянна. Читатель: Поскольку сила сопротивления воздуха для обоих тел одинако- ва, то ее можно не принимать во внимание. Следовательно, оба тела одновременно упадут на пол. Автор: Вы ошибаетесь. Вы не имеете права не учитывать силы сопротив- ления воздуха. Рассмотрим, например, тело массой М На него дей- ствуют две силы: сила тяжести G и сила сопротивления F Равно- действующая этих сил (О — F). Отсюда, учитывая, что G = Mg, находим ускорение а = (Mg — F)/M, или а = g — F/M Таким образом, тело с большей массой имеет большее ускорение и, следовательно, скорее достигнет пола. Мне хочется еще раз подчеркнуть, что при вычислении ускорения тела необходимо учитывать все силы, приложенные к телу, т. е. необходимо находить равнодействующую сил. В этой связи следует осудить приме- нение термина «движущая сила». Этот термин неудачен. Применяя его к какой-то силе (или нескольким силам), мы как бы выделяем роль этой силы (этух сил) в сообщении телу ускорения. Можно подумать, что другие силы менее существенны. Это совершенно неверно. Движе- ние тела есть результат действия всех без исключения сил, приложен- ных к телу (плюс, конечно, начальные условия). Рассмотрим пример на третий закон Ньютона Лошадь тянет телегу. В результате лошадь и телега движутся с некоторым ускорением. Из третьего закона Ньютона следует, что с какой силой лошадь тянет телегу, с такой же точно силой, но в обратную сторону, телега дей- ствует на лошадь. Почему же в таком случае лошадь и телега все- таки движутся с ускорением? Читатель- Я никогда не задумывался над этим, однако не вижу здесь никаких противоречий Наличие ускорения было бы трудно объяснить, если бы сила, с которой лошадь действует на телегу, уравновешива- лась силой, с которой телега действует на лошадь Но эти силы не могут взаимно уничтожаться, поскольку они приложены к разным телам: одна к лошади, другая к телеге Автор: Ваше объяснение верно для случая, когда телега не привязана к лошади, лошадь отталкивается от телеги, в результате чего телега дви- жется в одну сторону, а ло- шадь в другую Я же предло- жил Вам иной случай: ло- шадь и телега связаны друг с другом, они движутся вместе как единая система (рис. 5.4) Силы взаимодействия телеги и лошади приложены к разным частям одной и той же системы и при движении 34
этой системы как целого они могут рассматриваться как взаимно уравновешивающиеся силы. Читатель: Тогда я не понимаю, в чем здесь дело. Может быть, здесь действие не полностью уравновешивается противодействием? Все-таки лошадь — это живой организм... IflTop: Не фантазируйте. Стоило Вам встретиться с затруднением, как Вы уже готовы принести в жертву один из основных законов механики. Чтобы ответить на мой ворос, не надо «пересматривать» третий закон Ньютона. Напротив, давайте положим этот закон в основу наших рас- суждений. Согласно третьему закону, взаимодействие лошади и телеги не может привести к движению этой системы как целого (точнее: не может сообщить ускорение системе как целому). В таком случае необ- ходимо существование дополнительного взаимодействия. Иными сло- вами, кроме лошади и телеги в задаче должно участвовать по крайней мере еще одно тело. Им является Земля. В результате имеется уже не одно, а целых три взаимодействия: 1) лошади и телеги (сила 7о); s2) лошади и Земли (сила F) — лошадь отталкивается от поверхности Земли; 3) телеги и Земли (сила f) — трение телеги о поверхность Зем- »ли. На рис. 5.4 изображены три тела; лошадь, телега, Землж. К каждому Из них приложено по две силы — результат взаимодействия данного тела с двумя другими. (Для простоты на рис. 5.4 не показаны сила тя- жести и сила реакции опоры, так как эти силы взаимно уравновешены и В рассматриваемом движении не участвуют.) Ускорение системы ло- <,шадь — телега вызывается равнодействующей всех сил, приложенных к этой системе. Этих сил четыре, их равнодействующая (F — ~f). Она и 'обусловливает ускорение рассматриваемой системы. Как видите, это ускорение не связано с взаимодействием телеги и лошади. \татель: Получается, что земная поверхность является не просто местом, На котором происходят те или иные события, но и активным «участни- жом» событий. ^гор: Ваше образное замечание справедливо. Кстати говоря, поставьте лошадь с телегой на идеальный лед, исключите тем самым горизонталь- ное взаимодействие этой системы с Землей, и никакого движения не •Судет. Следует особо подчеркнуть: никакие внутренние взаимодействия не «могут сообщить ускорение системе как целому, для этого обязательно 'необходимы воздействия извне (нельзя самого себя поднять за воло- сы). Это есть важное практическое следствие из третьего закона Ньютона. КАК ВЫ РЕШАЕТЕ КИНЕМАТИЧЕСКИЕ ЗАДАЧИ? фвтор: С некоторой высоты падают два тела — одно без начальной скорости, другое с некоторой начальной скоростью, направленной гори- зонтально. Сравните времена падения указанных тел. Сопротивлением воздуха здесь и далее будем пренебрегать. Читатель: Движение тела, брошенного горизонтально, можно рассматри- вать в виде совокупности двух движений: по вертикали и горизонтали. Время полета связано с вертикальной составляющей движения. Так как вертикальное перемещение тел в обоих случаях определяется 35
Если вы хорошо знаете механику, то вы легко решаете задачи. Не менее очевидно и обратное заключение: если вы легко решаете задачи, то вы хорошо знаете механику. Поэтому решайте как можно больше задач. одними и теми же данными (одной и той же высотой, отсутствием верти- кальной составляющей начальной скорости), то отсюда следует, что время падения обоих тел одно и то же. Оно равно y2H/g, где И — высо- та исходного положения. Автор: Совершенно верно. Д теперь рассмотрим более сложный случай Предположим, что два тела падают с высоты Н без начальной скорости, но одно из них встречает на своем пути закрепленную площадку, на- клоненную под углом 45° к горизонту. В результате удара о площадку направление скорости тела становится горизонтальным (рис. 6.1) Место удара о площадку находится на высоте h. Сравните времена падения указанных тел. Читатель: До уровня площадки оба тела падают одно и то же время В результате удара о площадку одно из тел приобретает горизонталь- ную составляющую скорости. Однако горизонтальная составляющая скорости не может повлиять на вертикаль- ную составляющую движения тела. Отсюда следует, что время падения тел и в этом слу- чае одинаково. Автор: Это неправильный ответ. Вы правы в том, что горизонтальная составляющая ско- рости не влияет на вертикальное перемеще- ние тела, а следовательно, и на время его па- дения Однако удар о площадку приводит не только к появлению горизонтальной состав- ляющей скорости тела, но и к исчезновению 36
вертикальной составляющей скорости. Это, конечно, не может не от- разиться на времени падения тела. Читатель: А почему в результате удара о площадку исчезает вертикаль- ная составляющая скорости? Автор: При упругом ударе о закрепленную площадку скорость тела после удара равна по модулю скорости тела до удара. Направление же ско- рости после удара определяется правилом «угол падения равен углу отражения». Но продолжим рассмотрение задачи. При ударе о площадку тело теряет вертикальную скорость и падает с высоты h без начальной вертикаль- ной скорости. Площадка задерживает вертикальное перемещение тела и вследствие этого увеличивается время падения. Время падения тела, не испытавшего удара о площадку, равно ~\[2H/g, время падения тела, испытавшего удар о площадку, равно V2 (Я — h)/g + ~\l2h/g. В связи с этим я хочу задать следующий вопрос: при каком отношении высот h/H время падения тела оказывается максимальным? Иначе го- воря, на какой высоте надо поместить площадку, чтобы она наиболее эффективно задерживала падение тела? Читатель: Я затрудняюсь дать точный ответ. Мне кажется, что отношение h/Н не должно быть близко ни к 1, ни к 0, так как обращение этого отношения в 1 или в 0 эквивалентно просто отсутствию площадки. Площадку надо расположить где-то посередине между поверхностью земли и исходной верхней точкой. Автор: Ваши качественные замечания справедливы. Нетрудно получить и точный ответ. Запишем время падения тела: t = x[2H/g(x/1 — х + х[х), где х ~ h/H Надо найти х, при котором функция f (х) максимальна. Читатель: Я помню из курса алгебры, что для нахождения максималь- ного значения функции надо взять производную и приравнять ее ну- лю; но я затрудняюсь в вычислении производной от такой сложной функции. Автор: В данном случае можно обойтись без нахождения производной. Возведем время падения в квадрат: t2 = 2Н (1 + 2л/(1 -х) х) /g. Если время максимально, то максимален и квадрат времени. Из по- следнего равенства видно, что значение t2 максимально тогда, когда максимальна функция у — (1 — х) х. Таким образом, задача сводится к исследованию на максимум квадратного трехчлена У = — х2 х = — (х — '/г)2 -Ь 1 /4- Этот трехчлен имеет максимальное значение при % = ’/2. Итак, высота h должна быть вдвое меньше высоты Н. Дальнейшее рассмотрение ха- рактерных приемов решения кинематических задач проведем на при- мере тела, бросаемого под углом к горизонту. Читатель: Я не очень хорошо разбираюсь в подобных задачах. Автор: Начнем с простой задачи: тело брошено под углом а к горизонту с начальной скоростью v0. Надо определить время Т полета, макси- мальную высоту Н подъема и дальность L полета. Как обычно, задача 37
начинается с выявления сил, действующих на тело. На тело действует только сила тяжести, поэтому в горизонтальном направле- нии оно перемещается равномерно, а в верти- кальном — равноперемен- но с ускорением g. Будем рассматривать вертикаль- ную и горизонтальную со- ставляющие движения те- ла по отдельности, для этого разложим вектор на- чальной скорости на вертикальную (v0sin а) и горизонтальную (u0cos а) составляющие. Горизонтальная составляющая скорости тела во время полета постоянна, тогда как вертикальная составляющая скорости из- меняется, что показано на рис. 6.2. Начнем рассматривать вертикаль- ную составляющую движения. Время полета Т = 1\ + Г2, где — время подъема, Т2 — время спуска. Вертикальная скорость тела в наи- высшей точке траектории (в момент времени t = 7\) равна, очевидно, нулю. С другой стороны, эта скорость может быть выражена с помощью формулы зависимости скорости равнопеременного движения от време- ни: 0 = v0 sin а — gT}. Знак минус объясняется тем, что ускорение g противоположно по направлению оси У. Отсюда получаем vn sin а (6.1) о Зная Г], находим Н = (v0 sin а) — gT^/2, или v* sin2 а Н= 2Г~ (6.2) Время Т2 спуска можно вычислить, рассмотрев падение тела с извест- ной высоты Н без начальной вертикальной скорости: Сравнивая этот результат с (6.1), видим, что время спуска равио времени подъема. Полное время полета 2vn sin а T=~^g------' (6'3) Для нахождения дальности L полета необходимо обратиться к горизон- тальной составляющей движения тела. Как уже отмечалось, по гори- зонтали тело перемещаемся равномерно. Отсюда находим L = (v0 cos а) = (ио sin 2а)/g. (6.4) Из (6.4) видно, что если углы бросания тела в двух случаях состав- ляют в сумме 90°, то при равенстве модулей начальных скоростей тело упадет в одну и ту же точку (см. рис. 5.1). 38
Уитатель: Мне хотелось бы уточнить вопрос о траектории тела в рассмат- риваемой задаче. Нам говорили, что эта траектория — парабола. Мож- но ли это показать? 'Автор: Да, конечно. Рассмотрим движение в системе координат, показан- ной на рис. 6.2. Характер составляющих движений по горизонтали и вертикали мы уже выяснили. Поэтому можно записать следующие выражения для зависимости координат хну тела от времени (уравне- ние движения): х = (и0 cos а) /, у = ( Vq sin а) t — gt2/2. (6.5) Исключая время из этих уравнений, находим У = - — х2 + х tg а 2v2 cos2 а Это есть уравнение параболы. Читатель: Уравнение (6.5) для у имеет вид, отвечающий равнозамед- ленному движению. Однако во время спуска тело движется по верти- кали равноускоренно Можно ли и тогда пользоваться Вашим урав- нением? Автор: Можно. Действительно, обозначим: у0 — координата тела в наи- высшей точке траектории и т — время, отсчитываемое от момента прохождения этой точки. Легко видеть, что во время спуска тела Уо — У = /2- Так как т=/—-T} = t—uosina/g и zyo=//=UoSin2a/(2gj, то u2sin2a/(2g)-— — у = (g/2) (/ — v0 s*n a/g)2, или окончательно у = (vQ sin a) t — gt2/2. Все ли Вам понятно в задаче? Читатель: Да, все понятно. Автор: В таком случае усложним задачу. Предположим, что на тело массой М действует с постоянной силой F попутный ^горизонтальный ветер. Требуется, как и в предыдущем случае, найти время Т полета, высоту Н и дальность L. Читатель: В отличие от предыдущей задачи горизонтальное перемещение тела не является теперь равномерным: тело движется по горизонтали с ускорением а — F/М. Автор: Изменилось ли что-нибудь в вертикальной составляющей движе- ния тела? Читатель: Поскольку сила ветра действует горизонтально, то она не может влиять на вертикальное перемещение тела. Автор: Хорошо. А теперь сообразите, какие из искомых величин должны иметь такие же значения, как и в предыдущей задаче. Читатель: Очевидно, время Т полета и высота Н. Ведь эти величины определяются из рассмотрения вертикального перемещения тела. По- этому они будут такими же, как и в предыдущей задаче. 'Автор: Прекрасно. Вам остается найти дальность полета. 39
Читатель: Зная по горизонтали ус- корение и время полета, находим дальность полета L=(v0 cos а) Т -[-аТ2/2= = vl/g' sin2a+2/:’uo sin2a/ Mg2). Автор: Правильно. Только ответ лучше записать в ином виде: v2n sin 2a / f \ L = —T~ (1+wtga) (66) Далее рассмотрим задачу: тело массой М бросают под углом а к на- клонной плоскости, которая образует с горизонтом угол р (рис. 6.3) Нанальная скорость тела равна vQ. Найти расстояние L от точки броса- ния до точки падения тела. Читатель: Я уже пробовал как-то решить подобную задачу. У меня ничего не получилось. Автор: Не замечаете ли Вы некоторое сходство между этой задачей и предыдущей задачей, в условии которой на тело действовал ветер? Читатель: Нет, не замечаю. Автор: Давайте выберем ось Ох вдоль наклонной плоскости, а ось Оу перпендикулярно ей (рис. 6.4, а). Разложим векторы начальной скоро- сти v и силы тяжести G на направления этих осей: = уо C0S a’ U0y = vo s*n a> Gx — G sin $,Gy=G cos p. Легко видеть, что получена предыдущая задача. Но в отличие от нее здесь ускорение по оси Ох вызывается не силой ветра, а составляющей силы тяжести G sin р, а ускорение по оси Оу силой G cos р. Поэтому для определения расстояния L можно воспользоваться результатом (6.6) при условии, что в нем будут произведены замены: F Mg sin р, g g cos p. Таким образом, находим 40
При р = 0 этот результат совпадает с выражением (6.4). Интересно указать еще один способ решения предложенной задачи. Введем оси координат Ох и Оу с началом в точке бросания тела (рис. 6.4, б). Профиль наклонной плоскости описывается в этих осях линейной функцией у{ = — х tg р, а траектория тела описывается пара- болой z/2 = ах2 + Ьх, где а и Ь — некоторые коэффициенты, выражаю- щиеся через и0, аир. Найдем координату хА точки пересечения А функ- ций у{ и z/2> Для чег0 приравняем эти функции: — х tg р = ах2 + Ьх. Отсюда получаем хА — (tg р + &)/( — а). Зная хА, находим, искомое расстояние |(?Л| = L: L = Ха = fg Р + cos р — a cos Р Остается выразить коэффициенты аи & через и0, аир. Для этого сле- дует взять две точки параболы — точки Б(хб, уБ) и В(хв, ув) (см. рис. 6.4, б). Запишем для каждой из этих точек уравнение параболы: Учв — ахв + Ьхв> У2Б = «4 + ЬхБ- (6-8) Эта система позволяет определить коэффициенты а и Ь. Я предлагаю Вам на досуге самостоятельно довести это решение до конца и привести его к виду (6.7). Читатель: Мне больше нравится первый способ решения. [втор: Это дело вкуса. Указанные два способа решения существенно раз- личны по своему характеру. Первый способ можно назвать «физи- ческим». Он использует характерное для физического подхода модели- рование (мы немного изменили точку зрения и свели нашу задачу к рассмотренной ранее задаче с ветром). Второй способ может быть назван «математическим». Здесь использовались две функции и нахо- дились координаты точки пересечения этих функций. На мой взгляд, первый способ более изящен, нежели второй. Но зато он обладает меньшей общностью. Область применимости второго способа заметно шире. Так, его можно использовать в случае, когда профиль горы, с которой бросают тело, не является прямым. При этом вместо линей- ной функции у{ будем иметь какую-то другую функцию, отвечающую данному профилю горы. Первый же способ в подобных случаях принци- пиально непригоден. В связи с этим напомним, что более широкая область применимости математических методов связана с тем, что они более абстрактны. Читатель: А нельзя ли рассмотреть задачу, где участвуют два тела, сталкивающиеся в полете? \втор: Я Вам предложу следующую задачу: тело А бросают вертикально вверх со скоростью и0А = 20 м/с. На какой высоте Н находилось тело Б, которое, будучи брошено с горизонтальной скоростью и0Б = 4 м/с одно- временно с телом А, столкнулось с ним в полете? Расстояние по гори- зонтали между исходными положениями тел равно I = 4м. Найти также время Т движения тел до столкновения и скорость v каждого тела в момент столкновения. Читатель: Наверное, надо сначала составить уравнения движения. Прав- 41
Рис 6 5 да, не совсем понятно, как выбрать оси ко- ординат; ведь тела начинают движение из разных точек. Автор: На тела действует только сила тяжести, поэтому проще всего выбрать систему ко- ординат, показанную на рис. 6.5, связав на- чало отсчета с поверхностью земли; ось Оу — вертикальная, ось Ох — горизонталь- ная. Напишите теперь уравнения движения для каждого тела. Читатель: Тело А из точки с координатами х0А = — Ум = 0 движется вдоль оси Оу равнопе- ременно с начальной скоростью цол, коорди- ната его по оси Ох не изменяется; я запишу хА = /, Уа — им1\ £1/2. Автор: Это правильно. Читатель: Тело Б из точки с координатами х0Б = 0 и у0Б = Н движется вдоль оси Ох равномерно, а вдоль оси Оу равноускоренно, тогда ХБ = и0Б^2^ . Уб = £^/2- Автор: В последнем уравнении Вы сделали две ошибки. Во-первых, не учли, что тело Б в начальный момент времени находилось на высоте Н. Во-вторых, вектор ускорения g направлен в отрицательном направле- нии оси Оу, его проекция на ось Оу отрицательна. Вот правильное урав- нение: Уб = Н - «4/2- Читатель: А что делать дальше? В наших уравнениях очень много не- известных. Автор: Тела начали движение одновременно, поэтому t{ = t2. Известно также, что тела столкнулись в полете; следовательно, в момент столк- новения при t=T равны их х- и //-координаты. Таким образом, ( = и0Б?’ (vMT-gT1/2=H-gT1/‘l. Читатель: Получилась система из двух уравнений с двумя неизвестными. Ее легко решить: Г = 1/и0Б = 1 с; Н = и0АТ = и0А1/и0Б = 20 м. Автор: Правильно. Найдите теперь скорости тел в момент столкновения. Читатель: Скорость тела, движущегося из точки А, определяется v= v0 — gt (эту формулу можно получить, воспользовавшись тем, что скорость тела равна производной координаты по времени). В нашей задаче: иА = У0Л ё1/и0Б — м/с. 42
Определить скорость тела Б я затрудняюсь. $.втор: У тела Б две составляющие скорости: иБх — и0Б’ иБу — gt- Результирующая скорость определится по теореме Пифагора, так как обе составляющие перпендикулярны друг другу: V = + V<5y — и0Б + (gT) ~ 10,6 м/с. (АДАЧИ |Из точек А и Б, находящихся на высотах соответственно 2 и 6 м, одновременно сают навстречу друг другу два тела: одно горизонтально со скоростью 8 м/с, гое вниз под углом 45° к горизонту с такой начальной скоростью, чтобы оба а столкнулись в полете. Расстояние между точками А и Б по горизонтали но 8 м. Вычислить начальную скорость тела, брошенного под углом 45°, рдинаты х и у точки столкновения, время Т движения тел до столкновения корости vА и v Б обоих тел в момент столкновения. Траектории тел лежат в одной (СКОСТИ. Из одной точки брошены два тела под углами а, и а2 к горизонту с началь- ли скоростями соответственно v, и и? На каком расстоянии L друг от друга ,ут находиться тела через время /? Рассмотреть два случая: 1) траектории тел <ат в одной плоскости, причем тела брошены в разные стороны; 2) траектории лежат во взаимно перпендикулярных плоскостях. , Тело падает с высоты Н без начальной скорости. На высоте h оно упруго 1ряется о закрепленную площадку, расположенную под углом а = 30° к гори- ту. Найти время Т падения тела и дальность L полета. , Под каким углом а к горизонту надо бросить тело массой М, чтобы максималь- [ высота его подъема была равна дальности полета? На тело действует с по- янной силой F горизонтальный попутный ветер. , На наклонной плоскости с углом 0 к горизонту бросают камень с начальной ростью v0перпендикулярно плоскости. На каком расстоянии L отточки бросания щет этот камень? . Мальчик ростом h — 1,5 м, стоя на расстоянии L — 15 м от забора высотой = 5 м, бросает камень под углом а = 45° к горизонту. С какой минимальной •ростью У0надо бросать камень, чтобы он перелетел через забор? , С башни высотой Н = 3,48 м под углом а, = 30° к горизонту вниз бросают ка- ib со скоростью v г Одновременно с поверхности земли под углом а2 = 30° к гори- ту бросают второй камень со скоростью v2 навстречу первому. На каком рас- стоянии L от подножия башни находится место бросания второго камня, если оба камня столкнулись в воздухе? т.8. Шарик бросают под углом а = 30° к горизонту с начальной скоростью ,р0 = 14 м/с. На расстоянии I = 11 м от точки бросания шарик упруго ударяется вертикальную стенку. На каком расстоянии D от стенки шарик упадет на землю? ®.9. Тело бросили с высоты И = 19,6 м горизонтально со скоростью v0= 10 м/с. Тело упруго ударилось о землю, а затем о вертикальную стенду, расположенную На расстоянии I = 40 м по горизонтали от места бросания. Определить максималь- ную высоту h, ‘на которую поднимается тело после удара о стенку. На каком рас- стоянии s от стенки тело упадет на землю? 6.10. Вверх по горе с углом наклона к горизонту £ = 30° едут с постоянной одина- ковой скоростью v = 14 м/с грузовик и за ним легковой автомобиль. Определить, на каком минимальном расстоянии s от грузовика должен ехать автомобиль, чтобы камень, застрявший в промежутке между задними спаренными колесами грузовика, не попал в него. 43
7. КАК ВЫ РЕШАЕТЕ ЗАДАЧИ НА ЗАКОНЫ НЬЮТОНА? Автор: При решении так называемых «динамических» задач особенно важно уметь правильно выявлять силы, приложенные к телу (см § 3). Читатель: В связи с этим я хочу задать один вопрос. Предположим, что я правильно выявил все силы, приложенные к телу. Что следует делать дальше? Автор: Если векторы сил не направлены вдоль одной прямой, то их следует разложить на два взаимно перпендикулярных направления и рассмот- реть составляющие сил отдельно для каждого из этих направлений, которые будем называть «направлениями разложения». При этом мне хочется сразу же дать несколько практических советов. Во-первых, чтобы не запутаться, при выполнении разложения сил следует силы изображать на чертеже крупным планом, не мельчить. Обычно уча- щиеся стремятся изобразить силы в виде микроскопических стрелочек. Сравните в связи с этим свой рисунок (см. рис. 3.2) и мой рисунок (см. рис. 3.3). Во-вторых, не торопитесь разлагать векторы сил раньше времени. Сначала надо выявить все без исключения силы, приложенные к телу, и изобразить их на рисунке. Только после этого можно присту- пать к разложению тех или иных сил. В-третьих, помните, что, после того как Вы разложили какую-нибудь силу, надо «позабыть» о сущест- вовании этой силы и пользоваться вместо нее ее составляющими. Или сама сила, или ее составляющие. Читатель: А по какому принципу выбираются направления разложения? Автор: При выборе направлений разложения надо обратить внимание на характер движения тела. Возможны два варианта: 1) тело покоится или движется равномерно и прямолинейно; 2) тело движется с ускоре- нием, причем направление ускорения известно (по крайней мере с точ- ностью до знака). В первом случае Вы можете выбирать направления разложения произ- вольно, руководствуясь (или не руководствуясь) соображениями прак- тического удобства. Рассмотрим следующую задачу: определить силу, при действии которой тело, находящееся на наклонной плоскости, будет равномерно перемещаться вверх (см. рис. 3.4). Коэффициент трения скольжения k и угол а известны. Решая эту задачу, с одинаковым успехом можно выбрать как вертикальное и горизонтальное направле- ния разложения (рис. 7.1, а), так и направления разложения вдоль наклонной плоскости и перпендикулярно ей (рис. 7.1, б). После того как разложение сил выполнено, надо приравнять нулю алгебраические суммы составляющих сил для каждого из направлений разложения. Получаем системы уравнений соответственно для случаев, показанных на рис. 7.1, а, б: {К cos а — Етр sin а — G = О, F — Етр cos а—N sin а = 0; ( N — G cos а — F sin а= 0, | Етр + G sin а — F cos а — 0. Читатель: Эти системы уравнений различны. 44
Рис 7 1 Цртор; Но они приводят к одинаковым результатам. Учитывая, что FTp — = kN [см. (3.1)], находим ( N (cos а — k sin а) — G — О, | F — N (k cos а + sin а) = 0. Из первого уравнения этой системы находим N = G/(cos а — /г sin а) Подставляя этот результат во второе уравнение, получаем ответ: k cos а -|- sin а г = G--------г----. ' cos а — я sin а Такой же точно ответ получается из выражений (7.2), в чем нетрудно убедиться самостоятельно. Читатель: А как быть, если тело движется ускоренно? $втор: В этом случае выбор направлений разложения сил зависит от на- правления ускорения тела (направления равнодействующей сил). Сле- дует разлагать силы на направления вдоль ускорения и перпендику- лярно ему. При этом алгебраическая сумма составляющих сил на на- правление, перпендикулярное ускорению, приравнивается нулю, а ал- гебраическая сумма составляющих сил на направление вдоль ускоре- ния, согласно второму закону Ньютона, равна произведению массы тела на ускорение. Обратимся к задаче с телом на наклонной плоскости, полагая, что тело движется по плоскости с каким-то ускорением вверх. В соответ- ствии со сделанными замечаниями следует произвести разложение сил так же, как и в случае, изображенном на рис. 7.1, б. При этом вместо системы (7.2) будет следующая система уравнений (G = mg): ( N — mg cos а — F sin а = 0, | F cos а — Fjp — mg sin а = ma. Используя выражение (3.1), определяем ускорение тела: а = [F cos а — (mg cos а + F sin а) k — mg sin a]/m. (7.3a) Читатель: Можно ли при наличии ускорения производить разложение сил 45
a Рис. 7 2 на направления, отличные от на- правлений вдоль ускорения и перпендикулярно ему? Насколь- ко я понял из Ваших разъясне- ний, этого вроде бы делать нельзя? Автор: В связи с Вашим вопросом я должен уточнить свои разъясне- ния. Конечно, даже при наличии ускорения Вы вправе разлагать силы на любые два взаимно перпенди- кулярных направления. Но тогда придется производить разложение не только сил, но и вектора ускорения. При этаком способе решения Вам встретятся дополнительные трудности. Чтобы избежать их, делайте именно так, как я Вам советовал. Вы всегда знаете в задаче направле- ние ускорения тела (хотя бы с точностью до знака), вот и исходите из этого направления. Неумение экзаменующихся рационально выбрать направление разложения сил — одна из причин их беспомощности при решении более или менее усложненных задач по динамике. Читатель: Мы говорим все время о разложении на два направления. Но, наверное, в общем случае следует говорить о разложении на три взаимно перпендикулярных направления? Ведь пространство трех- мерно. Автор: Вы абсолютно правы. Два направления в наших рассуждениях объясняются тем, что мы рассматриваем задачи в плоскости (двумер- ные задачи). В общем случае надо разлагать на три направления. При Этом все замечания, сделанные выше, сохранят силу. Правда, я должен заметить, что, как правило, на экзаменах предлагаются двумерные задачи. Хотя, конечно, возможны несложные обобщения на трехмерный случай. Читатель: Часто встречаются задачи, где рассматривается движение не одного, а нескольких тел, связанных невесомой нерастяжимой нитью. В таких задачах требуется ндйти ускорение системы тел, а также силы натяжения нити (или нитей, еслй их несколько). Насколько я теперь понимаю, надо сначала выявить все-силы, которые приложены к каждому телу в отдельности. Автор: Именно так и надо поступить. При этом, естественно, к опре- деленным телам будет приложена дополнительная сила со Стороны нити, ее часто называют силой натяжения нити, хотя точнее было бы говорить о силе реакции нити, поскольку это есть сила, с которой нить действует на тело, а не наоборот. Когда говорят о невесомой нити, то подразумевают под этим тот факт, что массой нити можно пренебречь, и поэтому можно считать, что нить действует с одинаковыми по модулю силами на оба тела, которые она соединяет. Читатель: Я полагал, что нить всегда действует с одинаковой силой на эти тела; причем здесь масса нити? Автор: Для пояснения рассмотрим рис. 7.2. Тела 1 и 2 соединены нитью массой т. Благодаря наличию силы F система трех тел (оба тела плюс нить) движется с некоторым ускорением а, направленным слева направо. Будем считать, что трения нет и взаимно скомпенсированы все силы, кроме тех, что показаны на рисунке. Ведь этот рисунок мы приводим лишь для того, чтобы пояснить определенную мысль. Мы 46
видим, что нить действует на тело 1 с силой Г, ис такой же (по мо- дулю) силой действует это тело на нить. На тело 2 нить действует с си- лбй Г2; с такой же (по модулю) силой тело 2 действует на нить. Можно написать, что та — Т2 — 1\. Если же пренебрегать массой нити (т — 0), то из приведенного выше равенства следует, что 1\ — Т2. Читатель: Я Вас понял. Автор: Тогда вернемся к вопросу, как решать задачи. Итак, Вы выявили все силы, приложенные к каждому телу в отдельно- сти. Что надо делать дальше? Читатель: Наверное, надо, используя второй закон Ньютона, составить уравнения движения для каждого тела. Автор: И, кроме того, необходимо учесть направление ускорения каждого тела и тот факт, jto модуль ускорения для каждого тела в системе один и тот же. Напишите уравнение для тел, представленных на рис. 3.6, предполагая, что тело 3 опускается вниз. Читатель: Я воспользуюсь рис. 3.7, на котором мы уже выявили все силы, приложенные к телам, причем направление силы трения как раз соответствует движению тел 1 и 2 вверх по наклонной плоскости: для тела 1(G' = mTg) имеем Г Г — m{g sin a — Frp = m{a, ( N' — m{g cos а = 0; для тела 2( G" = m2g) Г Т" — Т’ — F^p — m2g sin а = m2a, ( 2V" — m2g cos а = 0; для тела 3( G'" — m3 g) mAg — Т” = mAa. (7.6) Автор: Далее учтем, что F1V ~ kN', FTp = kN" [см. (3.1)]. Используя эти соотношения и системы (7.4) — (7.6), приходим в итоге к системе урав- нений: {T' — mxg sin a— km{g cos a=m{a, T"— у— m2g sin a—km2g cos a=m2a, (7.7) mAg—T" — mAa. Складывая почленно правые и левые части уравнений системы (7.7), получаем ускорение m , — (m. 4- m 2) (sin a 4- k cos a) z = g------i------------------------ (7.8) m j + rn1 + 7 Теперь найдем силы натяжения нитей [подстановкой (7.8) в (7.7)]: тА (т 2 + т ]) 0 + sin a + k cos a) g tn, + m2 mA путл(\ 4- sin a 4- cos a) g m! 4- m2 4~ m i 47
Рис 7 3 t = 10 с после начала действия силы и ЗАДАЧИ 7.1. По горизонтальной плоскости дви- жется тело массой М — 5 кг под дейст- вием силы F = 30 Н, приложенной к телу под углом а — 30° к горизонту. Коэффициент трения скольжения k = = 0,2 Вычислить скорость v тела через 1боту А против силы трения за указан- ное время 7.2. Человек везет двое связанных между собой саней, прикладывая к веревке силу F — 120 Н под углом а = 45° к горизонту (рис 7 3) Массы саней одинаковы и равны т = 15 кг Коэффициент трения полозьев по снегу k = 0,02. Найти ускоре- ние а саней и силу натяжения Т веревки между санями, а также силу F , с которой должен тянугь веревку человек для того, чтобы сани двигались равномерно. 7.3. Через неподвижный блок перекинута нить, к которой подвешены три одина- ковых груза массой М = 2 кг каждый (рис 7 4) Найти ускорение а системы и силу натяжения Т нити, связывающей грузы 1 и 2 - 7.4. Вычислить ускорение а грузов и силы натяжения Т нитей в случае, изображен- ном на рис 7 5 Дано а = 30°, т} = 4 кг, /п2 = 2 кг, тл = 8 кг Трением о плоскость пренебречь 7.5. Дана система грузов (рис 7 6, Afj = 1 кг, М2 = 2 кг, М3 — 5 кг, М4 = 0,5 кг, а — 30°) Коэффициент трения грузов о плоскость k ~ 0,2 Найти ускорение а си- стемы грузов, силы натяжения Т нитей и силу f, с которой груз М4 давит на груз М3 7.6. Дана система грузов (рис 7 7, известно, что т} = 2т, т2 = Зт, тл = т) Вычислить ускорения грузов а и силы натяжения Т нитей 8. НАСКОЛЬКО УСЛОЖНЯЕТ РЕШЕНИЕ ЗАДАЧ УЧЕТ СИЛЫ ТРЕНИЯ? Автор- Учет силы трения может весьма сильно усложнить решение задач. Читатель: Но мы уже’обсуждали вопрос о силе трения (см. § 4). Если тело движется, то сила трения определяется через силу реакции опоры (Етр = kN)-, если же тело покоится, то сила трения равна по модулю той силе, которая стремится вывести тело из состояния покоя. Все это нетрудно понять и запомнить. Автор: Это так. Однако Вы упускаете из виду одно важное обстоятель- ство. Вы предполагаете, что заранее известны ответы на вопросы: 1) покоится или движется тело; 2) в какую сторону движется тело. Рис 7 4 48 Рис 7 5 Рис 7 6
Если это заранее известно, то тогда действительно //////, все относительно просто. Однако если это неизве- I стно, то задача сразу становится сложной и требу- Гр ет специальных исследований. О Читатель: Да, я вспоминаю. Мы с Вами говорили об этом в § 3 в связи с обсуждением выбора направ- ления силы трения. Автор: Теперь я хочу остановиться на этом вопросе JL подробнее. Я полагаю, что вопрос об усложнениях, wiU вносимых в динамические задачи учетом силы тре- ГР ния, явно недооценивается как учащимися, так и некоторыми составителями задач. Рассмотрим пример: тело находится на наклонной ± плоскости (рис. 3.4). Даны угол а наклона плос- т2>-1 кости, силы G и F, коэффициент трения скольже- О ния k Для простоты будем полагать, что k0 = k (здесь' k0 — коэффициент, определяющий макси- рис 77 мально возможную силу трения покоя). Требуется выяснить характер движения тела, найти ускоре- ние. Предположим, что тело движется вверх по наклонной плоскости. Раз- ложим все силы на направления вдоль плоскости и перпендикулярно ей (рис. 7.1, б) и воспользуемся результатом (7.3а), полученным для ускорения в § 7: а = g [F cos а— G sin а— (G cos а-f- F sin a) k]/G. (8.1) Из (8.1) следует, что для движения тела вверх по наклонной плоскости необходимо, чтобы выполнялось условие F cos а — G sin а — (G cos а -f- F sin а) k О Перепишем его в виде _ _ „ k cos а -f- sin а G--------7----, cos а — я sin а или (8 2) 1 — я tg а При этом предположим, что угол наклона плоскости не слишком велик, так что (1 — k tg а) >• 0, или tga<l/Ai. (8.3) Далее допустим, что тело движется вниз по наклонной плоскости. Раз- ложим все силы в соответствии со схемой на рис. 7.1, б, но теперь на- правим силу трения в противоположную сторону. В результате полу- чаем для ускорения тела следующее выражение: а = g [ G sin a — F cos a — (G cos a -f- F sin a) k]/G. (8.4) Из (8.4) следует, что для движения тела вниз по наклонной плоскости необходимо, чтобы выполнялось условие G sin a — F cos a — (G cos a -f- F sin a) k 0. 49
Перепишем его в виде „ _ „ sin а — k cos а F < G---------—--------, cos а + k sin а или tga-fe 1 + k tg а' (8.5) При этом предположим, что угол наклона плоскости не слишком мал, так что (tg а — /г) > О, или tga>/j. (8.6) Объединяя условия, выраженные неравенствами (8.2), (8.3), (8.5), (8.6), можно сделать следующие заключения: 1. Пусть для данной задачи выполняется условие k < tg a< \/k, тогда: ' а) если F > G (k + tg a)/( 1 — k tg a), то тело движется вверх с ускоре- нием (8.1); б) если F = G (/г + tg a)/(l — k tg а), то тело движется вверх равно- мерно или покоится; в) если F < G (tg а — /г)/( 1 + k tg а), то тело движется вниз с ускоре- нием (8.4); г) если F — G (tg а—&)/(! 4- k tg а), то тело движется вниз равно- мерно или покоится; д) если G (tg«a — /г)/(1 + k tg а) < F < G (k + tg а)/(1 — k tg а), то тело покоится. Заметим, что с увеличением силы F от G(tga — Zs)/( 1 + k tg а) до G (k + tg a)/(l — k tg а) сила трения покоя постепенно уменьшается от k (G cos a-f-F sin a) до нуля и затем, изменив направление на обратное, возрастает от нуля до k (G cos a+F sin a). Все время, пока совершается этот процесс, тело покоится. 2. Пусть для данной задачи выполняется условие О < tg a k, тогда: а) если F > G (k + tg a)/(l — k tg a), то тело движется вверх с ускоре- нием, определяемым по формуле (8.1); б) если F = G (k + tg a)/(l — k tg a), то тело движется вверх равно- мерно или покоится; в) если F < G (/г + tg a)/(l — k tg а), то тело покоится; движение тела вниз по наклонной плоскости вообще невозможно (даже если сила F обратится в нуль). 3. Пусть для данной задачи выполняется условие tg 1 //г, тогда: а) если F < G (tg a — fe)/(l + k tg а), то тело движется вниз с ускоре- нием, определяемым по формуле (8.4); 50
б) если F = G (tg а — Ze)/(1 + k tg а), то тело движется вниз равно- мерно или покоится; в) если F > G (tg а — &)/( 1 + k tg а), то тело покоится; движение тела вверх по наклонной плоскости вообще невозможно. На первый взгляд это кажется непонятным: ведь мы можем увеличивать силу F беспре- дельно! Однако наклон плоскости настолько велик, что с увеличением силы F будет сильнее возрастать эффект прижимания тела к плоскости. Читатель: Нам никогда ничего подобного не доказывали. Автор: Именно поэтому я и хотел обратить на это Ваше внимание. Конеч- но, на экзамене Вы столкнетесь, по-видимому, с более упрощенным случаем; трения вообще нет или трение есть, но заранее известен характер движения тела (например, известно, покоится тело или дви- жется). Однако, даже не плавая на глубоких местах, полезно знать, где ‘ они находятся. Читатель: А чго будет, если в Вашем исследовании положить k = О? Автор: В отсутствие трения все сразу же упростится. В этом случае для любого угла наклона плоскости будем иметь результаты: а) при F > G tg а тело движется вверх с ускорением а = g (F cos а — G sin a)/G; (8.7) б) при F — G tg а тело движется равномерно (вверх или вниз) либо покоится; в) при F < G tg а тело движется вниз с ускорением a=g(Gsina—Fcosa)/G. (8.8) Заметьте, что результаты (8.7) и (8.8) совпадают с точностью до знака. Поэтому при решении задачи можно спокойно предположить любое направление движения тела, найти а и обратить внимание на знак ускорения. Если а>0, то движение тела происходит вдоль вы- бранного направления, если же а<0, то движение тела происходит в обратном направлении (при этом ускорение равно |а|). Рассмотрим еще одну задачу. Даны два тела ml и m2, связанные нитью, перекинутой через блок. Тело находится на наклонной плоскости с углом наклона а, коэффициент трения о плоскость равен k\ тело т2 висит на нити (рис. (8.1). Найти ускорение системы. Предположим, что система движется слева направо. Рассматривая движение системы как целого, находим для ее ускорения следующее выражение: m2 ~ т\ s’n a —' ^т\ COs a В предположении, что система движется справа налево, находим mt sin a — т2 — km] cos a Проведем исследование для заданных а и k. При этом варьируется отношение p = m2/mi- Из формулы (8.9) следует, что для движения системы слева направо необходимо, чтобы выполнялось условие р sin a + k cos a. 51
Рис 8.1 Из (8.10) следует, что для движе- ния системы справа налево необхо- димо, чтобы выполнялось условие р <2 sin а — k cos а. При этом необходимо дополнитель- ное условие, чтобы угол наклона плоскости не был слишком мал: tg а >• k. Если же tg а k, то, как бы велико ни было отношение р, система не будет двигаться справа налево. Если tga>/s, то тела покоятся при выполнении неравенств sin a — /г cos a < р < sin a k cos a. Если же tga^/г, то тела покоятся при р < sin a + k cos a. ЗАДАЧИ 8.1. Два груза с одинаковыми массами связаны между собой нитью, перекинутой через невесомый блок. Плоскости, на которых находятся грузы, составляют с гори- зонтом углы аир (рис. 8.2). Найти ускорение а грузов и силу натяжения Т нити. Коэффициент трения скольжения грузов о плоскость одинаков и равен k. При каком коэффициенте трения k । грузы будут находиться в покое? 8.2. Тормозу автомобиля удерживают его на склоне горы с уклоном 0,3 Опреде- лить тормозной путь автомобиля на горизонтальной дороге, если перед торможе- нием он двигался со скоростью v = 72 км/ч. Коэффициент трения скольжения на горизонтальной дороге в п = 0,8 раз меньше, чем на склоне горы. 8.3. На наклонной плоскости с углом наклона а= 30° находится брусок массой m = 3 кг. К нему приложена сила F = 9,8 Н, направление которой показано на рис. 8.3. Определить ускорение а движения бруска и силу трения Атр, если коэффи- циент трения скольжения бруска о плоскость равен k = 0,3 8.4. Санки можно удержать на ледяной горе с уклоном 0,3 силой, не меньшей F = 60 Н, а предоставленные сами себе, они скатываются с ускорением а = 2 м/с2 Какую силу F, надо приложить к санкам, чтобы тянуть их в горку равномерно? Считать силы Р и Р i направленными вдоль наклонной плоскости. 8.5. Небольшой брусок находится на наклонной плоскости с углом наклона а. Коэффициент трения скольжения между бруском и плоскостью k. Наклонная плоскость движется с ускорением а в направлении, указанном на рис. 8.4, а, б. При каком минимальном ускорении брусок начнет скользить по плоскости? 8.6. Брусок массой М, находится на доске массой М? расположенной на горизон- тальной плоскости. Коэффициент трения скольжения между бруском и доской Рис. 8 2 52
равен k}, а между доской и плоскостью k2. Найти минимальную горизонтальную силу F, при действии которой на доску происходит сдвиг бруска относительно доски. 8.7. Три тела одинаковой массы m = 1 кг находятся друг на друге (рис. 8.5) Коэффициент трения между телами 1 и 2 равен =0,1, а между телами 2 и 3 — k2 — 0,2. Коэффициент трения между полом и телом 3 равен kA = 0,1. Тело 2 тянут с некоторой горизонтальной силой F. При какой силе F возможно такое движение этих трех тел, при котором тела 1 и 3 остаются в покое друг относительно друга? Определить ускорения а, а2 и а3 всех трех тел при этом движении. 9. СУЩЕСТВУЕТ ЛИ ЦЕНТРОБЕЖНАЯ СИЛА? Автор: Опыт показывает, что вопросы и задачи, связанные с движением тела по окружности, для многих экзаменующихся оказываются чрез- мерно трудными. В ответах на эти вопросы содержится множество характерных ошибок. Чтобы продемонстрировать это, давайте пригла- сим к участию в нашей беседе еще одного читателя, которому неизве- стно все то, о чем мы говорили до сих пор. Назовем его условно «читателем Б» (своего прежнего собеседника я буду называть теперь «читателем А»). Я предлагаю читателю Б указать силы, действующие на спутник, дви- жущийся вокруг Земли. Условимся пренебрегать сопротивлением атмо- сферы и притяжением Луны, Солнца и прочих небесных тел. Читатель Б: На спутник действуют две силы: притяжение Земли (сила тяжести) и центробежная сила. Автор: Против притяжения Земли у меня возражений нет. А вот появле- ние центробежной силы мне непонятно. Объясните. Читатель Б: Если бы этой силы не было, спутник не мог бы удержаться на орбите. Автор: И что же с ним стало бы? Читатель Б: Он упал бы на Землю. Автор (обращаясь к читателю А): Вспомни- те, что я говорил раньше! Вот пример, когда существование той или иной силы пытаются вывести не из взаимодействия тел; а обратным ходом — из характера движения тела. Видите ли, спутнику надо удержаться на орбите, значит, надо вво- дить «удерживающую» силу. Кстати го- воря, если бы эта центробежная сила дей- ствительно существовала, то именно F 2 Рис 8.5 53
Движение по окружности — простейший тип криволинейного движения. Тем более важно разобраться в специфике этого движения. Ведь мир полон криволинейных движений! Рассмотрим равномерное движение материальной точки по окружности, движение спутников, а в связи с этим обсудим причины невесомости тел. тогда-то спутник и не смог бы удержаться на орбите. Ведь в этом случае силы, действующие на спутник, взаимно уничтожились бы и спутник должен был бы лететь равномерно по прямой линии. Читатель А: Центробежная сила никогда не прикладывается к вращаю- щемуся телу. Она прикладывается к связи. А к вращающемуся телу, прикладывается центростремительная сила. Читатель Б: Получается, что к спутнику приложен только вес? Автор: Да. Только не вес, а сила тяжести. Читатель Б: И он при этом не падает на Землю? Автор: Движение тела под действием силы тяжести называют падением. Значит, спутник падает. Однако его «падение» имеет вид движения по окружности вокруг Земли и поэтому продолжается неограниченно долго. Мы уже обсуждали этот факт, что направления движения тела и действующей на него силы не обязательно должны совпадать (см. § 5). Читатель Б: Говоря о притяжении Земли и центробежной силе, я исходил из формулы 7—=—. (9.1) где в левой части равенства стоит сила притяжения (щ — масса спутника, М — масса Земли, г — радиус орбиты, считая от центра Земли, у — гравитационная постоянная), а в правой части — центро- бежная сила (и — скорость спутника). Выходит, что эта формула неверна?
Автор: Нет, эта формула верна. Неверно Ваше толкование формулы. Вы рассматриваете соотношение (9.1) как уравнение равновесия двух сил. Однако на самом деле это выражение второго закона Ньютона F = та, где F — утМ/г2, aa = v2/г — центростремительное ускорение. Читатель Б. Я согласен, что Ваше толкование позволяет обойтись без центробежной силы. Но если нет центробежной силы, то должна быть по крайней мере центростремительная сила. Однако Вы о ней не упо- минаете. Автор: В рассматриваемом случае центростремительная сила есть сила притяжения спутника к Земле. Подчеркиваю, что речь не идет о каких- то двух силах. Нет. Это одна и та же сила. Читатель Б: Но тогда зачем вообще вводят понятие центростремительной силы? Автор: Вот в этом я с Вами согласен. Я думаю, что термин «центро- стремительная сила» ничего, кроме лишней путаницы, не дает. То, что понимается под центростремительной силой, вовсё не является какой- то самостоятельной силой, приложенной к телу наряду с прочими сила- ми. Это равнодействующая всех сил, приложенных к телу, равномерно движущемуся по окружности. Величина mu2lr не является силой, а представляет произведение массы ш тела на центростремительное ус- корение о2/г. Это ускорение направлено к центру, значит, и равнодей- ствующая всех сил, приложенных к телу, равномерно движущемуся по окружности, направлена к центру. Итак, есть центростремительное ус-' корение и есть силы, которые сообщают телу центростремительное ускорение. Читатель Б: Должен сознаться, что такой подход к рассмотрению движе- ния тела по окружности мне нравится. Действительно, указанное дви- жение -- это не статический случай, для которого характерно равнове- сие сил, а динамический случай. Читатель А: Если отказаться от понятия центростремительной силы, то тогда, наверное, следует отказаться и от термина «центробежная сила» даже в применении к связи? Автор: Введение термина «центробежная сила» является неоправданным еще в большей мере. Центростремительная сила все-таки существует сила» Рис. 9.1 55
как равнодействующая сил. Центробежная же сила даже не всегда существует. Читатель А: Последнее замечание мне непонятно. Центробежная сила вводится как противодействие по отношению к центростремительной силе. Если она не всегда существует, то получается, что не всегда выполняется третий закон Ньютона? Автор: Третий закон Ньютона выполняется только для реальных сил, определяемых через взаимодействия тел, а вовсе не для равнодейст- вующих этих сил. Я поясню это утверждение на примере конического маятника (рис. 9.1). На шарик действуют две силы: сила тяжести G и сила реакции Т нити. Векторная сумма этих сил обеспечивает центро- стремительное ускорение шарика, ее и называют центростремительной силой. Сила G обусловлена взаимодействием шарика с Землей. Проти- водействием этой силы служит сила G{, она приложена к Земле. Сила Т обусловлена взаимодействием шарика с нитью. Противодействием этой силы служит сила Т}, она приложена к нити. Если формально сложить силы G{ и 1\, то получится сила, под которой принято пони- мать центробежную силу (см. пунктир на рис. 9.1). Но к чему прило- жена эта сила? Можно ли говорить о ней, как о силе, если одна из ее составляющих приложена к Земле, а другая совсем к другому телу — к нити? Ясно, что в данном случае понятие центробежной силы не имеет физического смысла. Читатель А: А в каких случаях центробежная сила существует? Автор: Например для спутника, когда взаимодействуют только два тела: Земля и спутник. Центростремительная сила есть сила притяжения спутника Землей. Центробежная сила есть сила притяжения Земли спутником. Читатель Б: Вы до сих пор говорили о равномерном движении по окруж- ности. А если тело движется по окружности неравномерно? Например, тело соскальзывает с верхушки вертикально поставленного обруча. Пока оно скользит по обручу, оно движется по окружности. Однако это движение явно не равномерное, так как скорость тела возрастает. Как тут быть? Автор: Мы касались этого в § 1. Напомним еще раз основные выводы. Если тело движется по окружности равномерно, то равнодействующая всех сил, приложенных к нему, направлена к центру — она сообщает телу центростремительное ускорение. В более общем случае неравно- мерного движения по окружности равнодействующая сил уже не на- правлена к центру. В этом случае она имеет составляющие, одна из которых направлена по радиусу к центру, а другая — по касательной к траектории тела (т. е. к окружности). Первая составляющая опреде- ляет центростремительное ускорение тела, связанное с изменением на- правления скорости; вторая — определяет тангенциальное (касатель- ное к окружности) ускорение тела, связанное с изменением модуля ско- рости. Следует подчеркнуть, что поскольку модуль скорости изменяет- ся, то должен изменяться и модуль центростремительного ускорения V2/г. Читатель Б: Значит, для каждого момента времени центростремительное ускорение определяется формулой а = и2/г, где и — мгновенная ско- рость для этого момента? 56
Автор: Совершенно верно. Если при равномерном движении по окружнос- ти центростремительное ускорение постоянно по модулю, то при нерав- номерном движении по окружности оно меняется в процессе движения тела. Читатель А: Чем следует руководствоваться, чтобы узнать, как именно изменяется скорость v при неравномерном движении тела по окруж- ности? Автор: Обычно для этой цели применяют закон сохранения энергии. Рас- смотрим конкретный пример. Пусть тело массой М соскальзывает без трения с верхушки вертикально поставленного обруча радиусом R. С какой силой оно будет давить на обруч, проходя точку, высота кото- рой меньше высоты верхушки обруча на h? Начальная скорость тела на верхушке обруча равна нулю. Прежде всего надо выяснить, какие силы действуют на тело? Читатель А: На тело действуют две силы: сила тяжести б = Mg и сила реакции опоры N. Они показаны на рис. 9.2. Автор: Правильно. Что Вы будете делать дальше? Читатель А: Я буду делать так, как Вы говорили. Найду равнодействую- щую этих сил и затем разложу ее на две составляющие: на направле- ния вдоль радиуса и по касательной к окружности. Автор: Все правильно. Однако проще, очевидно, не искать равнодействую- щую, а сразу же разложить приложенные к телу силы на два ука- занных направления, тем более что разлагать придется только одну силу — силу тяжести. Читатель А: Я выполню это разложение на рис. 9.2. Автор: Сила 62 обусловливает тангенциальное ускорение тела, она нас не интересует. Равнодействующая сил и N вызывает центростреми- тельное ускорение тела, т. е. G{—N = Mv1/R. (9.2) Скорость тела в интересующей нас точке (точка А на рис. 9.2) найдем из закона сохранения энергии: Mgh = Му2/2. (9.3) (Считаем потенциальную энергию в точке А равной нулю; см. § 13.) Объединяя (9.2) и (9.3) иучитывая, что G, — Mg cos a—Mg (R — h)/R, получаем Mg (R - h)/R - N= 2Mgh/R. Искомая сила давления тела на обруч равна в соответствии с третьим зако- ном Ньютона силе реакции опоры: N = Mg(R-3fi)/R. (9.4) Читатель Б: В Ваших рассуждениях предполагается, что в точке А тело все еще находится на поверхности обруча. Но ведь может оказаться, что оно оторвалось от обруча, не дойдя до точки А. Автор: Можно найти точку, в которой Рис. 9.2 57
тело должно оторваться от обруча. Эта точка соответствует крайнему случаю, когда сила давления тела на обруч уменьшается до нуля. Поэтому надо в (9.4) положить N = 0 и затем из получившегося урав- нения найти расстояние по вертикали от верхней точки обруча до точки отрыва: h0 = /?/3. (9.5) Если в предложенной задаче значение h удойлетворяет условию h < hQ, то верен результат (9.4); если же h hQ, то N = 0. Читатель А: Насколько я понял, в задаче были использованы два физи- ческих'закона (9.2) и (9.3). Автор: Хорошо, что Вы обратили на это внимание. Действительно, в зада- че использовались два закона: второй закон Ньютона [см. (9.2)] и за- кон сохранения энергии [см. (9.3)]. К сожалению, экзаменующиеся не всегда достаточно четко представляют, какие именно физические законы применяются при решении той или иной задачи. А это очень важно понимать. Приведу пример. Телу сообщают начальную скорость v0, чтобы оно смогло попасть из точки А в точку В. Предлагаются два варианта пути от А до В (рис. 9.3, а, б). В обоих случаях тело должно преодолеть одну и ту же высоту И, но по-разному. Найти минимальную начальную ско- рость у0 для обоих случаев. Трением можно пренебречь. Читатель Б: Я думаю, что в обоих случаях минимальная начальная скорость должна быть одна и та же, ведь трения нет, и надо преодо- леть одну и ту же высоту. Эту скорость можно вычислить из закона сохранения энергии mgH = hwq/2, откуда и0 = л/2g//. Автор: Ваш ответ неправилен. В первом случае тело в верхней точке будет иметь скорость щ, соответствующую центростремительному ускорению, которое создается равнодействующей силы тяжести и силы реакции опоры. Поскольку рассматривается задача на минимум, то следует считать, что в точке Б давление на опору исчезает, а следовательно, и реакция опоры сводится к нулю. При этом на тело Действует только сила тяжести, которая и сообщает ему центростремительное ускоре- ние, т. е. Рис. 9 3 Рис 9 4 58
где R = Н/2. Добавляя к динамическому равенству (9.6) энергетическое равенство mu2t/2 = mv2/2 mgH, (9.7) получаем, что минимальная начальная скорость равна V5/z gH- Во вто- ром случае тело может проходить верхнюю точку траектории при ско- рости, бесконечно близкой к нулю, так что можно ограничиться энерге- тическим равенством. И тогда справедлив предложенный Вами результат. Читатель Б: Я Вас понял. Если бы в первом случае тело не имело в точке Б скорости, то оно просто свалилось бы вниз. Автор: Если бы в первом случае тело имело начальную скорость ц0= предложенную Вами, то оно не дошло бы до точки Б и оторвалось бы от опоры несколько раньше. Я предлагаю найти высоту h точки отры- ва при условии, что vQ = ^2gH. Читатель А: Разрешите мне попробовать решить эту задачу. Автор: Пожалуйста. Читатель Д: В точке отрыва реакция опоры равна, очевидно, нулю. Поэто- му в этой точке на тело действует только сила тяжести G = mg. Разло- жим ее на составляющие вдоль радиуса (mg cos а) и перпендикулярно ему (mg sin а), как показано на рис. 9.4 (Д— точка отрыва). Со- ставляющая вдоль радиуса сообщает телу центростремительное уско- рение, определяемое из равенства mg cos а = mv2/R, (9.8) где и2 — скорость тела в точке Д. Для ее нахождения используем энергетическое равенство mv2/2 + mgh. = mv^/2. (9.9) Объединяя динамическое (9.8) и энергетическое (9.9) равенства и учи- тывая, что cos а = (h — R)/R, получаем mg (h — R)= тцц — 2mgh. Отсюда находим Л = (2“о + g^)/(6g). (9.Ю) Подставляя и20 = 2gH в (9.10), получим искомый результат: h - 5/б И. Автор: Все правильно. Замечу, что, используя (9.10), можно найти такую начальную скорость ц0, которая позволила бы телу описать мертвую петлю. Для этого надо в (9.10) принять h — Н, тогда // = (2^ + g//)/(6g), откуда следует уже известный нам результат: Читатель А: Условие (9.10) получено в случае, когда тело от опоры отрывается. Как же можно применять (9.10) к случаю, koi ла тело проходит петлю, не отрываясь? 59
Автор: Отрыв в самой верхней точке мертвой петли фактически означает, что тело не отрывается, а, пройдя эту точку, продолжает движение по окружности. Читатель Б: Можно сказать, что тело оторвалось как бы на одно мгно- вение. Автор: Да, Вы правы. В заключение я предлагаю рассмотреть следующую задачу. На краю наклонной плоскости с углом наклона а лежит тело. Плоскость равномерно вращается вокруг вертикальной оси с угловой скоростью и. Расстояние от тела до оси вращения плоскости равно R. Требуется найти наименьший коэффициент k0 (напомним, что этот коэффициент характеризует максимально возможное значение силы трения покоя), при котором тело удерживается на вращающейся на- клонной плоскости (рис. 9.5, а). Читатель А: К телу приложены три силы: сила тяжести G, сила реак- ции опоры N и сила трения FTp. Автор: Правильно. Хорошо, что Вы не добавили к ним центростреми- тельную силу. Что Вы будете делать дальше? Читатель А: Дальше я разложу силы на направления вдоль плоскости и перпендикулярно ей (рис. 9.5, б). Автор: Здесь я Вас остановлю. Мне не нравится Ваше разложение. Ска- жите, куда направлено ускорение тела? Читатель А: Ускорение направлено по горизонтали. Это центростреми- тельное ускорение. Автор: Правильно. Поэтому следует разлагать силы на направления по горизонтали (т. е. вдоль ускорения) и по вертикали (т. е. перпендику- лярно ускорению). Вспомните, о, чем мы говорили в § 7. Читатель А: Я понял. Разложение сил на горизонтальное и вертикальное направления показано на рис. 9.5, в. Вертикальные составляющие сил уравновешиваются, а горизонтальные составляющие обусловливают центростремительное ускорение тела: {W cos а + FTp sin а — G = О, FTp cos а — N sin а = mv2/R. Учитывая, что FTp — k0N, v2/R = w2/? и G —^mg, перепишем эти соотно- шения в следующем виде: Рис. 9.5 60
{N (cos a + sin a) = тё’ N (k0 cos a — sin a) = mu?R. Читатель Б: Здесь только два уравнения, а неизвестных три: /г0, m, N. Автор: Тем не менее эти уравнения решить можно. Ведь нам надо найти не все эти три неизвестных, а только одно — коэффициент /г0. Неизвест- ные m и N можно легко исключить, разделив первое уравнение на второе. Читатель А: После деления уравнений получаем cos a + k() sin a g kQ cos a — sin a a»2R ' Отсюда находим искомый результат: k = cos a + g sin a j 0 g cos a — sin a Автор: Из формулы (9.11) видно, что должно быть выполнено условие (g cos a — u>2R sin a) >• 0, которое можно записать в виде tgaCg/o)2/?. (9.12) Если условие (9.12) не выполнено, то никакая сила трения не в со- стоянии удержать тело на вращающейся наклонной плоскости. В за- ключение хочу обратить Ваше внимание на два частных результата, вы- текающих из (9.11). Первый частный случай, когда а — 0: тело нахо- дится на горизонтальном вращающемся диске. Тогда из (9.11) полу- чаем /г0 = <*2R/g- Второй частный случай, когда со = 0; тело находится на неподвижной наклонной плоскости и вот-вот заскользит вниз. Тогда из (9.11) по- лучаем *о = tg a. Читатель А: Последний результат мне уже известен. Это есть коэффи- цент трения скольжения тела, равномерно соскальзывающего с на- клонной плоскости. ЗАДАЧИ 9.1. Небольшое тело, подвешенное на нити, равномерно движется по окружности в горизонтальной плоскости (рис. 9.6). Определить отношение сил натяжений нити и угловых скоростей для двух разных значений угла а: а, — 30° и = 45°. 9.2. Тело массой М, подвешенное на нити длиной /, отклонили на 90° от верти- кального положения и отпустили. Найти зависимость силы натяжения нити и ускорения тела от угла между направлением нити и вертикалью. 9.3. Найти отношение сил, с которыми танк давит из середины выпуклого и вог- 61
нутого мостов. Радиус кривизны мостов в обоих случаях равен R = 40 м, а ско- рость движения танка v ~ 45 км/ч. 9.4. Тело соскальзывает без трения с высоты Н = 60 см и описывает мертвую петлю радиусом R = 20 см (рис. 9.7). Найти отношение сил, с которыми тело давит на опору в точках А, Б, В. 9.5. С какой минимальной высоты Н (см. рис. 9.7) должно соскальзывать без трения тело, чтобы оно описало мертвую петлю радиусом R = 30 см? 9.6. Тело массой М движется с постоянной скоростью v по профилю, изображен- ному на рис. 9.8 (радиус кривизны R). Найти силы, с которыми тело давит на опору в точках А, Б, В. 9.7. Тело может двигаться по окружности в вертикальной плоскости на нити дли- ной /. Какую горизонтальную скорость v0 необходимо сообщить телу в верхнем положении, чтобы сила натяжения нити в нижнем положении оказалась в п= 10 раз больше силы тяжести тела? 9.8. По желобку, изогнутому в виде окружности радиусом R, может без трения скользить тело массой т. На какой высоте h будет находиться тело, если желобок равномерно вращается с угловой скоростью со (рис. 9.9)? С какой силой F тело давит на желобок? 9.9. Шарик подвешен на нити длиной I. На расстоянии 1/2 по вертикали от точки подвеса в стену вбит гвоздь. Шарик отводят так, что нить занимает горизонтальное положение, и отпускают. Определить высоту h, до которой поднимается шарик. 9.10. Резиновый шнур, концы которого соединены, свободно насажен на диск, вращающийся в горизонтальной плоскости вокруг вертикальной оси с частотой п = 20 об/с. Принимая форму шнура за окружность, определить силу натяже- ния 7* шнура. Масса шнура т = 15 г, длина шнура / = 60 см. 9.11. Обруч радиусом R укреплен вертикально на полу. С вершины обруча со- скальзывает без трения тело (рис. 9.10). На каком расстоянии / от точки закреп- ления обруча упадет тело? 10. КАК ВЫ ОБЪЯСНЯЕТЕ НЕВЕСОМОСТЬ ТЕЛ? Автор: Как Вы понимаете следующее выражение: «На экваторе планеты, например Земли, тело весит меньше, чем на полюсе»? Читатель Б: Я понимаю это так. На экваторе сила притяжения тела к Земле меньше, чем на полюсе, что объясняется двумя причинами. Во- первых, Земля немного сплющена у полюсов и поэтому расстояние от центра Земли до полюса несколько меньше расстояния от центра до экватора. Во-вторых, Земля вращается вокруг своей оси, вследствие Рис. 9.6 62
чего сила притяжения на экваторе оказывается ослабленной за счет центробежного эффекта. Читатель А: Поясните, пожалуйста, Ваше последнее замечание. Читатель Б: Из силы притяжения надо вычесть центробежную силу. Читатель А: Я не согласен с Вами. Во-первых, центробежная сила не прикладывается к телу, движущемуся по окружности, о чем уже гово- рилось в § 9. Во-вторых, даже если бы эта сила существовала, она все равно не помешала бы силе притяжения оставаться точно такой же, как и при отсутствии вращения Земли. Ведь сила притяжения равна утМ/г2 и сама по себе она не изменяется от того, действуют или не действуют на тело какие-либо другие силы. Автор: Как видите, вопрос о «весомости тел» оказывается не таким уж простым. Недаром он относится к числу вопросов, на которые экзаме- нующиеся довольно часто дают неправильные ответы. Действительно, если условиться понимать под термином «вес тела» силу притяжения тела Землей, т. е. силу ymM/r2, то «уменьшение веса на эква- торе» следует связывать только со сплющенностью земного шара у полюсов. Читатель Б: Но нельзя же не учитывать вращения Земли! Автор: Я с Вами согласен. Только сначала я хочу подчеркнуть, что обычно на практике под «весом тела» понимают не силу притяжения тела Землей, что вполне логично, а ту силу, которая измеряется с помощью пружинных весов, т. е. силу, с которой тело давит на Землю. Иначе говоря, измеряют силу реакции горизонтальной опоры (сила, с которой тело давит на опору, и сила реакции опоры равны согласно третьему закону Ньютона). Отсюда следует, что выражение «тело на экваторе весит меньше, чем на полюсе» фактически означает, что на экваторе тело давит на горизонтальную опору с меньшей силой, чем на полюсе. Силу притяжения и силу реакции на полюсе обозначим соответственно и а на экваторе — бЛ и N2. На полюсе тело покоится, а на экваторе оно движется по окружности. Отсюда получаем G{ — = О, G2 — N2 — mau, где alL — центростремительное ускорение. Перепишем эти соотноше- 63
Отсюда ясно, что сила N2 меньше силы так как, во-первых, G2 меньше G, (эффект сплющенности Земли), и, во-вторых, из G2 вычи- тается еще тай (эффект вращения Земли). Читатель Б: Значит, выражение «тело потеряло в весе вдвое» не означает, что сила притяжения его к Земле (или к какой-либо другой планете) уменьшилось в два раза? Автор: Да, не означает. Сила притяжения может вообще не меняться. Это выражение означает, что вдвое уменьшилась сила, с которой тело давит на опору (иначе говоря, реакция опоры). Читатель Б: Но тогда получается, что я могу распоряжаться «весомостью» тела довольно свободно. Что мешает мне вырыть под телом глубокую яму? И пусть оно вместе со своей опорой падает туда. В этом случае не будет никакого давления на опору. Значит, тело полностью «потеряло вес»? Значит, оно находится в состоянии невесомости? Автор: Вы сами пришли кверному'заключению. Действительно, состояние невесомости есть состояние падения тела. В связи с этим я хочу сделать несколько замечаний. Мне приходилось встречаться с трактовкой неве- сомости как состояния, в котором сила притяжения Земли уравнове- шена какой-то другой силой. В качестве этой уравновешивающей силы в случае спутника выдвигалась центробежная сила. Говорилось так: сила притяжения спутника Землей и центробежная сила взаимно урав- новешены; в результате равнодействующая сил, приложенных к спут- нику, равна нулю, что и соответствует невесомости. Вы, конечно, понимаете, что такая трактовка неверна уже потому, что на спутник не действует центробежная сила. Кстати говоря, если по- нимать под невесомостью состояние, когда сила притяжения уравнове- шена другой силой, то тогда логичнее назвать невесомым тело, которое просто покоится на горизонтальной плоскости. Ведь здесь сила тяжести как раз уравновешена силой реакции опоры! На самом деле для неве- сомости не требуется никакого уравновешивания силы притяжения Напротив, чтобы тело стало невесомым, надо создать такие условия, прй которых на него не действовала бы никакая другая сила, кроме си- лы притяжения. Иными словами, необходимо, чтобы' реакция опоры равнялась нулю. Движение же тела под действием силы притяжения есть падение тела. Следовательно, невесомость — это состояние паде- ния, например падение лифта в шахту, равномерное движение спутника вокруг Земли. Читатель Л: В § 9 уже отмечалось, что движение спутника вокруг Зем- ли есть не что иное, как падение его на Землю, продолжающееся неограниченно долго. Автор: В том, что движение спутника вокруг Земли есть падение, можно очень наглядно убедиться следующим образом. Представьте, что Вы находитесь на вершине горы и бросаете камень горизонтально. Сопро- тивлением воздуха при этом будем пренебрегать. Чем больше началь- ная скорость камня, тем дальше от основания горы он упадет. На рис. 10.1, а показано, как постепенно изменяется траектория камня по мере возрастания начальной скорости. При некоторой скорости У; траектория падающего камня превращается в окружность и камень 64
Рис. 10.1 становится спутником Земли. Скорость называют первой космичес- кой скоростью. Ее значение находят из соотношения (9.1): = ^уМ/г. (10.2) Если принять радиус г орбиты спутника приблизительно равным ра- диусу Земли, то в этом случае и, ж 8 км/с. Читатель А: А что будет, если, бросая камень с горы, продолжать увели- чивать начальную скорость камня? Автор: В этом случае камень будет двигаться вокруг Земли по все более и более вытянутому эллипсу (рис. 10.1, б). При некоторой скорости у2 траектория камня превращается в параболу и камень перестает быть спутником Земли. Скорость v2 называют второй космической ско- ростью. Как показывают расчеты, скорость и2 приблизительно равна 11 км/с, что примерно в V2 раз больше скорости . Читатель А: Вы определили состояние невесомости как состояние падения. Однако если начальная скорость камня достигает значения второй кос- мической скорости, то камень уходит от Земли. В этом случае уже нельзя говорить о том, что он падает на Землю. Как следует при этом трактовать невесомость камня? Автор: Очень просто. Невесомость в этом случае — это падение камня на Солнце. Читатель А: Значит, невесомость космического корабля, находящегося где-то в межзвездном пространстве, следует связывать с состоянием падения этого корабля в поле тяготения тех или иных небесных тел? Автор: Совершенно верно. Читатель Б: Мне все же кажется, что определение невесомости через состояние падения требует какого-то уточнения. Автор: Вы правы. Невесомость — это не всякое падение. Невесомость — это свободное падение, т. е. движение тела под действием только (!) силы тяжести. Я уже говорил, что, для того чтобы тело стало невесо- мым, надо создать условия, при которых на это тело не действовала бы никакая другая сила, кроме силы притяжения. Например, в случае же 3 - Л. В. Тарасов, А. Н. Тарасова g5
падения парашютиста имеется дополнительная сила — сила сопротив- ления воздуха. Читатель А: Теперь мне понятно, что такое невесомость. Автор: Для проверки я задам Вам один вопрос. Пассажиры некоего космического корабля просят капитана создать состояние невесомости Что должен сделать капитан? Читатель А: Я думаю, он должен выключить двигатели. Автор: Совершенно верно. Читатель Б: Но позвольте. Я недавно читал роман Жюля Верна «Вокруг Луны». Там вообще не было никакого двигателя. А между тем путе- шественники пролетели больше половины расстояния от Земли до Луны, не испытывая невесомости. Они даже гадали — летят или не летят? Автор: Вы натолкнулись на ошибку писателя. Кстати говоря, в этом романе подобных ошибок немало. Жюль Верн неправильно представ- лял себе, что такое невесомость. Он, например, ошибочно полагал, что невесомость есть состояние, когда сила притяжения Земли уравнове- шена силой притяжения Луны. Поэтому он считал, что путешествен- ники должны испытывать невесомость только в определенной точке между Землей и Луной. Между прочим, саму картину невесомости он описал достаточно правильно. Читатель Б: Путешественники Жюля Верна должны были бы в течении всего полета находиться в состоянии невесомости. Автор: Конечно. ЗАДАЧИ 10.1. Вычислить плотность р шарообразной планеты, если спутник движется вокруг нее по круговой орбите с периодом Т на расстоянии от поверхности планеты, равном половине ее радиуса 10.2. Какой продолжительности Т должны быть сутки на Земле, чтобы тела на экваторе не имели веса? Радиус Земли /? = 6400 км 10.3. Вычислить плотность р вещества шарообразной планеты, сутки которой равны Т = 10 ч, если известно, что на экваторе планеты тела невесомы 10.4. Два спутника движутся вокруг Земли по круговым орбитам на высотах hy и h2 от ее поверхности Найти отношения скоростей движения vjv2 и периодов обращения /Т спутников Радиус Земли /? 10.5. Определить первую космическую скорость для планеты, масса и радиус кото- рой в три раза больше, чем у Земли 11. ЧТО ВЫ ЗНАЕТЕ О РАБОТЕ? Автор: Поговорим подробнее о работе как о физическом понятии. Читатель А: Мне кажется, что здесь не о чем говорить. Работа силы F на перемещении s есть произведение Fs, если направления силы и пе- ремещения совпадают. Если же они не совпадают, то надо использо- вать более общую формулу для работы А = (F cos ос) s (111) 66
Здесь а — угол между направлениями силы и перемещения, F cos а — проекция силы F на направление перемещения тела. ггор: И все же тут есть о чем поговорить. Начнем с простого случая. На тело массой т, находящееся на горизонтальной плоскости, действует в течение промежутка времени Т горизонтальная сила F. Трение отсут- J ствует. Чему равна работа силы?? Цтатель А: За время Т тело переместится на расстояние s = at2/2, где > а = F/т. Следовательно, искомая работа есть Л = Fs = F2T2/(2m). prop (обращаясь к читателю Б): Нет ли у Вас замечаний по поводу полученного результата? цтатель Б: Вроде бы все правильно. втор: Все правильно, если считать, что тело покоилось до того, как на ' него начала действовать сила ?. Предположим, однако, что до начала действия силы тело двигалось прямолинейно и равномерно с некоторой скоростью v0. Пусть направление силы ? совпадает с направлением скорости v0. Чему теперь равна работа А силы? за время Т? цтатель А: Теперь s = v0T -}- аТ2/2 и, следовательно, I А = F (v0T + at2 /2). цтатель Б: Получается, что одна и та же сила, действующая на одно и то же тело в направлении его перемещения, может производить за одно : и то же время разную работу — в зависимости от того, какова была начальная скорость тела! втор: Именно так и получается. г Теперь обратимся к рис. 11.1, а. Тело массой m скользит вниз по наклонной плоскости с углом наклона а. Коэффициент трения скольже- ния тела о плоскость k. На тело все время действует горизонтально направленная сила ?. Чему равна работа этой силы при перемещении тела на расстояние s (от точки В в точку С)? ’итатель А: Учтем все силы, действующие на тело. Кроме силы F на него | действуют силы тяжести G, сила реакции опоры N, сила трения ?хр I (рис. 11.1, б). Далее найдем равнодействующую всех сил... |втор (перебивая): Остановитесь, пожалуйста. Силы Вы указали верно, ! однако зачем нам все это? Мы интересуемся работой силы ? на пере- ’ мещении s. Достаточно воспользоваться формулой (11.1), так как нам известны и сила F, и модуль перемещения $, и угол а между направ- лениями силы и перемещения. Искомая работа есть Fs cos а. Только и всего! '{итатель А (растерянно): Меня подвела уже укоренившаяся привычка всякий раз указывать все силы, приложенные к телу. И потом Вы ведь указали массу тела и коэффициент трения. Автор: Привычка, конечно, хорошая. Однако надо помнить, что каждая сила «работает сама по себе», независимо от других сил. Воспользу- емся вашим рисунком (рис. 11.1, б) и рассмотрим работу каждой из сил на перемещении s. Читатель А: Я Вас понял. Работа силы F есть Л, = F cos а • s. Работа силы тяжести б есть А2 = G cos (90° — а) • s = mg sin а • s. Работа силы У равна нулю, так как эта сила перпендикулярна направ- 67
лению перемещения тела. Наконец, работа силы трения £тр есть Ал = = F^ • s = N ks = kmg cos a • s. Автор: Но проекция силы трения на направление перемещения тела есть — £тр (а не ведь сила трения направлена противоположно перемещению. Следовательно, Л3 = — kmg cos a • s. В ос1альном Ваш ответ верен. Читатель Б: Какой смысл в юм, чю работа силы, действующая на тело, оказывается отрицательной? Автор: Это означает, что абсолютное значение работы данной силы не до- бавляется к энергии тела, а вычитается из нее. Иначе говоря, результат действия такой силы оказывается отрицательным, если рассматривает- ся приращение энергии тела как следствие работы сил, приложенных к телу. Кстати, давайте поработаем с законом сохранения энергии, ис- пользуя пример, показанный на рис. 11^1. Читатель Б: В нашем примере силы совершили суммарную работу, равную А = Л] Л2 4- А3 — F cos a • s 4- tng sin a • s — kmg cos a • s. Обозначим через Ев и Ec — энергию тела в точках В и С соответ- ственно. Используя закон сохранения энергии, запишем 4~ == ^с- , (1Ь2) Автор: А что Вы подразумеваете под энергией Е тела в той или иной точке? Читатель Б: Сумму кинетической и потенциальной энергий. Я могу энер- гию тела в каждой точке его положения записать так: Ев = mv2B/2 4- 4- WB, Ес = mv2c/2 4- гДе ив и vc — скорости тела в точках В и С соответственно, а WB — Wc = mgh — разность потенциальных энергий тела в точках В и С. Таким образом, вместо (11.2) я могу записать mvl/2 + re + А = т^с/2 + Fc. (11.3) Автор: На первый взгляд, все как будто бы хорошо. Однако равенство (11.3), а следовательно, и (11.2) все же неверно. Читатель Б: Почему? Автор: В этом нетрудно убедиться. Пусть t — время движения тела из В в С. Тогда vc = vB 4- at, s = vBt 4- at2/2. Исключая из этих двух Рис. 11 1 68
уравнений время t, находим известный из кинематики равноускорен- ного движения результат: ис = и<в + 2 as. (11.4) Ускорение а определим, используя второй закон Ньютона: F cos a mg sin a — kmg cos a = ma. (11.5) Подставляя (11.5) в (11.4), находим mv2 mv2 ----= —-----р (F cos a + rng sin a — kmg cos a) s. 2 2 (11-6) Учитывая, что (F cos a-}- rng sin a — kmg cos a) s = А, перепишем (11.6) в виде mv2 mv2 (117) Результат (11.7) отличается от (11.3). Тем самым мы убеждаемся в ложности результата (11.3). Читатель Б: Как же так? Куда же у Вас исчезла потенциальная энергия тела? Автор: А она никуда не исчезала. Дело в том, что изменение потенциаль- ной энергии тела в поле сил тяготения связано с работой силы тяже- сти. Иначе говоря, Л2 = mg s sin а= mgh = WB — 1ГС. Учитывая это, перепишем (11.7) в виде mvl/2 +( - И7С) + А, + Аа = т<£/2 или, иначе, mvB/2 -f- WB + Д| 4- А3 — mvc/2 -J- Wc. (H-8) Читатель Б: Я понял свою ошибку. Автор: Сделаем еще один шаг. Учтем, что трение скольжения приводит к выделению теплоты. Абсолютное значение работы силы трения есть количество выделившейся теплоты: |А3|=—А3 = Q. Итак, окончательно запишем: mv2 mv2 __L4_U7b4-X1-Q = ^4-«7c. (11.9) Читатель Б: Признаюсь, разговор о работе получился довольно содержа- тельным. Автор: К тому же он еще не окончен. До сих пор мы полагали, что сила, работа которой рассматривается, постоянна на всем перемещении тела. Предположим теперь, что это не так. Пусть упругая пружина сжата на длину s (рис. 11.2, а). Пружина будет действовать на неко- торое тело с силой Р, которую называют силой упругости (рис. 11.2, б). Под действием этой силы тело перемещается, причем по мере переме- щения тела сила упругости Р уменьшается и обращается в нуль, когда тело переместится на расстояние х = s (рис. 11.2, в). График изменения 69
силы упругости по мере распрямления сжатой пружины показан на рис. 11.3; здесь Fo— сила упругости, отвечающая сжатию на s. Изображенная на рисунке зависимость F (х) является линейной: F = = Fo —(Fq/s) • хили, иначе, F=F0-fex, (11.10) где k = Fq/s — жесткость пружины или коэффициент упругости, этот коэффициент характеризует упругие свойства пружины. Читатель А: Насколько я помню, из закона Гука сила упругости описы- вается немного другой формулой: F= —£х. (ИИ) Автор: Тут нет никакого недоразумения. Формула (11.11) получается, если в качестве начала координат выбрать положение тела не в тот момент, когда пружина сжата, а в тот момент, когда она распрямилась. Пусть X = х — s. График зависимости F от X показан на рис. 11.4. Переходим от х к X, перепишем (11.10) в виде F = Fo — k (X + s) или F = ks — k (X + s') или, окончательно, F = — kX. Как видите, мы пришли к результату (11.11). Читатель А: Мне понятна левая половина графика на рис. 11.4 (при Х^О); она повторяет рис. 11.3. А что означает правая половина (при Х>0)? Читатель Б: Левая половина графика соответствует сжатию пружины, а правая — растяжению. Если тело находится в точке X — 0, то в этот момент пружина не сжата и не растянута. Рис. 11 4 70
Автор: Верно. А теперь вернемся к рис. 11.3 и подумаем, чему равна работа силы F на перемещении от х — 0 до х = s. Читатель Б: Во всяком случае эта работа не равна Fos. Автор: Существует общее правило. Предположим, нам известна зависи- мость F от х. Работа силы F при перемещении тела из точки х = х{ в точку х = х2 равна площади под графиком F (х) на отрезке [ х{; х2]; см. заштрихованную площадь на рис. 11.3. Читатель Б: Следовательно, работа силы F при перемещении тела от х = 0 до х = s равна площади всего прямоугольного треугольника, показанного на рис. 11.3: А = F0s/2 = ks2/2. (11.12) Автор: Это есть работа, которую производит упругая пружина жестко- стью k, сжатая на $. Теперь мы можем сделать окончательный вывод: сжимая упругую пружину жесткостью k на величину X (X — измене- ние длины пружины), мы производим работу, равную kX2/2\ эта работа превращается в потенциальную энергию сжатой пружины. Читатель А: Во всех этих рассуждениях мы не учитывали угол между направлениями силы и перемещения. Автор: В примере с пружиной этот угол равен нулю. В общем же слу- чае надо, конечно, рассматривать зависимость от перемещения не са- мой силы Р, а проекции Feos а этой силы на направление перемещения. Может, например, оказаться, что по мере перемещения тела модуль силы остается постоянным, но меняется ее направление, т. е. меняется cos а. Впрочем, подобных усложнений мы касаться не будем. ЗАДАЧИ 1 ’ll 1.1. Определить работу А, которую надо совершить, чтобы поднять столб массой m = 50 кг за один из его концов; другой конец при этом закреплен. Высота столба 5м. 11.2. Определить, какую работу А надо совершить, чтобы лежащий на палубе канат длиной I =*2 м поднять за один из его концов на высоту, равную длине Каната. Масса каната пг = 2 кг. 11.3. Сани массой m — 20 кг съезжают с горки высотой h = 2 м и останавливаются на горизонтальной поверхности. Определить работу А, которую надо совершить, чтобы затащить их обратно в гору по прежнему пути. 11.4. Какую работу А надо совершить, чтобы сжать пружину на А/ = 2 см? Извест- но, что сила Fo = 50 Н сжимает эту пружину на А/о = 1 см. 11.5. Для остановки вагонов применяют пружинные буфера. Найти наибольшее сжатие s буферов и работу А силы упругости, если вагон массой m — 2 т, движу- щийся со скоростью v = 1 м/с, ударяется о два неподвижных буфера. Известно, что буфер сжимается на А/ = 4 см при действии силы F — 105 Н. 11.6. Мальчик бросил мяч массой m = 0,3 кг вертикально вверх. Определить рабо- ту А, которую совершил мальчик, если мяч находился в воздухе в течение времени t = 4 с. 11.7. Найти работу всех сил, действующих на математический маятник, за один период его колебаний. Массой нити, растяжимостью нити, а также сопротивле- нием воздуха пренебречь. 71
12. ЧЕМ ОТЛИЧАЕТСЯ МГНОВЕННАЯ МОЩНОСТЬ ОТ СРЕДНЕЙ МОЩНОСТИ? Автор: На этот вопрос можно ответить, по сути дела, очень кратко: мгновенная мощность отличается от средней мощности тем же, чем мгновенная скорость отличается от средней скорости. Не будем, однако, торопиться и поговорим о мощности более обстоятельно. Итак, что такое мощность? Читатель А: Это работа, деленная на время, за которое она совершена. Автор: Мне не очень нравится такой ответ. Читатель А: Чем же он плох? Автор: Вам бы понравилось, если на вопрос «что такое скорость?» я отве- тил бы, что скорость — это физическая величина, измеряемая отноше- нием перемещения к промежутку времени, затраченному на это переме- щение? Читатель А: Такой ответ был бы верен лишь в случае движения с по- стоянной скоростью. Автор: Точно так же и ваш ответ верен лишь в случае, когда при совер- шении работы мощность остается постоянной во времени. Чтобы во всем этом разобраться, рассмотрим работу как функцию от времени. Пусть: А (/) — работа, совершенная за время /; A (J + А/) — работа, совершенная за время t + А/; тогда [А (/ + А/) — А (/)]/А/ — средняя мощность за промежуток времени от t до t 4- А/ (А/ — длительность этого промежутка). Предел последовательности значений таких сред- них мощностей при А/->0 есть мгновенная мощность, т. е. мощность в момент времени t: N(f) = Um Л + Ад! ~ Л . (12.1) м — о Читатель А: Такая же формула использовалась для определения мгновен- ной скорости [см. (1.2)]. Только вместо работы там фигурировал пройденный путь. Автор: Вы подметили глубокую аналогию. Речь идет об аналогии между пройденным за некоторое время путем и скоростью, с одной стороны, и совершенной за некоторое время работой и мощностью, с другой. Именно об этой аналогии я и говорил в начале параграфа. Читатель А: Теперь я понял, что формула для мощности N = A/t, (12.2) где А — работа, совершенная за время t, должна восприниматься так же, как формула и = s/t. Формула (12.2) справедлива тогда, когда мощность не изменяется во времени. Автор: Ее можно применять и тогда, когда мощность изменяется во вре- мени. Но тогда N = A/t должна рассматриваться как средняя мощ- ность за промежуток времени t. А кстати, в каких случаях мощность постоянна во времени? Читатель А: Когда рассматривается работа силы, не изменяющейся во времени. 72
Рис. 12.1 Автор: Неверно! Предположим, что сила постоянна во времени и при этом, для простоты, cos а— 1. Ис- пользуя (11.1), перепишем (12.1) в виде (123) Д/->0 Сила F не изменяется со временем и может быть вынесена за знак пре- дела, тогда г 1- S (/+ А/) — S (/) N (t) = F lim —i---г2----— = Fv. м -> о Итак, N(t)=F-v(t). (12.4) Отсюда видно, что для постоянства мощности недостаточно постоянст- ва силы, надо чтобы оставалась постоянной также и скорость тела. Читатель А: Формула (12.4) мне нравится: она выражает мгновенную мощность через мгновенную скорость тела. Можно ли в общем случае определить мгновенную мощность не формулой (12.1), а просто как производную от работы по времени: А (/) = дЛ/At. Автор: Конечно, можно. При этом учитываются зависимости от времени всех величин, входящих в формулу для работы. ЗАДАЧИ 12.1. Автомобиль движется по горизонтальной дороге со скоростью vt = 60 км/ч, а вверх по горе с уклоном 0,05 — со скоростью у2 = 48 км/ч. Определить коэффи- циент трения k, если мощность двигателя постоянна. 12.2. Автомобиль массой пг = 1000 кг трогается с места и едет вверх по горе с уклоном 0,02. Автомобиль развивает скорость v = 54 км/ч, пройдя расстояние s = 20 м. Определить среднюю мощность (А) автомобиля, если коэффициент трения о дорогу k = 0,05. 12.3. Лифт массой пг = 1 т поднимается с помощью лебедки равноускоренно. На некотором участке пути длиной / = 1 м лифт двигался со средней скоростью (у) = 5 м/с и при этом его скорость возросла на Аи — 0,5 м/с. Определить ра- боту А, которую совершила лебедка, и среднюю мощность (А) на указанном участке пути, а также мгновенную мощность А2 в конце пути. 73
12.4. Спускаясь с горы с уклоном 0,05 при выключенном двигателе, автомобиль дви- жется равномерно со скоростью v = 72 км/ч. Определить мощность N двигателя, при которой автомобиль мог бы подниматься на такой же подъем с той же ско- ростью. Масса автомобиля т= 1,5 т. 12.5. Аэросани движутся вверх по горе, угол наклона которой а = 30°, с постоянной скоростью I»! = 72 км/ч. Когда они спускаются с горы, то при той же мощности двигателя устанавливается скорость v2 — 108 км/ч. Определить скорость и, с кото- рой аэросани будут двигаться при той же мощности двигателя по горизонталь- ному пути. 12.6. На рис. 12.1 представлена зависимость работы А от времени за которое работа была совершена. Определить среднюю мощность 1) за первые две секунды, 2) за первые три секунды, 3) за вторую и третью секунды. 12.7. На рис. 12.2, а—в даны различные зависимости мощности N от времени t Найти работу, выполненную в каждом из трех указанных случаев. 13. УМЕЕТЕ ЛИ ВЫ ПОЛЬЗОВАТЬСЯ ЗАКОНАМИ СОХРАНЕНИЯ ЭНЕРГИИ И КОЛИЧЕСТВА ДВИЖЕНИЯ? Автор: Я хочу предложить несколько весьма сложных задач. Первая задача. По двум наклонным плоскостям одинаковой высоты, Н, но с разными углами наклона а, и без трения соскальзывают тела. Начальные скорости тел равны нулю. Вычислить скорости тел в конце пути. Вторая задача. Известно, что формула, выражающая ко- нечную скорость тела через ускорение и путь: v = V2as, относится к случаю, когда начальная скорость отсутствует. Как будет выглядеть формула при наличии у тела начальной скорости ц0? Третья за- дача. С высоты Н бросают тело с горизонтально направленной ско- ростью и0. Найти скорость тела в момент приземления. Четвертая задача. Тело бросают под углом а к горизонту с начальной скоро- стью и0. Вычислить максимальную высоту подъема тела. Читатель А: Первую задачу я буду решать так. Рассмотрим одну наклон- ную плоскость, например, с углом наклона а,. К телу приложены две силы: сила тяжести G и сила реакции опоры Разложим силу G на направления вдоль плоскости ( G sin aj и перпендикулярно ей (Geos aj. Составим уравнения для сил, действующих перпендикулярно плос- кости, и для сил, действующих вдоль плоскости: G cos щ — G, G sin a, = Ga{/g. Здесь ах — ускорение тела. Из второго уравнения находим а{ — g sin оц. Путь, пройденный телом, ///sin а,. Используя формулу, о которой гово- рится во второй задаче, получаем, что скорость тела в конце пути равна = 3plg sin (Xj ///sin at = V2gH. Поскольку окончательный результат не зависит от угла наклона, то он применим также и к телу, движущемуся по второй наклонной плоскости с углом наклона а>. Со второй задачей мы уже встречались в § 11. Для ее решения можно воспользоваться известными кинематическими соотношениями: и = v0 + at, s = v{}t + at2/Ч. 74
Трудно переоценить роль физических законов сохранения. Они являются наиболее общими правилами, полученными человеком на основе длительного опыта. Умелое использование законов сохранения позволяет относительно просто решать многие задачи. Рассмотрим примеры на законы сохранения энергии и количества движения. Исключая из них время t, находим (13.1) Для решения третьей задачи я сначала найду горизонтальную V] и вер- тикальную и2 составляющие конечной скорости. Так как по горизонтали тело перемещается равномерно, то = v0. По вертикали тело движется с ускорением g, причем без начальной скорости. Поэтому можно вос- пользоваться известным соотношением v2 = Применяя далее теорему Пифагора, получаем искомый результат: v=^v2^vl=-yl2gH + v20. (13.2) Четвертая задача уже рассматривалась в § 6. Надо разложить началь- ную скорость тела на горизонтальную (y0cosa) и вертикальную (uosina) составляющие. Далее следует рассмотреть вертикальное пере- мещение тела и прежде всего найти время tx подъема тела из формулы зависимости скорости от времени при равнозамедленном движении (vB = vQ sin a — gt), учитывая, что при t = tx вертикальная составляю- щая скорости тела обращается в нуль. Таким образом, yosina—gtx = 0, откуда t{ —(vQ/g) sin a. Зная t{, найдем искомую высоту Н по формуле зависимости пути от времени при равнозамедленном движении: И = v0 sin a • — gt\/2 = Uq sin a/(2g). 75
Автор: Во всех четырех случаях Вы получили правильные ответы. Однако мне не очень нравится метод решения этих задач. Все они могут быть решены проще, если воспользоваться законом сохранения энергии. Я предлагаю убедиться в этом. Первая задача. Закон сохранения энергии имеет вид mgH = = mv2/2 (потенциальная энергия тела в верхней точке плоскости равна его кинетической энергии в нижней точке). Отсюда немедленно находим скорость тела в нижней точке: v = ^2gH. Вторая задача. Закон сохранения энергии имеет вид mv2/2 — — mu2/2— mas, где mas — работа силы, сообщающей телу ускорение а. Отсюда получаем и2 4- 2as = и2, или окончательно и = ~у/ 2as + v2q. Третья задача. Закон сохранения энергии mgH-\-mv2/2 — mu2/2 Отсюда получаем искомый результат: u = ^2gH + u2. Четвертая задач а. В точке бросания энергия тела равна ти^/2. В верхней точке траектории энергия тела mgH + ти\/2. Так как ско- рость щ в верхней точке v0cos а, то, используя закон сохранения энергии mu2/2=mgH+mu2 cos2a/2, находим Н=[ Уо/(2#)] (1—cos2 a), или окончательно и* sin2a Читатель А: Я убедился, что эти задачи можно решить значительно проще. Я просто не подумал о законе сохранения энергии. Автор: К сожалению, экзаменующиеся довольно часто забывают об этом. В результате они начинают решать задачи более громоздкими метода- ми, следствием чего является увеличение вероятности ошибки. Мой Вам совет: смелее и шире пользуйтесь законом сохранения энергии. В связи с этим хотелось бы выяснить, как Вы умеете пользоваться этим законом. Читатель А: Мне кажется, что особого умения здесь не требуется: закон сохранения энергии сам по себе очень прост. Автор: Умение пользоваться законом определяется не его сложностью или простотой. Давайте рассмотрим конкретный пример. Предполо- жим, что тело равномерно движется по окружности в горизонтальной плоскости (см., например, рис. 5.2, б). Исключены какие-либо силы тре- ния. Равнодействующая сил тяжести и реакции нити направлена к центру окружности. Часто эту силу называют центростремительной си- лой. Чему равна работа, совершаемая ею за один оборот? Читатель А: Работа равна произведению силы на путь. Значит, в данном случае она равна(nw2//?) • 2яР — 2яти2, где R — радиус окружности, т и и — соответственно масса и скорость тела. Автор: Согласно закону сохранения энергии работа не может бесследно исчезнуть. Куда делась вычисленная Вами работа? Читатель А: Она идет на вращение тела. 76
Автор: Не понимаю. Выразитесь точнее. Читатель А: На удержание тела на окружности. Автор: Ваши рассуждения ошибочны. Чтобы удержать тело на окружно- сти, работу совершать не требуется. Читатель А: В таком случае я не знаю, как ответить на вопрос. Автор: Обобщенная телу энергия может распределиться по следующим, как говорят физики, «каналам»: 1) на увеличение кинетической энергии тела, 2) на увеличение потенциальной энергии тела, 3) на работу, которую данное тело совершает над другими телами, 4) на теплоту, ко- торая выделяется в результате трения. Таково общее положение, ко- торое не все экзаменующиеся представляют себе достаточно четко. Теперь рассмотрим конкретный случай с работой центростремительной силы. Тело движется с постоянной по модулю скоростью, следователь- но,его кинетическая энергия не изменяется — первый канал закрыт. Те- ло движется в горизонтальной плоскости, следовательно, не изменяется и его потенциальная энергия — второй канал закрыт. Ни над какими телами данное тело работы не совершает — третий канал закрыт. На- конец, исключены какие-либо силы трения — четвертый канал тоже оказывается закрытым. Читатель А: Но тогда получается, что работе центростремительной силы просто некуда деваться? Автор: Да, так получается. Теперь Вам остается определить свою пози- цию. Либо Вы признаете, что закон сохранения энергии просто не выполняется, и тогда все заботы отпадают. Либо Вы исходите из спра- ведливости закона сохранения энергии, и тогда... Впрочем, попробуйте самостоятельно указать выход из этого положения. Читатель А: Я думаю, что в данном случае остается заключить, что ника- кой работы центростремительная сила не совершает. Автор: Совершенно логичное заключение. Мне хочется особо подчеркнуть, что это заключение есть непосредственное следствие из закона сохра- нения энергии. Читатель А: Но как же все-таки быть с формулой работы? Автор: В эту формулу кроме силы и пути должен входить еще косинус угла между направлениями силы и скорости [см. (11.1)]. В данном случае косинус равен нулю. Читатель А: Да, да. Я совсем забыл про косинус. Рис 13 1 77
Автор: Я хочу предложить еще один пример. Рассмотрим сообщающиеся сосуды, соединенные узенькой трубочкой с краном. Пусть вначале вся жидкость находится в левом сосуде и уровень ее имеет высоту Н (рис. 13.1, а). Откроем кран, и жидкость потечет из левого сосуда в правый. Конечное положение отвечает равным уровням в обоих сосудах, имеющим высоту Н/2 (рис. 13.1, б). Давайте вычислим потен- циальную энергию жидкости в начальном и конечном состояниях, для чего умножим силу тяжести жидкости в каждом сосуде на половину высоты столба жидкости. Если в начальном состоянии потенциальная энергия GH/1, то в конечном состоянии она равна (G/2) (Я/4) + (G/2) (Я/4) = GH/4. Итак, в конечном состоянии потенциальная энергия жидкости оказывается вдвое меньшей, чем в начальном состоя- нии. Куда же, спрашивается, исчезла половина энергии? Читатель А: Я попробую рассуждать так, как Вы предлагали. Потенциаль- ная энергия GH/4 могла пойти на работу жидкости над другими телами, на теплоту, выделяющуюся при трении, и на кинетическую энергию самой жидкости. Правильно ли я рассуждаю? Автор: Совершенно правильно. Продолжайте. Читатель А: В данном случае никакой работы над посторонними телами жидкость, перетекая из одного сосуда в другой, не совершает. Кине- тической энергии жидкость в конечном состоянии не имеет, так как она находится в состоянии покоя. Значит, остается заключить, что полови- на потенциальной энергии превратилась в теплоту, выделившуюся при трении. Правда, я не совсем четко представляю себе, что это за трение Автор: Вы рассуждали правильно и пришли к верному заключению. Те- перь несколько слов о природе трения. Можно представить себе, что жидкость разбита на слои, каждый из которых характеризуется опре- деленной скоростью перемещения жидкости; чем ближе слой к стенкам трубки, тем меньше его скорость. Между слоями происходит обмен молекулами, в результате чего молекулы с большей скоростью направ- ленного движения попадают в слой молекул с меньшей скоростью на- правленного движения, и наоборот. Это приводит к тому, что более «быстрый» слой оказывает ускоряющее воздействие на более «медлен- ный» слой, и наоборот, более «медленный» слой оказывает замедляю- щее воздействие на более «быстрый». Эта картина позволяет говорить о существовании в жидкости своеобразного внутреннего трения — тре- ния между слоями. Оно проявляется тем сильнее, чем больше различие в скоростях слоев в средней части трубки и вблизи ее стенок. Заме- тим, что на скорость движения слоев вблизи стенок трубки влияет характер взаимодействия молекул жидкости и молекул стенок. Если жидкость смачивает трубку, то слой, непосредственно примыкающий к стенке, фактически не движется. Читатель А: Значит, в конечном состоянии температура жидкости должна быть несколько выше, чем в начальном состоянии? Автор: Да, именно так. А теперь немного изменим условие задачи. Предпо- ложим, что между жидкостью и стенками трубки нет никакого взаимо- действия, в результате чего все слои имеют одинаковую скорость и внутреннее трение не проявляется. Как в этом случае будет проходить процесс перетекания жидкости из одного сосуда в другой? 78
Читатель А: В этом случае уменьшение потенциальной энергии следует отнес- ти за счет появления у жидкости кине- тической энергии. Иными словами, в ситуации, изображенной на рис. 13.1, б, жидкость не находится в со- стоянии покоя. Жидкость должна про- должать перетекать из левого сосуда в правый до тех пор, пока не возникнет ситуация, изображенная на рис. 13.1, в. В этом случае потенциальная энергия жидкости такая же, как и в на- чальной ситуации (рис. 13.1, а). Автор: Что будет происходить с жид- костью далее? Читатель А: Жидкость начнет перетекать в обратном направлении — из правого сосуда в левый. В результате будут наблюдаться колебания уровней жидкости в сообщающихся сосудах. Автор: Эти колебания можно наблюдать, если взять, например, ртуть в стеклянных сообщающихся сосудах. Как известно, ртуть не смачива- ет стекло. Разумеется, с течением времени эти колебания будут зату- хать, так как невозможно полностью^сключить взаимодействие моле- кул жидкости с молекулами стенок трубки. Читатель А: Я вижу, что при решении задач можно активно пользоваться законом сохранения энергии. Автор: Я предлагаю рассмотреть еще одну задачу. В ящик массой М, под- вешенный на тонкой нити, попадает пуля массой т, летевшая горизон- тально со скоростью и0 и застревает в нем. На какую высоту Н подни- мется ящик после попадания пули в результате отклонения нити от по- ложения равновесия (рис. 13.2)? Читатель А: Обозначим через скорость ящика с находящейся в нем пулей сразу после попадания пули в ящик. Для нахождения этой ско- рости воспользуемся законом сохранения энергии muo/2 = (m + М) У|/2, (13.3) откуда = u0Vm/(m + М). (13.4) Зная эту скорость, найдем искомую высоту Н подъема ящика, снова пользуясь законом сохранения энергий: (m + М) gH = (т + М) У1/2. (13.5) Можно объединить соотношения (13.3) — (13.5): (m + М) gH = mt>o/2, • откуда rr Ul Vo Н==ИГТ—(13.6) M + m 2g v ' Автор (к читателю Б): А как думаете Вы? Читатель Б: Я не согласен с предложенным решением задачи. Нам гово- 79
рили, что в подобных случаях следует пользоваться законом сохране- ния количества движения'. Поэтому вместо формулы (13.3) я бы ис- пользовал другое соотношение: mvQ = (т + М) vt (13.7) (количество движения пули до удара равно количеству движения ящика с пулей после удара). Автор: Вы правильно написали закон сохранения количества движения для данной задачи. Однако надо заметить, что количество движения — это вектор, направление которого совпадает с направлением скорости. Помните, что закон сохранения количества движения — это векторный закон. В данном случае количество движения системы пуля -f- ящик до удара есть mv0, а сразу после удара есть (т + М) v{. Оба вектора направлены в одну и ту же сторону. Поэтому векторное равенство mvQ = (т -|- М) щ (13.8) эквивалентно написанному Вами скалярному равенству (13.7). Читатель Б: Я продолжу. Из (13.7) следует, что = vQm/(M + m). (13.9) Если теперь воспользоваться законом сохранения энергии для дви- жения ящика с пулей после удара и подставить в (13.5) результат (13.9), то получим Автор: Налицо два разных мнения и два различных результата. Сущ- ность разногласий состоит в том, что в одном случае к удару пули о ящик применяется закон сохранения кинетической энергии, тогда как в другом случае используется закон сохранения количества движения. Кто же прав? (К читателю А): Что Вы можете сказать в обоснование своей позиции? Читатель А: Мне не приходило в голову обращаться к закону сохранения количества движения... Автор (к читателю Б): А что скажете Вы? Читатель Б: Я не знаю, как обосновать свою позицию. Я помню, что при рассмотрении ударов закон сохранения количества движения вы- полняется всегда, тогда как закон сохранения энергии выполняется не всегда. Поскольку в данном случае эти два закона приводят к разным результатам, то, по-видимому, справедлив именно мой результат. Автор: Действительно, справедлив именно Ваш результат. Однако необ- ходимо разобраться в этом получше. Удар, после которого оба сталки- вающихся тела движутся вместе слипшись (или одно внутри другого), называют абсолютно неупругим ударом. Для этого удара характерно наличие остаточной деформации столкнувшихся тел, в результате чего выделяется некоторое количество теплоты, и поэтому соотношение (13.3), включающее только кинетические энергии тел, оказывается не- приемлемым. В этом случае для нахождения скорости ящика с пулей 80
после удара необходимо пользоваться только законом сохранения коли- чества движения (13.7). Читатель А: Значит, при абсолютно неупругом ударе не выполняется за- кон сохранения энергии? Но ведь этот закон универсален! Автор: Закон сохранения энергии, безусловно, выполняется и при абсо- лютно неупругом ударе. При таком ударе не сохраняется кинетическая энергия, именно кинетическая энергия, но не полная энергия. Если обозначить через Q выделившуюся при ударе теплоту, то можно запи- сать следующую систему закона сохранения, относящуюся к абсо- лютно неупругому удару: ' mvg — (m М) v}, < mtf (m + Af) vf (13.11) ---L = 2---------!_ _i_ n 2 2 Здесь первое уравнение — закон сохранения количества движения, второе — закон сохранения энергии (включающий не только механи- ческую энергию, но и теплоту). Система (13.11) содержит два неизвестных: щ и Q. Определив и, из первого уравнения, можно из второго уравнения найти выделившуюся теплоту Q: mv? (пг + АГ tr&v^ ти*0 / m 2 2 (m + Aff 2 \ 1 m + M (13.12) Отсюда видно, что чем больше масса М, тем больше энергии пойдет на теплоту. В пределе бесконечно большой массы М получаем, что Q = = гт%/2, т. е. вся кинетическая энергия пули превращается в 'теплоту. Это совершенно естественно: представьте, что пуля застревает в стене. Читатель А: А возможны ли удары без выделения теплоты? Автор: Да, такие удары возможны. Они называются абсолютно упругими. Например, соударение двух стальных шаров может с хорошей степенью приближения рассматриваться как абсолютно упругий удар. При этом происходит чисто упругая деформация шаров, теплота не выделяется. После удара шары принимают прежнюю форму. Читатель А: Значит, при абсолютно упругом ударе закон сохранения энер- гии является законом сохранения кинетической энергии? Автор: Да, конечно. Читатель А: Но в таком случае я не понимаю, как «примирить» друг с другом законы сохранения количества движения и энергии? Ведь мы получаем два разных уравнения для скорости после удара. Или, может быть, при абсолютно упругом ударе не выполняется закон сохранения количества движения? Автор: При абсолютно упругом ударе'выполняются оба закона сохране- ния: и количества движения, и кинетической энергии. Причем беспо- коиться о «примирении» этих законов нет оснований, так как в резуль- тате упругого удара тела разлетаются с разными скоростями. Если пос- ле абсолютно неупругого удара столкнувшиеся тела движутся с одной скоростью (поскольку они слипаются), то после упругого удара каждое тело движется со своей определенной скоростью. А для двух неиз вест- 81
ных надо и два уравнения. Рассмотрим пример. Пусть тело массой m со скоростью ип упруго ударяется о покоившееся тело массой М. Опреде- лить скорости тел после абсолютно упругого удара. Предположим, что $ результате удара налетевшее тело отскакивает назад. Обозначим ско- рость тела m после удара через и,, а тела М — через и2- В этом случае законы сохранения количества движения и энергии могут быть соответ- ственно записаны в виде: {mv(t — Mv, — mv{, Mvj mv- (13.13) ~2~ = ~2~ + ~2~' Читатель А: Непонятно, почему в первом уравнении количество движения массы m после удара берется со знаком минус. Автор: Напомню, что закон сохранения количества движения — это век- торный закон: mvn = Mv2 4- mvx. (13.14) Векторы v0 и направлены в одну сторону, например, в положи- тельном направлении координатной оси. их проекции на эту ось поло- жительны. Вектор же как мы предположили, направлен в обрат- ную сторону (тело массой m после удара отскакивает назад), поэтому его проекция на выбранную координатную ось отрицательна. Читатель Б: Но ведь заранее не всегда можно знать, в каком направ- лении будет двигаться тело после удара. Разве не может оказаться, что после удара тело m будет продолжать двигаться в прежнем направле- нии, но с меньшей скоростью? Автор: Это возможно. В таком случае, решая систему уравнений (13.131, мы получим отрицательную скорость Читатель Б: Я думаю, что направление движения тела m после удара определяется отношением масс ш и М. Автор: Совершенно верно. Если m < М, то тело m отскочит назад; при m = М оно после удара остановится, при m > М будет продолжать движение в ту же сторону, но с меньшей скоростью. Однако в общем случае можно не знать о направлении движения; достаточно предполо- жить какое-то направление и можно приступить к вычислениям, а знак ответа укажет на Вашу ошибку или на правильность ответа. Читатель Б: Известно, что при столкновении шары могут разлететься под углом друг к другу. Здесь же предполагалось, что движение проис- ходит вдоль одной и той же прямой. По-видимому, мы рассматривали частный случай? Автор: Вы правы. Это был частный случай, когда происходит централь- ный удар: шары до удара и после удара движутся по линии, проходя- щей через их центры. Более общий случай нецентрального удара будет продемонстрирован ниже. А пока я хотел бы знать, все ли понятно из того, что я объяснял? Читатель А: Мне, например, понятно. При любом ударе (и упругом, и не- упругом) выполняются два закона сохранения — и количества движе- ния, и энергии. Просто разный характер ударов обусловливает и раз- ный вид уравнений, которыми описываются законы сохранения. При >2
рассмотрении неупругих ударов необходимо наряду с механической энергией принимать во внимание теплоту, выделяющуюся при ударах. Автор: Ваше замечание справедлив^. Читатель Б: Насколько я понял, абсолютно упругий и абсолютно неупру- гий удары представляют собой два крайних случая. Всегда ли они при- годны для описания реальных ударов? Автор: Вы правы. Рассмотренные нами случаи ударов являются крайни- ми. При реальных ударах всегда выделяется какое-то количество тепло- ты (идеально упругих деформаций не бывает), при этом столкнувшиеся тела могут разлетаться с разными скоростями. Однако во многих слу- чаях реальные удары неплохо описываются с помощью упрощенных моделей: абсолютно упругого и абсолютно неупругого ударов. Рассмотрим пример с упругим нецентральным ударом. На горизонталь- ной плоскости покоится тело в виде, наклонной плоскости с углом наклона 45°. С этим телом массой М упруго сталкивается шарик мас- сой пг, летевший горизонтально со скоростью v0. В результате удара шарик подскакивает вертикально вверх, а тело М начинает без трения скользить по горизонтальной плоскости. Найти скорость, с которой шарик начинает свое вертикальное движение после удара (рис. 13.3). Кто желает попробовать свои силы в решении задачи? Читатель Б: Разрешите мне. Обозначим искомую скорость шарика через Up а скорость тела М — через и2. Поскольку удар упругий, то я вправе считать, что кинетическая энергия сохраняется: mv2 mv2 Mir2 -/• = V-+-7r' <13J5> Мне нужно еще одно уравнение, для составления которого надо, оче- видно, использовать закон сохранения количества движения. Я вос- пользуюсь формулой (13.14) и запишу для нашего случая: mv0 = Ми2 -Ь ти{. (13.16) Правда, я не знаю, как быть дальше, ведь скорости у0 и перпен- дикулярны. Автор: В общем случае необходимо разложить все скорости на взаимно перпендикулярные направления и векторное равенство (13.16) заме- нить двумя скалярными равенствами для проекции векторов количеств движения на два взаимно перпендикулярных направления. В данной задаче выберем горизонтальное и вертикальное направления. Проеци- руя векторное равенство (13.16) на горизонтальное направление, получаем ти0 = Ми2. Из уравнений (13.15) и (13.17) нахо- дим искомую скорость: и, = — т)/М. Читатель Б: А как же быть с вертикаль- ным направлением? Автор: На первый взгляд кажется, что Рис 13.3 для вертикального направления закон сохранения количества движения не 83
выполняется. В самом деле: до удара вертикальных скоростей не было, после же удара имеется направленное вертикально вверх количество движения mv{. Нетрудно сообразить, что в данной задаче участвует еще одно тело — Земля. Веды если бы не было Земли, то тело М не двигалось бы после соударения горизонтально! Обозначим: Л43 — массу Земли, аи3 — ско- рость, которую Земля приобретает в результате рассматриваемого уда- ра. Отсутствие трения позволяет считать, что взаимодействие тела М с поверхностью Земли осуществляется только по вертикальному на- правлению. Иначе говоря, скорость и3 Земли направлена вертикально вниз. Таким образом, участие Земли не изменяет вида уравнения (13.17), но приводит к уравнению, описывающему закон сохранения количества движения для вертикального направления: mv{— М3п3 = 0. (13.18) Читатель Б: Если в задаче участвует Земля, то, по-видимому, необходимо уточнить энергетическое соотношение (13.15). Автор: Что именно Вы хотите сделать с соотношением (13.15)? Читатель Б: Я хочу добавить слагаемое, связанное с движением Земли после удара: tnvl mu1. Mvl = + +_^. (1319) £ A A Автор: Ваше намерение вполне логично. Однако исправлять соотношение (13.15) не требуется. В самом деле, из (13.18) следует, что скорость Земли равна и3 = v{m/M3. Поскольку масса Л43 бесконечно велика по сравнению с массой пули, то отсюда следует, что скорость v3 Земли можно считать равной нулю. Теперь перепишем слагаемое М3и|/2 в уравнении (13.19) в виде (М3и3) • v3/2. Величина M3v3 в этом произведении в соответствии с (13.18) имеет конечное значение. Если это значение умножить на нуль (в данном случае на v3), то в итоге получится нуль. Отсюда заклю- чаем, что Земля весьма своеобразно участвует в задаче: получая не- которое количество движения, она в то же время практически не при- нимает энергии. Иначе говоря, при рассмотрении закона сохранения количества движения Землю учитывать надо, при рассмотрении закона сохранения энергии — не надо. Это обстоятельство особенно ярко под- черкивает тот факт, что законы сохранения энергии и количества дви- жения являются существенно разными, взаимно независимыми зако- нами. А теперь я хочу предложить следующую задачу. На горизонталь- ной плоскости покоится шар. С ним сталкивается другой шар такой же массы. Удар абсолютно упругий и нецентральный. Показать, что в ре- зультате столкновения шары разлетятся по двум взаимно перпендику- лярным направлениям. Читатель А: Я выберу ось Ох вдоль направления движения второго шара до столкновения. Обозначу его скорость и0. Пусть в результате столк- новения налетевший шар отскочил под углом а к оси Ох, имея скорость 84
v{, а покоившийся прежде шар — под углом 0 к оси Ох, имея скорость v2. Нам надо доказать, что а 4~ 0 = 90°. Автор: Вы неплохо переформулировали условие задачи. Читатель А: Запишем законы сохранения (в левых частях равенств я буду писать величины, относящиеся к системе тел до столкновения, а в пра- вых — после столкновения): а) закон сохранения энергии mi? тц2 ти? = "2“ + 2 ’ б) закон сохранения для х-проекции вектора количества движения тий = mv{ cos а 4- tnv2 cos 0; в) закон сохранения для (/-проекции вектора количества движения О == ти} sin а — mv2 sin 0. Сократив массу, я получу систему уравнений: ( U20 = + U2, < ц0 = Ц| cos а 4- v2 cos 0, \ 0 = у, sin а — v2 sin 0. Но что делать дальше? Читатель Б: Мне кажется, я понял. Надо возвести в квадрат второе и третье уравнения, а потом произвести сложение. Тогда получим = v2 -|- 2v}v2 (cos a • cos 0 — sin a • sin 0) 4- Сравнивая этот результат с первым уравнением, я нахожу cos a* cos 0— — sin а • sin 0 = 0, или cos (а 4- 0) = 0. Отсюда «4-0=90°, что и требо- валось доказать. Автор: Очень неплохо. Но можно проще. Давайте попробуем не расклады- вать векторы количества движения по осям Ох и Оу, а воспользуемся прямо векторным равенством: = mvj 4- mv2. Этому равенству отвечает на рис. 13.4 параллелограмм, построенный на векторах количества движения. Очевидно, что параллелограмм выра- жает закон сохранения количества движения. Сокращая массу и ис- пользуя известную теорему косинуса, запишем 2 2 , 2 о Уо = 4-^2 — ^ulu2 COS ф- 85
-г 2 2 I 2 Так как согласно закону сохранения энергии имеем v0 = и( + и2, то отсюда получаем ср =90°. Читатель Б: Да, это действительно проще. Автор: Заметьте, такой подход хорошо отражает векторный характер за- кона сохранения количества движения. ЗАДАЧИ 13.1. Тело массой m = 3 кг падает с некоторой высоты, имея начальную ско- рость п0 = 2 м/с, направленную вертикально вниз. Вычислить работу Л против сил сопротивления, совершенную в течение t — 10 с, если известно, что в конце этого промежутка времени тело имело скорость v = 50 м/с. Силу сопротивления считать постоянной. 13.2. Тело сначала скользит вниз по наклонной плоскости с углом наклона а = 30°, а затем по горизонтальной поверхности. Расстояний, пройденные телом по горизон- тальной поверхности до остановки и по наклонной плоскости, одинаковы. Опреде- лить коэффициент трения k, считая его постоянным, на горизонтальной и на- клонной поверхностях. 13.3. Из точки Л по двум разным наклонным плоскостям (рис. 13.5) соскальзывают без начальной скорости два одинаковых тела. Найти отношение скоростей v}/v2 тел у основания наклонных плоскостей для двух случаев: 1) без учета трения; 2) при наличии трения, причем коэффициент трения для обеих плоскостей одинаков и равен k. 13.4. Мальчик, опираясь о барьер, бросил камень с горизонтальной скоростью :'0= 14 м/с. Какую скорость v относительно поверхности земли он сообщит камню, если будет бросать с прежней силой, стоя на коньках на гладком льду? Какое расстояние I при этом проедет мальчик до остановки, если коэффициент трения о лед k — 0,02? Масса камня m = 1 кг, масса мальчика М = 36 кг. 13.5. Шарик массой m объемом V падает в воду с высоты Н, погружается на '’лубину h и затем выскакивает из воды (плотность шарика меньше плотности воды). Найти силу F сопротивления воды, полагая ее постоянной, а также вы- соту h р на которую поднимется шарик, выскочив из воды. Сопротивлением воздуха пренебречь. 13.6. Вагон массой М,= 50 т движется со скоростью и0 = 12 км/ч и встречает стоящую на пути платформу массой М2=30 т. Найти скорость v совместного движения вагона и платформы непосредственно после того, как сработала авто- сцепка. Вычислить путь /, пройденный вагоном и платформой после сцепле- ния, если сила сопротивления составляет п = 5% от веса. 13.7. Из пушки массой М, находящейся у подножия горки, вылетает в горизонтальном направлении сна- ряд массой m с начальной скоростью v0. На какую вы- соту Н поднимется пушка по горке в результате отда- чи, если угол наклона горки а, а коэффициент трения пушки о плоскость /г? 13.8. Сваю массой М, == 100 кг забивают в грунт ба- бой копра, масса которой М2= 300 кг. Баба копра свободно падает с высоты Н = 4 м и при каждом уда- ре свая опускается на h = 10 см. Определить силу со- противления F грунта, считая ее постоянной для двух случаев: 1) удар о сваю абсолютно упругий; 2) удар неупругий. 13.9. По горизонтальному стержню без трения сколь- зит со скоростью v0 шар массой m, = М и сталкивает- ся с другим шаром массой т? который до этого находился в покое. Удар абсолютно неупругий. Найти скорость v шаров после удара и выделившуюся при ударе теплоту Q для случаев: 1) m2= Af/2; 2) т2 — — А4; 3) m2 — 2М. 86
13.10. На горизонтальной плоскости покоится шар массой Л1. С ним сталкивается шар массой т, имевший перед ударом скорость v . Удар абсолютно упругий и нецентральный. В результате удара шар т приобретает скорость в направлении, перпендикулярном направлению его первоначального движения. Определить ско- рости vt и у2 шаров и направление шара М после удара, а также изменение энер- гии ДЕ и количества движения А (гпо) шара т в результате удара. 13.11. На гладкой горизонтальной поверхности на расстоянии / = 3 м от верти- кальной стенки находится шар массой М. Другой шар массой т скользит с неко- торой скоростью по направлению от стенки к шару М. После абсолютно упругого удара шаров шар т достигает стенки и, упруго отразившись от нее, догоняет шар М. Определить, на каком расстоянии s от стенки произошло второе соударе- нЖ если М/т = п = 5. 13.12. Два шара М и 2М подвешены в одной точке на нитях длиной /. Шар.мас- сой М отклонили на угол а и отпустили, сообщив ему при этом касательную ско- рость v, направленную к положению равновесия. На какую высоту h поднимутся шары после соударения, если удар: 1) абсолютно упругий, 2) абсолютно неупру- гий (шары в результате удара слипаются)? 13.13. Шар массой М висит на нити длиной I. В шар попадает горизонтально летящая пуля массой т и застревает в нем. С какой минимальной скоростью v должна лететь пуля, чтобы в результате попадания пули шар мог сделать на нити полный оборот в вертикальной плоскости? 13.14. На горизонтальной плоскости лежат два клина с углами наклона 45°, мас- са каждого М (рис. 13.16). С высоты Н свободно падает шарик массой т(т.^М), ударяется сначала об один клин, затем о другой и подскакивает вертикально вверх. Найти высоту А, на которую подскочит шарик. Принять, что оба удара упругие и что трение между клиньями и плоскостью отсутствует. 13.15. На горизонтальной плоскости лежит клин с углом наклона а= 30°, масса которого М. С высоты Н свободно падает шарик массой т, упруго ударяется о клин и подскакивает под углом 30° к горизонту. На какую высоту h поднимется шарик? Трением клина о плоскость пренебречь. 14. ЗНАЕТЕ ЛИ ВЫ, ЧТО ТАКОЕ ГАРМОНИЧЕСКИЕ КОЛЕБАНИЯ? Автор: Некоторые экзаменующиеся недостаточно четко представляют се- бе гармонические колебания. В связи с этим обсудим прежде всего определение гармонических колебаний. Читатель А: Гармоническими называются колебания, которые происходят по закону синуса: величина х отклонения тела от положения равновесия изменяется со временем следующим образом: х = A sin (со/ + а). Здесь А — амплитуда колебаний (макси- мальное отклонение тела от положения равновесия), ш—круговая частота (ш = = 2л/Т, где * Т — период колебаний), а а—начальная фаза (она показывает отклонение тела от положения равнове- сия в момент времени t — Q). Представле- ние о гармонических колебаниях можно дать, исходя из равномерного движения точки по окружности радиусом А с угло- вой скоростью (о: гармонические колеба- ния, описываемые выражением (14.1), совершает проекция этой точки на неко- торое направление Ох (рис. 14.1). 87
Окружающий человека мир заполнен колебаниями и волнами. Помните об этом, изучая раздел физики, посвященный этим явлением! Обсудим гармонические колебания и, как частный случай, рассмотрим колебания математического маятника. Проанализируем поведение маятника в неинерциальных системах. Читатель Б: Я знаю другое определение гармонических колебаний. Как известно, колебания происходят под действием возвращающей силы, т. е. силы, направленной в сторону положения равновесия и увеличи- вающейся по мере отклонения тела от положения равновесия. При этом гармоническими называются такие колебания, для которых возвра- щающая сила F прямо пропорциональна отклонению х тела от положе- ния равновесия: F=kx. (14.2) Такая сила называется упругой. Автор: Я вполне удовлетворен обоими предложенными определениями. В первом случае гармонические колебания определяются по тому при- знаку, как именно они происходят, во втором — какой причиной они обусловливаются. Иначе говоря, если первое определение использует пространственно-временное (кинематическое) описание колебаний, то второе — причинное (динамическое). Читатель Б: Но какое из двух определений более предпочтительно? Или может быть, оба эквивалентны? Автор: Нет, они не эквивалентны, причем более предпочтительно первое (кинематическое), оно более полно. Читатель Б: Но ведь каков характер возвращающей силы, таков, очевид- но, и характер колебаний. Я не понимаю, почему мое определение менее полное. 88
Автор: Вы не совсем правы: характер возвращающей силы не опреде- ляет полностью характера колебаний. Читатель А: По-видимому, здесь надо вспомнить, что характер движения тела в данный момент определяется не только силами, действующими на тело в этот момент, но и начальными условиями, т. е. положением и скоростью тела в начальный момент. Автор: Совершенно верно. В применении к рассматриваемому случаю это утверждение означает, что характер колебаний определяется не только возвращающей силой, но и теми условиями, при которых эти колебания начались. Очевидно, что колебания можно возбудить раз- личным образом. Например, можно отклонить тело от положения равновесия на некоторое расстояние и затем спокойно отпустить его, оно начнет колебаться. Если принять момент начала колебаний за нуле- вой, то из (14.1) получим, что а = п/2; при этом расстояние, на которое отклонено тело, будет являться амплитудой колебаний. Можно откло- нять тело на различные расстояния от положения равновесия, тем самым задавая различные амплитуды колебаний. Другой способ воз- буждения колебаний состоит в том, чтобы телу, находящемуся в поло- жении равновесия, сообщить некоторую начальную скорость (толкнуть его), тело начнет колебаться. Принимая момент начала колебаний за нулевой, получаем в этом случае из (14.1), что а= 0. При этом в зависи- мости от сообщенной телу начальной скорости получим ту или иную амплитуду колебаний. Можно, очевидно, предложить бесчисленное множество способов возбуждения колебаний, являющихся по отноше- нию к указанным двум способам промежуточными: тело отклоняется от положения равновесия и в то же время получает толчок. Каждый из этих способов будет обусловливать определенные значения амплиту- ды А и начальной фазы а колебаний. Читатель Б: Получается, что А и а не зависят от характера возвращающей силы? Автор: Вот именно. Этими величинами распоряжаетесь Вы сами, когда тем или иным способом возбуждаете колебания. Возвращающая сила определяет только круговую частоту w, или, иными уловами, период колебаний тела. Можно сказать, что период колебаний есть собствен- ная характеристика колеблющегося тела, тогда как амплитуда А и начальная фаза а зависят от внешних условий, возбудивших данные колебания. Рис 14 2 хч
Возвращаясь к определениям гармонических колебаний, мы видим, что динамическое определение не содержит никаких сведений ни об ампли- туде, ни о начальной фазе колебаний, тогда как кинематическое опре- деление содержит сведения об этих величинах. Читатель Б: Но если мы можем по своему усмотрению распоряжаться амплитудой, то, может быть, она как характеристика колеблющегося тела не так уж и важна? Автор: Вы ошибаетесь. Амплитуда — очень важная характеристика колеблющегося тела. Чтобы показать это, рассмотрим конкретный при- мер. Шарик массой m прикреплен к двум упругим пружинкам и совер- шает гармонические колебания амплитуды А по горизонтали (рис. 14.2). Возвращающая сила определяется коэффициентом упру- гости k, характеризующим упругие свойства пружинок. Найти энергию колеблющегося шарика. Читатель А: Для нахождения энергии шарика можно рассмотреть положе- ние его крайнего отклонения (х = А), где скорость шарика равна нулю, и поэтому полная энергия шарика есть его потенциальная энергия, которую можно найти как работу против возвращающей силы F, совершенную при удалении шарика на расстояние А от положения равновесия, т. е. W = FA. (14.3) Отсюда, учитывая, что согласно (14.2) F = kA, получаем W= kA2. Автор: Вы правильно рассуждали, но допустили одну ошибку. Формула (14.3) применима лишь при условии, что сила постоянна. Однако в данном случае сила F изменяется с расстоянием, что показанб на рис. 14.3. Работа этой силы на расстоянии х = А равна площади, заштрихованной под графиком силы. Это есть площадь треугольника, равная kA2/2. Таким образом, W = kA2/2. (14.4) Обращаю внимание на то, что полная энергия колеблющегося тела пропорциональна квадрату амплитуды колебаний. Отсюда видно, что амплитуда действительно является важной характеристикой колеблю- щегося тела. Если 0<х<А, то полная энергия W складывается из кинетической и потенциальной энергий: №= kA2/2 = mv2/2kx2/2. (14.5) Соотношение (14.5) позволяет найти скорость v колеблющегося шари- ка на любом расстоянии х от положения равновесия. Далее поставим такой вопрос: чему равен период колебаний шарика, изображенного на рис. 14.2? Читатель Б: Чтобы установить формулу периода колебаний, надо исполь- зовать дифференциальное исчисление. Автор: Строго говоря, Вы правы. Однако если воспользоваться одновре- менно и кинематическим, и динамическим определениями гармониче- 90
ских колебаний, то можно обойтись без дифференциального исчисле- ния. В самом деле, исходя из рис. 14.1, являющегося графическим выражением кинематического определения, можно заключить, что ско- рость тела в момент прохождения им положения равновесия Vj — мА = 2лА/Т. (14.6) Используя результат (14.4), вытекающий из динамического определе- ния, можно заключить, что скорость может быть найдена из энер- гетического соотношения mv\/2 = kA2/2 (14.7) (в момент прохождения положения равновесия вся энергия шарика есть кинетическая энергия). Объединяя (14.6) и (14.7), получаем 4л2А2аи/7'2 = kA2, откуда Т = 2лд/ m/k. (14.8) Как уже указывалось, период колебаний полностью определяется свой- ствами самой колеблющейся системы и не зависит от способа возбуж- дения колебаний. Читатель А: Обычно мы рассматриваем колебания не шарика на пружи- нах, а маятника. Нельзя ли обобщить полученные результаты для рассмотрения колебаний маятника? Автор: Чтобы выполнить это обобщение, необходимо выяснить, что именно в случае маятника играет роль коэффициента упругости k, ведь в дей- ствительности маятник колеблется не под действием упругой силы, а под действием силы тяжести. Рассмотрим шарик, подвешенный на нити длиной / (рис. 14.4). Отклоним нить от равновесного положения на угол а. На шарик действуют две силы: сила тяжести G и сида реакции Т нити. Их равнодействующая есть возвращающая сила. Из рисунка видно, что она равна G sin а = mg sin а. Читатель А: А что следует принимать за отклонение от равновесного положения в случае маятника — отрезок АБ или отрезок АВ? Автор: Мы рассматриваем гармонические колебания маятника. Для этого необхо- димо, чтобы угол максимального отклоне- ния нити от положения равновесия был весьма мал: а<1 (14.9) (замечу, что здесь угол а выражен в ра- дианах; если пользоваться градусами, то угол а должен во всяком случае не превы- шать 10°). При выполнении условия (14.9) можно пренебречь различием меж- ду длинами отрезков АБ и АВ тогда |ДБ| = I sin а « |АВ| = I tg а. Поэтому Ваш вопрос становится несуще- Рис. 14.4 91
ственным. Для определенности выберем: х = |ЛБ| = I sin а. После этого соотношение (14.2) принимает для маятника следующий вид: mg sin а = kl sin а. (14.10) Отсюда находим k — mg/l. (14.11) Подставляя это равенство в (14.8), получаем формулу периода гармо- нических колебаний маятника: Т = 2nV//g. (14.12) Рассмотрим также вопрос об энергии маятника. Его полная энергия, очевидно, равна mgh, где h — высота поднятия маятника в крайнем положении (см. рис. 14.4). Таким образом, W = mgh. = mgl (1 — cos а) = 2mgl • sin2 (а/2). (14.13) Соотношение (14.13) справедливо, очевидно, для любых углов а. Чтобы этот результат переходил в соотношение (14.4), необходимо выполнение условия гармоничности колебаний маятника, т. е. нера- венства (14.9). В этом случае sina можно приближенно заменить углрм, выраженным в радианах, после чего формула (14.13) принимает вид W к 2mgl (a/2)2 ж mgl a2/2. Отсюда, учитывая выражение (14.11), получаем W= k(laf х k(\AB\f/2. Таким образом, мы пришли к результату (14.4). Читатель Б: Насколько я помню, мы ранее, как правило, рассматривали колебания маятников, не задаваясь требованием малости угла откло- нения. Автор: Это требование не нужно, если рассматривается энергия шарика или сила реакции нити. В данном случае речь идет фактически не о маятнике, а о движении шарика по окружности в вертикальной пло- скости. Однако если в задаче участвует формула периода колебаний (14.12), то колебания маятника обязательно должны быть гармони- ческими и, следовательно, угол отклонения нити должен быть мал [см. (14.9)]. Так, например, в задаче 14.2 условие малости угла отклонения маятника несущественно, тогда как в задаче 14.3 это усло- вие крайне важно. Читатель Б: В одной книге, в разделе о колебаниях, я встретился с поня- тием «фигура Лиссажу». Что это такое? Автор: Фигура Лиссажу — это траектория тела, участвующего одновре- менно в двух взаимно перпендикулярных колебаниях. Предположим, что складываются колебания вдоль оси Ох и вдоль оси Оу, причем для простоты считаем частоты и амплитуды колебаний одинаковыми. Пусть в момент t — 0 отклонение от равновесия по оси Ох равно амплитуде колебаний, а отклонение по оси Оу равно нулю. В этом случае {х = A sin (со/ 4- л/2) = A cos со/, у = A sin wt 92
Возведя в квадрат каждое из уравнений и сложив, получим х2 + 1/2 = Л2. Это уравнение окружности радиусом А. Следовательно, в данном случае тело описывает в плоскости хОу фигуру Лиссажу в виде окруж- ности радиусом А. А теперь попробуйте сами найти фигуру Лиссажу в случае, когда в момент t = 0 отклонения по обеим осям равны нулю. Читатель Б: Понимаю. В данном случае мы можем записать: | х = A sin со/, I у = A sin со/. Отсюда видно, что у = х. Значит, фигура Лиссажу есть прямая линия. Автор: Верно. Если разность начальных фаз х- и {/-колебаний имеет зна- чения в промежутке между нулем и л/2, то мы получим фигуру Лис- сажу в виде эллипса. Если частоты х- и {/-колебаний различаются, то фигуры Лиссажу станут еще сложнее — они будут состоять из нескольких петель. Читатель Б: У меня остался один неясный вопрос. Вы говорили, что частота колебаний маятника есть его собственная характеристика и мы ею не можем распоряжаться. Однако ведь я же могу раскачивать качели с такой частотой, с какой мне захочется? Автор: Можете. Дело в том, что до сих пор мы всякий раз, запустив маятник, предоставляли ему колебаться самостоятельно. Это были собственные колебания. Частота <о0 этих колебаний действительно не зависит от нас, она определяется длиной маятника. Если же вы рас- качиваете маятник с частотой со (т. е. все время действуете на него периодической силой, характеризуемой частотой со), то в этом случае надо говорить о вынужденных колебаниях с частотой со. Этой частотой вы действительно «управляете». Примечательно, что если частота со вынуждающей силы совпадает с частотой собственных колебаний (со = (о0), то наблюдается особенно большая амплитуда колебаний. Это явление называют резонансом. ЗАДАЧИ 14.1. Шарик совершает гармонические колебания (рис. 14.2). Найти отношение скоростей и /ь» шарика в точках, удаленных от положения равновесия соответ- ственно на половину и на одну треть амплитуды. 14.2. Шарик, подвешенный на нити, отклонен от положения равновесия на угол а — 60° и затем отпущен. Найти отношение сил натяжения Т\/Т2 нити для положе- ний равновесия и максимального отклонения шарика. 14.3. Маятник в виде шарика на тонкой нити отклоняется на угол а = 5°. Найти скорость v шарика в момент прохождения равновесия, ес- ли круговая частота колебаний маятника со = 2 с - 1. 14.4. На гладком горизонтальном столе лежит шар массой М, прикрепленный к пружине с ко- эффициентом упругости k (рис. 14.5). В шар попадает пуля массой т, имеющая в момент удара скорость vQ, направленную вдоль оси Рис 14.5 93
ружины. Определить амплитуду А и период Т колебаний шара, считая удар абсо- ютно неупругим и пренебрегая массой пружины. 4.5. К пружине подвесили груз, под действием которого пружина растянулась а А/ = 3 см. Определить период свободных колебаний грузика. 4.6. Маятниковые часы на поверхности Земли спешат на А/= 1,5 мин в сутки. 1а какой высоте над поверхностью Земли они будут идти верно? Радиус Земли * = 6400 км. 5. ЧТО БУДЕТ С МАЯТНИКОМ В СОСТОЯНИИ НЕВЕСОМОСТИ? \втор: В стенку лифта вбит гвоздь. Повесим на этот гвоздь шарик на нити длиной I и заставим его совершать гармонические колебания. Пусть лифт движется вверх с ускорением а. Чему равен период колебаний маятника? Читатель А: Когда мы поднимаемся в лифте с ускорением вверх, то ощу- щаем некоторое увеличение веса. По-видимому, это же должен «ощу- щать» и маятник. Думаю, что период его колебаний будет опреде- ляться в этом случае формулой Т = 2л~\---1—. (15.1) V g + а Однако я не могу строго обосновать эту формулу. \втор: Ваша формула правильна. Для ее обоснования, правда, придется принять необычную для нас точку зрения. До сих пор мы рассматри- вали движение тел только в инерциальных системах отсчета и избегали неинерциальных систем отсчета. Более того, я даже предостерегал от использования неинерциальных систем (см. § 5). Однако в этом параграфе удобно пользоваться именно неинерциальной системой от- счета, которая в данном случае связана с ускоряющимся лифтом. Напомню, что при рассмотрении движения тела массой m в неинер- циальной системе отсчета, имеющей ускорение а, надо приложить к телу дополнительную силу, так называемую силу инерции, которая равна та и направлена в сторону, противоположную ускорению. После того как к телу приложена сила инерции, можно забыть, что система движется с ускорением, и рассматривать движение так же, как в инерциальной системе. В данном случае с лифтом надо прило- жить к шарику дополнительную силу та, которая, как и сила тяжес- ти G = mg, постоянна по модулю и направлению, причем их направ- ления одинаковы. Отсюда следует, что в выражении (14.12) надо заменить ускорение g арифметической суммой ускорений (g + а). В результате получим формулу (15.1). /игатель: Значит, если лифт будет двигаться с ускорением а, направлен- ным вниз, то период колебаний маятника будет определяться раз- ностью ускорений (g — а), ведь теперь сила инерции та будет направ- лена противоположно силе тяжести? ]нюр: Конечно. В этом случае период колебаний маятника равен = (15‘2) 4
Эта формула имеет смысл при условии, что а < g. Чем ближе значение ускорения а к значению g, тем больше период колебаний маятника. При а = g наступает состояние невесомости. Что будет с маятником в этом случае? Читатель А: В эт.ом случае согласно формуле (15.2) период колебаний становится бесконечно большим. Это означает, что маятник оказыва- ется неподвижным. Автор: Я хочу уточнить Ваш ответ. Итак,-маятник колеблется в лифте. Вдруг лифт оборвался и стал свободно падать вниз (сопротивлением воздуха пренебрегаем). Что будет с маятником? Читатель А: Как й уже сказал, маятник станет неподвижным. Автор: Ваш ответ не совсем правилен. Маятник действительно будет неподвижным (разумеется, по отношению к лифту), если в момент обрыва лифта он находился в крайнем положении. Если же в момент обрыва он не находился в крайнем положении, то в состоянии неве- сомости шарик будет равномерно вращаться на нити в вертикальной плоскости со скоростью, которую он имел р момент обрыва. Читатель А: Я понял. Автор: В таком случае нарисуйте мне картину поведения маятника (шари- ка, привязанного к нити) внутри космического корабля, находящегося в состоянии невесомости. < Читатель А: В космическом корабле шарик на нити либо покоится (по отношению к кораблю), либо равномерно вращается по окружности, радиус которой определяется длиной нити (если, конечно, не помешают • стейки или потолок). Автор: Ваша картина не совсем полна. Предположим, что мы внутри космического корабля, находящегося в состоянии невесомости. Возь- мем шарик с нитью и закрепим где-нибудь свободный конец нити так, чтобы ни стенки, ни потолок не мешали движению шарика, после чего осторожно выпустим шарик из руки. Шарик останется неподвижным. При этом будем различать два случая: 1) нить не натянута, 2) нить на- тянута. Рассмотрим первый случай (позиция 1, рис. 15.1, а). Сообщим шарику некоторую скорость v0. В результате шарик будет двигаться прямолинейно и равномерно до тех пор, пока нить, к которой он при- вязан, не натянется (позиция 2). В этот момент на шарик подействует сила реакции нцти, которую можно рассматривать по аналогии с силой реакции стенки, действующей на мячик, ударяющийся о стенку. В ре- зультате шарик резко изменит направление движения и снова полетит равномерно и прямолинейно (позиция 3). При этом своеобразном «от- ражении» должно выполняться правило равенства углов падения и от- ражения. Теперь рассмотрим второй случай: мы сначала натянем нить, а затем осторожно выпустим шарик из руки. Как и в первом случае, шарик будет неподвижно висеть в фиксированном положении (пози- ция 1, рис. 15.1, б). Сообщим шарику некоторую скорость и0, направ- ленную перпендикулярно нити. В результате шарик начнет равномерно вращаться на нити. Плоскость вращения задается нитью и вектором скорости, сообщенной шарику. Рассмотрим следующую задачу. Нить длиной I с шариком на конце за- креплена на тележке, которая без трения соскальзывает с наклонной 95
плоскости с углом наклона а (рис. 15.2, а). Найти период колебаний маятника, находящегося в системе, которая движется с некоторым ускорением. В отличие от предыдущих задач с лифтом ускорение системы направлено под некоторым углом к ускорению земного тяготе- ния. Поэтому в данном случае возникает дополнительный вопрос: каково равновесное направление нити маятника? Читатель А: Я пытался разобраться в такой задаче, но запутался. Автор: Период колебаний маятника в указанном случае описывается формулой (14.12), в которой вместо g надо, как и в случаях с лифтом, использовать некоторое эффективное ускорение (обозначим его £эф), равное векторной сумме ускорений земного тяготения и данной сис- темы. Кроме того, надо учесть, что в указанную сумму вектор ускорения тележки должен входить с обратным знаком, так как сила инерции направлена в сторону, противоположную ускорению системы. Векторы ускорений показаны на рис. 15.2, б, где g sin а — ускорение тележки. Определим £эф: £эф=л/£ф.х+£ф.у=^ (g sin а cos a)2+(g-g sin2 a)2cos a. (15.3) Отсюда находим (15.4) T= 2л\ ---------. у g cos a 96
Читатель А: А как определить равновесное направление нити? Автор: Это направление есть направление ускорения £эф. Исходя из ре- зультата (15.3), нетрудно заключить, что это направление образует с вертикалью угол а. Иначе говоря, в равновесном положении нить маятника на тележке, скатывающейся с наклонной плоскости, направ- лена перпендикулярно плоскости. Читатель Б: А нельзя получить последний результат иначе? Автор: Этот результат можно получить непосредственно, рассмотрев рав- новесие шарика относительно тележки. К шарику приложены сила тяжести G = mg, сила реакции Т нити и сила инерции F — — та (рис. 15.3). Обозначим угол, образуемый нитью с вертикалью, через 0. Разложим все указанные силы на вертикальное и горизонтальное на- правления и запишем условия равновесия составляющих сил для каж- дого из этих направлений: | Т cos р + ma sin а = mg, (15 5) ( Т sin р = ma cos а. \ Учитывая, что а = g sin а, перепишем систему (15.5) в виде Г Т cos р = mg (1 — sin2 а), ( Т sin р = mg sin а cos а. Разделив одно уравнение на другое, получим ctg р = ctg а. Таким образом, углы Р и а оказываются равными. Следовательно, равновесное направление нити маятника перпендикулярно наклонной плоскости. Читатель Б: Я внимательно следил за Вашими объяснениями и прихожу к выводу, что был не так уже неправ, когда, отвечая на вопрос о силах, приложенных к спутнику, указал силу тяготения и центробежную силу (см. § 9). Просто мой ответ следует относить к системе отсчета, свя- занной с самим спутником, и под центробежной силой следует понимать силу инерции. Ведь в неинерциальной системе отсчета, связанной со спутником, мы имеем не динамическую, а статическую задачу на рав- новесие сил, из которых одна — центробежная сила инерции. Автор: Такой подход к задаче о спутнике допустим. Однако, говоря в § 9 о центробежной силе, Вы не рассматривали ее как силу инерции — просто Вы опасались, как бы спутник не свалился на Землю. Кроме того, в упомянутом случае не было никакой необходимости переходить к системе отсчета, связанной со спутником: физическая сущность задачи более наглядно передавалась без введения центробежной силы инерции. Мой прежний совет остается в силе: если нет особой необ- ходимости, то не пользуйтесь неинерциальной системой отсчета. Читатель А: Я хочу попробовать решить нашу задачу в инерциальной сис- теме отсчета, т. е. рассмотреть движение маятника относительно плос- кости. Нам известно, что маятник вместе с тележкой движется с ускорением а = g sin а. Силы, действующие на шарик, я разложу на 4 - Л. В. Тарасов, А. Н. Тарасова 97
G a) 6) Рис. 15.4 направлениями g и g3$ нам направления вдоль плоскости и пер- пендикулярно ей, вот мой риёунок (рис. 15.4, а). Тогда уравнения дви- жения запишутся | Т cos р — mg cos а = О, ( Т sin р + mg sin а = ma. Заменив в последнем уравнении а = g sin а, получим sin р = 0, р = 0. Следовательно, нить перпендикуляр- на плоскости. Эффективное ускорение теперь легко определить, так как угол между известен (см. рис. 15.4, б). Треуголь- ник АВС прямоугольный, откуда следует £эф = g cos а, и мы приходим к формуле (15.4). Автор: Все совершенно верно. ЗАДАЧИ 15.1 . Как изменится период колебаний секундного маятника при перенесении его с Земли на Луну? Масса Луны в 81 раз меньше Земли, а радиус Луны в 3,7 раза меньше радиуса Земли. 15.2 . Секундный маятник в вертикально движущемся лифте совершает за время 1= 1 мин М= 100 колебаний. С каким ускорением а движется лифт? Как направлено это ускорение? 15.3 . Маятник длиной I = 1 м, подвешенный в кабине самолета, совершает гар- монические колебания. Определить период колебаний Т маятника при движении самолета в горизонтальном направлении с постоянным ускорением а — 5 м/с2. Каково при этом равновесное направление нити маятника? 15.4 . Определить, на какую часть длины надо уменьшить длину математического маятника, чтобы период колебаний маятника в кабине самолета оставался одина- ковым при равномерном движении самолета и при его движении с ускорением а = 5 м/с2, направленным горизонтально. 16. ЧТО ВАМ ИЗВЕСТНО О ВОЛНАХ? Автор: Как известно, волны бывают различной природы: волны на поверх- ности воды, упругие волны (в частности, звуковые волны), электро- магнитные волны (в частности, световые волны). К сожалению, экзаме- нующиеся часто имеют о волнах лишь некоторые общие представления. Мне кажется полезным разъяснить некоторые понятия, связанные с волнами, избрав в качестве объекта рассмотрения упругие волны. Если внутри среды (газа, жидкости, твердого тела) поместить некий источник колебаний, то в среде возникнут упругие волны, распростра- няющиеся от источника в разные стороны. Волны могут быть попереч- ными или продольными. Чем отличается поперечная волна от про- дольной? Читатель А: Это я знаю. В поперечной волне частицы среды отклоняются (смещаются) перпендикулярно направлению распространения волны; 98
в продольной - смещения частиц происходят вдоль направления рас- пространения волны. Продольные волны — это процесс распростране- ния сжатий и разрежений среды. Автор: Какие волны распространяются в газах? Читатель А: Продольные. Автор: Правильно. А внутри твердого тела? Читатель А: Поперечные. Автор: И продольные тоже. А внутри жидкости? Читатель А; Тут я не уверен. Может быть, как и -в твердом теле,— продольные и поперечные? Или, скорее, только продольные. Читатель Б: Поперечные волны в жидкости легко наблюдать: достаточно бросить камень в воду. Автор: Внесем ясность. Прежде всего в газах распространяются только продольные упругие волны, тогда как внутри конденсированных сред (т. е. внутри твердых тел и жидкостей) распространяются как про- дольные, так и поперечные упругие волны. Далее, не надо путать волны внутри воды с волнами на поверхности воды, возникающими от брошенного камня. Природа поверхностных и упругих волн различ- на. Какие параметры характеризуют волну? Читатель А: Длина волны Л, период колебаний Т, их частота v, скорость распространения и. Они связаны друг с другом соотношениями: Л = vT;T = 1/v. (16.1) Автор: Хорошо. Но чаще используется круговая частота ю = 2nv, тогда Т = 2л/а). Пусть волна порождается гармонически колеблющимся источником с периодом Т и распространяется вдоль оси Ох со скоростью и. Напи- шите уравнение этой волны. Читатель А: Я не могу. Нам этого не показывали. Автор (к читателю Б): А Вы что можете сказать об уравнении волны? Читатель Б: Я знаю только, что уравнение волны — это зависимость отклонения частиц среды от координаты и времени. Записать это уравнение я не смогу. Автор: Будем рассуждать вместе. Поместим в точку х — 0 источник гармо- нических колебаний. Пусть в момент tx в точке х = 0 отклонение равно ух = A sin со/,. (Отклонения происходят вдоль оси Оу, а волна распространяется вдоль оси Ох.) В какой момент времени в точке х=/=0 будет наблюдаться отклонение ух? Читатель Б: Очевидно, в момент tx 4- x/v, ведь, чтобы дойти до точки х, волне потребуется время x/v. Автор: Совершенно верно. А теперь обозначим отклонение в точке х в момент / через у (х, t) и попробуем сообразить, в какой момент вре- мени такое же отклонение наблюдалось в точке х = О? Читатель Б: В момент t — x/v. Автор: Правильно. Значит, мы можем записать так: У О = У\^~ x/v), 99
где через ух я обозначил отклонения, в точке х = 0. Эти отклонения про- исходят по известному нам гармоническому закону. Читатель Б: Я понял. Уравнение волны будет иметь вид у(х, t) = A sin [<о (/ — х/и)]. (16.2) Автор: Вы правы. Воспользовавшись соотношениями (16.1), перепишем (16.2) в виде у (х, t) = A sin (<о/ — 2лх/А,). (16.3) Заметим, что 2л/1 обычно обозначают через k и называют волновым числом. Окончательно у (х, /) = A sin (<о/ — kx); k = 2л/Л,. (16.4) Теперь ответьте на вопрос: что надо изменить в (16.4), если волна распространяется не в положительном, а в отрицательном направ- лении оси Ох? Читатель Б: Надо изменить знак перед скоростью. Вместо (16.2) получим у (х, /) = A sin [ со (/ + x/v)] (16.5) и окончательно у (х, t) = A sin (со/ + kx). (16.6) Автор: Совершенно верно. Предположим далее, что вдоль оси Ох рас- пространяются две одинаковые волны навстречу друг другу. Что полу- чим в результате сложения этих волн? Читатель Б: Я выполню сложение: у (х, t) = A sin (со/ — kx) + A sin (со/ + kx) = 2А sin со/ • cos kx. (16.7) А втор: Что Вы можете сказать о свойствах волны, описываемой уравнен- нием (16.7Р Читатель А: У нее в два раза больше амплитуда, а остальные характе- ристики прежние. Автор: Это не главное. Давайте рассмотрим точки _ л 1 Зл 1 5л 1 Х~ТТ~Т’ТТ’ т. е. х = '/Д, 3/Д, 5/Д, ••• • В этих точках cos kx = 0. Следовательно, в Рис 16 1 100
Рис 16 2 пропорциональна квадрату них отклонение равно нулю во все мо- менты времени. Читатель Б: Значит, в волне (16.7) есть неподвижные точки? Автор: Вот именно. Поэтому волну (16.7) называют стоячей волной, а указанные точки — узлами стоячей волны. Волны (16.2) — (16.6) называют бегущими волнами. На рис. 16.1, а, б хорошо про- демонстрировано различие между по- ведением во времени бегущей и стоя- чей волн. А теперь поговорим о звуко- вых волнах. Чем характеризуется звук? Читатель А: Громкостью, высотой и темб- ром. Автор: Как эти характеристики связаны с характеристиками волны? Читатель А: Я знаю, что громкость звука ’ амплитуды звуковой волны, а высота связана с частотой: чем больше частота, тем выше звук. Автор: А что Вы можете сказать о тембре? Читатель А: Этого я не знаю. Автор: Тембр связан с «профилем» звуковой волны. Мы до сих пор говори- ли лишь о синусоидальных волнах. Синусоидальные звуковые волны называют простыми (чистыми) тонами. В музыкальном отношении они невыразительны. Рис. 16.2 поясняет понятие «профиля» звуковых волн, имеющих одинаковые громкость и высоту, но разный тембр (волны для удобства графического представления рассматриваются как попе- речные) : а — простой тон (его может воспроизвести, например, колокол), б — тембр пианино, в — тембр скрипки. Своеобразие и вы- разительность различных музыкальных инструментов обусловлены спе- цификой тембров, порождаемых ими звуковых волн. Читатель Б: Я слышал, что в акустике существует эффект Доплера. Что это? Автор: Эффект Доплера наблюдается не только в акустике, но и в применении к волнам любой природы. Предположим, что Вы удаляе- тесь от источника звука со скоростью v0. Пусть в момент до Вас дош- ло максимальное «отклонение» в звуковой волне, а Вы при этом находи- лись в точке Хр Следующее максимальное «отклонение» дойдет до Вас в некоторый момент tx + т. При этом Вы окажетесь в точке xl + тв0. Легко сообразить, что время т складывается из периода колебаний Т и времени, которое потребовалось волне, чтобы пройти расстояние,ти0: т = Т + т • Vq/v, (16.8) где v — скорость звука; т — период звуковых колебаний, восприни- маемых Вами как наблюдателем. Обозначим: со=2л/Г — истинная частота звука, = 2л/т — частота звука, воспринимаемого наблюда- телем. Тогда из (16.8) получаем _L = ± + _LA to' (О (1)' V 101
или со' = co (1 — v0/v). (16.9) Итак, если Вы удаляетесь от источника звука, то слышите более низкий звук с частотой, определяемой выражением (16.9), и наоборот, если вы приближаетесь к источнику, то слышите более высокий звук с частотой w' = (О ( 1 4" Vo/v) В этом и заключается эффект Доплера. Заметим, что к тем же выводам можно прийти, если считать, что дви- жется не наблюдатель, а источник звука. Важно, очевидно, относи- тельное движение источника и наблюдателя. Читатель Б: А в оптике эффект Доплера наблюдается? Автор: Да. Я уже указывал, что этот эффект можно наблюдать в приме- нении к волнам любой природы. Читатель Б: Значит, если источник света движется от нас, то частота све- товой волны уменьшается? Но тогда, например, желтый свет должен казаться красным? Автор: В принципе Вы правы. Однако надо учесть, что световые волны распространяются с очень большой скоростью, равной 3 • 108 м/с. Поэтому при обычно наблюдаемых скоростях движения источников света (или наблюдателей) практически невозможно уловить изменение частоты света. Астрономы улавливают изменение частоты света, иссле- дуя излучение звезд. Так, например, по смещению спектра излучения звезды в сторону красного света заключают, что данная звезда удаля- ется от нас. А теперь поговорим о сверхзвуковых явлениях. Как, по-Вашему, можно ли по звуку отличить сверхзвуковой самолет от обычного? Читатель А: Если летит обычный самолет, то мы сначала услышим звук от него, а уже потом над нами пролетит сам самолет. В случае сверх- звукового самолета все должно происходить наоборот. Читатель Б: Я слышал, что при полете сверхзвукового самолета образу- ется звуковая ударная волна. Слышится не гул, а взрыв. Могут даже вылететь стекла в домах. Автор: Вы правы, образуется ударная звуковая волна. Проследим, как она возникает. Предположим, что сверхзвуковой самолет летит со ско- ростью, в три раза большей скорости звука. Рассмотрим несколько последовательных его положений (рис. 16.3): в момент самолет в точке А; в момент t2 — в точке Б; фронт звуковой волны от точки А имеет при этом вид сферы, показанной в позиции а. В момент само- лет в точке В, фронты звуковых волн от точек А и Б показаны в пози- ции б. В момент t4 самолет в точке Г; фронты звуковых волн от то- чек А, Б и В показаны в позиции в. Если сложить все звуковые волны, например, для момента /4, то получится фронт звуковой волны в виде конуса. По мере движения самолета этот фронт распространяется со скоростью звука по направлениям, показанным на рисунке стрелками. Это и есть ударная звуковая волна. Как только она дойдет до наблю- дателя, последний услышит резкий взрыв. Чем больше скорость самоле- та, тем более узким будет звуковой конус. 102
Рис. 16.3 Читатель Б: Я понимаю теперь, почему сверхзвуковые самолеты не летают над городами. Автор: После моих объяснений Вы можете без особых затруднений решить задачу: самолет летит горизонтально со сверхзвуковой скоростью v0. Наблюдатель услышал звук от самолета через время т после того, как увидел самолет над головой. На какой высоте летел самолет? Читатель Б: Пусть в точке А на рис. 16.4 находится наблюдатель. Он услышит звук, когда самолет окажется в точке Б. Из рисунка видно: Н = ут/cos а; и/и0 = sin а. Следовательно, Н ~ vx/Nl— (у/у0)2. Автор: Вы очень хорошо справились с задачей. Читатель Б: В этой задаче я полагал, что свет от самолета дошел до наблюдателя мгновенно. Но на самом деле свет распространяется с конечной скоростью. Автор: Вы правы. Я уже говорил, что скорость света равна 3 • 108 м/с. Что вы хотите спросить? Читатель Б: А не наблюдаются ли в оптике сверхсветовые явления? Читатель А: Это же невозможно: в природе не бывает скоростей больше скорости света. Автор: Задан интересный вопрос. Дело в том, что скорость распростране- ния световых волн в среде всегда несколько меньше скорости света в вакууме. Поэтому возможна ситуация, когда частица (например, элек- трон) движется внутри среды со скоростью, превышающей скорость света в этой среде. Тогда получаем типичное сверхсветовое явление: летящий электрон порождает световой конус — световую «ударную» волну. Это явление известно в физике как эффект Вавилова—Черен- кова. ЗАДАЧИ 16.1. Ультразвуковой сигнал, посланный кораблем вертикально вниз, возвратился через t = 8 с. Определить глубину моря Н. Скорость звука в воде v — 1450 м/с. 16.2. Скорость звука в воде v = 1450 м/с. Частота колебаний v = 725 Гц. Опреде- лить, на каком расстоянии Ах находятся ближайшие точки, совершающие колеба- ния: 1) в противоположных фазах; 2) в одинаковых фазах; 3) при разности фаз Д<р = л/4. 16.3. На шнуре длиной 1=2 м, один конец которого привязан к стене, а другой колеблется с частотой v — 5 Гц, возбуждаются стоячие волны. При этом между стеной и свободным колеблющимся концом образуется три узла. Найти скорость v распространения волн вдоль шнура. 103
Законы статики — это законы равновесия. Внимательно изучите эти законы. Помните об их практической важности. Ведь невозможно представить себе строителя, не знакомого с основными законами статики. Рассмотрим примеры, иллюстрирующие правило разложения сил. Обсудим условия равновесия тел, которые применяются, в частности, для отыскания центра тяжести. 17. УМЕЕТЕ ЛИ ВЫ ПОЛЬЗОВАТЬСЯ РАЗЛОЖЕНИЕМ СИЛ? Автор: При решении задач ho механике приходится часто пользоваться разложением сил. Поэтому нелишне остановиться на этом вопросе не- сколько подробнее. Прежде всего напомню основное правило: чтобы разложить силу на какие-либо два направления, надо провести через начало и конец вектора силы по две прямых, каждая из которых парал- лельна соответствующему направлению разложения. В результате получится параллелограмм, стороны которого и будут искомыми со- ставляющими данной силы. Это правило поясняет рис. 17.1, на котором сила F раскладывается на два направления : AAj и ББХ. Рассмотрим несколько примеров, для которых характерно использование разложения сил. Первый пример (рис. 17.2): к середине двух нитей подвешены грузы, причем силы тяжести обоих грузов одинаковы (обозначим через G). Нити провисают под тяжестью грузов и образуют с горизонталью углы, соответственно и о^. Какая из нитей натянута сильнее? Читатель А: На том же рисунке я выполню разложение силы тяжести каждого груза на направления нитей. Из разложения следует, что сила натяжения нити равна Т= 0/(2 sin а). Таким образом, сильнее натянута та нить, которая провисает в меньшей степени. Автор: Совершенно правильно. А можно ли натянуть нить так, чтобы она нисколько не провисала под тяжестью груза? 104
Читатель А: А почему бы и нет? Автор: Не торопитесь с ответом. Используйте результат, только что полу- ченный Вами. Читатель А: Да, я понял. Натянуть нить так, чтобы не было провисания, нельзя. Ведь с уменьшением угла а возрастает сила натяжения нити. Как бы ни была велика прочность нити, все равно при достаточно малом а сила натяжения разорвет нить. Автор: Следует отметить, что провисание нити под действием подвешен- ного к ней груза происходит за счет упругих свойств нити, обуслов- ливающих ее растяжение. Если бы нить не могла деформироваться (растягиваться), то к ней нельзя было бы подвешивать груз. Отсюда видно, что в строительной технике учет прочности различных конструк- ций теснейшим образом связан со способностью этих конструкций к упругим деформациям (как говорят, конструкция «должна дышать»). Чрезмерно жесткие конструкции оказываются непригодными, так как возникающие в них напряжения могут при малых деформациях стать чрезмерно большими. Такие конструкции могут разрушаться уже под действием собственного веса. . Если пренебрегать массой нити в рассмотренном выше примере, то нетрудно найти соотношение между углом а провисания нити и силой тяжести G. Для этого надо воспользоваться законом Гука для упругого растяжения нити (см. задачу 17.4). Второй пример. Как известно, «клин клином вышибают». Убе- димся в этом, используя разложение сил. На рис. 17.3, а клин 1 пытаются «вышибить» из щели, забивая в нее клин 2, на который действуют силой F. Даны углы а и Требуется найти силу, дей- ствующую на клин 1 и способствующую его «вышибанию» из щели. Читатель А: Я затрудняюсь решить эту задачу. Автор: Перенесем силу F по линии ее действия в точку Д и разложим силу на горизонтальное направление и направление, перпендикулярное стороне АБ клина 2. Получающиеся при этом составляющие обозначим соответственно F, и F2 (рис. 17.3, б). Составляющая F2 уравнове- шивается силой реакции левой стенки щели; составляющая равная по модулю l^l/tga, действует на клин 1. Разложим эту силу в точ- Рис. 17.1 Рис. 17.2 105
ке Е на вертикальное направление и направление, перпендикулярное стороне ВГ клина 1; получим соответственно составляющие Х3 и Е4 (рис. 17.3, в). Составляющая Р4 уравновешивается силой реакции правой стенки щели, а составляющая F3 способствует «вышибанию» клина 1 из щели. Это и есть искомая сила. Нетрудно видеть, что она равна = ?i tg 0 = ? tg 0/tg a- Третий пример (рис. 17.4, а): на нити, средняя часть которой горизонтальна, подвешены два груза Gx и G2- Найти угол р, если угол a известен, и силу натяжения каждой части нити (ТАБ, ТБВ и Таг). Читатель А: Я разложу вектор силы тяжести (jx на направления АВ и БВ (рис. 17.4, б). Из соответствующих треугольников получаем: ТАБ — — Gx/sin а и ТБВ = Gj /tg а. Таким образом, две силы натяжения уже найдены. Далее я разложу вектор силы тяжести G2 на направления БВ и ВГ (рис. 17.4, в) и аналогично получаю: ТБВ = G2/tg р и = G2/sin р. Приравнивая друг другу силы натяжения участка ни- ти БВ, находим G^tg a = G2/tg р, откуда р= arctg(G2 tg a/G,). Подставляя затем этот результат в выражение для ТВГ, находим силу натяжения Твг. Автор: Разве трудно довести задачу до конца — получить выражение для силы Твг? Читатель А: В это выражение будет входить синус от arctg р, т. е. G2 Т ~-------------i---------- вг sin [ arctg(G2/(G, tg а))] Рис. 17.3 106
Автор: Ваш результат верен, но его можно записать в более простом виде, если выразить sin 0 через tg 0: . „ tg 0 sin 0 = * р . Vl + tg20 Так как tg 0 = tg a( O2/gi)> т0 получим G l fG2\ Читатель Б: Я вижу, что, прежде чем сдавать экзамен по физике, надо довольно основательно повторить математику. Автор: Ваше замечание весьма справедливо. ЗАДАЧИ 17.1. В щель забиты два клина (см. рис. 17.3). При действии на клин 2 некоторой силой клин 1 начинает двигаться вверх. При каком коэффициенте трения k между клиньями и поверхностью щели это возможно? Массой клиньев и трением между нити пренебречь. 107
17.2. Однородный стержень массой т = 6 кг подвешен на нитях (рис. 17.5). Масса груза М = 10 кг. Определить силы натяжения нитей и силу реакции F вдоль стерж ня, если а = 20 = 60°. 17.3. Определить силу натяжения Т нити и силу давления N шарика массой т на наклонную плоскость (рис. 17.6). Угол наклона плоскости а, угол между нитью и вертикалью равен 0 Трением между шариком и плоскостью пренебречь. 17.4. Упругая нить, натянутая в лифте, провисает под действием груза, подвешен- ного в ее середине (рис. 17.7). Угол провисания а — 30° для покоящегося лифта и а,2 ==45° для лифта, движущегося с ускорением. Найти модуль и направление ускорения а лифта. Массой нити пренебречь. 17.5. Шарик массой т = 100 г подвешен на нити длиной I = 1 м к кронштейну (рис. 17.8; а = 30°). Шарику в положении равновесия сообщили горизонтальную скорость v — 2 м/с, после чего он стал колебаться. Вычислить силы F, действую- щие в стержнях АБ и БВ, когда шарик находится в точках максимального от клонения от положения равновесия 18. ЧТО ВЫ ЗНАЕТЕ О РАВНОВЕСИИ ТЕЛ? Автор: На рис. 18.1 показаны два положения равновесия кирпича. Оба равновесных положения являются устойчивыми, однако степень их устойчивости различна. Какое из положений является более устой чивым? Читатель А: Очевидно, что более устойчиво положение кирпича на рис. 18.1, а. Автор: Почему? Читатель А: В этом случае центр тяжести кирпича ближе к земной по верхности. Автор: Дело не только в этом. Читатель Б: Здесь площадь опоры больше, чем в положении на рис. 18.1, б Автор: Опять-таки дело не только в этом. Чтобы разобраться в существе вопроса, рассмотрим равновесные положения двух тел: прямоугольного параллелепипеда с квадратным основанием и цилиндра (рис 18.2, а) Пусть параллелепипед и цилиндр имеют одинаковые высоты Н и одина ковые площади оснований S. Центр тяжести тел находятся на одной и той же высоте и, кроме того, одинаковы площади опоры тел. Однако степень устойчивости тел оказывается различной. Мерой устойчивости конкретного равновесного состояния является энергия, которую надо затратить, чтобы оконча тельно вывести тело из данного состояния. Читатель Б: Что означает слово «окончательно»? Автор: Оно означает, что предоставленное Рис. 18.1 самому себе тело не сможет возвратить ся в исходное состояние. Указанная энер гия равна произведению силы тяжести на высоту, на которую надо приподнять центр тяжести тела, чтобы оно не могло вернуться в исходное щитояние. В пред ложенном примере е цилиндром и парал лелепипедом радиус цилиндра R = Vs/л, сторона основания параллелепипеда а = 108
a) Рис. 18 2 = л/S. Чтобы вывести цилиндр из равновесного положения, надо при- поднять его центр тяжести на высоту hx = У(Н/2)2 + R2 - (Я/2); чтобы вывести параллелепипед из равновесного положения, надо приподнять его центр тяжести на высоту Л2 = -/(H/if+Wif - (Н/2) _ а/2 л/nS/2 л/л (рис. 18.2, б). Так как—£-= —-г=— = ~2~< 1, "у 5 то отсюда следует, что h2 Таким образом, из двух рассмотренных тел цилиндр более устойчив. После этих замечаний я предлагаю вернуться к примеру с двумя положениями кирпичей. Читатель А: Если кувыркать кирпич, то он будет последовательно пере- ходить из одного равновесного положения в другое. На рис. 18.3 пунктиром показана траектория, которую при этом описывает центр тяжести кирпича. Чтобы вывести кирпич из стоячего положения, надо приподнять его центр тяжести на высоту hx, т. е. затратить энергию mghi, а чтобы вывести кирпич из лежачего положения, надо-шрипод- нять его центр тяжести на высоту h2, т. е. затратить энергию mgh2. Бо'льшая степень устойчивости тела в лежачем положении объясняется тем, что mg/ц С mgh2. Автор: Вот теперь Ваше обоснование тела в лежачем положении совер- шенно правильно. Читатель Б: Но ведь на высоты h{ и h2 как раз и влияют высота центра тя- жести от пола и площадь опоры. Значит, при обсуждении степени ус- тойчивости тел правомерно сравни- вать высоты центров тяжести и площадь опоры? Автор: Да, правомерно, но лишь в той мере, в какой эти величины влияют на различие между высотами hx и h2. Так, в примере с цилиндром и большей степени устойчивости Рис. 18.3 109
параллелепипедом сравнение высот центров тяжести и площадей опоры тел не позволяло решить вопрбс о том, какое из двух тел более устойчи- во. Кроме того, я хочу обратить Ваше внимание еще на одно обстоя- тельство. До сих пор молчаливо предполагалось, что сравниваемые тела сделаны из одного и того же материала. В этом случае для выпол- нения неравенства (18.1) было достаточно соблюдения геометрическо- го условия < h2. Однако в общем случае сравниваемые тела могут быть сделаны из различного материала, при этом неравенство (18.1) может выполняться даже при условии >h2 за счет соответствующего различия плотностей тел. Так, например, пробковый кирпич в лежачем положении менее устойчив, чем такой же кирпич из свинца, находящий- ся в стоячем положении. Рассмотрим теперь условия равновесия тела. Какие условия равнове- сия тела Вы знаете? Читатель А: Сумма всех сил, приложенных к телу, должна равняться нулю. Кроме того, вектор силы тяжести должен пересекать площадь опоры тела. Автор: Хорошо. Однако лучше сформулировать условия равновесия в ином виде, более общем и удобном для практического использования. Следует различать два условия равновесия. Первое: проекции всех сил, приложенных к телу, на любое направ- ление должны взаимно компенсироваться. Иными словами, алгебраи- ческая сумма проекций всех сил на любое направление должна рав- няться нулю. Это условие позволяет составить столько уравнений, сколько имеется в задаче независимых направлений: для одномерной задачи — одно, для двумерной — два, в общем случае — три (выбира- ются взаимно перпендикулярные направления). Второе (условие моментов): алгебраическая сумма моментов сил относительно любой точки должна равняться нулю. При этом все моменты сил, стремящиеся повернуть тело около выбранной точки в одАу сторону (например, по часовой стрелке), берутся со знаком плюс, все моменты сил, стремящиеся повернуть тело в другую сто- рону (против часовой стрелки), берутся со знаком минус. Чтобы за- писать условие моментов, надо: а) выявить все силы, приложенные к телу; б) выбрать точку, относительно которой предполагается рас- сматривать моменты сил; в) найти моменты всех сил относительно выбранной точки; г) составить алгебраическую сумму моментов сил и приравнять ее нулю. При использовании условия моментов необхо- димо учитывать, что, во-первых, приведенная выше формулировка от- носится к случаю, когда все силы и расстояния в задаче лежат в одной и той же плоскости (задача не является трехмерной), и во-вторых, равенство нулю алгебраической суммы моментов сил должно выпол- няться относительно любой точки, взятой как внутри тела, так и вне его. Следует подчеркнуть, что от выбора точки (относительно которой рас- сматриваются моменты сил) зависят значения отдельных моментов сил; однако алгебраическая сумма моментов всегда остается равной нулю Чтобы лучше понять условия равновесия, рассмотрим конкрет- ную задачу Балка закрепление точках Б и В (рис. 18.4, а). В точке Г 110
к балке подвешен груз G2. Даны расстояния: | А Б | — а, |БВ|=2а, \ВГ\ =а. Требуется найти силы реакции опоры NB и NB. Считать, что силы реакции опоры действуют по вертикальному направлению. Как обычно, укажите сначала силы, приложенные к телу. Читатель А: Телом в данной задаче является балка. К ней приложены че- тыре силы: силы тяжести G, и G2 и силы реакции NB и NB. Автор: Изобразите эти силы на чертеже. Читатель А: Но я не знаю, вверх или вниз направлены силы реакции. Автор: Предположите, что обе силы реакции направлены вверх. Читатель А: В таком случае вот мой рисунок (рис. 18.4, б). Я запишу первое условие равновесия в виде уравнения NB^NB = GrA-G2. (18.2) Автор: Я не имею ничего против этого уравнения. Однако в данной задаче проще воспользоваться вторым условием равновесия (условием момен- тов), применяя его один раз относительно точки Б, а другой раз— точки В. Читатель А: Хорошо, я так и сделаю. Момент силы NB относительно точки Б равен произведению этой силы на расстояние от точки Б до точки приложения силы В... Автор: В данном случае Ваше определение момента силы справедливо, но следует помнить, что момент силы равен произведению модуля силы на кратчайшее расстояние от точки, относительно которой рассматри- вается момент, до направления силы. Обычно это расстояние назы- вают плечом силы. Читатель А: Значит, чтобы определить моменты силы F относительно точки О (рис. 18.5), надо из точки О опустить на направление силы F перпендикуляр [ ОЛ], и тогда момент будет равен F • |ОЛ| = F • (OOj • sin а, Автор: Совершенно верно. Но давайте вернемся к нашей задаче. Читатель А: Я составлю два уравнения моментов. В результате получаю следующие уравнения: {G, а — NB2a 4- G23a = 0 (относительно точки Б), ' (18 3) N в2а — Gxa -|- G2a = 0 (относительно точки В). 111
Рис 18.5 Автор: Вот видите, у Вас в каждом уравне- нии содержится только одно из неизвест- ных. Его можно сразу вычислить. Читатель А: Из уравнений (18.3) находим: ^=(Gj-G2)/2, (18.4) tfB=(G, +3G2)/2. (18.5) Автор: Выражение (18.5) всегда положи- тельно. Значит, сила реакции NB всегда направлена вверх (как мы и предположи- ли). Выражение (18.4) положительно, если G, > G2, отрицательно, если Gt<zG2, и обращается в нуль, если Gj = G2. Это означает, что при G! > G2 сила МБ на- правлена так, как мы и предположили, т. е. вверх (см. рис. 18.4, б); при G,<G2 сила МБ направлена вниз (см. рис. 18.4, в), при G, = G2 сила реакции $Б отсутствует. Уравнение (18.2) можно использовать для проверки правильности решения системы (18.3). ЗАДАЧИ 18.1. К плоскому телу, имеющему форму квадрата, приложены пять сил: F = 2 Н, F2 = 2 Н, F3 = 2Н, F4 = 1 Н, F5 = 1 Н (рис 18.6) Сторона квадрата а — 2 м. Опре- делить моменты указанных сил относительно точек Б и В. Как движется это тело? 18.2. К плоскому телу, имеющему форму квадрата со стороной а = 2 м, приложено четыре силы по 2 Н (рис. 18.7). Определить равнодействующую R всех сил и мо- мент сил М. относительно точки А. А 18.3. Однородный квадрат в точке А опирается о выступ в стене, а в точке Б под- вешен на нити (рис. 18 8) Определить силу натяжения Т нити БВ, силу реакции R в точке А и направление этой силы Масса квадрата равна т, а угол между нитью и вертикалью а 18.4. На горизонтальной поверхности стола лежит цепочка длиной I Какова макси- мальная длина свешивающейся со стола части цепочки, если коэффициент трения между цепочкой и столок равен № 18.5. Какую работу А надо совершить, чтобы перевернуть тяжелый куб массой М вокруг ребра на другую грань’ Плотность материала куба равна р Рис. 18.6 112
18.6. Под каким наименьшим углом а к стене надо поставить на горизонтальный шероховатый пол лестницу массой т, чтобы человек массой М мог подняться по ней доверху’ Коэффициент трения скольжения лестницы о пол равен k 19. КАК ВЫ ОТЫСКИВАЕТЕ ПОЛОЖЕНИЕ ЦЕНТРА ТЯЖЕСТИ? Автор: Экзаменующиеся иногда испытывают затруднения в отыскании центра тяжести тела или системы тел. Есть ли у Вас какие-нибудь неясности в этих задачах? Читатель Б: Мне кажется, здесь все просто. Для симметричных тел центр тяжести определяется по положению оси симметрии; для несим- метричных тел положение центра тяжести определяется из условия: сумма моментов сил относительно центра тяжести равна нулю. Автор: Давайте сначала уточним, что такое центр тяжести. Любое тело можно рассматривать как совокупность материальных точек, на каж- дую из которых действует сила тяжести. Равнодействующая этих эле- ментарных сил тяжести и есть сила тяжести тела, а точка приложения этой равнодействующей называется центром тяжести тела. Читатель Б: Я знаю другое определение центра тяжести: центр тяжести тела — это точка пересечения прямых, вдоль которых действуют силы, сообщающие телу только поступательное движение. Автор: Вообще говоря, Ваше определение относится к центру масс, но поскольку под действием только силы тяжести тело движется поступа- тельно, то часто центр масс называют центром тяжести. Центр тяже- сти обладает следующим свойством: если в этой точке тело закрепить, то оно будет находиться в состоянии равновесия. Используя это свой- ство, обычно и определяют положение центра тяжести тела. При этом составляют уравнения моментов элементарных сил тяжести относи- тельно оси, проходящей через центр тяжести тела. Давайте рассмотрим конкретные примеры. 113
Определить положение центра тяжести однородной пластинки, пока- занной на рис. 19.1, а. Читатель Б: Сначала я разобью пластинку на два прямоугольника (рис. 19.1, б; пунктир; центры тяжести прямоугольников 1 и 2 нахо- дятся в точках А и Б). Эти точки совпадают с центрами симметрии этих прямоугольников. Силы тяжести прямоугольников пропорцио- ' нальны их площадям, сила тяжести G! прямоугольника 1 численно рав- на 6 ед (1 ед. пропорциональна площади 1 см2); сила тяжести G2 пря- моугольника 2 численно равна 10 ед. А что делать дальше, я не знаю. Автор: Спроецируем точки А и Б на координатные оси Ох и Оу, проекции обозначим буквами А,, Б} на оси Ох и А2, Б2 на оси Оу. Далее рас- смотрим «стержни» и А2Б2, полагая, что масса сосредоточена в концах «стержней», причем масса конца равна массе соответствующего прямоугольника (рис. 19.1, в). В результате задача по определению положения центра тяжести нашей пластинки сводится к отысканию центров тяжести «стержней» А1Б1 и А2Б2. Положения этих центров тяжести и будут являться координатами искомого центра тяжести пластинки. Доведем задачу до конца. Определим сначала положение центра тяжести «стержня» А^ (точки Cj). Составим уравнение моментов сил относительно точки С] (рис. 19.1, в): (хс — хА) 6 — (хБ — — хс) 10 = 0, где хА = 1 см, хБ = 3 см, хс — координата центра тяжести пластинки. Находим хс = 9/4 см. Аналогично определяем положение центра тяжести «стержня» А2Б2 :(ус — уА) 6 — (уБ — ус) 10 = 0, где уА = 1,5 см, уБ = 2,5 см. Вторая координата искомого центра тяжести равна ус = 5/8 см. Читатель Б: Я понял. Координату хс центра тяжести пластины я именно так и находил бы. Но я сомневался, можно ли таким же образом отыскивать координату ус. Автор: Вот еще пример. Определить положение центра тяжести тонкого однородного диска радиусом R, из которого вырезан круг радиусом г = R/2 (рис. 19.2, а). Вместо заданного круга с одним отвер- стием будем рассматривать систему двух тел: круга с двумя симметрич- ными отверстиями и кружка, который вставлен в одно из отверстий Рис 19.2 114
(рис. 19.2, б). Центры тяжести этих тел нахоштся в их центрах сим- метрии. Учитывая, что сила тяжести G круга с двумя отверстиями про- порциональна его площади (л/?2 — 2л/?2/4) = з/?2/2, а сила тяжести G\ маленького кружка пропорциональна плвщадп т/?2/4, приходим к про- стой задаче отыскания точки приложения равп > щйствующей двух сил, изображенных внизу на рис. 19.2, б. Обозна шм через х расстояние от искомого центра тяжести до геометрии i кого центра большого круга. Тогда согласно рис. 19.2, б можем запш 1гь(л/?2/4) (/?/2 — х) = = ( л А*2/2) х, откуда х — R/6. Возможен также иной подход к предложенной тадаче. Заменим данный круг с отверстием сплошным кругом (без каких-либо отверстий) плюс кружок, находящийся на том месте, где было отверстие, и имеющий отрицательную (т. е. направленную вверх) силу тяжести (рис. 19.2, в), которая будет компенсировать положительную силу тяжести соответ- ствующего участка сплошного круга, что в целом отвечает кругу с отверстием. В этом случае задача сводится к отысканию точки прило- жения равнодействующей сил, изображенных внизу на рис. 19.2, в. Согласно этому рисунку запишем л/?2х = л/? • (/?/2 4~х)/4, откуда, как и в предыдущем способе, х = /?/6 Читатель А: Мне больше нравится первый подход: в нем не приходится вводить отрицательную силу тяжести. Автор: Мне хотелось бы еще предложить задачу по отысканию центра тяжести системы грузов, изображенной на рис. 19 3, а. Даны шесть разных грузов (G\, G2..., G^), размещенных ы)оль стержня на одина- ковых расстояниях а друг от друга. Силой тяжести самого стержня пренебрегаем. Как Вы стали бы решать эту задачу? Читатель А: Сначала я рассмотрел бы две силы тяжести, например (щ и б2, нашел бы точку приложения их равнодействующей, нарисо- вал бы на чертеже эту равнодействующую (она равна сумме Gi + Cr2) и вычеркнул бы из дальнейшего рассмотрения силы G\ и (j2. Теперь вместо шести сил осталось только пять Далее я нашел бы точку при- ложения равнодействующей еще двух сил и т. д. Таким образом, путем последовательных операций я в конце концов пришел бы к искомой равнодействующей, точка приложения которой и есть центр тяжести данной системы. 115
Автор: Такой метод решения правилен. Однако он слишком громоздок Можно указать более изящный метод. Для этого мысленно укрепим стержень в его центре тяжести (в точке Б на рис. 19 3, б) Читатель Б (перебивая): Но ведь Вам пока еще неизвестно положение центра тяжести Откуда Вы знаете, что он находится между точ- ками приложения сил G5 и Gt? Автор: Мне безразлично, где именно находится центр тяжести. Я не буду пользоваться тем обстоятельством, что центр тяжести оказался между точками приложения сил и G4. Итак, мысленно укрепим стержень в его центре тяжести. В результате стержень будет находиться в сос- тоянии равновесия. При этом кроме шести сил на стержень будет действовать еще одна сила — сила реакции опоры N. Поскольку стержень уравновешен, можно воспользоваться условиями равнове- сия (см. § 18) Применим сначала первое условие равновесия для проекций сил на вертикальное направление: = Gj + G2G5 + G(G5 + Gfi, (19.1) затем второе уеловие равновесия (условие моментов), рассмотрев моменты сил по отношению к точке А на рис. 19.3, б (т е. к левому концу стержня) В этом случае все силы стремятся повернуть стер- жень по часовой стрелке, а сила реакции опоры стремится повер- нуть стержень против часовой стрелки Запишем N = G2a 4- G, • 2о + G4 • За + G- • 4а + G6 • 5а (19 2) Объединяя условия равновесия (19.1) и (19.2), находим отрезок |4В|, т. е. положение искомого центра тяжести, отсчитываемое от левого конца стержня: G,a 4- G • 2а 4 G • За 4- Gr • 4а 4- G, • 5а G, 4 G, -I- G, + G,4- Gh U ; Читатель А: Да Этот метод значительно проще. Автор: Обратите внимание, что Ваш способ решения задачи весьма чув- ствителен к количеству грузов на стержне (добавление каждого нового груза существенно усложняет решение), а мой способ ничуть не усложняется от увеличения числа грузов. При добавлении нового груза в числителе и знаменателе выражения (19.3) прибавится по одному слагаемому, только и всего. Читатель Б Можно ли определить положения центра тяжести стержня, используя только условие моментов? Автор: Да, можно Для этого надо записать условие равновесия момен- тов сил относительно двух различных точек Давайте именно так и сде- лаем. Будем рассматривать условие моментов сил относительно точек А и В (<м рис 19 3. б) Относительно точки А условие моментов выражает, я уравнением (19.2), а относительно точки В уравнение имеет вид N • (5а - 14Б|) - G\n 4 G, • 2а + Gt • За + G2 • 4а + G, • 5а (19 4) Разделив (19 2) на (19 4), получаем Gta 4- G • 2а 4- Gj • За 4- G. • 4а 4- G( • 5а 5а - |Д7П‘ ” G а 4 G'- 2а 4- G - За 4- G, • 4а 4- G, • 5а ’ 116
откуда находим |j45i (G5O 4- ^4 * 2а 4~ G^ ' За 4~ Gg ’ 4а 4- G] • 5а -I- G2^ 4~ G^ • 2а -|- 4- G4 • За -|- G5 * 4а -|- G6 • 5а) = 5а (G2a + G ( • 2а 4- G• За f- О5 • 4а -|- 4~ Gf, • 5а), z или |45| • 5a(G, + G2 + G3 + G4 + G5 + G6) ={G2a 4- G( • 2a-(-G4 • За-j- 4~ G5 • 4a 4- G6 • 5a) 5a. Итак, приходим к (19.3). Результат (19.3) можно обобщить. Если предположить, что в точке А взято начало координат оси Ох, то нетрудно видеть, что расстояния а, 2а, За и т. д. есть не что иное как координаты центров тяжестей грузов, |4Б| — х0 — координата центра тяжести системы грузов в выбранной системе координат. Кроме того, учтем, что G — mg Тогда (19.3) можно записать так: mlxi + т2х? + mixi 4 ... X, =-------;----:-----:----- 4 т2 -f т 4 (19.5) Для тела произвольной формы координату //0 центра тяжести также можно записать: (19 6) В формулах (19.5) и (19.6) знак обозначает суммирование по всем элементарным массам, х, и у, - координаты соответствующей элемен- тарной силы тяжести. ЗАДАЧИ 19.1. Определить положение центра тяжести ди<ка, в котором сделаны два круго- вых отверстия (рис. 19.4). Радиусы вырезанных отверстий равны соответственно половине и четверти радиуса R диска. 19.2. Определить центр тяжести фигуры, представляющей собой равносторонний треугольник, стороны которого равны а и сделаны из однородного материала, а в вершинах расположены грузы т, 2т, т Рассмотреть два случая: I) массой сторон треугольника пренебречь; 2) масса всех сторон М = 4m 19.3. В свинцовом шаре при выплавке об- разовалась сферическая полость, поверх- ность которой касается поверхности шара и проходит через его центр. Найти положение , центра тяжести получившегося тела, если радиус полости равен г = R/2 Рис. 19 4
19.4. Из квадратной однородной пластинки с длиной ребра а вырезали равнобед- ренный треугольник, основание которого равно а, а высота h Определить центр я жести получившейся фигуры 19.5. Найти координаты центра тяжести плоских однородных фигур, показанных н<1 рис 19 5, а—в (размеры на рисунке даны в сантиметрах). 20. ЗНАЕТЕ ЛИ ВЫ ЗАКОН ПАСКАЛЯ? Автор: Рассмотрим следующий опыт. Возьмем стеклянный колпак с отверстием в донышке. Перевернем колпак отверстием кверху и поста- вим на лист стекла (края колпака должны плотно прилегать к по- верхности стекла). Через отверстие, имеющееся в донышке, наполним колпак водой доверху и затем плотно закроем отверстие пробкой (рис. 20.1). Если поднять колпак, то вместе с ним поднимется и стекло (словно бы приклеенное), вода из-под колпака не выльется Если же пробку, закрывающую отверстие в донышке, вынуть, то при попытке поднять колпак стекло останется на месте, вода из-под колпака тут же выльется. Как Вы объясните это? Читатель А: Я догадываюсь, что надо воспользоваться законом Паскаля. Но я не очень представляю, в чем заключается сущность этого закона. Читатель Б: Закон Паскаля гласит: давление, оказываемое на жидкость в какой-либо ее точке, передается этой жидкостью одинаково во всех направлениях. Автор: Если отверстие в донышке колпака закрыто пробкой, то на лист стекла действует сверху давление, обусловленное тяжестью воды под колпаком, а снизу действует атмосферное давление. Второе существен- но больше первого, поэтому стеклянная пласти- на как бы «приклеивается» к колпаку. Когда пробка вынута, то атмосферное давление на по- Г верхность воды в отверстии в соответствии с - —— ---- - законом Паскаля передается водой на верхнюю _ _ поверхность стеклянной пластины... 1 ~----- Читатель А: Понимаю. Теперь на стеклянную - - —пластину действует сверху давление, обуслов- I ленное тяжестью воды, плюс атмосферное дав- ление, а снизу — только атмосферное давление рис 20 I Давление сверху оказывается больше, чем снизу. 118
Г~----------------1 Закон Архимеда обычно не привлекает к себе особого внимания. Совершенно напрасно. На основе этого закона можно построить весьма интересные вопросы U задачи. Обсудим вопрос о применимости этого закона в состоянии невесомости тел. Читатель Б: Признаться, все это удивительно. Отверстие в донышке кол- пака маленькое, площадь стекла, накрытая колпаком, большая. И тем не менее давление, оказываемое атмосферой на маленькую по площади поверхность воды в отверстии, передается на всю большую площадь стекла. Автор: И не только на поверхность стекла, но и на всю поверхность сте- нок колпака. Рассмотрим обыкновенный большой бидон, наполненный доверху водой (рис. 20.2, а). На поверхность воды в горлышке бидона Рис. 20 2 119
действует атмосферное давление ра. Покажите стрелочками давление на стенки и дно бидона. Читатель Б: По закону Паскаля давление ра передается на все точки стенки и Дйа (рис. 20.2, б). Читатель Аг.ррс. 20.2, б, по-Моему, неверен. Известно, что давление на стенки увеличивается с глубиной. Читатели Бт -Я забыл учесть давление, обусловленное тяжестью воды. На глубине h это дополнительное так называемое гидростатическое давление рйвно pgh, где р — плотность воды, a g — ускорение свобод- ного падения. Правильный рисунок есть рис. 20.2, в. Автор: Теперь все хорошо. Правда, если измерить длину ваших стрелок на рисунке, то можно сделать вывод, что давление около дна сосуда (и на само дно) примерно вдвое больше атмосферного. Какой же высоты должен был бы быть бидон, чтобы это наблюдалось на самом деле? Читатель Б: Я понимаю Ваше замечание. Такой бидон должен был бы иметь высоту около 10 м. Ведь атмосферное давление есть давление, какое оказывал бы на дно водяной столб высотой около 10 м. Автор: Но вернемся к закону Паскаля. Этот закон хорошо проявляется в гидравлическом прессе. Рассмотрим сначала очень простую задачу. Пусть поршень 1 имеет площадь а поршень 2 — площадь s2. На пор- шень 1 действуем силой F. Какую уравновешивающую силу надо приложить к поршню 2? Задачу поясняет рис. 20.3. Читатель А: На поршень 1 действует давление рх = F/sx. По закону Паска- ля это давление передается на поршень 2; оно действует на него снизу. Поэтому для уравновешивания надо поставить на поршень 2 тело мас- сой М, определяемой из соотношения F/s, = Mg/s2. (20.1) Автор: Правильно. Итак, в данном случае давление на поршень / сверху равно давлению на поршень 2 снизу. Поэтому уравновешивающая си- ла, приложенная к поршню 2, будет во столько же раз больше силы F, во сколько раз площадь поршня 2 больше площади поршня 1. А теперь рассмотрим более сложную задачу с гидравлическим прессом. Во-пер- вых, учтем массу поршней, а во-вторых, предположим, что поршень / находится выше поршня 2. Поршень 1 имеет площадь $! и массу пг{ \ на него поставили груз мас- сой Мх. Поршень 2 имеет площадь s2 и массу т2. Какова масса урав- новешивающего груза, который надо поставить на поршень 2, если из- вестно, что поршень 1 выше поршня 2 на высоту h и плотность жидкости равна р? Задачу поясняет рис. 20.4, а. 2 Читатель Б: Поршень / вместе со стоящим на нем грузом оказывает на жидкость давле- ние (Л^! + гщ) g/sx плюс дополнительное давление pgh, обусловленное тяжестью жидкости. По закону Паскаля все это давление передается на нижнюю поверх- рис 20 3 ность поршня 2. Оно должно быть уравно-
Рис. 20.4 вешено давлением (М2 + т2) g/s2, где М2 — искомая масса груза, который надо поставить на поршень 2. Таким образом, (Л1, + тх) g/sx + pgh = (М2 + т2) g/s2, (20.2) откуда М2 = (Мх + s2/st + phs2 — т2. Автор: Соотношение (20.2) и окончательный ответ правильные. Однако в Ваших рассуждениях нет достаточной четкости. Давайте выделим мысленно некоторую горизонтальную плоскость; на рис. 20.4, б сечение такой плоскости показано прямой ААХ. Рассмотрим давление, оказы- ваемое на плоскость соответственно в левом и правом коленах гидрав- лического пресса. Давление рх в левом колене складывается из дав- ления (М, + тх) g/sx, атмосферного давления ра и давления pgh. Дав- ление р2 в правом колене есть давление(Л42 + т2) g/s2 плюс атмосфер- ное давление ра. По закону Паскаля давление рх передается из левого колена пресса в правое (как и давление р2 передается из правого коле- на в левое). Эти давления уравновешены: Pi = Р2- (20.3) Атмосферное давление входит в обе части равенства (20.3); оно сокра- щается. После этого приходим к соотношению (20.2). Читатель А: По-видимому, закон Паскаля следует использовать также при решении задач с сообщающимися сосудами. Автор: Конечно. Рассмотрим задачу. Два сообщающихся сосуда с пло- щадью s сечения соединены внизу короткой узкой трубкой. В сосуды налита жидкость плотностью р0. Высота уровня жидкости равна Н (рис. 20.5, а). Найти высоту уровня жидкости в каждом сосуде, если: а) в левый сосуд долить объем У, жидкости плотностью р, (р, <р0); б) после этого в правый сосуд долить объем У2 жидкости плотностью Рг (р2 < Ро)- &се жидкости считать несмешивающимися. Читатель Б: Полагаю, что здесь нет сложностей. Достаточно посмотреть на рисунок (рис. 20.5, б). Давление на мысленную плоскость ААХ сле- ва равно pigVi/s + poghx, а справа равно pog(2// — /г,). Атмосфер- ное давление не учитываем, поскольку оно одинаково действует на 121
Рис. 20 5 жидкость в левом сосуде и на жидкость в правом сосуде. Таким обра- зом, hx— Н — К, p1/(2sp0), откуда нетрудно найти искомые высоты. Автор: Все совершенно правильно. Но это Вы рассмотрели первый вопрос задачи. Теперь обратитесь ко второму вопросу. Читатель Б: Второй вопрос иллюстрирует рис. 20.5, в. Давление на плос- кость АА{ слева pigV,/s + pQgh2, а справа p2gV2/s + p0g(2// —/i2). Сле- довательно, h2 = H — VrIp1/(2sp0) + V2p2/(2sp0). Автор: После того как мы вполне разобрались с законом Паскаля, обра- тимся к закону Архимеда. Эти два физических закона составляют основу гидростатики. ЗАДАЧИ 20.1 . Какая работа затрачена на подъем груза массой m = 1,5 т с помощью гидрав- лического пресса, если при этом малый поршень переместился на высоту /г, = 40 см? Площадь малого поршня в п — 20 раз меньше площади большого поршня. Опре- делить также силу F, приложенную к малому поршню. 20.2 . В два сообщающихся сосуда налита ртуть. Сечение одного из сосудов в п = 2 раза больше сечения другого. Узкий сосуд доливают до края водой. На сколько поднимется при этом уровень ртути в широком сосуде? Первоначально уровень ртути был на расстоянии I — 20 см от верхнего края сосудов. Плотность ртути ррт = 13,8 г/см3, плотность воды рв = 1 г/см3. 20.3 . В два сообщающихся сосуда налиты ртуть и вода одинакового объема V. Площадь сечения одного сосуда s, второго ns. Определить, на каких уровнях рас- положатся в сосудах ртуть и вода. Рассмотреть два случая — вода налита: 1) в широкий сосуд; 2) в узкий сосуд (п = 2). 21. ЗАЧЕМ НАДО ЗНАТЬ ЗАКОН АРХИМЕДА? Автор: Знаете ли Вы закон Архимеда? Читатель Б: Конечно. На погруженное в жидкость тело действует вытал- кивающая сила, равная весу жидкости, вытесненной телом. Здесь все очень просто. Автор: Не спешите с выводами относительно простоты. 122
Читатель А: Я полагаю, что при формулировке закона Архимеда надо говорить не «весу жидкости», а «силе тяжести жидкости». Автор: Почему Вы так полагаете? Читатель А: По аналогии с механикой. Ведь там мы везде подменили прежний термин «вес» термином «сила тяжести», а под «весом» услови- лись понимать силу нормального давления, действующую на горизон тальную опору. Автор: В отношении закона Архимеда Вы неправы. Здесь надо употреб- лять именно термин «вес». В этом можно убедиться’ ознакомившись со следующим параграфом. Забегая вперед, замечу, что в лифте, дви- жущемся с ускорением, выталкивающая сила зависит от ускорения Ну а сила тяжести не может зависеть от ускорения лифта. Итак, вернемся к закону Архимеда. Читатель Б: По-видимому, следовало бы добавить, что на тело, «погру- женное» в газ, тоже действует выталкивающая сила. Автор: Правильно. А теперь не могли бы Вы доказать справедливое! i> закона Архимеда? Читатель А: Доказать справедливость закона Архимеда? Автор: Да. Читатель А: Но закон Архимеда получен непосредственно из опьиа Автор: Это верно. Однако его можно вывести из простых энергетических соображений. Будем мысленно поднимать некоторое тело объемом Г и плотностью р на высоту Н, делая это один раз в вакууме, а другой раз в жидкости с плотностью р0. В первом случае на указанный подъем надо затратить энергию pgVH (где pV = т). Во втором случае затра- ченная энергия оказывается меньше, поскольку поднятие тела объемом V на высоту Н в жидкости сопровождается одновременным опусканием на такую же высоту объема V жидкости. Поэтому теперь для подъем;, тела необходимо затратить энергию (pgVH — pQgVH). Рассматривая вычитаемое pQgVH как работу некоторой силы, можно заключить, что по сравнению с вакуумом на тело в жидкости действует дополни тельная сила /7=ро^К которая и облегчает подъем тела. Эту си.и называют выталкивающей или архимедовой силой. Нетрудно виден что она равна весу жидкости в объеме V погруженного в жидко» и. тела. (Заметим, что в приведенных рассуждениях не принимались ы внимание энергетические потери, связанные с трением при реальны» перемещениях тела в жидкости.) К закону Архимеда можно прийъ также несколько иным способом. Предположим, что погруженное । жидкость тело имеет форму цилиндра высотой h с площадью основаh.i . S (рис. 21.1). Допустим, что давление жидкости на верхнее дно iih.iHh ра равно р. Тогда давление на нижнее дно равно р 4- pogh. Таки я об;»а зом, существует разность давлений на нижнее и верхнее основания цилиндра, равная pQgh. Если эту разность умножить на площадь <S о. нования, то получим силу F = poghS, которая стремится вытолкнут!, о- ло вверх. Так как hS = V — объем цилиндра, то легко видеть, то ио . есть выталкивающая сила, которая фигурирует в законе Архимед.< Читатель А: Да, я вижу, что к закону Архимеда можно прийти чш г» умозрительно. Автор: Прежде чем решать задачи, давайте вспомним условие плавании тел. 123
Рис. 21.1 Читатель А: Я помню это условие. Сила тяжести должна быть уравновешена выталкивающей силой, действующей на тело согласно закону Архимеда. Автор: Правильно. Рассмотрим пример. В сосу- де с водой плавает кусок, льда. Что будет с уровнем воды в сосуде после того, как лед растает? Читатель А: Положение уровня не изменится, так как сила тяжести, действующая на лед, уравновешена выталкивающей силой и, следовательно, равна весу жидкости (в данном случае воды), вытесненной льдом. Когда лед рас- тает, он превратится в воду, объем которой он вытеснял ранее. Автор: Совершенно верно. А теперь предположим, что внутри куска льда находился, например, кусочек свинца. Что будет с уровнем воды после таяния льда в этом случае? Читатель А: Если я не ошибаюсь, то уровень воды в сосуде должен немного понизиться. Однако я затрудняюсь доказать это утверждение. Автор: Обозначим: V — объем куска льда вместе со свинцом; v — объем , кусочка свинца; V\ — объем воды, вытесненной подводной частью кус- ка льда; pQ — плотность воды; pt — плотность льда; р2 — плотность свинца. Сила тяжести, действующая на кусок льда со свинцом, равна Pi£(V- ^) + р2^- Эта сила тяжести уравновешена выталкивающей силой р0^И|. Таким образом, Pi£(V — у) + Р2^ = Ро£^1- (21-1) После таяния лед превратится в воду, объем К2 которой определяется из равенства p,g (V — v) = p0gV2. Подставляя это равенство в (21.1), находим Ро£^2 + Р2^= Po^i- Отсюда следует, что объем воды, образовавшейся в результате таяния льда, равен И2=Г, -о(р2/р0). ' (21.2) Итак, до таяния льда был вытеснен объем воды Vp Затем свинец и вода от таяния льда стали занимать объем (У2 4- у). Чтобы ответить на во- прос о положении уровня воды в сосуде, надо сопоставить эти объемы. Из формулы (21.2) получаем К2 + ^=К1 у(р<2 Ро)/Ро- (21-3) Так как р2 >• р0 (свинец тяжелее воды), то из (21.3) видно, что (К2 4- v) <. Vi. Следовательно, в результате таяния льда уровень воды в сосуде понижается. Разделив разность объемов V, — (К2-(-о) на площадь S поперечного сечения сосуда (для простоты полагаем, что сосуд имеет цилиндрическую форму), найдем высоту h, на которую опустится уровень воды после таяния льда: 124
л=4(~-1\ (2L4) S \ Po / Понятно ли Вам решение задачи? Читатель А: Да, понятно. Автор: Давайте теперь вместо кусочка свинца внутри льда rioMei тим кусо- чек пробки объемом и и п /нтгностью р3. Чю стане! < уровнем воды после таяния льда9 Читатель А: 7ь думаю, что уровень воды в сосуде немною поднимется. Автор: Почему? Читатель А: В задаче со свинцом уровень воды опускался. Свинец тяже- лее воды, а пробка легче воды. Поэтому в случае с пробкой надо ожи- дать противоположного эффекта: уровень воды должен повыситься. Автор: Вы ошибаетесь. Ваш ответ был бы верен, если бы пробка после таяния льда оставалась внутри воды. Но это, очевидно, невозможно. Поскольку пробка легче воды, она обязательно всплывет на поверх- ность. Поэтому случай с пробкой (как и с любым другим телом, которое легче воды) требует специального рассмотрения. Используя результат (21.2), найдем разность объемов воды, вытесненной куском льда с пробкой, и воды, образующейся вследствие таяния льда: V! - V2= у(р3/ро). : (21.5) Далее воспользуемся условием плавания для пробки: р3у=р0У1, (21.6) где щ — объем пробки, погруженной в воду. Подставляя это равен- ство в (21.5), находим V] = V2 + ^1. Таким образом, объем воды, вытесненной куском льда, оказывается в точности равным сумме объемов воды, образующейся от таяния льда, и воды, вытесняемой погруженной частью пробки. Отсюда следует, что уровень воды в сосуде остается неизменным. Читатель А: А если бы вместо пробки в куске льда был просто пузырек воздуха? Автор: После таяция льда этот пузырек выйдет наружу. Нетрудно убе- диться, что при этом уровень воды в сосуде примет в точности то же положение, которое он имел до таяния льда. Одним словом, случай Рис 21.2 125
с пузырьком воздух.' >.о льду анало!ичен случаю с кусочком пробки во льду. Читатель А: Я вижу, чк> h.i основании закона Архимеда можно придумать довольно интересньи вопросы и задачи. Автор: К сожалению, некоторые экзаменующиеся относятся к этому за- кону пренебрежите и>но и не дают себе труда подумать над ним в про- цессе подготовки к экзамену. Рассмотрим следующий пример. На одной чаше весов стоит сосуд с водой, а на другой - штатив, на котором подвешен груз. Чаши весов уравновешены (рис 21.2, а). Штатив поворачивают так, чтобы подве- шенный на нем гру < оказался целиком погруженным в воду. Очевидно, что равновесие чаш нарушится, так как чаша со штативом станет легче (рис. 21.2, б) Определить вес гири, которую надо дополнительно положить на чаши < о штативом для того, чтобы восстановить равно- весие весов? Читатель А: На груз н-йствует выталкивающая сила, равная весу Р воды в объеме груза. Следовательно, для восстановления равновесия надо положить на чашу со штативом гирю весом Р. Автор: Вы ошибаетесь. Тут не мешает вспомнить третий закон Ньютона. Согласно ему, с какой силой вода в сосуде действует на груз, с такой же силой, но npoiивоположно направленной, будет действовать груз на воду. Следова юльно, одновременно с уменьшением веса чаши со штативом увеличивается вес чаши с сосудом. Поэтому для восстанов- ления равновесия потребуется гиря весом 2Р. Читатель А: Я что-то не совсем понял Ваше рассуждение. Все-таки взаимодействие груза и воды в сосуде не похоже на взаимодействие двух тел в механике. Автор: Область деиовия ipc .< к» »акона Ньютона не ограничена меха- никой. Выражение < юн нше равно противодействию» относится к самым различным и ыимодействиям. Однако здесь нетрудно привести иное рассуждение, против которого Вы наверняка не станете возра- жать. Будем рассматривать штатив с грузом и сосуд с водой как еди- ную систему: ее вес складывается, очевидно, из веса левой чаши и веса правой чаши. Общий вес сис- темы не может измениться от того, что ее части взаимодейст- вуют друг с другом. Поэтому если в результате взаимодейст- вия вес правой чаши умень- шился на Р, то это же взаимодействие должно при- вести к увеличению веса левой чаши на то же самое значение веса Р. Поэтому после погруже- ния груза в сосуд с водой раз- ность между весом левой и весом правой чаши должна составлять 2Р. 126
Читатель Б: Я давно хотел разобраться в одной практической задаче. Известно, что подводной лодке не рекомендуется опускаться на глини- стое ровное дно — она может потом не всплыть. Насколько я понимаю, в этой задаче надо использовать закон Архимеда. Автор: Вместе с законом Паскаля. Прежде всего, как Вы сами объясните, почему подводной лодке опасно ложиться на глинистое дно? 1 Читатель Б: Когда подводная лодка ложится на такое дно, может оказать- ся, что между ее днищем и морским дном вообще не окажется воды. В этом случае выталкивающая сила, помогающая лодке всплыть, исчезает. Автор: Вы правы: выталкивающая сила в данном случае действительно исчезает. Однако дело не только в этом. На рис. 21.3, а изображена подводная лодка, которая находится почти над самым дном, но не лежит на нем. Сверху на лодку действует давление р,, а снизу давле- ние р2. Легко сообразить, что р^ = pgHi + ра, где ра — атмосферное давление (оно передается на погруженный объект в соответствии с законом Паскаля), Р2=Р£Я24-ра. Таким образом, существует разность давлений: р2 — р] = pg(H2 — Я]), которая и создает выталкивающую силу (архимедову силу). Читатель Б: Мы могли бы здесь вообще не принимать во внимание атмос- ферное давление, слагаемое ра сокращается, когда мы находим раз- ность р2 — р]. Автор: Вы правы. Однако при погружении лодки на глинистое дно атмо- сферное давление уже нельзя не учитывать. Лодка плотно лежит на глинистом дне (рис. 21.3, б), на нее действует только давление сверху, равное р = pgH + ра. Вы видите, как существенно изменилась ситуация. В предыдущем слу- чае на лодку действовало результирующее давление снизу, равное р^(Я2 ~ Я]). Теперь же этого давления снизу нет (нет выталкивающей силы). Но дело не только в этом! Теперь на лодку действует неуравно- вешенное мощное давление сверху, складывающееся из гидростатиче- ского давления pgH и атмосферного давления ра. Именно это, а не тот факт, что отсутствует выталкивающая сила, и представляет Серьезную опасность для подводной лодки. ЗАДАЧИ 21.1. Вес тела, погруженного в жидкость с плотностью рг равен Р а погружен- ного в жидкость с плотностью р2 — Р2. Определить плотность р тела 21,2. Некоторое тело, весящее в воздухе Р, будучи погружено в жидкость с плот- ностью р,, весит Р, а жидкости неизвестной плотности — Р2. Определить плот- ность р2 второй жидкости 21.3. В сосуд, имеющий цилиндрическую форму с площадью поперечного сечения 3, налита вода, в которой плавает кусок льда со свинцовым шариком внутри. Объем куска льда вместе с шариком равен V; над водой выступает 1 /20 часть этого объема На какую высоту h опустится уровень воды в сосуде после того, как лед растает? Плотность воды рв = 1 г/см3, льда рп = 0,9 г/см3, свинца р = 11,3 г/см3. 21.4. Ареометр, состоящий из шарика, заполненного дробью, и цилиндрической 127
трубки с площадью S поперечного сечения, плавает в жидкости с плотностью р Ареометр погрузили еще немного в жидкость и отпустили, после чего он начал совершать свободные колебания около положения равновесия. Определить период Т этих колебаний. Масса ареометра М. 22. ДЕЙСТВУЕТ ЛИ ЗАКОН АРХИМЕДА В КОСМИЧЕСКОМ КОРАБЛЕ? Автор: Действует ли закон Архимеда в космическом корабле, когда он находится в состоянии невесомости? Читатель А: По-моему, не действует. Ведь сущность закона Архимеда связана с тем, что из-за различия плотностей тела и жидкости (взятых, разумеется, в одинаковых объемах) требуется различная работа по поднятию их на одну и ту же высоту. В состоянии же невесомости различие в указанных работах, очевидно, отсутствует, так как и работа по поднятию тела, и работа по поднятию равного ему объема жидко- сти равны нулю. К такому же заключению приходим, если использовать силы давления жидкости на тело, погруженное в нее, так как выталкивающая сила обусловлена различием сил давлений на нижнее и верхнее основания тела. В состоянии невесомости это различие исчезает, а вместе с ним исчезает и выталкивающая сила. Можно заметить, что в состоянии невесомости нет различия между «верхом» и «низом», поэтому нельзя даже указать, какое основание тела является верхним, а какое — нижним. Таким образом, в состоянии невесомости на тело внутри жидкости не действует выталкивающая сила, значит, закон Архимеда не выпол- няется. Читатель Б: Я не согласен с окончательным выводом читателя А. Я счи- таю, что закон Архимеда действует также и в состоянии невесомости. Будем рассуждать более осторожно. Не будем сразу переходить к неве- сомости, а рассмотрим сначала движение лифта с некоторым ускоре- нием а, направленным так же, как и ускорение свободного падения g При этом пусть а < g. Нетрудно сообразить, что в данном случае на тело, погруженное в жидкость, будет действовать выталкивающая сила F=p.{g-a) У; (22.1) вес жидкости в объеме тела тоже равен р0 {g — а) V. Таким образом, выталкивающая сила по-прежнему равна весу жидкости, вытесненной телом, т. е. закон Архимеда выполняется. Далее будем постепенно уве- личивать ускорение а, приближая его значение к значению g. В соот- ветствии с (22.1) выталкивающая сила при этом будет постепенно уменьшаться, но одновременно и точно так же будет уменьшаться вес жидкости в объеме тела. Иначе говоря, при приближении ускорения а к ускорению g закон Архимеда будет неизменно оставаться в силе В пределе а = g наступит состояние невесомости. При этом выталки- вающая сила обратится в нуль, но обратится в нуль и вес жидкости, вытесненной телом. Поэтому ничто не мешает утверждать, что закон Архимеда действует также и в состоянии невесомости. Я хочу проиллюстрировать свои рассуждения на следующем нагляд- ном примере. Предположим, что в сосуде с водой плавает пробка В соответствии с (21.6) отношение объема пробки, погруженного в во- 128
ду, ко всему объему пробки равно отношению плотности пробки к плотности* воды: ^Л = Рз/ро- (22.2) Предположим, что сосуд находится в лифте и лифт начинает падать с некоторым ускорением а. Так как при этом плотности пробки и воды не изменяются, то соотношение (22.2) остается справедливым. Иначе говоря, при движении лифта с ускорением положение пробки относи- тельно поверхности воды в сосуде остается точно таким же, как и в отсутствие ускорения. Очевидно, что это положение не изменится и в предельном случае, когда а= g, отвечающем состоянию невесомости. Таким образом, определяемое законом Архимеда положение пробки относительно поверхности воды оказывается не зависящим от ускоре- ния лифта, и никакого различия между случаями отсутствия и наличия невесомости мы в данном случае не обнаруживаем. Автор: Оба рассуждения были вполне аргументированы. Однако я должен согласиться с читателем А: в состоянии невесомости закон Архимеда не действует. Читатель Б: Но тогда опровергните мои доказательства. Автор: Сейчас я и попытаюсь это сделать. Ваши рассуждения построе- ны на двух основных соображениях: 1) при наличии ускорения a<Zg тело выталкивается из жидкости в полном соответствии с законом Архимеда; 2) это утверждение должно оставаться в силе и в предель- ном случае, когда а = g, т. е. наступает невесомость. Против первого соображения я не возражаю, однако со вторым не согласен. Читатель Б: Но ведь пробка в сосуде остается в том же положении и в состоянии невесомости! А ведь ее положение обусловлено законом Архимеда. Автор: Да, пробка остается в том же положении и в состоянии невесомо- сти. Однако в этом состоянии ее положение относительно поверхности жидкости уже не обусловливается законом Архимеда. Погрузите в этом случае пробку в глубь воды, и она будет неподвижно висеть на той глубине, на которой Вы ее оставите, тогда как при наличии хоть самой малой разности (g — а) пробка обязательно будет всплывать и занимать положение, определяемое законом Архимеда. Таким обра- зом, между случаями невесомости и наличия хотя бы ничтожной «весомости» существует принципиальное различие. Иначе говоря, при переходе в состояние невесомости «в самый последний момент» про- исходит скачок, качественно меняющий всю ситуацию. Читатель Б: Но с чем может быть связан такой скачок? Откуда он взялся? Ведь мы же плавно приближали значение ускорения а к g. Автор: Этот скачок связан с тем, что при а = g появляется симметрия: исчезает различие между «верхом» и «низом», на что, кстати говоря, очень правильно указывал читатель А. Если разность (g — а) сколь угодно мала, но все же не равна нулю, в задаче есть физически выде- ленное направление «снизу вверх». Именно по этому направлению и действует выталкивающая сила. Однако при а = g это направление исчезает и все направления становятся физически эквивалентными. Вот Вам и скачок. Уничтожение либо появление симметрии всегда происходит скачком. 129
Современная физика в основе своей является молекулярной физикой. Поэтому особенно важно, хотя бы на простейшем примере идеального газа, ознакомиться с основами молекулярно-кинетической теории вещества. Отдельно обсудим вопрос об особенностях теплового расширения воды. Подробно проанализируем газовые законы и применим их к решению конкретных технических задач. 23. ЧТО ВЫ ЗНАЕТЕ О МОЛЕКУЛЯРНО-КИНЕТИЧЕСКОЙ ТЕОРИИ ВЕЩЕСТВА? Автор: В экзаменационных билетах есть такой вопрос: основные положе- ния молекулярно-кинетической теории вещества. Как Вы стали бы отвечать на этот вопрос? Читатель А: Я указал бы два основных положения: 1) все тела состоят из молекул; 2) молекулы находятся в состоянии хаотического тепло- вого движения. Автор: Ваш ответ весьма характерен: лаконичен и неполон. Я заметил, что учащиеся привыкли формально относиться к этому вопросу. Как правило, они не знают, что надо говорить об основных положениях молекулярно-кинетической теории, и отделываются несколькими общи- ми фразами. В связи с этим я считаю необходимым более подробно побеседовать о молекулярно-кинетической теории вещества. Сначала я отмечу те положения молекулярно-кинетической теории, которые можно считать основными. I. Вещество имеет «зернистую» структуру: оно состоит из молекул (атомов). В одном моле вещества содержится NA = 6 • 1023 молекул независимо от агрегатного состояния вещества (число NA называется постоянной Авогадро). 130
2. Молекулы в веществе находятся в непрекращающемся тепловом движении. 3. Характер теплового движения молекул зависит от характера взаи- модействия молекул и изменяется при переходе вещества из. одного агрегатного состояния в другое. 4. Интенсивность теплового движения молекул зависит от степени нагретости тела, характеризуемой абсолютной (термодинамической) температурой Т В теории доказано, что средняя энергия (е) отдельной молекулы пропорциональна температуре Г; так, например, для одно- атомных молекул <е>=3/2/гГ, (23.1) где k= 1,38 • 1023 Дж/К — физическая константа, называемая постоян- ной Больцмана. 5. С точки зрения молекулярно-кинетической теории, полная энергия Е тела является суммой следующих слагаемых: Е = £к + Еп + U, (23.2) где Ек — кинетическая энергия тела как целого, Еп — потенциальная энергия тела как целого в некотором внешнем поле, U — энергия, связанная с тепловым движением молекул тела. Энергию U называют внутренней энергией тела. Учет внутренней энергии тела при рас- смотрении различных энергетических балансов является характерной чертой молекулярно-кинетической энергии. Читатель Б: М.ы привыкли к тому, что вопрос о моле и постоянной Аво- гадро относится к химии. Автор: По-видимому, поэтому учащиеся на экзамене по физике довольно часто не знают, что такое моль, и, как правило, всегда уверены, что постоянная Авогадро относится только к 'газам. Запомните: моль — это количество вещества, содержащее столько же молекул, сколько содержится атомов в 0,012 кг углерода (а отнюдь не масса одной молекулы, выраженная в граммах, как это часто можно услышать). Модярная масса М = mQNA, где т0 — масса одной молекулы, NA = = 6 • 1023 моль- 1 — постоянная Авогадро (число молекул или атомов в одном моле любого вещества независимо от агрегатного состояния). Мне хотелось бы отметить, что постоянная Авогадро играет роль своеобразного «мостика» между макро- и микрохарактеристиками вещества. Так, например, пользуясь постоянной Авогадро, можно выразить через плотность и молярную атомную массу вещества такую микрохарактеристику, как среднее расстояние между молекулами (атомами) вещества. Возьмем для примера твердое железо. Его плот- ность р = 7,8 г/см3, молярная масса А = 56 г/моль. Найдем среднее рас- стояние между атомами в железе. Будем рассуждать следующим обра- зом: в А г железа содержится NA атомов; значит, в 1 г железа содер- жится Na/A атомов; отсюда следует, что в 1 см3 железа содержится pNA/A атомов. Таким образом, на один атом в железе приходится /Объем А/(рАл) см3. Искомое среднее расстояние между атомами приблизительно равно кубическому корню из этого объема: 131
X»^A/(pNA) = '^56/(7,8 • 6 • 102J) <m 2 • 10 \m Читатель Б: Ранее Вы указывали, чю харам ip ип.юьою движения молекул зависит от межмолекулярного взаимодействия и изменяется при переходе из одного агрегатного состояния в другое-. Расскажите об этом подробнее. Автор: Взаимодействие двух молекул можно качеовенно описать с по мощью кривой, изображенной на рис. 23.1. Эта кривая передает за- висимость потенциальной энергии 1Г взаимодействия молекул от рас- стояния г между центрами молекул. При достаточно большом рас- стоянии между молекулами кривая W (г) асимптотически приближа- ется к нулю, т. е. молекулы практически перестают взаимодействовать По мере сближения молекул между ними возникает притяжение, кривая W (г) понижается. Затем при достаточно сильном сближении молекулы начинают отталкиваться, кривая W (г) начинает возрастать (это отталкивание означает, что молекулы не могут свободно прони- кать одна внутрь другой). Легко видеть, что кривая W(г) имеет характерный минимум. Читатель Б: Что означает отрицательность энер]ии? Автор: Как известно, энергию можно измерять от любою значения Например, можно измерять потенциальную энергию камня от уровня данной местности, а можно измерять и от уровня моря — это безраз- лично. В данном случае за нуль принимается энергия взаимодей ствия молекул, находящихся друг от друга на бесконечно большом расстоянии. Поэтому отрицательность энергии молекулы означает, что она находится в связанном состоянии (с другой молекулой). Чтобы «освободить» эту молекулу, ей необходимо добавить некоторую энер- гию, с тем чтобы энергия молекулы увеличилась до нулевого уровня Предположим, что молекула имеет отрицательную энергию ?! (рис. 23.1). Из рисунка видно, что в этом случае молекула не может уйти от своей соседки далее точки Б и не может приблизиться к ней бли- же точки А. Иначе говоря, она будет совершать колебания между точ- ками А и Б в поле соседней молекулы (точнее говоря, возникают относи- тельные колебания двух молекул, образующих связанную систему). В газе молекулы в среднем находятся на таких больших расстояниях друг от дру- га, что их можно считать практически не взаимодействующими. Каждая из моле- кул движется свободно от других моле- кул, испытывая относительно редкие столкновения. При этом каждая молеку- ла участвует в трех типах движения- поступательном, вращательном (молеку- ла вращается вокруг самой себя) и колебательном (атомы внутри молекулы колеблются друг относительно друга). Если молекула одноатомная, то имеет место только поступательное движение. 132
в крнсыллс молекулы все вместе образуют единую связанную систему. Каждая молекула совершает колебания в общем силовом поле, об- условленном взаимодействием коллектива молекул. Для кристалла ха- рактерно наличие пространственной упорядоченной структуры — кри- сталлической решетки. Узлы этой решетки представляют собой поло- жения равновесия отдельных молекул. Около этих положений и совер- шают молекулы свои сложные колебательные движения. Цитатель Б: Получается, что существуют колебания молекул в поле кри- сталла и колебания атомов внутри каждой отдельной молекулы? Автор: Это так при условии, что энергия связи атомов внутри молекулы заметно больше энергии связи самих молекул в кристаллической решет- ке. В этом случае молекулы, объединенные в кристалл, как бы продол- жают сохранять свою индивидуальность. Однако часто при образова- нии кристалла молекулы своей индивидуальности не сохраняют, так что кристалл оказывается построенным фактически не из молекул, а из от- дельных атомов. При этом внутримолекулярные колебания отсутст- вуют; имеются только колебания атомов в поле кристалла. Читатель Б: Вы ничего не сказали о характере теплового движения моле- кул в жидкости Автор: Характер тепловою движения в жидкости наиболее сложен. Жид- кость, занимая промежуточное положение между газом и кристал- лом, обладает наряду с сильным межчастичным взаимодействием в значительной мере разупорядоченной структурой. Трудности рас- смотрения кристаллов, обусловленные сильным взаимодействием час- тиц, в значительной степени компенсируются наличием упорядоченной структуры — кристаллической решетки. Трудности рассмотрения га- зов, обусловленные разупорядоченностью положений отдельных час- тиц, компенсируются практически полным отсутствием межчастичного взаимодействия. В случае же жидкостей имеются обе указанные труд- ности при отсутствии в то же время соответствующих компенсирующих факторов. Можно сказать, что в жидкости молекулы, как правило, вполне сохраняют свою индивидуальность. Для жидкости характерно существование самых различных типов движений: перемещения моле- кул, вращения молекул, колебания атомов внутри отдельных молекул, колебания молекул в поле соседних молекул. Самое сложное состоит в том, что все эти типы движений нельзя, строго говоря, рассматривать порознь (как говорят, «в чистом виде»): существует сильное взаимное влияние движений. Читатель Б: Я не понимаю, как можно совместить поступательное переме- щение молекулы и колебания молекулы в поле соседей. Автор: Существуют разные модели, в которых делаются попытки совме- стить указанные движения. Например, в одной модели предполагается, что молекула ведет себя так: она некоторое время колеблется в поле соседей, затем делает скачок, переходит в новое окружение, колеблется в этом новом окружении, затем снова делает скачок и т. д. Такая мо- дель называется «моделью диффузии скачком». Читатель Б: Мне кажется, что именно так должна происходить диффузия атомов в кристаллах. Автор: Вы правы. Только учтите, что в случае кристаллов этот процесс 133
л) (•> @ ® ©•(•)(•)(•)(•) более замедленный: скачки в новое окружение происходят значительно реже. Существует также другая мо- о) дель, согласно которой моле- г\ кула в жидкости ведет себя ~ следующим образом: она со- вершает колебания в окруже- рис 2з 2 нии своих соседей, а само это окружение плавно перемеща- ется в пространстве («плы- вет») и при этом постепенно деформируется. Такая модель называется «моделью непрерывной диффузии». Читатель Б: Вы говорили, что жидкость занимает промежуточное поло- жение между кристаллами и газами. К чему она ближе? Автор: А как Вы сами думаете? Читатель Б: Мне кажется, что жидкость все же ближе к газу Автор: Однако в действительности жидкость скорее ближе к кристаллу; на это указывает количественная близость плотностей, удельных тепло- емкостей, коэффициентов объемного расширения жидкостей и кристал- лов. Известно также, что теплота плавления существенно меньше теплоты парообразования. Все эти факты свидетельствуют б заметном сходстве сил межчастичного сцепления в кристаллах и жидкостях. Следстием указанного сходства является также установленное в опы- тах по рассеянию рентгеновских лучей наличие элементов упорядочен- ности в расположении атомов жидкости, известное под термином «ближний порядок». Читатель Б: Что это такое? Автор: Ближний порядок означает упорядоченное расположение около произвольно выбранного атома (молекулы) некоторого числа его ближ- них соседей. В противоположность кристаллу это упорядоченное рас- положение по отношению к выбранному атому не сохраняется по мере удаления от него и не приводит к образованию кристаллической решет- ки. Однако на малых расстояниях оно вполне сходно с расположением атомов данного вещества в твердой фазе. На рис. 23.2, а показан дальний порядок для цепочки атомов, который сравнивается с ближ- ним порядком, изображенным на рис. 23.2, б. Наличие сходства между жидкостями и кристаллами привело к появлению термина «квазикри- сталличность» жидкости. Читатель Б: Но в таком случае жидкость, по-видимому, можно рассматри- вать по аналогии с кристаллами? Автор: Следует предостеречь от злоупотребления понятием квазикристал- личности жидкости и приписывания ему неоправданно большего зна- чения. Во-первых, надо иметь в виду, что жидкому состоянию соответ- ствует широкая область температур, и нельзя ожидать, что структурно- динамические свойства жидкости будут одинаковы (или даже примерно одинаковы) по всей этой области. Вблизи критического состояния жидкость, очевидно, должна утрачивать всякое сходство с твердым телом и плавно переходить в газовую фазу. Таким образом, если поня- тие квазикристалличности жидкости и обосновано, то во всяком случае 134
не слишком далеко от точки плавления. Во-вторых, характер межмо- лекулярного взаимодействия в разных жидкостях различен, в связи с чем понятие квазикристалличности оказывается применимым к раз- ным жидкостям в различной степени. Так, например, вода оказывается более квазикристаллической жидкостью, нежели расплавленные ме- таллы, и это обусловливает ряд особенностей ее свойств (см. § 24). Читатель Б: Я вижу, что простой картины тепловых движений молекул в жидкости не существует. Автор: Вы правы. Относительно просты только крайние случаи. Промежу- точный случай всегда сложен. Читатель А: В программе для поступающих стоит вопрос: опытное обо- снование молекулярно-кинетической теории вещества. По-видимому, здесь надо рассказать о броуновском движении? Автор: Да, броуновское движение является ярким экспериментальным обоснованием основных положений молекулярно-кинетической теории. Но знаете ли Вы, что такое броуновское движение? Читатель А: Это есть тепловое движение молекул. Автор: Вы ошибаетесь: ведь броуновское движение наблюдают в обычные микроскопы! Броуновское движение есть движение отдельных частиц вещества под воздействием ударов молекул среды, совершающих тепловые движения. Эти частицы, с молекулярной точки зрения, сами являются макротелами. Однако, с точки зрения наших обычных мас- штабов, они очень малы. В результате случайных нескомпенсиро- ванных ударов молекул броуновские частицы совершают хаотические отклонения и тем самым перемещаются по среде, в качестве которой используют какую-нибудь жидкость. Читатель Б: А почему броуновские частицы должны быть все же относи- тельно малы? Почему броуновское движение не наблюдается для заметных крупиц вещества, например для чаинок в стакане? Автор: Это объясняется двумя причинами. Во-первых, число ударов молекул о поверхность частицы пропорционально площади этой по- верхности; масса же частицы пропорциональна ее объему. Таким обра- зом, с увеличением размера R частицы число ударов молекул о ее поверхность возрастает по закону R2, тогда как масса частицы, которая под действием удара должна сдвигаться с места, увеличивается по за- кону R3. Поэтому молекулам становится все труднее и труднее сдвинуть частицу. Для большей ясности я привожу на рис. 23.3 два графика: у = R2 и у = R3. Легко видеть, что квадратичная зависимость преобладает при малых R, а кубичес- кая — при больших R. Следовательно, при малых R должны преобладать по- верхностные эффекты, тогда как при больших R должны преобладать объем- ные эффекты. Во-вторых, броуновская частица должна быть малой, так как не- обходимо чтобы были нескомпенсирова- ны удары молекул, т. е. число ударов о частицу в единицу времени слева 135
и число ударов в единицу времени справа должны быть существенно различными. Но отношение указанной разницы числа ударов ко всему числу ударов будет тем больше, чем меньше поверхность частицы. Рассмотрим на основе молекулярно-кинетических представлений явле- ния испарения и кипения жидкости. Как Вы объясните явление испа- рения? Читатель А: Это явление состоит в том, что наиболее быстрые молекулы жидкости преодолевают притяжение со стороны других молекул и вылетают из жидкости. Автор: Что способствует усилению испарения? Читатель А: Во-первых, увеличение поверхности жидкости, во-вторых, на- гревание жидкости. Автор: Следует иметь в виду, что процесс испарения не односторонний, а двусторонний процесс: наряду с уходом части молекул из жидкости имеет место также частичное возвращение молекул обратно в жид- кость. Испарение будет происходить тем эффективнее, чем сильнее преобладает процесс ухода молекул из жидкости над обратным про- цессом. Нагревание жидкости и увеличение ее поверхности усиливают процесс ухода молекул из жидкости. Наряду с этим для ускорения испарения можно принять меры по уменьшению процесса возвращения молекул обратно в жидкость. Например, при ветре молекулы, выле- тевшие из жидкости, относятся в сторону и тем самым уменьшается вероятность их обратного попадания в жидкость. Именно поэтому на ветру мокрые вещи высыхают быстрее. Если процессы ухода молекул из жидкости и возвращения их в жид- кость взаимно компенсируются, то наступает состояние динамического равновесия, пар над поверхностью жидкости становится насыщенным. В некоторых случаях выгодно затормозить процесс испарения. Напри- мер, нежелательно, чтобы из хлеба быстро испарялась влага. Для предотвращения преждевременного засыхания хлеба его помещают в закрытый сосуд (ящик, футляр, мешочек), тем самым создается прег- рада для ухода испарившихся молекул и над поверхностью хлеба воз- никает слой насыщенного пара, который препятствует дальнейшему испарению воды из хлеба. А теперь объясните процесс кипения. Читатель А: Процесс кипения — это тот же процесс испарения, но про- текающий более интенсивно. Автор: Ваше определение процесса кипения мне совершенно не нравится. Я должен заметить, что многие экзаменующиеся не понимают сущно- сти этого процесса. При нагревании жидкости понижается раствори- мость содержащихся в ней газов, в результате чего внутри жидкости (на дне и стенках сосуда) возникают пузырьки газа. В эти пузырьки происходит испарение; пузырьки наполняются насыщенным паром, давление которого с увеличением температуры возрастает. При некото- рой температуре давление насыщенного пара внутри пузырьков стано- вится равным давлению на пузырьки извне (это давление равно сумме атмосферного давления и давления, производимого слоем воды). Начиная с этого момента пузырьки устремляются вверх, и жидкость кипит. Как видите, кипение жидкости существенно отличается от испарения. 136
Заметим, что испарение происходит при любой температуре, кипение же — при определенной температуре, называемой точкой кипения. Напомним, что если процесс кипения начался, температура жидкости, несмотря на продолжающееся сообщение теплоты, не повышается, она так и останавливается на точке кипения до тех пор, пока не выкипит вся жидкость. Из приведенных рассуждений видно, что с уменьшением внешнего давления должна понижаться точка кипения жидкости. В связи с этим я предлагаю рассмотреть следующий вопрос. В колбе находится небольшое количество воды при комнатной температуре. Начинаем с помощью насоса откачивать воздух, находившийся в колбе над водой. Что будет происходить с водой? Читатель А: По мере откачивания воздуха уменьшается давление над колбой. С уменьшением давления понижается точка кипения воды. Когда точка кипения понизится до комнатной температуры, вода в колбе закипит. Автор: А может вода не закипеть, а, напротив, замерзнуть? Читатель А: Не знаю. Думаю, что не может. Автор: Все дело в том, как происходит процесс откачивания воздуха из колбы. Если этот процесс происходит достаточно медленно, то вода должна рано или поздно закипеть. Если же процесс откачки про- исходит очень быстро, то, вода, напротив, замерзнет. В результате откачки воздуха, а вместе с ним и паров воды усиливается процесс испарения. Так как при испарении воду покидают молекулы с более высокой энергией, то это приводит к охлаждению воды. В случае мед- ленной откачки охлаждение воды компенсируется притоком теплоты извне, в результате температура воды остается прежней. В случае же весьма быстрой откачки охлаждение воды не успевает скомпенсиро- ваться притоком теплоты извне, в результате температура воды нач- нет понижаться. Как только температура воды начала понижаться, возможность закипеть стала также уменьшаться. Продолжение быстрой откачки воздуха из колбы приведет к понижению температуры воды до точки отвердевания и последующему превращению неиспарив- шегося остатка воды в лед. 24. ЧЕМ ОБЪЯСНЯЕТСЯ ОСОБЕННОСТЬ ТЕПЛОВОГО РАСШИРЕНИЯ ВОДЫ? Автор: В чем состоит особенность теплового расширения воды? Читатель А: При нагревании от 0 до 4°С вода уплотняется. Лишь при дальнейшем повышении температуры (от 4°С) вода начинает расши- ряться. Автор: Чем объясняется эта особенность? Читатель А: Этого я не знаю. Автор: Для объяснения указанной особенности воды надо рассмотреть атомную структуру воды. Молекулы воды взаимодействуют друг с другом направленным образом: каждая молекула может присоединить к себе только четыре соседние молекулы, центры которых в результате 137
о Рис. 24.1 присоединения молекул образуют тет- раэдр (рис. 24.1). В результате образует- ся довольно рыхлая, ажурная структура, свидетельствующая о квазикристаллич- ности воды. Разумеется, говорить о структуре воды, как и всякой другой жид- кости, можно только в рамках ближнего порядка (см. § 23). С увеличением рас- стояния от выделенной молекулы будет наблюдаться постепенное искажение по- рядка, обусловливаемое изгибанием и разрывами межмолекулярных связей. Связи между молекулами по мере повышения температуры нарушаются чаще, возрастает число молекул со свободными связями, заполняющих пустоты тетраэдрической струк- туры; степень квазикристалличности воды уменьшается. Указанная ажурная структура воды как квазикристаллической субстанции хоро- шо объясняет аномалии физических свойств воды и, в частности, осо- бенность теплового расширения воды. С одной стороны, повышение температуры приводит к увеличению средних расстояний между атома- ми внутри молекулы вследствие усиления внутримолекулярных колеба- ний, т. е. молекулы как бы слегка «разбухают». С другой стороны, с повышением температуры происходит ломка ажурной структуры воды, что, естественно, приводит к более плотной упаковке самих моле- кул. Первый эффект (эффект колебаний) должен обусловливать умень- шение плотности воды. Второй эффект (эффект ломки структуры) должен, напротив, приводить к увеличению плотности воды по мере на- гревания. При нагревании до 4°С преобладает эффект структуры, поэтому плотность воды растет. При дальнейшем нагревании начинает преобладать эффект колебаний, поэтому плотность воды уменьшается. 25. ЗНАЕТЕ ЛИ ВЫ, ЧТО ТАКОЕ ИДЕАЛЬНЫЙ ГАЗ? Автор: Знаете ли Вы, что такое идеальный газ? Читатель А: Это газ, для которого справедливы все известные нам газо- вые законы: закон Бойля—Мариотта, закон Гей-Люссака, закон Шар- ля, объединенный газовый закон. Читатель Б: Это такой газ, молекулы которого не взаимодействуют друг с другом. Идеальный газ — это упрощенная модель реального газа. Автор: Верно, что все известные Вам газовые законы применимы именно к идеальным газам. Верно также, что идеальный газ — это упрощенная модель реального газа. Однако не вполне верно замечание об отсут- ствии взаимодействия молекул в идеальном газе. В случае идеального газа пренебрегают не самим взаимодействием молекул, а энергией их взаимодействия, в результате чего внутренняя энергия U газа представляется как сумма кинетических энергий молекул. Пусть, например, имеется W молекул газа, каждая из которых характеризуется 138
средней кинетической энергией <е). В общем случае внутренняя энер- гия газа может быть записана в виде U = N <е> + U', (25.1) где U' — потенциальная энергия взаимодействия молекул. В случае идеального газа вместо (25.1) имеем I/= А (е) (25.2) или с учетом (23.1) для одноатомного газа U = 3/2NkT. 4 (25.3) Читатель Б: Всегда ли можно пренебрегать энергией взаимодействия молекул (/'? Автор: Разумеется не всегда. Чтобы выполнялось условие А <е> » U', (25.4) необходимо потребовать увеличения энергии N (е) и уменьшения энергии U'. Первое достигается при достаточно высоких температу- рах, второе — при достаточно сильном разрежении газа. Иными сло- вами, газ может рассматриваться как идеальный, если он достаточно нагрет и разрежен. При охлаждении и сжатии газа мы рано или поздно приходим к ситуации, когда уже необходимо учитывать энергию взаи- модействий молекул. Такие газы, как азот или кислород, в обычных условиях с хорошей точностью могут рассматриваться как идеальные газы. Читатель А: Что Вы понимаете под «обычными условиями»? Автор: Условия, близкие к нормальным. Напомню Вам, что нормальная температура — это 273 К (0°С), нормальное давление — это 76 мм рт. ст., или 1,02 • 105 Па [1 Па (паскаль) = 1 Н/м2]. А теперь представим себе, что имеется некоторое количество идеаль- ного газа. Какие параметры определяют состояние газа? Читатель А: Давление р, объем V, температура t. Читатель Б: Наверное, удобнее пользоваться температурой в абсолютной шкале (Г). Кроме того, есть еще один параметр — масса m газа. Автор: Вы правы. Добавлю лишь, что вместо массы m можно рассматри- вать число N молекул газа. Далее подумайте над следующим вопросом: можно ли изменять какой- либо из перечисленных четырех параметров (р, V, Т, N), не меняя при этом других параметров? Читатель А: По-моему, нельзя... Читатель Б: Можно изменять два параметра, не меняя остальных пара- метров. Автор: Да, правильно. Можно также изменять три параметра, оставляя неизменным четвертый. Можно изменять все четыре параметра. Однако нельзя изменять один параметр, не меняя при этом других параметров. Это означает, что между четырьмя параметрами р, V, Т, N существует определенная связь. Выражение, описывающее эту связь, называют уравнением состояния идеального газа. Учитывая важность этого урав- нения, приведем упрощенный вывод его. Будем для простоты полагать, что все молекулы газа имеют одинаковую скорость и и движутся в од- 139
Рис. 25.1 ном из шести направлений: вдоль осей х, у, z и противоположно им. Давление р есть отношение нормаль- ной составляющей силы F к площади поверхности, на которую она действу- ет. Согласно закону Ньютона, сила равна изменению количества движе- ния в единицу времени Значит, давле- ние на стенку равно суммарному изме- нению количества движения молекул, которые ударились в единицу времени об единичную поверхность стенки. Предположим, что перпендикулярно стенке летит молекула массой т со скоростью и. В результате упругого удара о стенку молекула изменит направление своего движения на про- тивоположное и улетит от стенки с той же скоростью. Изменение коли- чества движения молекулы равно 2mv. Предположим, что о поверх- ность стенки площадью S в единицу времени ударяется п молекул, причем все молекулы движутся перпендикулярно пенке и имеют скорость и. Тогда р = F/S = 2mvn/S. (25.5) Рассмотрим площадку S, перпендикулярную оси Ох (рис. 25.1). В со- ответствии с упрощающими предположениямй будем полагать, что 1 /б всех молекул движется вдоль оси Ох по направлению к площадке. В единицу времени о площадку ударятся все молекулы, находящиеся в объеме vS прямоугольного параллелепипеда, показанного па рисунке, и 1 N ' ~ Число таких молекул равно -g--у и8. Это и есть величина и, которая фигурировала в выражении (25.5). Таким образом, 2mvn 1 , N P = -s- = ym^- Так как ти2/2 — <ь> = 3/2 kT, то pV = NkT или р = nokT. (25.6) Это и есть искомое уравнение состояния идеального газа. Необходимо отметить, что хотя уравнение (25.6) получено в упрощающих предпо- ложениях, однако окончательный результат оказывается строгим (такой же результат мы могли бы получить, не прибегая к указанным выше упрощениям). Читатель А: Мне не приходилось встречать такое уравнение. Автор: Напротив, оно Вам должно быть известно. Представим: N = NAm/M\ (25.7) здесь Na — постоянная Авогадро, m — масса газа, М -- молекулярная масса, m/М - число молей газа Подставляя (25.7) в (25.6), получаем рУ = Nv 140
Вводя обозначение NAk=R, (25.8) где R называют универсальной (молярной) газовой постоянной, нахо- дим РИ = 21/?Г. (25.9) Читатель Б: Это уравнение нам известно. Его называют уравнением Мен- делеева—Клапейрона. Автор: Вы правы. Это есть одна из форм записи уравнения состояния идеального газа. Читатель А: Оно описывает объединенный газовый закон. Автор: Вот здесь позвольте сделать весьма существенное замечание. Объединенный газовый закон тут ни при чем. Дело в том, что все газо- вые законы описывают какие-либо изменения состояния газа и устанав- ливают связь между параметрами исходного состояний и параметрами конечного состояния. А выражения (25.6) и (25.9) описывают связь между параметрами одного и того же состояния. Читатель А: А как же тогда записывается уравнение объединенного газового закона? Автор: К газовым законам мы скоро перейдем. А пока я хотел бы сделать еще одно замечание о состоянии газа. Предположим, что газ имеет определенную массу. Тогда для задания состояния газа надо фиксиро- вать всего два параметра. Это могут быть: р, V, или V, Т, или р, Т. Таким образом, данному состоянию га »а соответствует конкретная точка в координатных осях р и V. либо V и Т, либо р и Т 26. КАКИЕ ГАЗОВЫЕ ЗАКОНЫ ВЫ ЗНАЕТЕ? Автор: Еще раз подчеркну: любой газовый закон описывает какое-то изменение состояния газа. Иными словами, газовый закон всегда связывает друг с другом параметры начального и конечного состояний газа. Воспользуемся уравнением состояния идеального газа (25.9) и выве- дем выражение для объединенного газового закона. Рассмотрим два различных состояния некоторого i аза- начальное с параметрами ри V,, 7\, nil и конечное с параметрами р>. I/.. Т,. ш > Тогта для начального состояния уравнение имеет вил Для конечного состояния m2 PiV2 R = (26 16) 1 i М m2T, М 1 Так как правые части выражений (26.1а) и (26.16) равны, то должны быть равны друг другу и левые части Pi |/1/(;п1 Т|) — т?Т?). (26.2) 141
Таким образом,'получена в наиболее ‘общем- виде связь между пара- метрами начального и конечного состояний. Это и есть выражение для объединенного газового закона. Его можно, очевидно, записывать и в таком виде: nV const. (26.2а) Читатель Б: Нам давали более простое выражение для объединенного газового закона: Р^ pV * . -у— = -у—, ИЛИ -уг- = const. (26.3) Автор: Выражение (26.3) получается из (26.2), если считать, что в про- цессе перехода от начального состояния к конечному масса газа не менялась (гщ = т2). Ниже мы будем полагать, что это условие вы- полнено и пользоваться именно выражением (26.3). Из выражения (26.3) можно прийти к более частным случаям — так называемым изо- процессам: а) не изменяется температура — изотермический процесс (закон Бой- ля-Мариотта): Pi — Рг^2> или pV — const; (26.4) б) пе изменяется давление — изобарный процесс (закон Гей-Люс- сака): V, V2 V у-=-у-.^ли —= const; (26.5) в) не изменяется объем — изохорный процесс (закон Шарля): ~ = у-, или у- = const. (26.6) Все эти изопроцессы выражаются в виде определенных графиков на плоскости (например, в осях р, У). На рис. 26.1 различные изопро- цессы показаны в различных координатных осях: р, У; V, Т и Г, р. За- мечу, что экзаменующиеся обычно умеют изображать изопроцессы в 142
осях р, V, но затрудняются, когда их просят изобразить эти же процес- сы в других осях. Читатель Л: А я и сейчас затрудняюсь. Мне не совсем понятно, почему, например, изобара в осях V, Т имеет вид прямой, выходящей из начала координат. Автор: Обратитесь к выражению (24.5) для изобарного процесса. Его, очевидно, можно переписать так: У = const • Т. Налицо прямая пропор- циональная зависимость объема V от температуры Т. Читатель А: Теперь мне понятно. Автор: На рис. 26.2 показаны две изотермы для одной и той же массы газа, но для разных температур (7\ и Т2). Какая из изотерм относится к более высокой температуре? Читатель Б: Кажется, я смогу ответить на вопрос. Надо провести на ри- сунке какую-нибудь изобару (см. пунктир на рис. 26.2). При постоян- ном давлении объем газа будет тем больше, чем больше его темпе- ратура. Значит, наружная изотерма (Т2) соответствует более высокой температуре. Автор: Правильно. Запомните: чем ближе изотерма к началу координат в осях р, V, тем более низкой температуре она соответствует. В этом смысле приближение к началу координат неизбежно связано с пониже- нием температуры газа, а удаление от начала координат — с повы- шением температуры газа. Обратимся к рис. 26.3. На нем сплошной линией показана некая изо- терма. Повышается или понижается температура газа в процессе рас- ширения, показанном на рис. 26.3 пунктирной линией? Читатель А: При движении по пунктирной линии мы удаляемся от начала координат. Значит, температура повышается. Автор: Верно. А теперь выберем в координатных осях р, V совершенно произвольно две точки: точку 1 (с параметрами р{, 1\) и точку 2 (с параметрами р2, V2, Т2). Будем переходить из точки 1 в точку 2; при этом всякий раз будем пользоваться какими-либо двумя изопроцесса- ми. Спрашивается, сколько таких различных переходов из 1 в 2 возможно? Читатель А: Наверное, три. Во-первых, изотерма — изобара; во-вто- рых, изотерма — изохора; в третьих, изобара — изохора. 143
Автор: А если, например, изобара — изотерма? Читатель А: А разве это не одно и то же, что и изотерма — изобара? Впро- чем, я никогда над этим не задумывался... Автор: Это разные переходы. Всего возможно шесть переходов; все они показаны на рис. 26.4: а) изотерма — изобара; б) изотерма — изо- хора; в) изобара — изохора; г) изобара — изотерма; д) изохора — изотерма, е) изохора — изобара. Легко видеть, что это весьма разные переходы; им соответствуют разные промежуточные состояния. В связи с рис. 26.4 полезно подчеркнуть два обстоятельства: во-первых из любого начального состояния в любое конечное можно перейти, используя два каких-либо изопроцесса (любые две точки плоскости всегда можно соединить, используя, например, изотерму и изобару); во-вторых, поскольку рассматриваются три изопроцесса, то возможны шесть различных переходов между двумя произвольными состояниями. А теперь давайте обратимся снова к объединенному газовому закону (26.3). Вспомните, как Вас учили выводить этот закон. Читатель А: Нас учили выводить объединенный газовый закон, исходя из законов Бойля—Мариотта и Гей-Люссака. Только я всегда путаюсь в этом выводе. Кроме того, Вы показали нам совсем иной вывод объединенного газового закона — из уравнения состояния газа. Автор: Вот я и хочу внести полную ясность. Запомните: в основе лежит уравнение состояния газа (25.9); оно, как Вы убедились, получается из рассмотрения движения молекул в газе. Затем на основе уравнения состояния (25.9) получают выражение (26.2) для объединенного газо- вого закона в наиболее общем виде (когда меняются все четыре пара- метра: р, V, Т, т). Далее, исходя из (26.2), приходят к уже более частным случаям — выражению (26.3) и, наконец, выражениям (26.4) — (26.6), описывающим изопроцессы. Здесь налицо переход от более общих выражений к более частным — это так называемый дедуктивный путь. Возможен, однако, и обратный путь — от более част- ных выражений к более общим (так называемый индуктивный путь). В этом случае полагают, что более частные выражения заведомо известны, например получены непосредственно из опыта; исходя из этих частных выражений, делается попытка путем обобщений перейти Рис 26 5 144
к более общим выражениям. Дедуктивный путь более последователен и логичен; индуктивный путь может оказаться подчас более простым. Следуя по индуктивному пути, будем исходить из законов Бойля— Мариотта и Гей-Люссака. В основе вывода выражения для объеди- ненного газового закона лежит уже знакомое нам утверждение: из любого начального состояния (рн Ир в любое конечное (р2, И2, Г2) можно перейти по изотерме и изобаре. Изобразим этот переход в ко- ординатных осях р, V (рис. 26.5). Параметры промежуточного сос- тояния: р2, У3, Гр Используя закон Бойля—Мариотта, свяжем друг с другом параметры начального и промежуточного состояний: Р1^1=Р2^3- (26.7) Используя закон Гей-Люссака, свяжем параметры промежуточного и конечного состояний: К К -I = -1. (26.8) / । 1 2 Обратите внимание: в выражениях (26.7) и (26.8) есть один общий параметр — промежуточный объем И3. Исключая его, приходим к иско- мому результату: Pi^i/^i = Рг^г/^2- Читатель Б: А если использовать сначала изобару, а затем уже изотерму? Автор: Тогда получится картина, изображенная на рис. 26.6: начальное состояние — plt Ир 7\, конечное — р2, V2, Т2, промежуточное (здесь всегда надо быть внимательным) — рн И4, Т2. Читатель А: Мы имеем здесь иное, чем ранее, промежуточное состояние. Автор: Да, начальное и промежуточное состояния связаны законом Гей- Люссака: Г1/Т1 = Г4/Т2. Промежуточное и конечное состояния связаны законом Бойля—Мариотта: Р1И4 = р2И2. Исключая общий параметр (объем И4), приходим, как легко убедиться, к искомому результату — выражению (26.3). Читатель Б: Очевидно, можно было бы взять изотерму и изохору или же изобару и изохору? Автор: Конечно. Всякий раз будет своя картина перехода в осях р, V, 145
свое промежуточное состояние. Но окончательный результат будет одним и тем же. Читатель А: Значит, если исходить из изопроцессов, существует шесть выводов объединенного газового закона? Автор: Совершенно правильно. Это заключение находится в полном со- ответствии с рис. 26.4. Читатель А: Вот теперь, с этими картинками в осях р, V я понял, как вы- водится объединенный газовый закон. Автор: Далее рассмотрим некий циклический (т. е. замкнутый) процесс, образуемый тремя разными изопроцессами. Пусть в осях р, V этот процесс имеет вид, показанный на рис. 26.7. Здесь: 1-^2 — изотерми- ческое расширение, 2 -> 3 — изохорное нагревание, 3-^1 — изобарное сжатие... Читатель Б (перебивая): Наверное, в случае 3 -> 1 можно также сказать «изобарное охлаждение»? Автор: Вы правы. Ведь при движении от точки 3 к точке 1 температура газа падает. Итак, вопрос: как будет выглядеть указанный цикли- ческий процесс в координатных осях V, Г? Читатель Б: Он будет выглядеть так, как показано на рис. 26.8. Автор: Все правильно. Вы действительно неплохо усвоили графическое изображение изопроцессов. Поэтому пойдем далее и рассмотрим некий конкретный процесс, в котором изменяются уже не дьа параметра (как это имело место в изопроцессах), а три. Пусть газ расширяется таким образом, что его давление и объем подчиняются условию pV2 — const. Известно, что объем газа увеличился в два раза. Как изменилась температура газа? С чего мы обычно всегда начинаем, рассматривая какой-либо про- цесс в газе? Читатель Б: Надо, по-видимому, обозначить параметры начального и конечного состояний. Пусть это будут соответственно V), и р2, У2, Т2. Автор: Какая связь существует между этими параметрами в общем слу- чае? Читатель Б: Понял. Эта связь нам хорошо известна: Pi У\/?\ — Рг^г/^2- Автор: Если бы у нас был изотермический процесс, то какое дополнитель- ное условие мы имели бы? Читатель Б: Условие 1\ = Т2. Автор: Следовательно, у нас была бы система уравнений: ( Pi^i/T'i = РтУч/Т'ъ ( Ti = Т2 Отсюда мы немедленно пришли бы к известному выражению для закона Бойля—Мариотта: р{ V! ,= р2У2. Однако в нашем случае процесс не изотермический: дополнительное условие у нас иное. Какое оно? Читатель Б: Это”условие дано в формулировке вопроса: р{У2 = р2У2- 146
Теперь мы имеем систему уравнений: Г P\V\/T\ = Рг^г/^г» t Pi = Рг^г- Разделив почленно эти уравнения друг на друга, найдем = v2T2. Следовательно, температура i аза должна уменьшаться во столько же раз, во сколько раз увеличился его объем. Автор: В заключение обсудим, как вычислять универсальную газовую постоянную R, входящую в (25.8). Читатель А: Надо воспользоваться выражением (25.8). Постоянная Аво- гадро МА=6' 1023 моль- ’, постоянная Больцмана k= 1,38-10-23 Дж/К. Следовательно, R = NAk = 8,3 Дж/ (моль • К). Читатель Б: Для определения R можно также воспользоваться выра- жением для объединенного газового закона: pV/T=p„V„/T,„ (26.9) где параметры р0, Уо, То относятся' к данной массе газа, взятой при нормальных условиях :р0 « 105 Па, Т0=273 К, V0=(m/M)-22,4-10-3м3 (хорошо известно, что 1 моль любого газа при нормальных условиях занимает определенный объем, равный 22,4 л; отношение m/М есть, очевидно, число молей, содержащихся в данной массе газа). Подстав- ляя все эти числа в (26.9) и сравнивая полученный результат с (25.9), находим R = 8,3 Дж/К. Автор: Оба ответа верны численно. Только в последнем вычислении надо помнить, что объем Ко относится к молю. Тогда получится правильная размерность универсальной газовой постоянной. 147
Рис. 26.12 Рис. 26.13 ЗАДАЧИ 26.1. При нагревании газа была получена зависимость давления от температуры (рис. 26.9). Определить, что производилось во время нагревания газа: сжатие или расширение. 26.2. При нагревании газа при постоянном давлении вследствие неисправности прибора график зависимости объема от температуры имеет вид, показанный на рис. 26.10. Определить, как изменилась масса газа, если V2= 1,5V|( Г2= ЗГГ 26.3. На рис. 26.11 дан график изменения состояния идеального газа в координа- тах р, V. Представить этот процесс графически в координатах р, Т и V, Г. 26.4. На рис. 26.12 дан график изменения состояния идеального газа в коорди- натах V, Т. Представить этот процесс графически в координатах V, р и р, Т. 26.5. На рис. 26.13 дан график изменения состояния идеального газа в коор- динатах р, Т Представить этот процесс графически в координатах V, Т и р, V. 27. КАК ВЫ РЕШАЕТЕ ЗАДАЧИ НА ГАЗОВЫЕ ЗАКОНЫ? Читатель А: Мне бы хотелось рассмотреть применение газовых законов к решению различных задач. Автор: Почти все предлагаемые экзаменующимся задачи на газовые законы, на мой взгляд, довольно просты. Большинство из них может быть отнесено к одной из следующих двух групп. Первая группа: задачи строятся на основе изменения состояния некоторой массы газа, причем значение этой массы не используется. В результате расширения, нагревания и других процессов газ перехо- дит из некоторого состояния Р], 1\ в состояние р2, У2, Т2. Параметры исходного и конечного состояний связаны друг с другом уравнением объединенного газового закона (26.3): Р\ ^i/^i = Рг^г/^г- В задаче требуется найти один из этих шести параметров. Вторая группа: состояние газа не изменяется, но зато в задаче участвует масса газа. Требуется найти либо эту массу по известным параметрам состояния газа, либо один из параметров состояния по известной массе газа и остальным параметрам. В подобных задачах должна быть известна молярная масса рассматриваемого газа. Читатель Б: При решении задач второй группы удобнее всего восполь- зоваться уравнением (25.9). 148
Актор: Вы правы. Решите, пожалуйста, следующую задачу: найти объем 58 г воздуха при давлении 8 ат и температуре 91 °C. Читатель Б: Из уравнения (25.9) получим V = mRT/(Mp), только необходимо все данные задачи привести к единицам в СИ: tn = 5,8 • 1(Г 2 кг; р = 8- 105 Па; Т = 9ГС + 273°С = 364 К, молярная масса воздуха М=0,029 кг/моль, 7?=8.3 Дж/(моль*К). Тогда получим |/= 7.5 • 10“ *м3 = 7,5л. Читатель А: При переводе давления Вы, вероятно, считали, что 1 атм — = 105 Па, однако в условии задачи имелись в виду технические атмо- сферы. Используя технические атмосферы, следовало бы положить 1 ат = 9,8 • 104 Па. Читатель Б: Вы правы.. Между физической атмосферой (соответствующей нормальному давлению) и технической атмосферой существует разли- чие. В данном случае я пренебрег этим различием. Читатель А: Не могли бы Вы указать характерные трудности, с которыми приходится сталкиваться при решении задач первой или второй группы? Автор: Я уже говорил, что эти задачи, на мой взгляд, довольно просты. Читатель 4: Но в чем обычно ошибаются экзаменующиеся? Автор: Если не учитывать обычной невнимательности, то основной причиной ошибок является, пожалуй, неумение вычислить давление газа в том или ином конкретном состоянии. Рассмотрим пример; в трубке/запаянной с одного конца, имеется столбик ртути, отделяю- щий от окружающей среды некоторый объем воздуха внутри трубки. Трубка может поворачиваться в вертикальной плоскости. В первом положении (рис. 27.1, а) столбик воздуха внутри трубки имеет длину Ц, а во втором положении (рис. 27.1, б) — длину 12. Найти длину столби- ка воздуха в положении, когда трубка наклонена под углом а к вертика- ли (рис. 27.1, в). Обозначим р0 — атмосферное давление в единицах длины ртутного столба, а А/ — длину ртутного столбика в трубке. В первом положении давление воздуха внутри трубки равно, очевидно, атмосферному давлению ра. Во втором положении давление воздуха в трубке равно разности(ра — А/), так как атмосферное давление в этом случае уравновешивается суммарным давлением ртутного столбика и воздуха внутри трубки. Используя закон Бойля—Мариотта, запишем '1РЯ = А>(Ра ~ */). Отсюда находим, что атмосферное давление равно Ра = А/./2/(/2-/(). (27.1) В третьем положении часть веса ртутного столбика уравновешена силой реакции стенок трубки. В результате давление воздуха внутри трубки оказывается равным (ра — А/ cos а). Используя закон Бойля— Мариотта для первого и третьего состояний газа, запишем /|Р.> = /з(Ра — V cos «). 149
Рис. 27 I Отсюда находим искомую длину: /3 = Отсюда находим, что атмос- ферное давление равно pa=A/-/3cosa/(/3—/,) (27.2) Приравнивая правые части выражений (27.1) и (27.2), получаем /2 /л cos a м2 4 -(4 - 0 cosa‘ Легко видеть, что если cos a= 1, то 1А = /2 (второе положение трубки); если cos a = 0, то 1А — Ц (первое положение трубки). Читатель А: В решении этой задачи мне все ясно, кроме одного, почему Вы при определении давления столбика ртути не использовали резуль- тат/полученный в § 20, р = pgA/, а записывали р = А/? Автор: Конечно, правильнее было бы воспользоваться результатом р = = pg&l, но тогда необходимо перевести атмосферное давление, выра- женное в единицах ртутного столба, в паскали, я же подчеркнул, что ра — атмосферное давление в единицах ртутного столба, а А/ — длина тоже ртутного столбика. Читатель А: Сами по себе первая и вторая группы задач, указанные Вами, мне понятны. Но на экзамене, наверное, будут предлагать зада- чи, являющиеся комбинацией задач первой и второй групп? Автор: Это не исключено. Рассмотрим следующую задачу: 16 г кислорода при давлении 2 ат занимают объем 5 л. Как изменилась температура газа, если при увеличении давления до 5 ат его объем уменьшился на 1 л? Читатель А: Зная массу кислорода, его давление и объем, я сразу найду его температуру 16 г кислорода — это 0,5 моля, т. е. объем 11,2 л при нормальных условиях. Далее определю, что 7' = 7« (пГ = 273 ' тЦгТ К = 244 К- <273> Г0 4 0 1 1 ’ Автор: Правильно. На данном этапе задача решалась как типичная зада- ча второй группы. Читатель А: Зная температуру Г, исходного состояния газа, я получу температуру Г2 конечного состояния: PoV2 5 • 4 72 = 71г3= 244488 к- Сравнивая этот результат с (27.3), находим, что температура газа по- высилась на 244 К- Автор: Все совершенно верно. Как видите, на втором этапе данная задача рассматривалась как типичная задача первой группы. Читатель Б: Я бы сразу решал эту задачу как задачу первой группы. Из (26.3) получу Tt/T2 = р, V, Др2К,) = 2 • 5/(5 4) = '/2; 150
температура увеличилась в два раза. Если бы в задаче спрашивалось, не как изменится температура газа, а на сколько, то я бы решал ее как задачу второй группы. Запишу уравнения Клапейрона—Менделе- ева (25.9) для начального и конечного состояния: р+ =-^-RTrP2V2 = -TTRT> 11 М 1 2 2 м и получу разность температур: Автор: Решение правильно. Вы справедливо заметили некоторую некор- ректность в постановке вопроса. Читатель Б: А существуют ли задачи, принципиально отличные от задач первой и второй групп? Автор: Да, существуют. До сих пор мы полагали, что масса газа оста- ется неизменной. Однако возможны задачи, в которых масса газа изме- няется (газ откачивают или, напротив, накачивают). Будем условно относить подобные задачи к третьей группе. Они не имеют готовых рецептов решения, а требуют сугубо индивидуального подхода. Однако в каждом конкретном случае задачу третьей группы можно свести к задачам первых двух групп. Вся трудность состоит в том, как это сделать. В этой связи я предлагаю рассмотреть две задачи. Первая: газ в сосуде находился под давлением 20 ат при темпера- туре 27°С. Найти давление газа в сосуде после того, как половина массы газа выпущена из сосуда, а температура повышена на 50 К. Эта задача похожа на задачу первой группы, поскольку в ней про- исходит изменение состояния газа. Однако с изменением состояния газа здесь изменяется и масса газа. Чтобы воспользоваться уравне- нием объединенного газового закона, надо проследить за изменением состояния одной и той же массы газа. Выберем в качестве этой массы ту массу газа, которая в конечном счете остается в сосуде. Обозначим ее конечные параметры р2, ^2» Т’2; 7"2=(273+27+50) К=350 К; К2 = К, где V — объем сосуда; р2 — искомая величина. Как выразить началь- ные параметры указанной массы газа? Читатель А: Эта масса будет характеризоваться такой же температурой, что и весь газ: = (273 + 27) К = 300 К; ее объем V[ равен половине объема сосуда V/2; ее давление такое же, как и давление всего газа: р, = 20 ат. Читатель Б: Я бы рассмотрел начальные параметры указанной массы газа немного иначе: Т\ = 300 К; объем такой же, что и всего газа (= V), но зато давление равно половине давления всего газа: pi = 10 ат. Автор: Поскольку давление и объем входят в виде произведения, Ваши предложения, хотя они и различны, приводят к одному и тому же ре- зультату. Поэтому можно было бы и не останавливаться на этих раз- личиях, если бы их обсуждение не представляло определенного физи- ческого интереса. Будем называть условно молекулы той части газа, которая в конечном счете останется в сосуде, «красными» молекулами, а молекулы той части газа, которая будет выпущена из сосуда — «чер 151
a) Рис. б) 27 2 ными» молекулами. Итак, красные молекулы останутся в сосуде, а черные уйдут из него. Исходное состояние газа допускает двоякую трактовку. 1) красные и черные молекулы разделены так, что можно выделить в сосуде макроскопические объемы, занятые только красными или только черными молекулами (рис. 27.2, а); 2) красные и черные молекулы взаимно перемешаны, так что в любом макроскопическом объеме внутри сосуда имеется практически одинаковое число молекул того и другого сорта (рис. 27.2, б). В первом случае каждый сорт молекул образует собственное газообразное «тело» с объемом V/2, которое давит на стенки и на воображаемую границу с другим «телом» с давлением 20 ат. Во втором случае молекулы каждого сорта оказы- ваются распределенными по всему объему V сосуда, при этом молекулы каждого сорта вносят половинный вклад в давление на стенки (в любом месте стенки половина ударов приходится на долю белых молекул, а половина — на долю черных). В этом случае = V, рх = 10 ат. В связи с последним замечанием напомним закон парциальных давлений (за- кон Дальтона): давление смеси газов равно сумме давлений отдельных компонентов смеси. Подчеркнем, что здесь речь идет именно о смеси га- зов: молекулы всех сортов перемешаны между собой. Читатель Б: Второй подход более правилен, ведь молекулы обоих сортов действительно перемешаны. Автор: В нашей задаче оба подхода совершенно равноправны. Не забы- вайте, что априорное разделение молекул на два сорта являлось условным. Возвращаясь к решению задачи, запишем уравнение объединенного газового закона для' массы газа, которая остается в сосуде: 10 V/300 = р2 V/350. Отсюда находим, что р2 = 11,7 ат « 11,9 • 105 Па. Читатель Б: Мне все же больше нравится решать такие задачи, исполь- зуя уравнение Клапейрона—Менделеева (25.9). Запишем уравнение для начального и конечного состояний газа, обозначив массу в началь- ном состоянии т, а в конечном ги/2: ,, т ,, т \ 2лГ Разделив одно уравнение на другое, получим Р\ 2Т । р {Т 2 "•’ИР1=^7 20 • 350 2 • 300 ат = 11,7 ат. 152
Автор: Ваш подход к этой задаче со- вершенно верен. Вторая задача: газ находит- ся в сосуде объемом V при давлении р0. Его откачивают из сосуда с по- мощью поршневого насоса с рабо- чей камерой объемом и (рис. 27.3). рис 273 Требуется найти число п ходов поршня, которое надо сделать, * чтобы давление газа в сосуде понизилось до значения рп. Читатель А: По-моему, эта задача очень несложная: п ходов поршня при- водят к n-кратному увеличению объема газа на значение и. Поэтому запишем уравнение Бойля—Мариотта в виде РоУ =Рп(У-У nv). Отсюда найдем число п ходов. Автор: К какой массе газа относится написанное Вами уравнение? Читатель А: К массе, которая была в сосуде. Автор: Но ведь уже после первого хода часть э,той массы уходит из системы: когда поршень начинает движение справа налево, он закры- вает клапан А и открывает клапан Б, через который газ и покидает систему (см. рис. 27.3). Иначе говоря, указанное Вами n-кратное увели- чение объема газа на v не относится к одной и той же массе. Поэтому написанное Вами соотношение является неправильным. Будем рассматривать каждый из ходов поршня по отдельности. Начнем с первого хода. Для массы газа, находившейся в сосуде первоначально, можно записать p0V = р, (V + у). где pi — давление газа, после того как поршень, заканчивая рабо- чий ход, занял крайнее правое положение. Затем поршень возвраща- ется в исходное левое положение. При этом, как уже указывалось, кла- пан А закрывается и в сосуде остается масса газа, меньшая по срав- нению с исходной массой; давление газа в сосуде есть рх. Для этой массы газа можно написать уравнение = р2 (у + у), где р2 — давление газа после окончания второго рабочего хода порш- ня. Рассматривая последовательно третий, четвертый и последующие ходы поршня, получаем систему п уравнений закона Бойля—Мариотта; ' p0V= pi (V + и), < PiV=p2(V + n), (27.4) < Р2У = Рз(У + у). pn_!V=p„(V + n). Каждое из этих уравнений относится к определенной массе га ia Ре шая систему (27.4), получаем
Рп = Pol V/(V + У)]"- Логарифмируя этот результат, находим ‘ё(.Р,,/Ро) П lg I УДУ + е)| В заключение рассмотрим еще одну задачу: три одинаковых шара А. Б, В (рис. 27.4) соединены трубками одинаковой длины и одинаковой площадью сечения S. Внутри трубок находятся капельки ртути, кото- рые в исходном положении (при температуре 7\) расположены посере- дине трубок. Объем воздуха в каждом шаре и части трубки до капельки ртути равен Уг Как передвинутся капельки, если шар Б нагреть на АТ, а шар В — на 2АТ? Примечание: считать, что АТ <С Т,; ввиду относительной малости изме- нений температуры и объема газа слагаемые типа ATAV во всех встре- чающихся при решении задачи соотношениях отбрасывать как вели- чины второго порядка малости. Читатель А: Я затрудняюсь решить эту задачу. Она представляется мне весьма сложной, комбинированной. Автор: Здесь масса газа не меняется, но зато в задаче участвует сразу не- сколько масс, и необходимо следить, к какой именно массе относится всякий раз то или иное соотношение. Кроме того, надо четко фикси- ровать в своем сознании начальное и конечное состояния каждой из трех масс газа (обозначим их пгА, тБ, тв). Мы полагаем, что объем массы тА в результате нагревания шара Б уменьшается на неко- торую величину АУ1( а объем массы тв в результате нагревания шара В увеличивается на АУ2; тогда объем массы тБ возрастает на А— АУ2. С учетом условия задачи и сделанных замечаний за- ключаем: 1) для газа пгА (р,, Ун Т, — начальные и р2, У( — AV,, ные параметры) р,у, р2(У, - АИ,) Г, Г, Т\ — конеч- (27.5) 2) для газа тБ (р{, Уи Тх — начальные и р2, АТ — конечные параметры) Р,У, р2(У, +АУ, -АУ) Г, Тх+ АТ V. + АУ, -АУ2, Т. - (27.6) 3) для газа mB (ph У,, Т{ — начальные и р2, + АУ2, Ti + 2АТ — конечные параметры) Тх Г, + 2АГ (27.7) Используя соотношения (27.5) и (27.6), запишем (V. - АУ,) (Г, + АТ) = ( У, + АУ, - АУ2) Т. Используя соотношения (27.6) и (27.7), запишем 154
(У, +Д1/, -ДУ2)(Т, +2A7)=(V,+AV2)(T1 +дт). Раскрывая скобки и отбрасывая слагаемые типа ДУДТ, находим: ( 2\ViTi = V^TbV2T\, ( 2Д1/2Т1 = V, AT + AV, Г,. Отсюда получаем AV, = ДУ2 = V.AT/T,; А/ = У.АТДЗТ,). Итак, каждая капелька ртути передвинется влево на одну и ту же величину А/, указанную выше. Объем газа в шаре Б не изменится. Читатель А: Действительно, здесь все сделано четко. Автор: Помните: для определенной массы газа необходимо рассматривать именно ее начальные и конечные параметры. Все весьма просто. ЗАДАЧИ 27.1 . Трубка с запаянным верхним концом полностью погружена в сосуд с ртутью (рис. 27.5). При этом столбик воздуха внутри трубки имеет длину Л = 10 см. На какую высоту Л2 над уровнем ртути в сосуде надо приподнять верхний конец трубки, чтобы уровень ртути внутри трубки сравнялся с уровнем ртути в сосуде? Атмосферное давление нормальное(р0 = 76 см рт. ст.). Вычислить массу m воздуха внутри трубки, если площадь ее поперечного сечения S — 1 см2, а температура t = 27°С. 27.2 . Трубка, один конец которой запаян, опущена открытым концерн в сосуд с ртутью (рис. 27.6). Как изменится уровень ртути в трубке, если температура повысится от t — 20°С до t2 = 77°С? Дано: 1Х — 10 см, t2 = 30 см; атмосферное давление нормальное. Тепловым расширением трубки пренебречь. Найти массу m воздуха внутри трубки, если площадь ее поперечного сечения S = 0,5 см2. 27.3 . Воздух в сосуде объемом V = 5 л находится при температуре t = 27°С под давлением рх = 20 ат. Какую массу Ат воздуха надо выпустить из сосуда, чтобы давление в нем упало до р2 = 10 ат? 27.4 . В трубке с площадью сечения S = 0,5 см2, ориентированной вертикально запаянным концом вверх, находится столбик воздуха длиной ±= 40 см, запертый снизу столбиком ртути длиной /i = 8 см; температура tx = 27°С. На сколько изме- нится длина А/’ воздушного столбика, если трубку отклонить от вертикали на угол а = 60° и одновременно повысить температуру на АГ = 30 К? Атмосферное 155
Рис. 27.8 давление нормальное. Найти м<нт у ш воздуха юк н<> ценного внутри трубки. 27.5. Два одинаковых шара А н Ь (рис. 27 7) сие ш йены тонкой трубкой. При температуре Г, капелька ртути находится посередине трубки. Объем во)д>ха в каждом шаре и части трубки до капельки pi ути равен Vr На какую длину А/ передвинется капелька, если шар А нагреть на а шар Б охладить на <С <С Г,, АГ2 <§; Г,)? Площадь поперечного сечения трубки равна S. Рассмотреть три случая: 1) АГ, > АГ2; 2.) АГ, < АГ2; 3) АГ, = АГ2. 27.6 . Условие задачи совпадает с условием задачи о трех шарах, рассмотренной в тексте (рис. 27.4). На какую длину А/ передвинется капелька, если шар Б на- гревают на АГ, а шары А и В охлаждают на АГ. 27.7 . Условие задачи совпадает с условием задачи о трех шарах, рассмотренной в тексте (рис. 27.4). На какое расстояние А/ передвинется капелька, если шар А нагревают на АГ, а шар В охлаждают на АГ. 27.8 . В двух частях цилиндра, разделенного поршнем А, находятся разные массы пг, и /п2 воздуха при одной и той же температуре (рис. 27.8). Правый конец ци- линдра закрыт подвижным поршнем В. На сколько сместится поршень А, если поршень В передвинуть вправо на расстояние 1=4 см? При равновесии в цилиндре устанавливается первоначальная температура. Отношение масс т{/т^ = 3. 27.9 . Шар-зонд имеет герметическую оболочку постоянного объема V = 50 м!. Масса шара т = 5 кг. Определить, на какую максимальную высоту Н сможет подняться этот шар-зонд, если известно, что атмосферное давление уменьшается в два раза через каждые h = 5 км высоты. Температура в стратосфере t = — 60°С. Давление у поверхности Земли р0 = 1(У’ Па. 27.10 . Из баллона объемом V = 10 л, заполненного водородом, вследствие неис- правности вентиля вытекает газ. При температуре /, = 6°С манометр показывал давление р, = 56 атм, а через некоторое время при температуре t, = 27°С давление р2 = 30 атм На сколько уменьшилась масса водорода в баллоне? 28. КОГДА ВЫПАДАЕТ РОСА? Читатель А: Мне бы хотелось коснуться задач на влажность. Они какие к» непонятные. Автор: В действительности здесь нет ничего трудного. Рассмотрим в ка честве примера следующую задачу. Воздух находится в замкнутом объеме V = 3 м3 при температуре t{ = 20°С. Относительная влажность = 60%. Сколько воды надо дополнительно испарить в этот объем, чтобы относительная влажность стала — 80%. Выпадет ли рос а, если воздух в рассматриваемом объеме охладить до /2 = Ю°С? Читатель В: Насколько я помню, относительная влажность определяеюя как отношение давления р0 паров воды, находящихся в воздухе при данной температуре, к давлению насыщенных паров воды: f = A..ioo%. (28Ц Р„ Читатель Д: Но как связать давление и массу водяных паров? Автор: Полагая, что пар подчиняется уравнению состояния идеи h.iioio газа, можем считать, что давление пара пропорционально массе пара в данном объеме. Из (25.9) следует: р = m/V = pM/(RT). (28.2) 156
Поэтому в (28.1) заменим отношение давлений р0/рн отношением плот- ностей пара р0/рн: Ро / = —.100%. (28.3) Р н Таким образом, относительную влажность можно определить как отно- шение плотности р0 паров, находящихся в воздухе при данной темпе- ратуре (эту плотность называют абсолютной влажностью), к плотности рн насыщенных паров при данной температуре. Читатель Б: В таком случае я попробую решить сформулированную выше задачу. Обозначим р, — плотность пара при после испарения m г воды плотность пара станет pj m/V. Используя (28.3), получаем Pi + 'n/IZ f I Р|/Рн‘> /2= n г II f2 Pi + m/V Разделив второе уравнение на первое, находим -т— ---------, откуда п Pi m = ₽, (h - А) V/f, = р„ (/2 - /,) V. (28.4) Читатель А: В этом соотношении есть неизвестная величина рн. Читатель Б: Я об этом как-то не подумал. Автор: Значение плотности насыщенных паров обычно находится из таб- лиц; если у Вас имеются таблицы для давления (упругости) насы- щенных паров, то можно воспользоваться соотношением (28.2) и найти плотность. Плотность насыщенных паров при = 20°С равна рн == = 17,3 г/м3. Подставляя это значение в (28.4) и используя данные из условия задачи, получаем m — 10,38 г. Читатель Б: Для ответа на вопрос о выпадении росы необходимо, по-види- мому, знать плотность насыщенных паров при t2 = 10°С. Автор: Вы правы. Эта плотность равна 9,4 г/м3. Читатель Б: В нашем объеме плотность паров р = pHf2 = 13,84 г/м3. Эта плотность больше, чем плотность насыщенных паров при температуре 10°С, следовательно, роса должна выпасть. Читатель А: А как определить, при какой температуре начинает выпадать роса? Читатель Б: Мне кажется, надо воспользоваться таблицей и найти, при какой температуре плотность насыщенных паров равна плотности парОв, зафиксированных в нашей задаче. Эта температура и будет точкой росы. Читатель А: Я понял. Точка росы — это температура, при которой плот- ность насыщенных паров равна абсолютной влажности при данных условиях. При достижении этой температуры находящиеся в воздухе водяные пары начинают конденсироваться, другими словами, начинает выпадать роса. Автор: Ну вот, мы и разобрались во всех «страшных» вопросах, связанных с влажностью. ЗАДАЧИ 28.1. Какова масса ш водяных паров в комнате объемом V = 105 м3, если при температуре /, = 15°С относительная влажность равна / = 55%? Выпадет ли роса, если температура воздуха понизится до /2== 10°С? Какую часть k составляет 157
vi.icca водяных паров от всей массы воздуха в комнате, если давление воздуха 1> — 75 см рт ci Плотность насыщенных паров при 15°С р = 12,8 г/м3, плотность насыщенных паров при 10°С рн2 = 9,4 г/м3 28.2. Сухой и влажный воздух при одинаковой температуре = 27°С и при оди- наковом давлении занимают одинаковый объем V — 1 м3 Определить, на сколько отличается масса сухого воздуха от массы влажного, относительная влажность которого f = 60% Давление насыщенного пара при 27°С равно рн = 26,74 мм рт ст 28.3. При какой максимальной относительной влажности f воздуха в комнате бутылка молока, взятая из холодильника, не будет запотевать? Температура в холодильнике t = 5°С, а в комнате — /2 — 25°С Давление насыщенных паров при 5°С рн| — 886 Па, а при 25°С рн2 — 3192 Па. 29. ЧТО ТАКОЕ ТЕРМОДИНАМИКА? Читатель Б: Мне приходилось встречать термин «термодинамика». Что это такое? Автор: Термодинамика — это раздел физики, в котором с наиболее общих позиций (без обращения к молекулярным представлениям) рассматри- ваются процессы обмена энергией между изучаемым объектом и окру- жающей его средой. Читатель А: В какой мере надо быть знакомым с термодинамикой при изучении газовых законов? Автор: Это станет ясным из дальнейшей нашей беседы. Мы в самых общих чертах познакомимся с элементами термодинамики идеального газа. Отмечу сразу же, что более или менее глубокое усвоение газовых законов требует обращения к термодинамике. Так, например, термоди- намика показывает, чем с энергетической точки зрения отличаются друг от друга изотермическое и изобарное расширения. Начнем с того, что напомним уже известное обстоятельство: внутрен- няя энергия U. одноатомного идеального газа пропорциональна массе m газа в температуре Т: U = ^RT (291> [Этот результат следует из (25.3) и (25.8).] При неизменной массе газа внутренняя энергия может изменяться в результате двух процессов (как говорят, существуют два способа энергообмена газа с окружающей средой, два «канала» энергообмена): а) вследствие совершения работы А; в этом случае объем газа должен изменяться (при расширении газ совершает работу над окружающими телами, например отодвигая поршень; при сжатии окружающие тела совершают работу над газом); б) вследствие передачи теплоты Q (от 1аза к окружающим телам или, наоборот, от окружающих тел к газу). Таким образом, приращение Д(/ внутренней энергии газа можно пред- ставить в виде \U = U2 - >±А ± Q. (29.2) }десь знаки «4-» относятся к случаям, когда газ приобретает энергию (по соответствующему «каналу» энергия поступает от окружающих тел к газу): знаки «—» относятся к случаям, когда энергия поступает от газа к окружающим телам. 158
В природе непрерывно и многообразно совершается обмен энергией между отдельными телами и их совокупностями. Общие задачи этого обмена исследует термодинамика. Читатель Б: Не является ли выражение (29.2) так называемым первым началом термодинамики? Автор: Да, это первое начало термодинамики. Отмечу, что оно фактиче- ски есть не что иное, как закон сохранения энергий: изменение энергии газа обусловлено работой и теплопередачей. Читатель Б: Таким образом, если газ не расширяется и не сжимается (т. е. рассматривается изохорный процесс), то энергообмен с окружаю- щими телами возможен только через теплопередачу? Автор: Правильно. Читатель А: А можно ли, зная начальное и конечное состояния газа, вычислить работу и рассмотреть теплопередачу? Автор: Можно. Рассмотрим сначала изобарное расширение. В этом случае работа, совершаемая газом над окружающими телами, определяется особенно просто. Пусть газ, находящийся в сосуде с подвижным порш- нем, расширяется при давлении р = const от объема V, до объема V2- При этом он отодвигает поршень с площадью S сечения на расстояние А/ (рис. 29.1). Работа, совершенная газом при отодвигании поршня, равна А = FM = pS\l = p\V= p(V2- V,). (29.3) При неизобарном расширении работа газа вычисляется более сложным путем, так как при этом давление в течение процесса расширения газа не остается постоянным: следовательно, сила, действующая на отодвигаемый поршень, изменяется во времени. Существует следующее правило для вычисления работы при произвольном процессе расшире- 159
ния: надо изобразить рассматриваемый процесс в осях р, Е; искомая работа равна площади под кривой на участке от Vi до V2 (рис. 29.2). Читатель Б: С вычислением работы все понятие. А как находить коли- чество переданной теплоты? Автор: Пользуясь выражением (29.1), можно найти изменение внутренней энергии газа в рассматриваемом процессе. Затем следует вычислить работу и, наконец, воспользоваться выражением (29.2). Читатель А: В изохорных процессах передача энергии через работу отсут- ствует. Наверное, возможны также ситуации, когда отсутствует тепло- передача? Автор: Конечно. В этих случаях говорят, что газ является теплоизолиро- ванным. Кстати, как Вы думаете, возможно ли в теплоизолированном газе изотермическое расширение? Читатель А: Почему бы и нет? Автор: Не торопитесь с ответом. Помните: в данном процессе теплопере- дачи нет, газ расширяется (следовательно, совершает работу), темпе- ратура газа не меняется. Читатель Б: Такой процесс невозможен. Если температура постоянна, то постоянна и внутренняя энергия газа. Значит, для совершения работы газ должен обязательно получать теплоту из окружающей среды. Автор: Правильно. Итак, мы убеждаемся в двух фактах: во-первых, изо- термическое расширение в теплоизолированном газе невозможно; во- вторых, при изотермическом расширении количество полученной извне теплоты равно совершенной газом работе. Читатель Б: Наверное, в теплоизолированном газе невозможны не только изотермические, но и изобарные и изохорные процессы? Автор: Ваша догадка верна. Процессы, происходящие в теплоизолирован- ном газе, специфичны. Они имеют специальное название — адиабатные процессы. При адиабатном расширении работа совершается за счет расходования части внутренней энергии газа — в результате темпера- тура газа понижается. При адиабатном сжатии за счет работы внешних сил увеличивается внутренняя энергия газа — в результате темпера- тура газа повышается. Если сжатие газа произвести достаточно быст- ро, то теплообмен произойти не успеет — в этом случае, даже без при- нятия мер к теплоизоляции газа, будет происходить адиабатное Рис 29 2 160
сжатие. При достаточно медленном сжатии, обеспечивающем тепло- обмен газа с окружающей средой, происходит изотермическое сжатие (при температуре среды). Я попрошу Вас нарисовать в осях р, V график изобары, изотермы и адиабаты для расширяющегося газа; начальное состояние газа и ко- нечный объем одинаковы для всех трех процессов. Рассмотрим энерго- обмен газа со средой для указанных трех процессов. В каком из этих случаев максимальна работа, совершаемая газом при расширении? В каком случае теплопередача является наибольшей? Питатель Б: Но Вы нам не показали, как изображается адиабата... Автор: Зато Вы знаете, что при адиабатном расширении температура газа падает. Читатель Б: Понял. Вот рисунок (рис. 29.3). Здесь кривая 1 — изобара, 2 — изотерма, 3 — адиабата. Автор: Рисунок хорош. Что Вы скажете о различиях в энергообмене газа со средой в этих процессах? Читатель Б: В точке 1 на рисунке Г, > Т. Следовательно, Д1/ > 0 — при изобарном расширении внутренняя энергия газа повышается. В точке 2 Т2 = Т. Следовательно, AU = 0. В точке 3 Т3<.Т. Следовательно, Л(/ <0. Автор: Пожалуйста, подробнее. Читатель Б: Я затрудняюсь. Мне не приходилось проводить подобные рас- суждения. Автор: Пусть А2, А3— работы, совершаемые газом при изобарном, изотермическом и адиабатном расширениях соответственно. Из сопо- ставления площадей под кривыми видно: At >Д2>Д3. Теперь рас- смотрим энергообмен в каждом из указанных процессов. При изобарном расширении Л(/ = Q! — Д^ газ получает извне тепло- ту часть полученной энергии идет на совершение газом работы Д,, другая часть — на приращение внутренней энергии газа. При изотер- мическом расширении Л£/ = 0, Q2 = Д2; газ получает извне теплоту Q2, такое же количество энергии в виде работы Д2 он возвращает в окру- жающую среду; внутренняя энергия газа не изменяется. При адиабат- ном расширении А1/ = — Д3, Q3 = 0; газ не получает теплоты, работа Д3 совершается за счет внутренней энергии газа. На рис. 29.4 схема- тически показан характер энергообмена для каждого из рассматривае- мых процессов (по левому «каналу» в газ поступает энергия через Рис 29 3 6 - Л. В. Тарасов, А. Н. Тарасова Рис. 29.4 161
теплопередачу, по правому «каналу» из газа уходит энергия через совершение работы; количество стрелок наглядно показывает интен- сивность соответствующего энергообмена). Хотелось бы подчеркнуть, что запас внутренней энергии газа нельзя даже мысленно разделить на «запас теплоты» и «запас работы». Тепло- передача и совершение работы — это просто два разных «канала», по которым может притекать (или утекать) энергия (подобно тому, как вода может поступать в бассейн по двум разным трубам). А теперь вернемся к рис. 26.7 и рассмотрим энергообмен газа со средой в изображенном на рисунке циклическом процессе. Читатель Б: Теперь я смогу это сделать. На участке 1—2 (изотерма) газ получает извне некое количество теплоты и целиком расходует его на совершение работы. На участке 2—3 (изохора) работа не совершается, внутренняя энергия газа увеличивается за счет поступающей извне теп- лоты. На участке 3—1 (изобара) над газом совершают работу, однако его температура понижается. Это означает, что газ весьма интенсивно отдает теплоту в окружающую среду (количество отданной теплоты равно сумме работы и уменьшения внутренней энергии). Автор: Вы правы. В заключение последний вопрос: на рис. 26.4 были изображены шесть разных процессов перехода из одного состояния в другое. Чем различаются эти процессы с точки зрения термодинамики? Читатель А: Они различаются характером энергообмена газа со средой. Автор: Правильно. В заключение отметим одно из общих положений учения о теплоте, называемое принципом Ле Шателье. Этот принцип гласит, что внешнее воздействие, выводящее систему из состояния равновесия, вызывает в этой системе процессы, стремящиеся ослабить результат этого воздей- ствия. Так, нагревание тела стимулирует процессы, связанные с погло- щением теплоты; наоборот, охлаждение способствует процессам, при которых теплота выделяется. Принцип Ле Шателье позволяет предска- зать, в каком направлении сместится состояние равновесия, если изме- нить какие-либо внешние параметры, определяющие состояние системы (например, давление). Этот принцип является термодинамическим ана- логом закона индукции Ленца (см. § 42). ЗАДАЧИ 29.1. Рассматриваются процессы, показанные на рис. 26.4. Даны параметры: р|5 Vt, Т,, р2, V2. Найти отношение количеств телоты, участвующих в переходах 1 7, 1 -► 8, 52, 4^2. 29.2. Условие такое же, как в задаче 29.1. Найти отношение количеств теплоты, участвующих в переходах 1 -* 6, 3 -* 2 и 1 5, 8^2. Рис 29.5 29.3. Вычислить работу А, которую совершит газ при изобарном нагревании от = 20°С до t2 = 100°С, если он находится в сосуде, закры- том подвижным поршнем с площадью сечения S = 20 см2 и массой М = 5 кг. Рассмотреть два случая расположения сосуда: 1) горизон- тальное (рис. 29.5, а); 2) вертикальное (рис 29.5, б). Начальный объем газа равен V — 5 л; атмосферное давление нормальное. 162
29.4. Три моля идеального газа, находящегося при температуре tQ — 27°С, охлаж- дают изохорно так, что давление падает в п = 3 раза. Затем газ расширяется при Постоянном давлении. В конечном состоянии его температура равна первона- чальной. Определить работу А, произведенную газом. 29.5. Чему равно изменение внутренней энергии воздуха в комнате при изобарном нагревании воздуха от температуры Z до температуры t2? 30. ЧТО ТАКОЕ ЦИКЛ КАРНО? Автор: Поговорим о коэффициенте полезного действия (КПД) тепловой машины. Известна ли Вам какая-нибудь формула для КПД? Читатель А: Да, известна: п = (Л-г2)/т, (30.1) (здесь т] — КПД). Автор: А что такое Тt и Г2? Читатель А: Я помню, что Т2— это температура холодильника, а Г] — температура нагревателя. Только я не очень ясно представляю себе, что называют холодильником и нагревателем. Автор: А для какого случая верна формула (30.1)? Читатель Б: Я где-то читал, что эта формула выведена для цикла Карно. Но я не знаю ничего о нем. Автор: Поэтому разрешите мне начать несколько издалека. Представим себе некую тепловую машину, действие которой основано на изменении состояний некоторой массы газа (так называемого рабочего тела). Очевидно, что эти изменения должны образовывать циклический про- цесс — в конце цикла газ всякий раз должен возвращаться в исходное состояние. В разных машинах указанный циклический процесс будет разным. Следовательно, разным будет и характер энергообмена рабо- чего тела (газа) с окружающей средой. Все это важно учитывать при рассмотрении КПД тепловой машины. Наибольший КПД достигается в идеализированной тепловой машине, где газ изменяет состояния по циклу Карно. Именно в этом случае КПД описывается формулой (30.1). Предположим, что существуют два больших тела — одно при температуре Т2, другое — при температуре Гр Тела должны быть большими, для того чтобы энергообмен между ними и рабочим телом не приводил к изменению температур Т2 и Гр Если Т2 < Гр то первое большое тело называют холодильником (тепло- приемником), а второе — нагревателем (теплоотдатчиком). Рабочее тело (газ) попеременно вступает в тепловой контакт с нагревателем и холодильником и обменивается энергией через теплопередачу. Цикл Карно состоит из двух изотерм и двух адиабат (рис. 30.1). На участке 1—2 (изотерма при Г() газ получает от нагревателя теплоту Qi и, расширяясь, расходует ее на работу Ар На участке 2—3 (адиаба- та) газ совершает работу А3; при этом его температура падает до Т2. На участке 3—4 (изотерма при Т2) газ отдает холодильнику теплоту Q2, равную работе А2 по сжатию газа. На участке 4—1 (адиа- бата) работа А4 по сжатию газа переходит во внутреннюю энергию, вследствие чего температура газа повышается до Гр В итоге возвра- 163
Рис. 30.1 щаемся в исходное состояние 1 (а следовательно, и к исходной внутрен- ней энергии рабочего тела). Предположим, что некая тепловая машина работает по циклу Карно. Газ получает от нагревателя теплоту Q, и отдает холодильнику тепло- ту Q2. Так как Q4 = и Q2 — т0 легко убедиться, что Q2 < QP Зна- чит, газ отдал теплоты меньше, чем ее получил. В то же время внут- ренняя энергия газа за цикл остается прежней. Следовательно, раз- ность теплот Q[ — Q2, отданная газом в окружающую среду, и есть полезная работа тепловой машины. Отсюда следует, что выражение для КПД машины может быть записано в виде t]=(Qi-Q2)/Qi- (30.2) В термодинамике показывается (мы это делать не будем), что выра- жение (30.2) приводится к более простому виду (30.1). Из (30.1), (30.2) видно, что КПД не зависит от свойств вспомогательного рабо- чего тела. Кроме того, даже в идеальной тепловой машине КПД меньше единицы; доля Т\/Т% энергии, отдаваемой нагревателем, беспо- лезно переходит в виде теплоты к охладителю. Коэффициент полезного действия реальной тепловой машины всегда меньше, чем КПД, описываемое формулой (30.1) из-за неизбежно происходящих в ней необратимых процессов. Читатель А: Встречаются ли реальные машины, работающие по циклу Карно? Автор: Да, такой машиной можно считать работу двигателя внутреннего сгорания. ЗАДАЧИ 30.1. Двигатель работает по циклу Карно и за один цикл получает от нагревателя = 700 кал. Температура нагревателя Т{ — 600 К, а температура охладителя Т2 = 300 К. Найти работу А, совершаемую за цикл, и количество теплоты Q2, отдаваемое при этом охладителю (1 кал = 4,19 Дж). 30.2. Тепловая машина периодического действия имеет коэффициент полезного действия т]( == 40%. В результате ее усовершенствования количество теплоты, получаемое от нагревателя за один цикл, увеличилось на b = 20%, а количество теплоты, отдаваемое холодильнику, уменьшилось на а = 20%. Каким стал коэффи- циент п2 полезного действия этой тепловой машины. 164
30.3. Определить коэффициент г| полезного действия' тепловой машины, рабо- тающей по циклу, изображенному на рис. 30.2. Рабочим веществом является моль.азота (Af = 28 • ICh ’ кг/моль). Известно р2 = 2р, = 4 • 105 Па, V2 = 2V, = = 20 л. Удельная теплоемкость азота сх = 748 Дж/(кг • К), с = 1045 Дж/(кг • К). Определить также работу А, совершаемую газом за один цикл. 30.4. Идеальная тепловая машина имеет полезную мощность N = 50 кВт. Темпе- ратура нагревателя при этом Г, = 373 К, а температура холодильника Т2 = 273 К Определить тепловую мощность 2V , получаемую машиной от нагревателя, и энер- гию Q2, отдаваемую холодильнику за t = 1ч работы. 31. СКОЛЬКО У ГАЗА ТЕПЛОЕМКОСТЕЙ? Автор: Что такое теплоемкость? Читатель А: Это количество теплоты, которое надо затратить, чтобы повы- сить температуру тела на 1 К. Автор: Верно. А сколько у газа теплоемкостей? Читатель А: Я не понимаю вопроса. Автор: В таком случае я задам тот же вопрос в более доходчивой форме. Предположим, что газ нагревают и повышают его температуру на АТ. Делают это дважды: один раз при постоянном объеме газа (изохорное нагревание), а другой раз—при постоянном давлении (изобарное нагревание). Одинаковое ли количество теплоты надо затратить на нагревание газа в указанных случаях? Читатель А: По-моему, одинаковое. Читатель Б: А по-моему, неодинаковое. При постоянном объеме работа не совершается и вся теплота идет на приращение внутренней энергии газа. Обозначим эту теплоту через Qy При постоянном давлении нагре- вание газа неизбежно сопровождается его расширением, а при этом совершается работа А = pAV. Подводимая в этом случае теплота Qp идет частично на приращение внутренней энергии газа, а частично на совершение газом указанной работы. Следовательно, Qp > Qv. Автор: Совершенно верно. Теперь еще раз вспомним, что такое теплоем- кость. Читатель А: Я, кажется, понял. У газа две теплоемкости — одна при постоянном объеме (Су), а другая — при постоянном давлении (Ср). Мы можем записать: Cv = QV/AT и Ср = Qp/AT. При этом Ср >• Cv. Автор: Верно. А не могли бы Вы выразить Ср через Су? Читатель Б: Разрешите мне. На приращение внутренней энергии газа идет теплота Qy = СуАГ. Значит, Qp=CyAT + pAV, отсюда Ср \Т С'- Р \Т' Автор: Далее надо воспользоваться уравнением состояния газа, из кото- рого следует pAV = (m/Af) R&T. Окончательно находим Ср = Су -|- (nt/М) R. (31.1) (31.2) 165
Из выражения (31.2) видно, что универсальная газовая постоянная R равна разности теплоемкостей Ср — Cv одного моля любого идеального газа. ЗАДАЧИ 31.1. Определить количество теплоты Q, которое необходимо сообщить трем молям одноатомного газа, чтобы нагреть его при постоянном давлении от температуры Г( — 300 К до температуры Т2 — 2Ту. 31.2. Кислород массой m = 7 г находится в цилиндрическом сосуде под тяжелым поршнем. Определить количество теплоты Q, которое необходимо сообщить кисло- роду, чтобы нагреть его на ДГ = 16 К- Молярная теплоемкость кислорода при постоянном объеме равна Су = 20,9 Дж/(моль • К). 31.3. В цилиндре под тяжелым поршнем находится пг = 40 г углекислого газа. Газ нагревают от = 20°С до t2 = 100°С. Определить работу А, которую соверша- ет углекислый газ, и количество теплоты Q, которое ему необходимо сообщить Удельная теплоемкость при постоянном объеме су = 0,655 кДж/(кг • К). 32. ДАВАЙТЕ ПОГОВОРИМ О ПОЛЕ Автор: Давайте побеседуем о поле — одном из основных физических понятий. Для определенности будем говорить об электростатическом поле. Как Вы представляете себе поле? Что это такое? Читатель А: Признаться, мне очень трудно представить себе поле. Поле — это что-то неуловимое, невидимое, вроде призрака. В то же время оно, как утверждают, присутствует везде. Я не возражаю, когда мне гово- рят, что поле материально. Но для меня это остается пустым звуком. Когда говорят о веществе, то я понимаю, о чем говорят. Но когда говорят о поле, тут я уже не понимаю, о чем собственно, идет речь. Читатель Б: Я вполне ощутимо представляю себе поле. В веществе мате- рия находится как бы в концентрированном виде. В поле, напротив, ма- терия как бы «размазана» по пространству. Тот факт, что мы не видим поле глазами, ничего не доказывает: поле можно прекрасно «видеть» с помощью относительно простых приборов. Поле выполняет роль пере- датчика взаимодействия между телами, например электрическое поле является передатчиком взаимодействия между неподвижными электри- ческими зарядами. Можно считать, что каждый заряд «создает» вокруг себя поле. Поле, созданное одним зарядом, влияет на другой и, наобо- рот, поле, созданное вторым зарядом, влияет на первый. Так осуществ- ляется кулоновское взаимодействие зарядов. Читатель А: Но разве нельзя обойтись без «посредников»? Что мешает нам полагать, что один заряд воздействует на другой заряд непо- средственно? Читатель Б: Против этого можно выдвинуть серьезное возражение. Пред- ставим себе, что один из зарядов в какой-то момент времени по какой- то причине сдвинулся, «вздрогнул». Если исходить из предположения о «непосредственном взаимодействии», то необходимо заключить, что второй заряд должен в тот же самый момент тоже «вздрогнуть». Это означало бы, что сигнал от первого заряда дошел до второго заряда мгновенно, что, как хорошо известно, противоречит основным представ- 166
Что такое поле? Как описывается поле? Как происходит движение в поле? Эти фундаментальные вопросы физики удобнее всего рассматривать на примере электростатического поля. Обсудим движение заряженного тела в электростатическом поле. Решим ряд задач, иллюстрирующих закон Кулона. лениям теории относительности. Если же есть передатчик взаимодей- ствия, т. е. поле, то в этом случае сигнал распространяется от одного заряда к другому по полю. Как бы ни велика была скорость распро- странения сигнала, она все же конечна, и поэтому может существо- вать некоторый промежуток времени, когда первый заряд уже кончил «вздрагивать», а второй еще не начинал. В течение этого промежутка времени только поле содержит сигнал о «вздрагивании». Читатель А: Все-таки я хотел бы услышать четкое определение того, что такое поле. Автор: Я с интересом выслушал Ваш диалог. Я чувствую, что чита- тель Б живо интересовался проблемами современной физики и загля- дывал в различные популярные книжки по физике. В результате у него вырабатывалось то, что можно назвать инициативным мышлением. По- нятие поля является для него вполне реальным, «рабочим» понятием. Его замечания о поле как о передатчике взаимодействий вполне пра- вильны Читатель А, по-видимому, ограничивался формальным прочте- нием учебника. В результате его мышление выглядит в значительной мере беспомощным. Я говорю это, конечно, не с целью обидеть собесед- ника, а для того, чтобы на его примере подчеркнуть, что многие экзаме- нующиеся чувствуют себя в подобных ситуациях довольно беспо- мощно. Как ни странно, многие учащиеся не любят читать научно- популярную литературу Однако вернемся к существу рассматриваемо- го вопроса. (К читателю А.) Вот Вы потребовали, чтобы Вам дали чет- кое определение поля Без такого определения Вы не можете предста- 167
вить себе поле. Однако Вы говорили, что представляете себе, что такое вещество. Но разве Вы знаете четкое определение понятия вещества? Читатель А: Понятие вещества не нуждается в таком определении. Веще- ство можно «потрогать» рукой. Автор: В таком случае и понятие поля не нуждается в определении: ведь поле тоже можно «потрогать», хотя и не рукой. Однако положение с определением гораздо серьезнее. Дать четкое, логически безупречное определение — значит выразить рассматриваемое понятие через какие- то более «первичные» понятия. Но что делать, если данное понятие относится к «первичным» понятиям? Попробуйте в геометрии дать оп- ределение прямой линии. Примерно такое же положение вещей имеет место в случае понятий «вещество» и «поле». Это настолько первичные, настолько фундаментальные понятия, что вряд ли можно надеяться дать какое-либо четкое определение, которое бы исчерпывало их сущ- ность. Читатель А: Однако, наверное, можно попытаться дать какое-то не очень четкое определение? Автор: Да, конечно. Только при этом надо иметь в виду, что всякое такое определение ни в коей мере не является исчерпывающим. Мате- рия может существовать в различных формах. Она может быть сосре- доточена в пределах ограниченной области пространства с более или менее четкой границей (или, как говорят, «локализована»), но может быть, напротив, «делокализована». Первое состояние материи можно сопоставить с понятием «вещество», второе состояние — с понятием «поле». И то, и другое состояния наряду со специфическими харак- теристиками имеют общие физические характеристики. Например, есть энергия единицы объема вещества и есть энергия единицы объема поля. Можно говорить о количестве движения единицы объема вещества и о количестве движения единицы объема поля. Всякое поле играет роль переносчика определенного типа взаимодействия: именно по этому взаимодействию и выявляются характеристики поля в той или иной его точке. Например, электрически заряженное тело образует вокруг себя в пространстве электростатическое поле. Чтобы обнаружить и изме- рить это поле в той или иной точке пространства, надо внести в эту точку другое заряженное тело и измерить силу, действующую на это тело. При этом предполагается, что указанное второе тело достаточно мало, так что вносимым им искажением измеряемого поля можно пре- небречь. Свойства материи неисчерпаемы, процесс познания бесконечен. Мы по- степенно продвигаемся все дальше по пути познания и практического использования свойств окружающей нас материи. Продвигаясь, мы вынуждены время от времени «наклеивать ярлыки», которые являются как бы вехами на пути познания. Вот мы что-то назвали «полем». Мы понимаем, что это «что-то» — само по себе бездна. Мы многое знаем об этой бездне, названной нами «полем», и поэтому более или менее удов- летворительно пользуемся введенным понятием. Мы знаем многое, однако далеко не все. Пытаться дать этому «что-то» четкое опреде- ление—это все равно, что пытаться измерить глубину бездонной пропасти. 168
Читатель Б: Я думаю, что понятие поля, как, впрочем, и всякое другое понятие, возникающее в процессе изучения материального мира, неис- черпаемо. Именно поэтому и невозможно исчерпывающее, четкое опре- деление поля. Автор: Я полностью с Вами согласен. Читатель А: Меня вполне удовлетворяли Ваши замечания о веществе и поле как о двух состояниях материи — локализованном и делокали- зованном. Зачем Вы стали говорить о неисчерпаемости физических по- нятий, о бесконечности процесса познания? Когда я слушал это, ясность снова исчезла и все стало каким-то расплывчатым. Автор: Ваша психология мне понятна. Вы ищете пусть не абсолютно точное, но спокойное определение поля. Вы готовы добросовестно вы- учить это определение и выдавать его по первому требованию. Вы не хотите признать, что ситуация вовсе не статична, а динамична. Не следует думать, что все становится расплывчатым. Я бы сказал, что все становится динамичным. Всякое четкое определение само по себе является жестким, законченным. Однако физические понятия следует рассматривать именно в развитии. То, что мы понимали под понятием поля вчера, заметно отличается от того, что мы понимаем под этим понятием сегодня. Так, например, современная физика в отличие от классической не проводит строгой границы между полем и веществом. В современной физике поле и вещество взаимно превращаются: веще- ство переходит в поле, а поле переходит в вещество. Однако говорить об этом более подробно сейчас значило бы забегать слишком далеко вперед. Читатель Б: Наша беседа по физике приняла характер явно философской беседы. Автор: Это совершенно естественно, потому что обсуждение физических понятий обязательно предполагает наличие достаточно развитого диа- лектического мышления. Если такое мышление еще не выработано, при- ходится поневоле делать отступления философского характера. Именно поэтому я настойчиво советую Вам почаще заглядывать в разные книж- ки. Тем самым Вы будете воспитывать свое мышление, делать его более гибким, более динамичным и, я бы сказал, менее бюрократичным. В этой связи неоценимую помощь всякому молодому человеку может оказать книга В. И. Ленина «Материализм и эмпириокритицизм». Я рекомендую прочитать ее. Читатель А: Но это очень сложная книга. Ее изучают в институте. Автор: Я не настаиваю на том, чтобы Вы изучили эту книгу. Она дей- ствительно не предназначена для легкого чтения. Вы просто попробуйте со вниманием прочитать ее. В зависимости от Вашей подготовленности она окажет на Ваше мышление более сильное или менее сильное влияние. В любом случае это будет полезно. В заключение я хотел бы сказать следующее: читателя А явно пугает «расплывчатость», он тре- бует четкости. Он полагает, что чем больше четкости, тем лучше. Он забывает, что все (и даже четкость) хорошо в меру. Попробуйте пред- ставить себе предельно четкий мир, о котором Вы имеете исчерпываю- щие сведения. Представьте это и скажите: разве не поразил бы Вас этот мир своей примитивностью, своей неспособностью к развитию? Поду- 169
майте над всем этим и не спешите с выводами. А пока давайте попро- буем подойти к проблеме с другой стороны. Поставим вопрос: как описывается поле? Я знаю, что многие люди, получив ответ на этот вопрос, скажут: «Теперь мы знаем, что такое поле». 33. КАК ОПИСЫВАЕТСЯ ЭЛЕКТРОСТАТИЧЕСКОЕ ПОЛЕ? Автор: Итак, продолжим беседу, начатую в предыдущем параграфе. Как описывается электростатическое поле? Читатель Б: Для описания электростатического поля используется век- торная силовая характеристика, называемая напряженностью электри- ческого поля. В каждой точке поля напряженность Е имеет определен- ное направление и числовое значение. Если перемещаться от одной точки поля к другой так, чтобы направления векторов напряженности были все время ориентированы по касательной к направлению переме- щения, то полученные в результате такого перемещения траектории называются «силовыми линиями» поля. Силовые линии весьма удобны для графического представления поля. Автор: Хорошо. Теперь давайте рассуждать более конкретно. Кулонов- скую силу взаимодействия двух зарядов q{ и q2, расстояние между кото- рыми равно г, запишем в виде Эту формулу можно переписать так: Fe = Е (г) q2, (33.2) где £ (г) = (33.3) Формула (33.3) говорит о том, что заряд q{ создает вокруг себя поле, напряженность которого на расстоянии г от заряда имеет величину qjE. Формула (33.2) говорит, что это поле действует на заряд q^ помещенный в точку на расстоянии г от заряда qx, с силой Е (г) q2. Новая запись формулы (33.1) произведена за счет введения «посред- ника» — величины Е, являющейся характеристикой поля. Попробуйте проанализировать область применимости соотношений (33.1) — (33.3). Читатель Б: Формула (33.1) применима для двух точечных зарядов. Зна- чит, такая же должна быть и область применимости формул (33.2) и (33.3). Ведь они получены из (33.1). Автор: Вы не совсем правы. В отличие от (33.1) и (33.3) формула (33.2) имеет значительно более широкую область применимости. Чем бы ни было создано поле Ё (точечным зарядом, совокупностью точеч- ных зарядов, заряженными телами произвольной формы), во всех слу- чаях сила, с которой поле действует на заряд q0, равна произведению этого заряда на напряженность поля в той точке, где находится за- ряд q0. Болбе общая запись формулы (33.2) имеет следующий вектор- ный вид: Fe = E(r)q() (33 4) 170
Из (33.4) видно, что направление си- t / у J лы, действующей на заряд q0 в данной \ точке поля, совпадает с направлением •* JEkj » напряженности поля в этой точке, если заряд q0 положителен. Если заряд q0 f \ Х7 \ отрицателен, то направление силы ' ~ противоположно направлению на- Риг чч 1 пряженности. гиъ- 001 Здесь можно почувствовать «само- стоятельность» понятия поля. Разные заряженные тела создают вокруг себя разные электрические поля, од- нако каждое из этих полей действует на помещенный в него заряд по одному и тому же закону (33.4). Чтобы найти силу, действующую на заряд, надо предварительно вычислить напряженность поля в точке, где находится этот заряд. Поэтому важно уметь находить напряжен- ность поля системы зарядов. Предположим, что имеются два заряда qx и q2. Напряженность поля от каждого из этих зарядов легко можно най- ти (и по значению, и по направлению) для любой интересующей Вас точки пространства. Допустим, что в некоторой точке, заданной векто- ром г, эти напряженности описываются векторами Ёх(г) и Ё2(г). Чтобы найти результирующую напряженность в точке г, Вы должны векторно сложить напряженности от отдельных зарядов: +^2(г). (зз.5) Подчеркиваю, что складывать напряженности надо именно векторно. (К читателю А) Вам это понятно? Читатель А: Да, я знаю, что напряженности складываются векторно. Автор: Хорошо. В таком случае давайте проверим, как Вы используете это на практике. Нарисуйте силовые линии поля двух зарядов разных зна- ков (-}- qx и — <?2) ПРИ условии, что один из зарядов (например, -f- q{) в несколько раз больше другого Читатель А: Я затрудняюсь это сделать. Ранее мы такие поля не рас- сматривали. Автор: А какие поля Вы рассматривали? Читатель А: Я знаю, как будет выглядеть картина силовых линий поля двух одинаковых по величине точечных зарядов. Я изобразил эту кар- тину на рис. 33.1. Автор: Ваш рисунок немного неточен, хотя качественно правильно пере- дает картину силовых линий поля, одинаковых по числовому значению и различных по знаку зарядов. Разве Вы не можете представить себе, как будет изменяться эта картина по мере того, как значение одного из зарядов начнет возрастать? Читатель А: Мы никогда этого не делали. Автор: В таком случае давайте воспользуемся правилом векторного сло- жения напряженностей. Начнем со случая, когда заряды одинаковы (рис. 33.2, а). Выберем три точки А, Б, Вк построим в каждой из них па- ру векторов напряженности: Ёх и Ё2 (Ёх —поле заряда q{, Ё2 — поле заряда — q2). Затем выполним для каждой из указанных точек сложение векторов Ёх и Ё2, получим суммарные векторы ЁА, ЁБ, Ёв. Они должны быть касательными к силовым линиям поля в соответст- 171
вующих точках. Эти три вектора подсказывают поведение силовых ли- ний, изображенных на рис. 33.3, а. Сравните этот рисунок с рис. 33.1, ко- торый был предложен Вами. Обратите внимание на допущенные Вами неточности в поведении силовых линий слева от заряда — q и справа от заряда + q Предположим, что заряд 4- q{ увеличился в два раза, а заряд — q2 уменьшился в два раза (рис. 33.2, б). Выберем такие же, как и в предыдущем случае, точки А, Б, В. Построим для этих то- чек сначала векторы напряженностей от каждого из зарядов, а затем их суммы: ЁА, ЁБ, Ёв. Соответствующая этим векторам картина сило- вых линий показана на рис. 33.3, б. Наконец, предположим, что заряд q{ увеличился еще вдвое, a q2 еще вдвое уменьшился (рис. 33.2, а). Построим для точек А, Б, В векторы ЁА, ЁБ, Ёв. Соответствующая картина силовых линий изображена на рис. 33.3, в. Вы видите, что с возрастанием относительного значения заряда его влияние становится сильнее: поле заряда + q} начинает явно подавлять поле заряда — q2. Ри( 33 2 172
Читатель А: Теперь мне понятно, как можно конструировать картину сило- вых линий поля системы из нескольких зарядов. Автор: Продолжим обсуждение электростатического поля. У этого поля есть одно важное свойство, роднящее его с полем сил тяготения, а именно: работа сил поля по любому замкнутому пути равна нулю. Иными словами, если заряд, перемещаясь в поле, возвращается в исходную точку, то работа сил поля по этому перемещению равна нулю. На одних участках nyiH ла работа положительна, на других — отрицательна, но в сумме получается работа, равная нулю. Из этого свойства электростатического поля вытекают интересные следствия. Не могли бы Вы указать эти следствия? Читатель Б: Нет, я что-то не соображу. Автор: Я помогу Вам. Вы, наверное, обращали внимание на то, что сило- вые линии электростатического поля не бывают замкнутыми. Они начи- наются и оканчиваются в зарядах (начинаются в положительных за- рядах и оканчиваются в отрицательных зарядах) или же уходят на бес- конечность (приходят из бесконечности). Не могли бы Вы связать это обстоятельство с указанным выше свойством электростатического поля? Читатель Б: Я, кажется, понял. Если бы силовая линия электростатиче- ского поля замыкалась сама по себе, то, перемещаясь все время вдоль нее, мы могли бы вернуться в исходную точку. При перемещении заряда вдоль силовой линии работа поля, очевидно, не меняется по знаку и, следовательно, не может быть равна нулю С другой стороны, работа по любому замкнутому пути должна равняться нулю. Значит, силовые линии электростатического поля не могут замыкаться сами на себя. Автор: Правильно. Из указанного выше свойства электростатического по- ля вытекает еще одно следствие: работа по перемещению заряда из од- ной точки поля в другую не зависит от пути, по которому происходит перемещение. В самом деле, будем перемещать заряд из точки А в точку Б по двум разным путям / и 2 (рис 33.4). Обозначим работу сил поля по перемещению заряда по пути 1 через Л1; а по пути 2 — через Л2. Совершим круговой обход: из точки А пойдем в точку Б по пути 1, а из точки Б в точку Л — по пути 2 При обратном прохождении пути 2 будет совершаться работа — А2. Полная работа поля при круговом обходе есть Л! +(—Л2) —Л, — Л2. Поскольку работа по любому замк- нутому пути должна равняться нулю, то отсюда получаем Л, = Л2. Тот факт, что работа по перемещению заряда не зависит от того, какой мы выбрали путь, а целиком определяется начальной и конечной точками пути, позволяет использовать эту величину в качестве характеристики поля (ведь она зависит только от выбора то- чек поля!). Так появляется еще одна харак- теристика электростатического поля — потенциал. В отличие от напряженности эта характеристика является скалярной, так как выражается через работу. Читатель Б: Нам говорили, что понятие потен- циала поля не имеет физического смысла. Физический смысл имеет только разность по- тенциалов двух каких-либо точек поля 173
Автор: Вы правы. Строго говоря, предыдущие рассуждения позволяют ввести именно разность потенциалов: разность потенциалов двух точек поля А и В (обозначим ее через фл — фв) определяется как отношение работы сил поля по перемещению заряда qQ из точки А в точку В к за- ряду qQ, т. е. Аа в (33.6) Однако если полагать, что на бесконечности поле отсутствует (ф^ = 0), то выражение (33.6) принимает вид А. Фл = (33.7) Таким образом, потенциал поля в данной точке может быть опреде- лен через работу, которую производят силы поля при перемещении положительного единичного заряда из данной точки в бесконечность. Если рассматривать работу, производимую не полем, а против сил поля, то потенциал поля в данной точке есть работа, которую надо произвести при перемещении положительного единичного заряда из бесконечности в данную точку. Ясно, что такое определение потенциала не позволяет выполнить экспериментальное измерение потенциала в данной точке поля, так как мы в действительности не можем удалиться на беско- нечность. Именно поэтому и говорят, что физический смысл имеет раз- ность потенциалов в двух точках поля, но не сам потенциал в той или иной точке. Можно сказать, что потенциал в данной точке определен с точностью до произвольной постоянной. В качестве этой постоянной принято рассматривать значение потенциала на бесконечности, именно от этого значения и отсчитывается потенциал. Для удобства полагают, что значение потенциала на бесконечности равно нулю. В рамках ука- занных предположений потенциал поля точечного заряда q{ в точке, на- ходящейся на расстоянии г от заряда, равен ф(Н = Я\/г- (33.8) Нетрудно сообразить, чему равен потенциал поля нескольких точечных зарядов в той или иной точке г. Читатель Б: Обозначим значения потенциала в точке г от каждого из зарядов в отдельности через фДг), ф2(/0. Суммарный потенциал ф(г) равен, очевидно, алгебраической сумме потенциалов от отдельных за- рядов: ф(г) = <Р1 (г) + ф2(г) + .... (33.9) Потенциал от положительного заряда берется в этой сумме со знаком плюс, от отрицательного со знаком минус. Автор: Правильно. Рассмотрим теперь понятие об эквипотенциальных по- верхностях. Геометрическое место точек поля, имеющих один и тот же потенциал, называют эквипотенциальной поверхностью (или поверх- ностью равного потенциала). Через каждую точку поля проходит одна силовая линия и одна эквипотенциальная поверхность. Как они ориен- тированы друг относительно друга? 174
Читатель Б: Я знаю, что в каждой точке поля "2 силовая линия и соответствующая эквмпо- тенциальная поверхность взаимно перпенди- кулярны. Автор: Мотли бы Вы доказать это? ^Х Читатель Б: Нет, пожалуй, не смогу! С"'. Автор: Это доказательство несложно. Пусть /'*''Х S через некоторую точку А проходят силовая / ^Хх**^ линия AAt и эквипотенциальная поверхность S (рис. 33.5). Напряженность поля в точке А описывается вектором ЁА. Переместим за- рнс 335 ряд q0 из точки А в некоторую точку Б, кото- рая лежит на эквипотенциальной поверхнос- ти S на малом расстоянии А/ от точки А. Работа по такому перемещению выражается формулой А = Fe&l cos а = EAq^Al cos а, (33.10) где а — угол между вектором ЁА и направлением перемещения. Эта же работа может быть выражена через разность потенциалов поля в точках А и Б. Таким образом, можно записать еще одно соотношение: 4 = <7о(<Рл ~ <₽б)- (33.11) Так как точки А и Б лежат на одной и той же эквипотенциальной поверхности, то = <р£. Значит, в соответствии с (33.11) работа А рав- на нулю. Подставляя этот результат в соотношение (33.10), получаем EAqQ\l cos а = 0. Из всех множителей в левой части этой формулы нулю может быть равен только cos а. Таким образом, заключаем, что а= 90°. Ясно, что этот результат получается для разных направлений перемещения АЁ, лишь бы все эти перемещения находились в пределах эквипотенциаль- ной поверхности S. Кривизна поверхности не нарушает рассуждений, поскольку перемещения А/ весьма малы. Для графического изображения электростатического поля наряду с силовыми линиями рисуют эквипотенциальные поверхности. Используя взаимную перпендикулярность линий и поверхностей, можно по извест- ному семейству силовых линий нарисовать семейство сечений эквипо- 175
тенциальных поверхностей, и наоборот. (К читателью А) Попробуйте нарисовать сечения эквипотенциальных поверхностей в случае, изобра- женном на рис. 33.3, а. Чтобы не спутать с силовыми линиями, изоб- ражайте сечения поверхностей пунктирными линиями. Читатель А: Я буду проводить пунктирные линии так, чтобы они все время пересекали силовые линии под прямым углом. Вот мой рису- нок (рис. 33.6). Автор: Ваш рисунок правильный. Читатель А: Какие виды электростатических полей встречаются в задачах, предлагаемых на экзамене? Хотелось бы как-то собрать воедино основ- ные формулы. Автор: Это сделать нетрудно. Встречаются поля трех видов: а) однородное поле (напряженность поля одинакова во всех точках): £(г) = const и — <р£ = Егаб (33.12) (здесь гАБ — проекция расстояния между точками А и Б на направ- ление напряженности поля); б) поле точечного заряда q: = y (33 13) (здесь г — радиус-вектор из заряда в точку наблюдения, п — еди- ничный вектор; п = г/г); в) поле системы из нескольких точечных зарядов. В последнем случае, как уже указывалось, напряженность есть векторная сумма напряжен- ностей от отдельных зарядов, а потенциал — скалярная сумма потен- циалов от отдельных зарядов. Далее я хочу немного поговорить об однородном поле. Как Вы пола- гаете, достаточно ли определить однородное поле как поле, в каждой точке которого напряженность одинакова по направлению? Читатель Б: При этом никаких утверждений о модуле напряженности не делается? Автор: Не делается. Читатель Б: Я думаю, недостаточно. Однородное поле — это поле, в каж- дой точке которого напряженность одинакова и по направлению, и по модулю. Автор: Это так. Но, может быть, из одинаковости направлений напря- женности следует одинаковость модулей напряженности? Читатель Б: С какой стати? Не понимаю. Автор: Тогда давайте рассмотрим поле, силовые линии которого показаны на рис. 33.7. Замечу, что чем гуще располагаются силовые линии, тем больше модуль напряженности. Судя по рисунку, Е{^>Е2. Требуется вычислить работу по перемещению некоторого заряда q по замкнутому прямоугольному контуру АБВГ. Читатель Б: А зачем это делать? Заранее ясно, что работа должна рав- няться нулю. Автор: И все же посчитайте работу по участкам контура. Читатель Б: Работа на участках АБ и ВГ равна нулю, так как здесь 176
силовые линии перпендикулярны направлению перемещения. Работа на участке БВ равна — qE2l, а на участке ГА равна qE\l. Автор: Каков суммарный результат? Читатель Б: Странно! Получается q(E{ —£2) I, что не равно нулю. Какой-то абсурд! Автор: Каковы Ваши выводы? Читатель Б. Я могу заключить только одно: поле, изображенное на рис. 33.7, не может существовать. Автор: Совершенно справедливо. Значит, если силовые линии параллель- ны, то густота их должна быть равномерной (если напряженность оди- накова во всех точках по направлению, то она должна быть одинакова и по модулю). Читатель Б: Довольно интересный результат. У меня есть еще один вопрос. В соотношениях (33.13) для напряженности и потенциала поля точеч- ного заряда у Вас отсутствует коэффициент k, связанный, насколько я помню, с е0. Автор: Множитель k зависит от выбора системы единиц. Наши соотно- шения (33.13) получены для системы СГС, в которой k — 1, если же Вы пользуетесь СИ, то k = 1 /(4ле0), где е0 = 8,85 • 10" 12 Кл2/(м2- Н), т. е. k = 9 • 109 м2 • Н/Кл2. Отметим также, если рассматривается поле в диэлектрике, то напряженность и потенциал уменьшаются в е раз (е — диэлектрическая проницаемость диэлектрика): £(r) = k —— • п, ф (г) = k — (33.14) er2 er ЗАДАЧИ 33.1 . На рис. 33.6 изображены четыре одинаковые по модулю точечные заряды q = 2 Кл каждый, сторона квадрата равна 2 м. Найти модули и направление напряженности поля системы зарядов в точках О и О, в четырех случаях: 1) все заряды положительны; 2) заряды А и Б положительны, В и Г отрицательны; 3) заряды Б и В положительны, А и Г отрицательны; 4) заряды Б и Г положительны, А и В отрицательны. 33.2 . Четыре одинаковых заряда q = 2 Кл каждый расположены в вершинах квад- рата со стороной а = 2 м (рис 33.9). Вычислить потенциал поля системы зарядов в точках А, Б, В, Г, Д. 177
33.3 . В вершинах острых углов прямоугольного треугольника расположены оди- наковые по модулю разноименные заряды по |д| = 2 Кл Определить напряжен- ность Ё и потенциал <р в вершине прямого угла. Катеты равны а = 3 см, b — 4 см. 33.4 . Диагонали ромба имеют длину d} — 2 см, d2 = 3 см. На концах короткой диагонали расположены заряды <?( = 2 мкКл, q2 = 6 мкКл, на концах длинной — заряды = 3 мкКл, <у4 = 12 мкКл. Определить модуль и направление напряженно- сти Ё электрического поля в центре ромба. 33.5 . В вершинах равностороннего со стороной а = 5 м треугольника расположены три одинаковых заряда q — 3 мкКл каждый. Определить модуль и направление напряженности Е электрического поля в центре треугольника и вершине тетра- эдра, построенного на этом треугольнике. 33.6 . Два точечных заряда q, = 9 мкКл и q2 — 25 мкКл расположены на рас- стоянии I = 20 см друг от друга. Определить, в какой точке на прямой, их соеди- няющей, напряженность поля равна нулю. 34. КАК ВЕДУТ СЕБЯ СИЛОВЫЕ ЛИНИИ ВБЛИЗИ ПОВЕРХНОСТИ ПРОВОДНИКА? Автор: Внесем в электростатическое поле какое-нибудь тело, являющееся проводником. Вы хорошо знаете, что проводник в поле характеризуется некоторой физической величиной, называемой электрической емкостью (или просто емкостью). Но задумывались ли Вы когда-нибудь над вопросом: почему мы говорим о емкости именно проводника, но не гово- рим о емкости диэлектрика? Читатель А: Я никогда не думал об этом. Автор: Как Вы определите емкость уединенного проводника? Читатель А: Как количество электричества, которое надо сообщить этому проводнику, чтобы увеличить его потенциал на единицу. А втор: Обратите внимание, что Вы говорите здесь о потенциале как о характеристике тела. Однако до сих пор потенциал рассматривался как характеристика поля, и в качестве таковой он изменялся от точки к точке. Потенциал — функция координат точки поля. Можно ли гово- рить о нем как о характеристике тела? А если можно, то почему? Читатель Б: Это возможно, если тело является проводником. Дело в том, что все точки проводника, помещенного в электростатическое поле, имеют один и тот же потенциал. Проводник представляет собой экви- потенциальное тело. Автор: Чем Вы можете обосновать свое утверждение? Читатель Б: В проводнике есть свободные заряды. Поэтому если бы между какими-нибудь точками в проводнике существовала разность потенциа- лов, то между этими точками должен был бы появиться электрический ток. Это, очевидно, невозможно. Автор: Правильно. Можно сказать, что когда проводник вносится в элект- ростатическое поле, имеющиеся в проводнике свободные заряды пере- распределяются таким образом, что напряженность поля внутри про- водника становится равной нулю. Это и означает, что все точки про- водника (как внутри его, так и на его поверхности) имеют один и тот же потенциал. Постоянство потенциала во всех точках проводника позволяет говорить о потенциале проводника как тела. На рис. 34.1, а показано, как меняется с расстоянием от центра заря- 178
женного металлического шара напряженность электростатического поля, создаваемого этим шаром. Внутри шара напряженность равна нулю. Вне шара она изменяется по закону feg/r2 — как если бы весь заряд шара был сосредоточен в его центре. На поверхности шара (при г = г0) имеем EQ = kq/rQ. На рис. 34.1, б показан график зависимо- сти потенциала (р поля от расстояния г до центра шара. Вне шара по- тенциал изменяется по закону kq/r\ во всех точках внутри шара и на его поверхности потенциал постоянен и равен <р0 = kq/rQ. Замечу, что в диэлектрике свободных зарядов нет, и поэтому указан- ного выше перераспределения зарядов там не происходит. Кстати, как именно перераспределяются свободные заряды в проводнике? Читатель Б: Они сосредоточиваются на его поверхности. Причем тем плот- нее, чем больше кривизна поверхности. Наибольшая плотность зарядов будет на конце острия. Автор: Совершенно верно. Именно поэтому вблизи острия или тонкой проволоки может возникнуть электрический разряд. Такой разряд, обусловленный сильно неоднородным электрическим полем в газе при давлениях порядка 105 Па, называют коронным. Вы встречаетесь с ним всякий раз, когда, проходя под высоковольтной линией электропере- дачи, слышите над головой характерное жужжание. В давние времена коронный разряд пугал моряков, видевших свечение на мачтах корабля (огни святого Эльма). Но вернемся к проводнику в электростатическом поле. Мы уяснили, что он представляет собой эквипотенциальное тело. Отсюда следует, что поверхность проводника должна быть эквипотенциальной поверх- ностью. Используя этот вывод, ответьте на вопрос: как ведут себя сило- вые линии электростатического поля вблизи поверхности проводника? Читатель Б: Так как силовые линии всегда перпендикулярны эквипотен- циальным поверхностям, то они должны подходить к поверхности про- водника под прямыми углами. Автор: К сожалению, экзаменующиеся довольно часто не знают об этом. Нарисуйте картину силовых линий поля плоского конденсатора, внутри которого находится металлический шарик. Читатель Б: Силовые линии должны подходить к пластинкам конденсато- ра и к поверхности шарика под прямыми углами. Поэтому картина силовых линий будет иметь вид, показанный на рис. 34.2. 179
Автор: Все правильно. Мне непонятно, почему некоторые экзаменующиеся считают, что силовые линии должны обходить шарик. А теперь давайте рассмотрим следующую задачу. На расстоянии г от поверхности Земли находится точечный заряд q. По индукции он будет наводить заряд противоположного знака в Земле. В результате появится сила электри- ческого притяжения заряда к Земле. Требуется найти эту силу. Предла- гаю подумать над этой задачей обоим читателям. Читатель А: Заряд, наведенный по индукции в Земле, должен равняться заряду + q. Отсюда следует, что искомая сила равна q2/г2. Читатель Б: Я не согласен с таким заключением. Читатель А предположил, что наведенный в Земле заряд сосредоточен в одной точке (точка А на рис. 34.3, а). Однако в действительности наведенный заряд не сосре- доточен в одной точке, а распределен по поверхности Земли. Поэтому заранее ясно, что искомая сила меньше q2/г2. Автор: Полностью с Вами согласен. Однако как же все-таки найти иско- мую силу притяжения заряда к Земле? Читатель Б: Мне кажется, что надо рассмотреть поле между зарядом и поверхностью Земли. Поверхность Земли есть, очевидно, эквипотен- циальная поверхность, следовательно, вблизи поверхности Земли экви- потенциальные поверхности поля должны быть близки к плоскостям. Вместе с тем вблизи точечного заряда эквипотенциальные поверхности должны иметь форму сфер. Это позволяет нарисовать качественную картину эквипотенциальных поверхностей (точнее, сечений поверхно- стей) . Имея эту картину, я могу по известному правилу провести и сило- вые линии (рис. 34.3, б; силовые линии - сплошные, а сечения поверх- ностей — пунктирные). Автор: Продолжайте свои рассуждения. Не напоминает ли Вам что- нибудь картина силовых линий на рис. 34.3, б? Читатель А: Ах, да, верно. Эта картина схожа с картиной силовых линий двух одинаковых по модулю и разных по знаку точечных зарядов. Я нарисую ее рядом (рис. 34.3, в). Теперь все понятно. В обоих случаях (рис. 34.3, б, в) вид поля вблизи заряда 4~ q один и тот же. В соответ- ствии с (33.4) это означает, что в обоих случаях на заряд + q должна действовать одна и та же сила. Таким образом, искомая сила равна q2/^2). Рис. 31 2 Рис 34 3 180
Автор: Ваши рассуждения правильны. На примере этой задачи хорошо видно, как важно иметь правильное представление об электростатиче- ском поле. ЗАДАЧИ 34.1. Металлический шар радиусом R == 4 см заряжен до потенциала <р0 = 2 В. Определить напряженность и потенциал электростатического поля на расстояниях I ~ 2 см и I = 6 см от центра шара Рассмотреть два случая: 1) шар находится в воздухе, 2) шар помещен в диэлектрик с диэлектрической проницаемостью е = 2. 34.2. Металлический шар радиусом R, = 2 см заряжен зарядом <7, = 2 мкКл. Шар окружен концентрической металлической оболочкой радиусом R, — 6 см, заряд которой qt = — 6 мкКл. Определить напряженность и потенциал поля на следую- щих расстояниях от центра шара- /( = 1 см, /, == 4 см; = 8 см. 34.3. Металлический шар радиусом /? = 2 см, заряженный до потенциала <р0 = 2 В, окружают сферической концентрической оболочкой радиусом R2 = 4 см. Чему станет равен потенциал <р шара, если внешнюю оболочку заземлить? 34.4. Два металлических шара радиусами = 2 см и /?, = 4 см заряжены оди- наковым по модулю и по знаку зарядом q{ = q, = 2 мкКл. Шарики соединяют длинным проводником. Определить количество электричества AQ, которое пройдет при этом по проводнику 35. КАК ВЫ РАССМАТРИВАЕТЕ ДВИЖЕНИЕ В ОДНОРОДНОМ ЭЛЕКТРОСТАТИЧЕСКОМ ПОЛЕ? Автор: Предположим, что тело заряжено и движется в однородном элект- ростатическом поле, т. е. в таком поле, в каждой точке которого напря- женность Ё одна и та же как по модулю, так и по направлению. Приме- ром может служить поле внутри плоского конденсатора. Не усматри- ваете ли Вы общего сходства задачи движения заряженного тела в однородном электростатическом поле с какими-либо задачами, рас- сматривавшимися ранее? Читатель Б: Мне кажется, что существует болыЯое сходство с задачей движения тела в поле сил тяготения, ведь на относительно небольших расстояниях поле сил тяготения Земли может рассматриваться как однородное! Автор: Совершенно верно. А в чем состоит различие между, указанными случаями — движениями в электростатическом поле и в поле сил тя- готения? Читатель Б: Различны силы, действующие на тело. В электростатическом поле на тело действует сила Fe = Eq (она сообщает заряженному телу ускорение ае = Eq/m), а в поле сил тяготения на тело действует сила (j = mg (она сообщает телу ускорение g). Здесь m— масса тела, a q — его электрический заряд. Автор: Хотелось бы, чтобы все экзаменующиеся усвоили простую истину, что движения тела в любом однородном поле кинематически одинако- вы, различие связано только со значением силы, действующей на тело в том или ином поле. Движение заряженного тела в однородном электро- статическом поле имеет такой же характер, что и движение обычного камня в поле тяготения Земли. Рассмотрим задачи, в которых движение тела происходит одновременно в двух полях: сил тяготения и электро- 181
статическом. Тело массой т зарядом Q брошено под углом а к го- ризонту с начальной скоростью и0. Движение тела происходит одновре- менно в поле сил тяготения и однородном электростатическом поле с напряженностью Ё. Силовые линии обоих полей направлены верти- кально вниз (рис. 35.1, а). Найти время дальность полета L{ и максимальную высоту Н{ подъема тела. Читатель Б: На тело действуют две силы: сила тяжести G = mg и электро- статическая сила Ёе = Eq. В данном случае обе силы параллельны. Как и в § 6, я разложу вектор начальной скорости п0 на два направ- ления... Автор (перебивая): Вы хотите повторить ход решения, демонстрировав- шийся в аналогичной задаче в § 6? Читатель Б: Да, хотя бы вкратце. Автор: В этом нет необходимости. Вы можете сразу воспользоваться результатами (6.2) — (6.4). Вообразите, что теперь тело движется в некотором «утяжеленном» поле сил тяготения, характеризующемся суммарным ускорением g + Eq/m. Произведите в соотношениях (6.2) — (6.4) замену + Eq/ri) и Вы сразу же придете к искомым результатам: 2v„ sin а и2 sin 2а sin2 а i _ U 1 __ у гг _ О | — g + Eq/m' g + Eq/m' '~2(q + Eq/m)' Читатель А: Мне непонятно одно место. По сравнению с соответствующей задачей из § 6 в данной задаче на тело действует дополнительная сила Ёе. Эта сила направлена вертикально, поэтому она не должна влиять на горизонтальное перемещение тела. Почему же в данном случае она повлияла на дальность полета £, тела? Автор: Дальность полета зависит от времени полета, а это время опреде- * ляется из рассмотрения вертикального перемещения тела. А теперь не- сколько изменим задачу: пусть силовые линии-электростатического по- ля направлены под углом 0 к вертикали (рис. 35.1, б). Как и прежде, требуется найти время t2 и дальность L2 полета, а также максималь- ную высоту Н2 подъема тела. Читатель А: Сначала я разложу силы Ёе на две составляющие: верти- кальную (Fe cos р) и горизонтальную (Fe sin (J). Данная задача напо- Рис 35 1 182
минает мне задачу с попутным ветром из § 6: в качестве «силы ветра» здесь выступает составляющая Fe sin 0. Автор: Это правильно. Только учтите, что в отличие от упомянутой задачи с попутным ветром здесь будет иная вертикальная сила, а именно: mg Fe cos Р- Читатель А: Я воспользуюсь соотношениями (6.2), (6.3) и (6.6), в которых произведу следующие замены: , .Г’ о / ч F Eq sin р р -+ (g + Eq cos Р m), —------—f-----s-- s v Mg mg + Eq cos p После этого я сразу получаю искомые результаты: 2у0 sin а *2 g + Eq cos P//n’ £ = 8'П 201 Г 1 + fg Sln P) 1 2 g 4- Eq cos p//n [ mg 4- Eq cos p J ’ v2 sin2 а 2 (g 4~ Eq cos P/m) ’ Автор: Все совершенно правильно. К сожалению, экзаменующиеся часто не умеют проводить аналогию между движением в поле сил тяжести и движением в однородном электростатическом поле. Поэтому подобные задачи оказываются для них чрезмерно трудными. Читатель А: Ранее мы таких задач не рассматривали. На эту тему мне встречались только задачи с движением электрона между пластинами плоского конденсатора, причем мы пренебрегали влиянием поля сил тяготения на электрон. Помню, что эти задачи казались мне весьма трудными. Автор: Все эти задачи представляют собой частный случай задачи, изоб- раженной на рис. 35.1, а, поскольку при движении электрона внутри конденсатора можно пренебрегать влиянием поля сил тяготения. Рас- смотрим одну из таких задач. Имея начальную скорость vit электрон влетает в плоский конденсатор под углом oq к его пластинам, а выле- тает из конденсатора под углом oq к пластинам (рис. 35.2). Длина пластин конденсатора равна L. Найти напряженность Е поля конденса- тора и кинетическую энергию элект- рона при вылете его из конденсато- ра. Массу m и заряд q электрона считать известным. Решая задачу, я обозначу скорость электрона, вылетевшего из конден- сатора, через у2. Вдоль пластин электрон летит равномерно, отсюда находим время его движения внут- ри конденсатора: Т = L/{ У] cos eq). Начальная и конечная составляю- щие скорости электрона, перпенди- Рис. 35.2 183
кулярные пластинам, связаны известным кинематическим соотноше- нием для равнозамедленного движения: и2 sin ag = у, sin CZ] —(Fe/tn) Т = и{ sin а, — EqL/{mv{ cos а(). Отсюда, учитывая, что составляющая скорость вдоль пластин не меня- ется cos cq = и2 cos °^)> получаем и, cos а] • tg «2 — U] sin — EqL/(mv{ cos оц). Из этого равенства находим напряженность поля конденсатора: E=(tg — tg otg) mv\ cos2 a.{/(qE). (35.1) Кинетическая энергия электрона после вылета из поля конденсатора равна mv2 mv2 cos2»] 2 2 cos 2a2 (35.2) Все ли понятно в этой задаче? Читатель А: Да, теперь я понимаю, как решаются такие задачи. Автор: Интересны задачи на колебания заряженного маятника, помещен- ного внутрь плоского конденсатора. Вот одна из них. Шарик массой иг с зарядом q подвешен на тонкой нити длиной I внутри плоского кон- денсатора с горизонтально ориентированными пластинами. Напряжен- ность поля конденсатора равна Ё, силовые линии направлены вниз (рис. 35.3, а). Требуется найти период колебаний маятника. Читатель Б: Так как в данном случае силовые линии электростатического Рис. 35.3 184
поля и поля тяготения направлены в одну и ту же сторону, то я могу воспользоваться результатом (14.2) для периода колебаний обычного маятника, заменив в нем ускорение q на арифметическую сумму уско- рений (g + Eq/m). Таким образом, искомый период колебаний T=2it^J-----, L . (35.3) V g + Eq/m Автор: Совершенно верно. Как видите, предложенная задача, по суще- ству, весьма проста, если только уметь пользоваться аналогией между движением в однородном электростатическом поле и движением в поле сил тяготения. Читатель А: По своей структуре формула (35.3) сходна с (15.1). Автор: Вы правильно подметили. Только в (15.1) дополнительное слагае- мое к ускорению g было связано с ускорением системы, в которой рассматривались колебания маятника, тогда как в (35.3) это допол- нительное слагаемое связано с наличием дополнительного взаимодей- ствия. Как изменится формула (35.3), если изменить знак заряда на пластинах конденсатора? Читатель А: В этом случае период колебаний равен Г=2лд/-------. (35.4) V g — Eq/m Автор: Правильно. А что будет с маятником, если в этом случае посте- пенно увеличивать напряженность поля конденсатора? Читатель А: Период колебаний будет возрастать, стремясь к бесконечнос- ти при Е = mg/q. Если продолжать и дальше увеличивать Е, то в этом случае надо закрепить нить не у верхней, а у нижйей пластины конден- сатора. Автор: И какой вид будет иметь формула периода колебаний в этом случае? Автор: Эта формула будет иметь вид Т = 2л "Л /-—1----. (35.5) V Eq/m — g Автор: Хорошо. А теперь усложним задачу: рассмотрим колебания заря- женного шарика внутри конденсатора, пластины которого ориентиро- ваны не горизонтально, а вертикально (рис. 35.3, б). В этом случае ускорения g и Fe/m направлены под прямым углом друг к другу. Как и прежде, требуется найти период Т колебаний маятника, а также угол а, который образует с вертикалью равновесное направление нити маятника. Читатель Б: Рассматривая эту задачу, я учту рассуждения, приведенные в данном параграфе и в § 15. В результате я могу сразу заключить: 1) период колебаний выражается через эффективное ускорение g9(t), являющееся векторной суммой ускорений земного тяготения и электро- статического поля; 2) равновесное направление нити совпадает с на- 185
правлением вектора упомянутого эффективного ускорения (это направ- ление показано на рис. 35.3, б пунктиром). Таким образом, tg а = 2л I ^g2 + (Eq/m)2 Ед mg' (35 6) (35.7) Автор: Совершенно верно. Я думаю, что теперь не представит труда рассмотреть общий случай, когда пластины конденсатора составляют с горизонтом угол р (рис. 35.3, в). Вопрос тот же: найти период коле- баний и угол а, между вертикалью и равновесным направлением нити. Читатель Б: Как и в предыдущем случае, эффективное ускорение явля- ется векторной суммой ускорений свободного падения и электрического поля, а его направление указывает равновесное направление нити маятника. Величину £эф можно найти, используя известную из тригоно- метрии теорему косинуса: £эф = £2 + (Eq/m)2 + 2g (Eq/m) cos p. Таким образом, T = 2л I - - 1 -........ V ^ё2 + (Eq/m)2 + 2g (Eq/m) cos p Значение tg а можно найти следующим образом: £эф х __ (Eq/m) sin р g^ у ~~ g + (Eq/m) cos р’ (35.8) (35.9) Автрр: Вы получили правильный ответ. Очевидно, что при р = 0 он дол- жен приводить к результатам для случая горизонтальных пластин, а при р = 90° — для случая вертикальных пластин. Убедитесь в этом. Читатель Б: Если р = 0, то cos р= 1, a sin р — 0. В этом случае формула (35.8) переходит в (35.3), a tga=0 (равновесное положение нити становится вертикальным). Если р=90°, то cos р = 0, а sin р = 1. В этом случае выражение (35.8) переходите (35.6), а формула (35.9) в (35.7). Автор: Таким образом, задача на колебания заряженного маятника внут- ри плоского конденсатора рассмотрена полностью. В заключение я хочу предложить вопрос: вычислить период колебаний заряженного шарика при условии, что в точке закрепления нити находится еще один точно такой же заряд (рис. 35.4). Никаких конденсаторов здесь нет. Читатель А: По закону Кулона, шарик отталкивается от точки подвеса с силой q2/I2. Эта сила сообщает шарику ускорение q2/(l2m). В фор- муле для периода колебаний надо учесть это ускорение. В результате получим следующее выражение: * g + q^^m)' (35 10) Автор (к читателю Б): Согласны ли Вы с этим результатом? Читатель Б: Нет, не согласен. Для справедливости формулы (35.10) необ- 186
ходимо, чтобы ускорение g2/(/2m) все время было направлено по вертикали вниз. В действительнос- ти оно направлено так только при прохождении маятником положения равновесия. Поэтому ясно, что формула (35.10) во всяком случае неверна. Однако я затрудняюсь указать правильный результат. Автор: Хорошо уже то, что Вы понимаете ошибоч- ность формулы (35.10). В данном случае электри- ческая сила направлена все время вдоль нити, и поэтому она все время уравновешивается силой реакции нити. Отсюда следует, что электрическая сила не приводит к появлению возрастающей силы и, следовательно, не может влиять на период колебаний маятника. Рис. 35 1 Читатель Б: Значит, в данном случае период колебаний описывается фор- мулой (14.12) для незаряженного маятника? Автор: Совершенно верно. В рассматриваемом случае поле электрических сил не является однородным и никаких аналогий с полем сил тяготения здесь проводить нельзя. ЗАДАЧИ 35.1 . Электрон влетает в плоский конденсатор параллельно его пластинам на расстоянии d = 4 см от положительно заряженной пластины; длина пластины / — 15 см. Через какое время t электрон упадет на эту пластину, если напряжен- ность поля конденсатора равна Е = 500 В/м? С какой минимальной скоростью у() должен влетать электрон, чтобы не упасть на пластину? Масса электрона m — 9 • 10~28г, заряде = 1,6 • 10~19Кл. 35.2 . Электрон влетает в плоский конденсатор параллельно его пластинам со скоростью у0=3- 106 м/с. Найти напряженность Е поля конденсатора, если электрон вылетает под углом а = 30° к пластинам. Длина пластины I = 20 см. Масса пг и заряд е электрона известны (см. задачу 35.1). 35.3 . Внутри плоского конденсатора с напряженностью Е поля равномерно вра- щается шарик массой m с зарядом + <7. подвешенный на нити длиной I (рис. 35.5). Угол отклонения нити от вертикали равен а. Найти силу натяжения Т нити и кинетическую энергию шарика. 35.4 . Два шарика массами m, и ш2 с зарядами соответственно + q ( и q2 связаны нитью, перекинутой через неподвижный блок. Вычислить ускорение а шариков и силу натяжения Т нити, если вся система помещена в однородное электростати- ческое поле с напряженностью Е, силовые линии которого направлены вертикально вниз. Взаимодействием между заряженными ша- риками пренебречь. 35.5 . В однородном электростатическом поле с на- пряженностью Е, силовые линии которого направ- лены вертикально вверх, может вращаться в вертикальной плоскости шарик массой m с заря- дом + q на нити длиной I. Какую горизонтальную скорость v надо сообщить шарику в верхнем поло- жении, чтобы сила натяжения нити в нижнем положении в п — 10 раз превосходила силу тя- жести шарика? Рис 35 5 187
36. УМЕЕТЕ ЛИ ВЫ ОБРАЩАТЬСЯ С ЗАКОНОМ КУЛОНА? Автор: Остановимся несколько подробнее на законе Кулона и задачах, связанных с его применением. Прежде всего я попрошу сформулиро- вать этот закон. Читатель Б: Сила взаимодействия двух зарядов пропорциональна произ- ведению этих зарядов и обратно пропорциональна квадрату расстоя- ния между ними. Автор: Ваша формулировка недостаточна. Она неполная. Читатель Б: Наверное, надо добавить, что сила взаимодействия обратно пропорциональна диэлектрической проницаемости е среды? Автор: Это добавление, конечно, не лишнее. Однако главное не в этом. Вы опять забываете, что сила — векторная величина. Поэтому, говоря о модуле силы, не забывайте в то же время указывать и ее направление (вспомните в связи с этим обсуждение второго закона Ньютона в § 5). Читатель А: Я понял. Значит, надо добавить, что сила, с которой взаимо- действуют заряды, направлена вдоль линии, соединяющей эти заряды? Автор: Этого мало. Вдоль линии существуют два направления. Читатель А: Надо сказать, что заряды отталкиваются, если они одинако- вого знака, и притягиваются, если они имеют противоположные знаки. Автор: Правильно. Вот теперь, если Вы соберете все эти добавления, Вы получите полную формулировку закона Кулона. Не мешает еще под- черкнуть, что он относится к взаимодействию точечных зарядов. Читатель А: А можно ли записать формулу закона Кулона так, чтобы в ней содержалась полная информация о законе? Ведь в обычной записи <7(<72 (36 1) er не содержится сведений о направлении силы. Автор: Такую запись закона Кулона дать можно. Для этого надо сначала уточнить, о какой именно силе идет речь. Предположим, что речь идет о силе, с какой заряд q{ действует на заряд q2 (а не наоборот). Введем оси координат с началом, в котором находится заряд q{. Приведем из начала координат вектор г в точку, где находится заряд q2. Этот век- тор называется радиусом-вектором заряда q2. В этом случае полная запись закона Кулона имеет следующий вид: Р = k 7, (36 2) erd где множитель k зависит от выбора системы единиц. Читатель А: Цо в этой формуле сила обратно пропорциональна не квад- рату, а кубу расстояния между зарядами! Автор: Ничего подобного. Ведь вектор г/г по числовому значению равен единице (безразмерной единице!). Его называют единичным вектором. Он служит только для указания направления. Читатель А: Значит, я могу просто написать формулу (36.2), если меня спросят о законе Кулона? И больше ничего не надо? Автор: Надо лишь пояснить смысл обозначений в формуле. Читатель А: А если я напишу не формулу (36.2), а формулу (36.1)? 188
Автор: Тогда Вы должны будете указать, ку- да направлена кулоновская сила. Читатель А: А как формула (36 2) учитыва- ет тот факт, что заряды притягиваются или отталкиваются? Автор: Если заряды одного знака, то произ- ведение qxq2 положительно. В этом слу- чае вектор F параллелен вектору г Р — это сила, приложенная к заряду q2, заряд q2 отталкивается зарядом qx. Если же заряды имеют разные знаки, то произ- ( ведение qxq2 отрицательно, и тогда вектор Р направлен антипараллельно вектору г, т. е. заряд q2 притягивается зарядом qx. Рассмотрим несколько задач на закон Кулона. Задача 1. Четыре одинаковых заряда q размещены в углах квадрата. Какой заряд Q про- тивоположного знака надо поместить в центр квадрата, чтобы вся си- стема заряда находилась в равновесии? Читатель А: В системе пять зарядов — четыре известных и один неизве- стный. Поскольку система находится в равновесии, то сумма сил, при- ложенных к каждому из пяти зарядов, равна нулю. Иначе говоря, надо рассмотреть равновесие каждого из пяти зарядов. Автор: Это лишняя работа. Нетрудно сообразить, что заряд Q независимо от своего значения будет находиться в равновесии вследствие своего геометрического положения. Поэтому условие равновесия для этого заряда ничего не дает. Оставшиеся четыре заряда q из-за симмет- рии квадрата полностью эквивалентны. Поэтому достаточно рассмот- реть равновесие всего лишь одного из этих зарядов, безразлично — какого именно. Выберем, например, заряд в точке А (рис. 36.1). Какие силы действуют на этот заряд? Читатель А: Сила Р{ со стороны заряда в точке Б, сила Р2 со стороны заряда в точке Г и, наконец, сила со стороны искомого заряда, распо- ложенного в центре квадрата. Автор: Простите, а почему Вы не учли заряд в точке В? Читатель А: Но его же «загораживает» заряд в центре квадрата. Автор: Ваша ошибка довольно наивна. Запомните: в системе зарядов на каждый из зарядов действуют силы со стороны всех без исключения остальных зарядов системы, так что следует добавить еще силу Е3, дей- ствующую на заряд в точке А со стороны заряда в точке В. Оконча- тельная картина сил показана на рис. 36.1. Читатель А: Далее все просто Я выберу направление АВ и спроецирую на это направление все силы, приложенные к заряду в точке А. Алгеб- раическая сумма проекций сил равна нулю, т. е. F4 = 2Е, cos 45° + F,. Обозначим через а сторону квадрата, перепишем это равенство в виде <2*7 ?2 , q2 а2/2 а2 2а2 189
Отсюда находим Q =(^/4)(2V2 4-1). Автор: Правильно. А как Вы думаете, будет ли равновесие системы заря- дов устойчивым? Читатель Б: Нет, не будет. Это неустойчивое равновесие. Достаточно одному из зарядов слегка сдвинуться, как все заряды придут в движе- ние и система разрушится. Автор: Вы правы. Оказывается, что вообще невозможно создать устой- чивую равновесную конфигурацию из неподвижных зарядов. Задача 2. Два шарика одинаковых масс и радиусов с одинаковыми зарядами, подвешенные в одной точке на нитях одинаковой длины, опускаются в жидкий диэлектрик, проницаемость которого е и плот- ность р0. Какова должна быть плотность вещества шариков р, чтобы угол расхождения нитей в воздухе и диэлектрике был один и тот же? Читатель Б: Расхождение нитей обусловлено кулоновским отталкиванием шариков. Пусть ТеХ — кулоновская сила отталкивания в воздухе, а ?е2 — в жидком диэлектрике. Автор: Как отличаются эти силы? Читатель Б: Так как по условию задачи угол расхождения нитей в обоих случаях один и тот же, то одинаково и расстояние между шариками. Поэтому различие между силами FeX и Ре2 обусловливается только на- личием диэлектрической проницаемости FeX = ^2. (36.3) Рассмотрим конкретно случай, когда шарики находятся в воздухе. Из равновесия шарика заключаем, что сумма силы FeX и силы тяже- сти б (векторная сумма!) должна быть направлена вдоль нити, так как в противном случае она не сможет быть уравновешена силой реакции нити (рис. 36.2, а). Отсюда следует, что Fei/G =tg а, Рис. 36.2 Рис. 36.3 190
где а — угол между нитью и вертикалью. Когда шарики погружены в диэлектрик, силу Fel надо заменить на силу Ре2, а (j надо заме- нить на разность — ?выт), где Рвыт — выталкивающая сила. Однако отношение новых сил должно по-прежнему равняться tga (рис. 36.2, б). Fei/(G — Лзыт) =tg a. Используя последние два равенства, получаем Подставляя сюда (36.3) и учитывая, что G = Vgp, а FBhl] = Vgp0, находим е _ 1 Р — Р — Ро ’ откуда искомая плотность Р = Рое/(е — !)• Автор: Ваш ответ правильный. Задача 3. Два одинаково заряженных шарика массой m подвешены в одной точке на нитях длиной I каждая. В точке подвеса находится третий шарик, заряженный так же, как и первые два (рис. 36.3). Вычислить заряд q шариков, если угол между нитями в положении равновесия равен а. Читатель Б: Рассмотрим шарик А. К шарику приложены четыре силы (рис. 36.3). Поскольку шарик находится в равновесии, то я разложу эти силы на два направления... Автор (перебивая): В данном случае можно поступить проще. Сила со стороны заряда в точке подвеса не оказывает никакого влияния на равновесное положение нити: сила ?е2 действует вдоль нити и при любом ее положении уравновешивается силой реакции нити. Поэтому данную задачу можно рассматривать так, как если бы заряда в точке подвеса вообще не было. Как правило, экзаменующиеся этого не по- нимают. Читатель Б: В таком случае не будем рассматривать силу fie2. Так как векторная сумма сил Fei и G должна быть направлена вдоль нити, то Fel/G=tg(a/2). (36.4) Автор: Обратите внимание, что результат (36.4) не зависит от нали- чия или отсутствия заряда в точке подвеса. Читатель Б: Так как Fel = q2/[ 4/2 sin2 (a/2)], то из (36.4) получаем q2/{ M2mg sin2 (a/2)] = tg(a/2). Отсюда приходим к искомому результату: q =21 sin (a/2)xl mg tg (a/2). Автор: Этот результат правильный. 191
Читатель А: А в каком случае наличие заряда в точке подвеса оказывается существенным? Автор: Когда, например, требуется найти силу натя- жения нити. ЗАДАЧИ 36.1. В вершинах правильного шестиугольника помешены одинаковые заряды + q. Какой заряд Q следует поместить в центр шестиугольника, чтобы вся система зарядов находи- лась в равновесии? 36.2. Шарик массой m с зарядом q, подвешенный на нити длиной /, вращается около неподвижного заряда, такого же, как и заряд шарика (рис. 36.4). Угол между нитью и вер- тикалью равен а. Найти угловую скорость ю равномерного вращения шарика и силу натяжения Т нити. 36.3. Шарик массой m с зарядом q может вращаться в вертикальной плоскости на нити длиной I. В центре вращения находится второй шарик с зарядом, равным по значению и знаку заряду вращающегося шарика. Какую минимальную гори- зонтальную скорость v надо сообщить шарику в нижнем положении, чтобы он смог сделать полный оборот? 37. ЗНАЕТЕ ЛИ ВЫ ЗАКОН ОМА? Автор: Знаете ли Вы закон Ома? Читатель А: Да, конечно. По-моему, закон Ома знают все. Я считаю, что это самый простой вопрос во всем курсе физики. Автор: Проверим это. На рис. 37.1, а изображен участок электрической цепи. Здесь 7Т— электродвижущая сила, она направлена направо; R\ и R2 — сопротивления; г — внутреннее сопротивление источника ЭДС; Фл и Фй — потенциалы на концах заданного участка цепи. Ток по участку течет слева направо. Требуется найти силу I этого тока. Читатель А: Но ведь у Вас разомкнутая цепь! Автор: Я предложил Вам рассмотреть участок какой-то большой цепи. 192
Электрический ток так глубоко вошел в нашу жизнь, что нет никакой необходимости указывать на важность законов Ома и Джоуля—Ленца. Однако хорошо ли вы знаете эти законы? Всей этой цепи Вы не знаете. Но Вам и не надо ее знать, поскольку заданы потенциалы на концах данного участка. Читатель А: Ранее мы рассматривали только замкнутую электрическую цепь. Для нее закон Ома имеет вид R + r (37.1) Автор: Вы ошибаетесь. Вам приходилось рассматривать также и участок цепи. Согласно закону Ома для участка цепи ток равен отношению напряжения на концах этого участка и его сопротивлению. Читатель А: Но разве это участок цепи? Автор: Конечно. Этот участок цепи изображен на рис. 37.1, б. Для этого участка цепи Вы можете записать закон Ома в виде при этом вместо разности потенциалов концов участка цепи(фл — фв). Вы ранее употребляли более простой термин «напряжение» и обозна- чали U. Читатель А: Однако участок цепи такого вида, как на рис. 37.1, а, мы не рассматривали. Автор: Итак, отметим, что Вы знаете закон Ома для частных случаев замкнутой цепи и простейшего участка, не содержащего ЭДС. Однако для общего случая закон Ома Вам неизвестен. Давайте разбираться вместе. 193
На рис. 37.2, а показано изменение потенциала вдоль заданного участ- ка цепи. Ток течет слева направо, поэтому от Л и В потенциал умень- шается. Падение потенциала на сопротивлении R{ равно 1R{. Далее предположим, что в точках В и Г находятся пластины гальванического элемента. В этих точках происходит скачок потенциала вверх; сумма скачков есть значение ЭДС, равное Между В и Г потенциал па- дает на внутреннем сопротивлении элемента; падение потенциала рав- но 1г. Наконец, от Г до Б потенциал падает на сопротивлении /?2, это падение потенциала равно IR2. Сумма падений потенциала на всех со- противлениях участка минус скачок потенциала вверх, равный и есть разность потенциалов на концах рассматриваемого участка цепи: / ( /?1 + /?2 + г) — — Фл — Фб< откуда получаем выражение для силы тока, т. е. закон Ома для данного участка цепи #+(фл-<рБ) /?, +/?2+г ' (37.3) Обратите внимание на то, что отсюда немедленно получаются знако- мые Вам частные случаи. Для простейшего участка без ЭДС надо положить в (37.3) = О, г = 0. Тогда получаем Фл - Ф5 _ /?, + r2’ что соответствует формуле (37.2). Для замкнутой цепи концы А и Б нашего участка следует соединить. Это означает, что <рл = <рБ. Отсюда получаем / =_____?.____ Rt + R2+r' что соответствует формуле (37.1). Читатель А: Да, я убедился, что не знал закона Ома. Автор: Точнее говоря, Вы его знали для частных случаев. Предположим, что к зажимам элемента на участке цепи, изображенном на рис. 37.1, а подсоединен вольтметр. Будем полагать, что вольтметр имеет доста- точно большое сопротивление, так что можно пренебрегать искаже- ниями, связанными с его подключением. Что покажет вольтметр? Читатель А: Я знаю, что вольтметр, подсоединенный к зажимам элемента, должен показывать падение напряжения во внешней цепи. Однако в данном случае внешняя цепь нам неизвестна. Автор: Оказывается, можно вполне обойтись и без знания внешней цепи. Если вольтметр подсоединен к точкам В и Г, то он будет показывать разность потенциалов между этими точками. Это, надо полагать Вам понятно? Читатель А: Да, конечно. Автор: Теперь посмотрите на рис. 37.2, а. Из него видно, что разность потенциалов между точками В и Г равна (^—/г). Обозначая U показание вольтметра, получаем формулу U—^—Ir. (37.4) 194
Советую пользоваться именно этой формулой, так как здесь не тре- буется знать никаких внешних сопротивлений. Это особенно ценно в случае более или менее усложненных цепей. Заметим, что из (37.4) следует известный частный результат: если цепь разомкнута и, следова- тельно, ток не течет (/ = 0), то U — В этом случае показание вольт- метра совпадает со значением ЭДС. Понятно ли Вам все это? Читатель А: Да, теперь мне это понятно. Автор: В качестве проверки я предложу Вам вопрос, на который экзаме- нующиеся довольно часто затрудняются дать ответ. Замкнутая цепь со- стоит из п последовательно соединенных одинаковых элементов с ЭДС % и внутренними сопротивлениями г. Сопротивление соединительных проводов полагают равным нулю. Что покажет вольтметр, подсоединен- ный к зажимам одного из элементов? Как обычно предполагается, что ток через вольтметр не идет. Читатель А: Я буду рассуждать аналогично предыдущему объяснению. Вольтметр покажет U = — 1г. Из закона Ома для данной замкнутой цепи находим силу тока: / = п^Дпг) — ^/г. Используя этот результат, получаем (^/г) г = 0. Таким образом, вольтметр ничего не покажет. Автор: Совершенно верно. Только помните, что эта ситуация была идеа- лизированной: с одной стороны, мы пренебрегали сопротивлением со- единительных проводов, а с другой стороны, полагали сопротивление вольтметра бесконечно большим, так что не пробуйте проверять этот результат на опыте. Рассмотрим случай, когда ток на участке цепи и ЭДС участка направ- лены не в одну сторону, а в разные (рис. 37.1, в). Изобразите изме- нение потенциала вдоль такого участка. Читатель А: А разве может ток течь навстречу ЭДС? Автор: Вы забываете, что перед Вами всего лишь участок цепи. В этой це- пи могут быть другие ЭДС, не входящие в рассматриваемый участок, под действием которых ток по данному участку может течь и навстречу данной ЭДС. Читатель А: Понимаю. Поскольку ток течет слева направо, то от А до В происходит падение потенциала на //?[. Так как ЭДС направлена те- перь в другую сторону, то скачки потенциала в точках В и Г должны не повышать, а, напротив, уменьшать потенциал. От точки В до точки Г потенциал падает на 1г; от точки Г до точки Б — на IR2. В результате приходим к графику, изображенному на рис. 37.2, б. Автор: Как же запишется теперь закон Ома? Читатель А: Он будет иметь вид Автор: Правильно. А каково показание вольтметра? Читатель А: Из рис. 37.2, б видно, что в этом случае (37.6) Автор: Совершенно верно. Рассмотрим следующую задачу. Схема элект- рической цепи изображена на рис. 37.3. Дано: г = 1 Ом, R = 10 Ом, со- 195
противление вольтметра Rv = 200 Ом. Вычислить относительную погрешность показаний вольт- метра, которая получается в предположении, что вольтметр имеет бесконечо большое сопро- тивление и, следовательно, не вносит искажений в измеряемую цепь. Обозначим показание реального вольтметра через U, а вольтметра с бесконечно большим - сопро- Рис. 37.3 тивлением — через иж. Искомая относительная по- грешность f^(Ux-U)/Ux=l-U/Ux. (37.7) Далее учтем, что U°° = ~r^7R' (37.8) _ s RRV U ~ г+ /?/?/(/? + R + R/ (37’9) Подставляя формулы (37.8) и (37.9) в (37.7), получаем Rv(R + r) Rv(R + r) i ' (R + Rv}r'+ RRV (/+R) Ry+rR T+r/?/[(r + /?)/?v] ’ Так как Rv^> R и R > г, то дробь в знаменателе последнего равенства много меньше единицы. Поэтому можно воспользоваться приближен- ной формулой, которую всегда полезно иметь в виду: (Ц-Х)“« 1+аХ, (37.10) справедливой при X <С 1 для любого а (целого и дробного, положи- тельного и отрицательного). Воспользуемся приближенной формулой (37.10), полагая в ней а = — 1 и 1 — rR (г 4~ R)~ 1 Ry 1. В результате получим f = rR/[ (г + R) Rv]. (37.11) Рис. 37.4 196
Подставляя в (37.11) числовые значений величин, данные в условии задачи, получаем, что искомая погрешность f ж 1 /220 = 0,0045. Читатель А: Значит, чем больше сопротивление вольтметра по сравнению с внешним сопротивлением, тем меньше указанная погрешность? Зна- чит, с тем большим основанием можно пренебрегать искажением цепи в результате подключения вольтметра? Автор: Да, это так. Только надо учесть, что неравенство R <С Rv есть достаточное, но не необходимое условие малости погрешности f. Из (37.11) видно, что погрешность f мала также и в случае, когда вы- полняется условие r<^iRv, т. е. сопротивление вольтметра много больше внутреннего сопротивления источника тока. При этом внешнее сопро- тивление может быть как угодно велико. Задача: Электрическая цепь изображена на рис. 37.4, а. Дано: 8Г= 6 В, г=2/з Ом, R = 2 Ом. Вычислить показание вольтметра. Читатель А: Сопротивление вольтметра можно считать бесконечно большим? Автор: Да, тем более что это сопротивление не дано в условии. Кроме того, следует пренебрегать сопротивлением проводов. Читатель А: Но тогда ток, наверное, не потечет через сопротивления в середине схемы, а пойдет прямо по участкам А^2 и Б1Б2? Автор: Вы неправы. Прежде чем рассматривать токи, я бы советовал несколько упростить схему. Поскольку участки AtA2 и BtB2 не имеют сопротивления, то отсюда следует, что <рЛ1 = <рЛ2 и Фвч = Фвг- Далее можно воспользоваться правилом: если в схеме две какие-то точки имеют одинаковые потенциалы, то можно свести эти точки одну к дру- гой, при этом токи через сопротивления останутся прежними. Приме- ним это правило к нашему случаю: совместим точку А} с точкой А2, точку Б{ с точкой Б2. В результате мы получим схему, изоб- раженную на рис. 37.4, б. А эту схему рассмотреть уже нетрудно. Поэтому я сразу приведу окончательный ответ: вольтметр покажет 4 В. Предоставляю Вам самостоятельно на досуге проделать выкладки. 197
ЗАДАЧИ 37.1. Амперметр включен в участок цепи (рис. 37.5) и показывает /, = 0,5 А. Найти силу /4 тока через сопротивление R4, если /?, = /?4 = 2 Ом, /?2= 4 Ом, /?3=/?5= 1 Ом 37.2. Электрическая цепь изображена на рис. 37.6. Дано: ^=4 В, г=1 Ом, R = 2 Ом Определить показание амперметра. 37.3. Что покажут вольтметр и амперметр в случаях, изображенных на рис. 37.7, а, б? Полагать известными ^иг. Сопротивлением соединительных проводов пренеб- речь. Нарисовать для указанных случаев графики изменения потенциала вдоль цепи 37.4. Сопротивление гальванометра равно /?г= 0,2 Ом. Параллельно с ним подклю- чено шунтирующее сопротивление /?ш = 0,05 Ом Какое добавочное сопротивле- ние Rx надо подключить последовательно с этой комбинацией, чтобы в целом получилось сопротивление, равное Rr? 37.5. К зажимам элементов с = 10 В и г = 1 Ом подсоединили вольтметр со- противлением Rv— 100 Ом Определить показание U вольтметра и вычислить относительную погрешность f его показания, которая получается в предположении, что вольтметр имеет бесконечно большое сопротивление. 37.6. В цепь с сопротивлением R — 49 Ом и с источником тока с Ъ — 10 В и г — 1 Ом включили амперметр с сопротивлением R, = 1 Ом Определить показание I ампер- метра и вычислить относительную погрешность f его показания, которая полу- чается в предположении, что амперметр не имеет сопротивления. 37.7. Три источника тока соединены, как показано на рис. 37.8. Определить токи, протекающие через сопротивление R и через источники, если известно, что «Г, = 1,3 В, ^2 = 1,5 В, = 2 В, г, = г2 = г3= 0,2 Ом, R - 0,55Ом. 37.8. Два элемента с ?, = 2 В и = 3 В соединены одинаковыми полюсами. Элементы замкнуты на внешнее сопротивление R = 2 Ом. Определить силы токов, протекающих через сопротивление и через элементы. Рассмотреть три случая: 1) внутренние сопротивления элементов равны г, = r2 = 1 Ом; 2) внутренние со- противления элементов различны и равны гх = 0,5 Ом, r2 = 1 Ом; 3) внутренними сопротивлениями элементов пренебречь. 38. МОЖНО ЛИ ВКЛЮЧАТЬ КОНДЕНСАТОР В ЦЕПЬ ПОСТОЯННОГО ТОКА? Автор: Рассмотрим следующую задачу. Дана цепь, изображенная на рис. 38.1; С — емкость конденсатора. Найти заряд Q на обкладках конденсатора, если ЭДС источника тока равна <9 и его внутреннее сопротивление г. Рис 37 7 Рис. 37.8 198
Читатель А: А разве можно в цепь с постоянным током включать кон- денсатор? Ведь ток через него все равно не пойдет. Автор: Ну и пусть. Зато по параллельным ветвям пойдет. Читатель А: Я, кажется, понял. Поскольку в схеме на рис. 38.1 ток через конденсатор не идет, то, следовательно, он не идет и через сопротивление . Во внешней части цепи ток идет только через сопро- тивление /?2. Силу тока находим из соотношения (37.1) / = ^/( /?2 -f- г), тогда разность потенциалов между точками А и Б равна падению напряжения на сопротивлении /?2, т. е. Фб ~ Фл = //?2 = &Я2/(#2 + г). (38.1) Заряд на конденсаторе определяется по формуле Q = CU, (38.2) где U — напряжение на обкладках конденсатора, С — его электро- емкость. Следовательно, чтобы найти заряд, я должен знать разность потенциалов между точками А и Г, но она мне неизвестна. Автор: Верно, что через сопротивление R{ ток не идет. Но в таком случае все точки этого сопротивления должны иметь один и тот же по- тенциал (вспомните обсуждения, проводившиеся в § 34). Значит, Фг — Фб- Отсюда, используя (38.1) и (38.2), получаем искомый ре-' зультат: Рассмотрим следующую задачу. Электрическая цепь изображена на рис. 38.2. Дано: 8Г=4В, г = 1 Ом, R{ = 3 Ом, R2 = 2 Ом, С\ = 2 мкФ, С2 = 8 мкФ, С3=4 мкФ, С4==6 мкФ. Найти заряд на обкладках каждого конденсатора. В связи с этим вспомните правила сложения емкостей при их парал- лельном и последовательном соединениях. Читатель Б: При параллельном соединении конденсаторов емкости скла- дываются, т. е. +С2 + С3 + ..., (38.3) тогда как при последовательном соединении конденсаторов складыва- ваются величины, обратные емкостям: 199
с с, + с2 + cd + (38 4) Читатель А: А нельзя ли пояснить, как эти соотношения получаются. Читатель Б: У параллельно соединенных конденсаторов напряжение на их обкладках одинаково; общий заряд равен сумме зарядов на каждом из конденсаторов, причем в соответствии с (38.2) заряд на каждом кон- денсаторе пропорционален его емкости. Отсюда следует, что UC — — UC\ UC2 4* UC6 + ... • Мы приходим к формуле (38.3). Читатель А: Мне непонятно, как можно складывать заряды конденсато- ров. Ведь одна из обкладок заряжена положительно, тогда как другая отрицательно, так что суммарный заряд равен нулю. Читатель Б: По-моему надо складывать только положительные заряды. Автор: Конденсатор — это система из двух изолированных проводников, заряженных разноименными и равными по модулю зарядами. В соотно- шение (38.2) входит модуль заряда. При параллельном соединении конденсаторов соединены их одноименные обкладки, следовательно, на эквивалентной емкости заряд равен сумме зарядов (на положитель- ной — сумме положительных, на отрицательной — сумме отрица- тельных) . Читатель А: А как быть в случае последовательного соединения конден- саторов? Автор: При последовательном соединении на всех конденсаторах уста- навливается одинаковый по модулю заряд, так как сумма зарядов пластин, соединенных друг с другом, равна нулю: потенциал этих пластин одинаков. Пластины 2 р 3 на рис. 38.3 имеют одинаковые по модулю, но разноименные заряды, при этом <р2 = <р3. Для замены эквивалентной емкостью необходимо сложить напряжения, которые обратно пропорциональны емкостям: = _2_ д_ _2_д_ С С{ • Отсюда получается результат (38.4). Читатель Б: Я попробую решить сформулированную выше задачу. Ис- пользуя правило (38.3), находим эквивалентную емкость между точ- ками А и Б и между точками В и Г: САБ = 2 мкФ 4- 8 мкФ = 10 мкФ, СБГ = 4 мкФ 4* 6 мкФ = 10 мкФ. Разность потенциалов между точками А и Г находим как падение напряжения на сопротивлении R{: ER Чг ~ Фл = /7?i = 4~г = 3 В- Ясно, что сопротивление R2 здесь не играет никакой роли; его можно вообще не принимать во внимание. Поскольку САБ — Свг и заряды на эквивалентных емкостях одинаковы (они соединены последовательно) QAB = QBr, то одинаковы и напряже- ния на этих емкостях 200
Рис. 38.3 Рис. 38.4 Фб — Фд = Фг ~ Фв = U5B. Далее находим искомые заряды: Qi = G (фб — <Рл) = ЗмкКл, Q2 = С2(фй — <рл) = 12 мкКл, Q3 — (Фг ~ Фе) т= 6 мкКл, <?4 = С4 (фг — <рв) = 9 мкКл, Автор: Все правильно. Рассмотрим следующую задачу. Два конденсатора с емкостями С! и С2 присоединены к двум источникам с и ^2 (рис. 38.4). Определить напряжение на каждом конденсаторе и раз- ность потенциалов между точками а и б. Читатель Б: Конденсаторы соединены последовательно, их эквивалентная емкость равна С = С} С2/(С}-]-С2'). Заряд на каждом конденсаторе оди- наков и равен заряду на эквивалентной емкости Q — С (+ ЗГ2). Напряжение на каждом конденсаторе можно определить, используя (38.2): +^2) ct+c2 ' W +^2) С,+С2 • (38.5) (38.6) Разность потенциалов между точками авб равна фа — фб = U2 -|- ^2. Автор: Ваш последний вывод неверен. Заменим цепь на рис. 38.4 (вернее ее участок асб) эквивалентной схемой (рис. 38.5). В соответствии с замечаниями, сделанными в предыдущем параграфе, можно записать Фа Фб = ^2 ^2 (38.7) (здесь падение напряжения на сопротивлении заменено падением на- пряжения на конденсаторе). Читатель Б: Я понял свою ошибку: я не учел направления ЭДС. Мне кажется, что эту задачу можно решить, не вводя эквивалентных схем. Обозначим напряжение на конденсаторе фа — фс = U2— Uc2. С другой 201
стороны, разность потенциалов между точками бис равна <рб—ф(. = #2; из этих двух соотношений получим <ра — <рб = = 172 *^2- Автор: Все правильно. Вот еще один способ: ис’ ’ мысленно включим между точками а и б вольтметр. Мы знаем (см. § 37), что по- казания вольтметра V равны электродвижущей силе Е источника минус падение напряжения на заданном участке цепи U, но V = U2, U = — Фа — Ф2’ ~ Приходим к уже полученному результату (38.7). Используя (38.6) и (38.7), находим % - % С, +С2 ’ ЗАДАЧИ 38.1. Вычислить полную емкость С системы конденсаторов, изображенной на рис. 38.6. Принять: С, = 2 мкФ, С2 = 4 мкФ. 38.2. Электрическая цепь изображена на рис. 38.7. Дано: ^=5 В, г = 1 Ом, R2 = 4 Ом, = 3 Ом, С == 3 мкФ. Найти заряд Q на обкладках каждого конден- сатора. 38.3. Дана электрическая цепь, изображенная на рис. 38.8. Считая известными все величины, показанные на рисунке, найти заряд Q на пластинах каждого кон- денсатора. 38.4. Плоский конденсатор с пластинами длиной I включен в цепь, как показано на рис. 38.9. Даны ЭДС Я? источника тока, его внутреннее сопротивление г и расстояние d между пластинами. В конденсатор параллельно пластинам влетает электрон со скоростью vQ. Какое сопротивление R надо подсоединить параллельно конденсатору, чтобы электрон вылетел из него под углом а к пластинам? Массу m и заряд е электрона считать известными. 38.5. Два одинаковых плоских конденсатора включены в цепь, как показано на рис. 38.10; = 4 Ом. В один из конденсаторов параллельно пластинам влетает электрон (стрелка на рисунке). При каком сопротивлении /?2 электрон вылетает из второго конденсатора параллельно пластинам? 38.6. Плоский конденсатор с пластинами длиной I и расстоянием между ними d включен в цепь, как показано на рис. 38.11 (ЭДС ЯГ и сопротивления R и г из- вестны). В конденсатор параллельно пластинам со скоростью у0 влетает электрон. Под каким углом а к пластинам вылетит электрон из конденсатора? Массу m и заряд е электрона считать известными. 202
38.7. Определить заряд батареи конденсаторов, изображенной на рис. 38.12, если к клеммам АВ приложено напряжение £7= 100 В, а емкости конденсаторов С = 2 мкФ, CQ = 1 мкФ. 38.8. Определить заряды конденсаторов емкостью С = 4 мкФ и CQ = 2 мкФ (рис. 38.13). Известно: /?( = 100 Ом, R2 = 300 Ом, = 15 В, /2 = 5 В. Внут- ренним сопротивлением источников пренебречь. 39. УМЕЕТЕ ЛИ ВЫ ВЫЧИСЛЯТЬ СОПРОТИВЛЕНИЕ РАЗВЕТВЛЕННОГО УЧАСТКА ЦЕПИ? Автор: Вычислите сопротивление участка, изображенного на рис. 39.1, а. Сопротивлением проводов можно пренебречь. Читатель А: Если сопротивлением проводов можно пренебречь, то эти про- вода можно совсем не принимать во внимание. Тогда искомое сопротив- ление участка равно 3/?. Автор: Вы ответили, не подумав. Пренебрегать сопротивлением прово- дов и пренебрегать проводами — это совершенно разные вещи (хотя некоторые экзаменующиеся полагают, что это одно и то же). Выки- нуть из схемы какой-нибудь провод — это значит заменить этот провод •бесконечно большим сопротивлением. Здесь же, наоборот, сопротивле- ние проводов равно нулю. Читатель А: Да, верно, я просто не подумал. Я буду рассуждать сле- дующим образом. У точки А ток разделится на два тока, направления которых я показал стрелками (рис. 39.1, б). В этом случае среднее сопротивление можно вообще не учитывать, так что полное сопротив- ление участка равно R/2. Автор: И этот Ваш ответ неверен. Я советую пользоваться следующим правилом: найдите на схеме точки с одинаковым потенциалом и сделай- те так, чтобы эти точки совместились друг с другом. При этом токи в разных ветвях схемы останутся прежними, а сама схема может суще- ственно упроститься. На это обстоятельство я уже указывал Вам в § 37. Поскольку в данной задаче сопротивления проводов равны нулю, то точки А и А{ имеют один и тот же потенциал. Точно так же имеют одинаковый потенциал точки Б и Б,. Учитывая это, будем видоизме- нять схему так, чтобы точки с одинаковым потенциалом в конце концов совместились друг с другом. Для этого будем постепенно сокращать длину соединительных проводов. Последовательные стадии этой опера- Рис. 38.9 203
Рис. 38.11 ции показаны на рис. 39.1, в [(1) ->(2) ->(3) ->(4)]. В результате обна- руживаем, что данное соединение соответствует параллельному соеди- нению трех сопротивлений, так что полное сопротивление участка рав- но 7?/3. Читатель А: Да, действительно. Из рис. 39.1, в хорошо видно, что это есть параллельное соединение. Автор: Перейдем к следующему примеру. Дан куб из проволочек, каждая из которых имеет сопротивление R (рис. 39.2, а). Куб включен в цепь, как показано на рисунке. Вычислить полное сопротивление куба. Применим указанное ранее правило. Найдите точки, имеющие одинако- вый потенциал. Читатель А: Я думаю, что одинаковый потенциал имеют три точки: А, At, Д2 (рис. 39.2, а), поскольку все три ребра куба (ГА, ГА,, ГА2) совершенно эквивалентны. Автор: Вы правы. Точно так же эквивалентны ребра БВ, БВ,, БВ2. Поэто- му точки В, В1( В2 тоже имеют одинаковый потенциал. Далее давайте порвем наш проволочный куб во всех указанных точках, согнем ребра- проволочки и соединим их так, чтобы точки с одинаковым потенциалом оказались совмещенными друг с другом. Какая картина получится при этом? Читатель А: Получится картина, изображенная на рис. 39.2, б. Автор: Совершенно верно. Показанная на рис. 39.2, б схема эквивалентна исходной схеме (кубу), но заметно проще последней. Теперь нетрудно вычислить полное сопротивление. Читатель А: Оно равно ₽/3 + R/& + ₽/3 = 5Я/6. Рис. 38.12 204
а) б) Рис. 39.1 Читатель Б: А как найти полное сопротивление проволочной фигуры в виде квадрата с диагоналями, включенной в цепь, как показано на рис. 39.3, а? Автор: Надо поискать точки с одинаковым потенциалом. В данном случае нетрудно подметить, что схема обладает осью симметрии. Я изображу ее на рис. 39.3, а пунктиром. Ясно, что все точки, лежащие на оси симметрии, должны иметь один и тот же потенциал, равный полусумме потенциалов точек А и Г. Таким образом, потенциалы точек О, и 02 равны друг другу. В соответствии с известным правилом можно совме- стить эти три точки друг с другом, в результате чего рассматриваемая комбинация сопротивлений разобьется на два последовательно соеди- ненных одинаковых участка, один из которых показан на рис. 39.3, б. Сопротивление такого участка определить нетрудно. Если каждая из проволочек в квадрате будет иметь одно и то же сопротивление R, то получим, что указанный участок должен иметь сопротивление 4/i5 R- Таким образом, искомое сопротивление равно 8/i5 R. 205
Рис 39.3 Читатель А: Значит, основное правило со- стоит в том, чтобы отыскать на схеме точки с одинаковым потенциалом и в результате совмещения этих точек упростить схему? Автор: Именно так. В заключение я хочу предложить один пример с бесконеч- ным участком. Дана цепь, составлен- ная из бесконечного числа повторяю- щихся секций с сопротивлениями R\ и /?2 (рис. 39.4, а). Найти полное сопро- тивление между точками А и Б. Читатель А: Может быть, здесь следует воспользоваться методом математиче- ской индукции? Рассмотрим сначала одну секцию, затем две, три и т. д. А потом попробуем обобщить результат для п секций на случай п -+ оо. Автор: Нет, здесь не нужен метод матема- тической индукции. Воспользуемся тем, что если от бесконечности отнять один элемент, бесконечность от этого не изменится. Отрежем от рассматриваемой схемы первую секцию (разрез произведем по пунктиру на рис. 39.4, а). Очевидно, по-прежне- му останется бесконечное число секций, так что сопротивление между точками В и Г будет равно искомому сопротивлению R. Таким образом, исходная схема имеет вид, показанный на рис. 39.4, б. Участок цепи на рис. 39.4, б имеет сопротивление R} 4- RR^/{R -f- R2). Так как этот участок эквивалентен исходной схеме, то его сопротивление равно иско- мому сопротивлению R. Таким образом, +/?/?2/(/? + Я2), т. е. получаем квадратное уравнение относительно R: r2 -ед -R.R2 -0. Решая его, находим R =(/?, /2) [ 1 +V 1 + 4(/?2//?1)] . 206
Читатель А: Такой метод решения действительно очень интересен. Автор: В заключение рассмотрим еще одну интересную задачу; между точками А и Б присоединено сопротивление R; кроме того, имеются еще (N — 2) точек, причем между каждой парой точек, включая сюда точки А и Б, также присоединено сопротивление R. Найти резуль- тирующее сопротивление между точками А и Б. Читатель Б' А я имею право разместить эти N — 2 точек на прямой линии, относительно которой точки А и Б симметричны? Автор: Конечно. Важно лишь не порвать при этом сопротивлений, при- соединенных к точкам. Читатель Б: Но тогда системы сопротивлений слева и справа от такой линии будут одинаковыми. А это означает, что все N — 2 точек имеют одинаковый потенциал и поэтому могут быть совмещены друг с другом. Значит, искомое сопротивление равно/?! = 2R/(N — 2). Автор: Хорошо, но вы забыли о сопротивлении R между точками А и Б. Оно присоединено параллельно найденному Вами R{. Поэтому искомое сопротивление равно R/(Rt -+-/?) = 2R/N. ЗАДАЧИ 39.1. Электрическая цепь изображена на рис. 39.5. Дано: ? = 4 В, г = I Ом, R = 45 Ом. Определить показания вольтметра и амперметра. 39.2. Найти сопротивление проволочного квадрата, изображенного на рис. 39.3, а, при условии, что он включен в цепь в точках А и В. Рис. 39.6 207
39.3. Из проволочек сделан правильный шестиугольник с диагоналями. Сопротив- ление каждой проволочки равно /?. Шестиугольник включен в цепь, как показано на рис 39.6, а. Найти его полное сопротивление. 39.4. Найти полное сопротивление шестиугольника, данного в задаче 39.3, при условии, что он включен в цепь, как показано на рис. 39.6, б. 39.5. Вычислить полное сопротивление шестиугольника, данного в задаче 39.3, при условии, что он включен в цепь, как показано на рис. 39.6, в. 39.6. Определить полное сопротивление цепи, показанной на рис. 39.7, если на- пряжение приложено к точкам А и Б. 40. ОТЧЕГО ПЕРЕГОРЕЛА ЛАМПОЧКА? Читатель А: Отчего перегорает электрическая лампочка — от большого напряжения или от большой силы тока? Автор: А как Вы думаете? Читатель А: Мне кажется, что от большого тока. Автор: Ваш ответ нехорош. Прежде всего замечу, что лампочка перего- рает в результате выделения чрезмерно большого количества теплоты в единицу времени, т. е., иначе говоря, в результате резкого увеличения мощности тока. Увеличение же мощности тока может быть следствием изменения различных факторов: напряжения, подаваемого на лампоч- ку, тока через лампочку, сопротивления лампочки. В связи с этим да- вайте вспомним все известные Вам формулы для мощности, выделяе- мой на некотором сопротивлении R. Читатель Б: Мне известны следующие формулы: А = (<₽! — ф2)/, (40.1) А = I2R, (40.2) А = (Ф1 — ф2)2//?, (40.3) где А — мощность, выделяемая на сопротивлении R; (Ф1 — ф2) — раз- ность потенциалов на его концах; I — сила тока через это сопротив- ление. Читатель А: Мы обычно пользовались только формулой (40.2), выражаю- щей мощность через квадрат силы тока и сопротивление. Автор: Легко видеть, что все три формулы эквивалентны, так как одна переходит в другую при применении закона Ома. Эквивалентность этих формул как раз и указывает на то, что при решении вопроса о перегора- нии лампочки не следует рассматривать по отдельности ток или напря- жение, а надо рассматривать в совокупности все три величины: и силу тока, и напряжение, и сопротивление. (R читателю А.) Кстати говоря, почему Вы отдаете предпочтение именно формуле (40.2)? Читатель А: Но ведь обычно напряжение, подводимое к лампочке, посто- янно. Поэтому зависимость мощности от напряжения не представляет интереса. Формула (40.2) является наиболее «рабочей». Автор: Вы неправы, ставя формулу (40.2) в привилегированное поло- жение. Давайте рассмотрим задачу. Электроплитка имеет три секции одинакового сопротивления R. Если все три секции соединены, парал- лельно, вода в чайнике закипает через tQ — 6 мин. Через какое время за- 208
кипит вода той же массы в чайнике при различных соединениях секций электроплитки (рис. 40.1)? Читатель А: Прежде всего найдем полное сопротивление электроплит- ки для каждого варианта соедине- ний секций. В исходном случае (параллельное соединение) полное сопротивление RQ = R/3. В случаях а, б, в (рис. 40.1) соответственно имеем: Ra = 3R, R6 = R + (R/ty = 3R/2, Rg = 2R2/(3R) = 2R/3. a) (40.4) Если обозначить через U напряжение, подводимое к электроплитке, то, пользуясь законом Ома, найдем силу полного тока через электро- плитку в каждом случае... Автор (перебивая): Не надо находить силу тока. Обозначим через t0, ta, t6, te время, требуемое для нагревания воды в чайнике в каждом из рассматриваемых случаев. Выделяемая теплота равна произведению мощности тока на времязнагревания. В каждом из указанных случаев эта теплота одна и та же. Используя для мощности тока формулу (40.3), получаем t/2/0//?0 = U\/Ra = U2t6/Re = Ub./R.. (40.5) Подставляя в (40.5) соотношения (40.4) и сокращая затем общие мно- жители ( U2 и 1 /R), находим 3/0 = ta/3 — 2/б/3 = 3/e/2. Отсюда сразу получаем значения искомых величин: ta = 9t0 = 54 мин, /б = 9/0/2 = 27 мин, tg — 2tQ — 12 мин. Замечу, что в данной задаче было удобно воспользоваться формулой (40.3) для мощности именно потому, что напряжение, подводимое к электроплитке, есть постоянная вели- чина. Далее рассмотрим следующий вопрос. Дан источник тока с ЭДС S и внутренним сопротивлением г, замкнутый на некоторое внешнее сопро- тивление R. Чему равен коэффициент полезного действия источника? Читатель Б: КПД источника тока есть отношение полезной мощности (т. е. мощности, выделяемой на внешнем сопротивлении) ко всей мощ- ности (т. е. сумме мощностей, выделяемых на внешнем и внутреннем сопротивлениях); П = I2R/[ I2 (R + г)] = R/(R + г). (40.6) Автор: Правильно. Предположим, что внутреннее сопротивление источ- ника постоянно, а изменяется только внешнее сопротивление. Дак будет изменяться при этом КПД источника тока? Читатель Б: При R — 0 (случай короткого замыкания) т| = 0. При R = г ц = 0,5. При бесконечном возрастании R КПД стремится к единице. 209
Автор: Совершенно верно. А как при этом изменяется полезная мощность (мощность, выделяемая на внешнем сопротивлении)? Читатель Б: Поскольку при возрастании R увеличивается КПД источника, то, следовательно, будет увеличиваться и полезная мощность. Короче говоря, чем больше R, тем больше полезная мощность. Автор: Вы неправы. Увеличение КПД источника тока означает, что уве- личивается отношение полезной мощности ко всей мощности источни- ка, а полезная мощность может при этом уменьшаться. В самом деле, полезная мощность У2 _ У2 х (/? + г)2 Г (х+1)2’ (40.7) где x—R/r. Если х<С 1, то Nn ~х. Если х 1, то Nn ~ 1/х. Максималь- ное значение Nn принимает при х= 1 (R = г), при этом Nn — ^2/(4г). На рис. 40.2 показан график функции у = х/(х -р I)2, иллюстрирующий изменение полезной мощности по мере возрастания внешнего сопротивг ления. Читатель Б: Я понял. А можно определить максимальное значение мощ- ности, исследуя функцию у=х (х-|-1)2 на максимум через производную? Автор: Конечно, только надо хорошо знать правила дифференцирования. Читатель Б: Производная равна у1 =[ (х I)2 —- 2 (х 1)] /(х + I)4. При- равнивая ее" нулю, получаем х= 1. Автор: Все правильно. Рассмотрим следующую задачу: двести параллель- но соединенных одинаковых ламп сопротивлением R — 300 Ом каждая подсоединены к источнику тока с ЭДС 100 В и внутренним сопро- тивлением г = 0,5 Ом. Вычислить мощность, выделяемую на каждой лампе, и относительное изменение мощности, выделяемой на лампе, если одна из двухсот ламп перегорела. Сопротивлением соединитель- ных проводов пренебречь (рис. 40.3). Читатель Б: Сила полного тока через внешний участок цепи равна /п = Ъ/(г + R/ri) = 50 А. Сила тока через одну лампу равна / = 1п/п = — 0,25 А. Отсюда находим мощность, выделяемую на одной лампе: N — I2R = 18,75 Вт. Чтобы вычислить относительное изменение мощ- 210
ности, выделяемой на лампе, если одна из двухсот ламп перегорела, я найду сначала мощность на одной лампе для п = 199, а затем рассчитаю отношение / = (#,-#)/#. (40.8) Автор: Я не одобряю такого пути вычисления искомой величины f. Ее надо выразить в общем виде через сопротивления /? и г и через число ламп п. Запишем сначала мощности: «2 (г + /?/н)2 N =—«______________. ' (П-1)2 [г + /?/(п- I)]2 Подставляя эти выражения в (40.8), получаем /«/Л- Л./ " V _ ! =Г ________!______У - | ' \W / \ nr — г + R / [ 1 — г/(иг-4-Z?) J Дробь в знаменателе последнего равенства много меньше единицы (вследствие того, что в цепи много ламп и сопротивление лампы много больше внутреннего сопротивления источника тока). Поэтому приме- ним приближенную формулу (37.10): / \ 2 9 г f = (l------г--) - 1 « ——------. (40.9) 1 \ nr + R} nr + R V ’ Подставляя в (40.9) числовые значения из условия задачи, находим f « 0,0025.' Читатель Б: А почему Вы возражаете против того, чтобы сначала вы- числить jVt, а затем, подставив числовые значения в (40.8), получить /? Автор: Вы видите, что f — 0,0025. Значит, чтобы получить этот резуль- тат по Вашему (числовому) методу, надо рассчитать значение с точностью до четырех знаков. Заранее Вы даже не знаете, со сколь высокой точностью Вам требуется рассчитывать ДО,. Если бы в данном случае Вы стали считать с точностью до двух знаков, то пришли бы к выводу, что мощность ДО, совпадает с мощностью N. ЗАДАЧИ 40.1. Электрическая цепь показана на рис. 40.4. Дано: У = 100В, г = 36Ом; КПД источника равен 50%. Вычислить сопротивление R и полезную мощность /V. 40.2. Источник тока замкнут через сопротивление, которое в k раз превосходит внутреннее сопротивление источника. Как изменится КПД источника, если парал- лельно внешнему сопротивлению подсоединить дополнительное сопротивление, которое в п раз больше внутреннего сопротивления источника? Принять k = 4, п = 2. 40.3. Несколько одинаковых сопротивлений R соединены в комбинацию, показан- ную на рис. 40.5. В одном случае эту комбинацию подсоединяют к источнику в точках 1 и 2, а в другом — в точках 1 и 3. Вычислить внутреннее сопротивление г источника тока, если отношение коэффициента полезного действия источника тока в первом и втором случаях равно 16/15. Найти эти значения КПД. 40.4. В электроплитке сопротивления соединены в комбинацию, показанную на 211
рис. 40.5, и включаются в сеть в точках 1 и 2. При этом за некоторое время удается довести до кипения 500 г воды. Сколько воды можно довести до кипения за то же время, когда комбинация сопротивлений Электроплитки включена в сеть в точках 1 и 3? Начальная температура воды в обоих случаях одинакова, тепло- выми потерями прнебречь. 40.5. 1,5 л воды, взятой при температуре 20/С, ставят на время ^=15 мин на электроплитку, имеющую две секции с одинаковым сопротивлением. Когда секции соединены параллельно, вода в указанное время закипает, при этом mt — 100 г превращается в пар. Что будет с водой, если соединить секции последовательно и вести нагревание /2 = 60 мин? Удельная теплота парообразования L — 22,6 X X 10* Джокг. Сколько времени /3 потребуется, чтобы нагреть эту воду до кипе- ния при включении одной секции? 40.6. Источник тока, ЭДС которого ?= 5 В, замыкается один раз на сопротивле- ние /?! = 4 Ом, другой раз на /?2= 9 Ом. В обоих случаях на внешнем сопротив- лении выделяется одинаковая мощность N. Найти эту мощность и внутреннее сопротивление г источника тока. 41. КАК ОПИСЫВАЕТСЯ МАГНИТНОЕ ПОЛЕ ТОКА? Автор: Когда возникает постоянное во времени магнитное поле? Читатель А: Оно возникает вблизи постоянного магнита или вблизи про- водника, когда по проводнику течет постоянный ток. Автор: Чтобы определить напряженность электростатического поля в не- которой точке пространства, надо поместить в эту точку пробный заряд и измерить силу, действующую на него. А как быть в случае маг- нитного поля? Что и как применяют для определения*характеристик магнитного поля? Читатель А: Можно насыпать слой железных опилок на лист бумаги. В магнитном поле опилки намагнитятся и будут ориентироваться по силовым линиям поля. Читатель Б: Можно воспользоваться стрелкой компаса или использовать рамку с током. Автор: Расскажите о рамке с током подробнее. Читатель Б: В этих вопросах я всегда разбирался неважно. Наверное, будет лучше, если Вы нам расскажите сами. 212
Рис. 41.1 Рис. 41.2 Автор: Для изучения магнитного поля используют и железные опилки, и стрелку компаса. Однако лучше всего воспользоваться маленьким контуром (рамкой) с током. Внося, рамку в ту или иную точку поля, определяют направление и модуль вектора магнитной индукции (обо- значается В), являющегося аналогом вектора напряженности в случае электростатического поля. Линии магнитной индукции — это линии, касательные к которым в каждой точке имеют направление вектора индукции В. Рассмотрим рамку площадью S с текущим по ней постоянным током /; назовем положительной нормалью п перпендикуляр к плоскости рам- ки, направленный вдоль1 осевого перемещения буравчика, рукоятка ко- торого поворачивается в направлении, совпадающем с направлением тока в рамке (рис. 41.1). Магнитное поле оказывает на рамку ориен- тирующее действие — поворачивает рамку, устанавливая ее в конеч- ном итоге в определенном положении. Направление положительной нормали в этом (равновесном) положении есть направление вектора В в данной точке поля. Если повернуть рамку, то возникнет момент сил М поля, стремящийся возвратить рамку в равновесное положение. Мо- мент М зависит от угла а отклонения нормали от направления вектора Ё. При а= 90° момент максимален; обозначим его через Мо. Отноше- ние Af0/(/S) в данной точке поля есть модули вектора В в этой точке: В = М„/(/3). (41.1) А теперь нарисуйте картину линий магнитной индукции в случаях, изображенных на рис. 41.2, а—г. Линии проводите в указанных плос- костях. Читатель Б: Вот эта картина (рис. 41.3, а—г). а) б) в) г) Рис. 41.3 213
Автор: Заметьте, что направление линий магнитной индукции отвечает правилу буравчика (осевое перемещение буравчика происходит по на- правлению тока). Чем отличается картина линий магнитной индукции магнитного поля от картины силовых линий электростатического поля? Читатель Б: Электрические силовые линии начинаются в положительных и оканчиваются в отрицательных электрических зарядах. Аналога заря- да у магнитного поля нет. Линии магнитной индукции магнитного поля — замкнутые линии. Автор: Хорошо. Далее я приведу несколько полезных формул для вычис- ления магнитной индукции В: а) поле бесконечно прямолинейного тока / на расстоянии R от тока В ~ ~2nR ' (4L2) б) поле тока /, текущего по окружности радиуса R, в центре этой окружности s = #; (41.3) в) поле тока /, текущего по соленоиду (радиус витка R, п витков на длине /), внутри соленоида: В = (41.4) (заметим, что поле внутри соленоида однородно). В этих формулах k — коэффициент пропорциональности, зависящий от выбора системы единиц. В СИ: k = ц0 = 4л • КГ 7 В • с/(А • м). (41.5) Величину |ы0 называют магнитной постоянной. Заметим, что если бы токи рассматривались в среде, то во всех формулах вместо ц0 следо- вало бы использовать где ц — магнитная проницаемость среды. В качестве единицы магнитной индукции в СИ берут магнитную индук- цию такого поля, в котором на рамку площадью 1 м2 при протекании по ней тока силой 1 А действует максимальный момент 1 Н • м. Эта еди- ница называется тесла (Тл): 1 Тл = 1Н/(А • м). Читатель Б: Я читал, что магнитное поле в произвольном случае описыва- ется законом Био—Савара—Лапласа. Как это делается? Автор: Предварительно обратимся к некоторым операциям над векторами. Что Вы умеете делать с векторами? Читатель А: Складывать, разлагать и умножать на число. Автор: Я познакомлю Вас еще с одной операцией: векторным умножением. Векторным произведением двух векторов А и В является вектор С, направленный по нормали к плоскости векторов А и В в сторону осе- вого перемещения буравчика, рукоятка которого поворачиваестя от вектора А к вектору В в направлении меньшего угла (рис. 41.4). Модуль векторного произведения С = АВ sin ср. (41-6) 214
Обозначается указанная операция так: ё=4АХВ). (41.7) Если векторы А и В взаимно перпендикулярны, то С = АВ. Если век- торы А и В параллельны или антипараллельны, то их векторное произ- ведение равно нулю. Заметим, что (5ХЛ) = -(АХ^). (41.8) Читатель Б: Это для меня ново. Получается, что произведение векторов может быть равным нулю, даже если один из векторов не равен нулю. Кроме того, здесь от перемены мест сомножителей изменяется произведение (точнее, оно меняет знак). Автор: Совершенно верно. А теперь перейдем к закону Био—Савара— Лапласа. Пусть по некоторому проводнику течет ток силой /. Выделим маленький участок длины А7 проводника (направление этого вектора совпадает с направлением тока, а модуль равен длине участка); будем называть /А7элементарным током. Выберем некоторую точку А наблю- дения и проведем из выделенного элементарного тока в точку А век- тор Л (рис. 41.5). Закон Био—Савара—Лапласа утверждает: маг- нитная индукция АВ поля, создаваемого в точке А элементарным током /А/, определяется выражением АВ = -^^-(а7х Я). (41.9) 4л Чтобы затем найти полный вектор индукции в точке А, надо выпол- нить сложение (интегрирование) вектора АВ от всех элементарных токов, на какие только можно разбивать рассматриваемый проводник с током. Выражения (41.2) — (41.4) являются результатом такой опе- рации. Рассмотрим в качестве простейшего примера случай поля в центре кру- гового тока. На рис. 41.6 приведена иллюстрация закона Био—Сава- ра—Лапласа для некоторого элементарного тока /А/. Легко видеть, что Но / АВ = —— —- А/. (41.10) 4л /?2 v ’ Если перемещаться по току (переходя к новым элементарным токам), то направление АВ и само выражение будут сохраняться. Поэтому о Но I 1 4л /?2 /, 4л /?2 0 2/? Итак, мы пришли к выражению (41.3). 215
a) Рис. 42.1 ЗАДАЧИ 41.1 . Найти магнитную индукцию В поля в точке посередине между двумя парал- лельными прямолинейными проводниками, если по проводникам течет ток силой /, расстояние между проводниками d Рассмотреть два случая: а) токи параллельны; б) токи антипараллельны. 41.2 . Найти магнитную индукцию В поля в центре двух концентрических круговых токов (сила тока /, радиусы R и 2/?). Рассмотреть случаи круговых токов: 1) в одной плоскости, в одном направлении; 2) в одной плоскости, в разных направлениях; 3) во взаимно перпендикулярных плоскостях. 41.3 . К двум произвольным точкам проволочного кольца подведены идущие ра- диально провода, соединенные с удаленным источником тока. Определить индук- цию магнитного поля в центре кольца. 41.4 . Два длинных параллельных провода находятся на расстоянии I = 5 см друг от друга. Найти модуль вектора магнитной индукции В в точке А, находящейся на расстоянии г = 4 см от одного и г2 = 3 см от другого провода. По проводам текут одинаковые токи силой / = 12 А. Рассмотреть два случая: 1) токи текут в одном направлении; 2) токи текут в противоположных направлениях. 41.5 . Два бесконечно длинных прямых провода скрещены под прямым углом По проводам текут токи силой = 80 А и /2 = 60 А. Расстояние между проводни- ками d = 10 см. Определить модуль вектора магнитной индукции в точке, лежащей посередине между проводами. 42. КАК ВЗАИМОДЕЙСТВУЮТ ТОКИ? Автор: Как действует магнитное поле на ток? Читатель А: По правилу левой руки: поле входит в ладонь, ток направ- лен вдоль вытянутых вместе четырех пальцев, а сила действует по на- правлению отставленного большого пальца. Рис. 42.2 216
a) б) в) Рис. 42.3 Автор: Это верно. Однако Вы ничего не сказали о модуле силы. Дейст- вие магнитного поля на ток описывается законом Ампера: пусть в неко- торой точке магнитного поля индукция задается вектором В; если в эту точку поместить элементарный ток /ЛТ, то поле действует на него с силой AF: AF = /(Д/Х В). (42.1) Здесь фигурирует уже знакомое Вам векторное произведение. Можете проверить, что направления векторов, входящих в (42.1), согласуются с правилом левой руки. А теперь определите направление силы, действующей на ток в магнит- ном поле, для случаев, изображенных на рис. 42.1. Читатель А: Я воспользуюсь правилом левой руки. Автор: Не возражаю. Но советую привыкать к выражению (42.1), учитесь пользоваться векторным произведением. Читатель А: Хорошо. В случае а) векторы А/ и В параллельны; значит, их векторное произведение равно нулю и поле на ток не действует. В случае б) будем поворачивать рукоятку буравчика от вектора А/ к вектору В — вращение происходит по часовой стрелке; осевое пере- мещение буравчика будет происходить перпендикулярно плоскости листа в направлении от нас. Так будет направлена сила. Автор: Хорошо. Далее проделайте манипуляции с буравчиком мысленно и сообщите готовый результат. N » токи силы Рис. 42.4 217
Читатель А: В случае в) сила перпендикулярна плоскости листа и направ- лена к нам. В случае г) сила направлена в плоскости листа горизон- тально слева направо. В случае д) направление силы по сравнению с предыдущим изменилось на противоположное. Автор: Все правильно. Перейдем к рассмотрению взаимодействия токов. Пусть имеются два элементарных тока: /]Д/] и /2Д/2; &F— сила, с которой первый ток действует на второй. Будем рассуждать следую- щим образом: элемент тока создает в месте расположения второго тока магнитное поле М3, которое по закону Био—Савара—Лапласа может быть представлено в виде (рис. 42.2, а) (42-2) Это поле в соответствии с законом Ампера действует на элемент тока /2М2 с силой AF (рис. 42.2, б): \F= I2(M2X Mi). (42.3) Подставляя (42.2) в (42.3) находим следующее выражение для силы, с которой элемент тока A/j действует на элемент тока: х х 4) Кстати, из рис. 42.2 легко усмотреть результат, который должен быть Вам хорошо известен: параллельные токи притягиваются друг к другу. Читатель Б: С элементарными токами все понятно. А как быть с реаль^ ными токами, реальными проводниками? Автор: Надо разбить (мысленно, конечно) каждый из проводников на элементарные токи, определить взаимодействие между каждой парой элементарных токов (выбранных, разумеется, на разных проводниках), а затем получить суммарный результат. Читатель Б: В принципе это понятно. А вот как проделать такую гро- моздкую операцию практически? Автор: Практически Вы будете это делать после того, как познакомитесь с высшей математикой и научитесь интегрировать. А пока ограничьтесь взаимодействием элементарных токов. На рис. 42.3 изображены четыре пары элементарных токов. Восполь- зуйтесь выражением (42.4) и нарисуйте направления сил, приложен- ных к каждому из изображенных токов. Читатель Б: Разрешите мне. Одну минуту... В случае а) токи взаимно притягиваются, в случае б) — отталкиваются, в случае в) они не взаи- модействуют, а вот в случае г) ... Получается какая-то нелепица: в слу- чае г) на правый ток сила действует, а на левый не действует. Вот мой рисунок (рис. 42.4). Автор: Ваш рисунок верен. Читатель Б: Но ведь в случае г) не выполняется третий закон Ньютона! 218
Здесь действие не равно противодей- ствию. Читатель А: А может быть, третий закон Ньютона справедлив только в механике. Ведь здесь рассматриваются электричес- кие токи. Читатель Б: Не согласен. К тому же многие силы, рассматриваемые в механике, имеют электрическую природу. Автор: Это так. Третий закон Ньютона явля- ется фундаментальным законом; из него следует закон сохранения количества движения. Следовательно, отказ от третьего закона Ньютона должен привести к отказу от закона сохра- нения количества движения. Читатель Б: Вот и получается, что в случае г) йа рис. 42.4 изобра- жена физическая нелепица. Автор: И все же рисунок верен. Ищите выход из положения. Читатель Б (после раздумья): Кажется, я понял, в чем дело. Мы рассмат- ривали элементарные токи. Эти токи не замкнуты. Реальные же токи должны быть замкнуты. Попробуем рассмотреть взаимодействие двух замкнутых токов, каждый из которых состоит, допустим, из четырех элементарных токов (рис. 42.5). Для удобства все они на рисунке перенумерованы. Возьмем, например, ток 1. Найдем силы, с какими на него действуют токи 5, 6, 7, 8. Сложим эти силы. Затем возьмем последовательно токи 2, 3 и 4. В итоге можно найти результирую- щую силу, приложенную к каждому из замкнутых токов. Я думаю, что результирующие силы будут равны по модулю и противоположны по направлению в соответствии с третьим законом Ньютона. Автор: Ваш вывод в отношении результирующих сил, приложенных к замкнутым токам, верен. Читатель Б: В итоге никакого парадокса не остается. Автор: А как быть с переменными электрическими токами? Ведь они-то могут быть незамкнутыми. Для них случай г) на рис. 42.4 может отра- жать реальную ситуацию. Читатель Б: Парадокс опять возродился! Автор: Не буду больше испытывать вашу сообразительность. Действи- тельно, в случае элементарных переменных токов можно встретиться с ситуацией, когда действие не равно противодействию. Может пока- заться, что при этом нарушается закон сохранения количества движе- ния. Однако не следует спешить с таким выводом. В природе никогда количество движения не исчезает и не возникает. Надо сделать вывод, что в рассматриваемой нами ситуации мы имеем дело еще с одним объектом, который уходит из системы и уносит с собой определенное количество движения. Этот объект — электромагнитная волна. Таким образом, мы приходим к заключению, что система переменных токов типа той, что изображена на рис. 42.4, г, представляет собой открытый колебательный контур и должна излучать электромагнитные волны. Читатель Б: Потрясающе! Автор: Заметьте, что этот вывод следует из самых общих положений. Здесь весьма уместно подчеркнуть важность овладения материалисти- 219
ческим мировоззрением.' Действительно, материалистически мыслящий человек будет исходить из того, что закон сохра- нения количества движения в любом слу- чае выполняется. Следовательно, кажу- щееся его нарушение может быть связано только с незамкнутостью рассматривае- мой системы, с наличием дополнительно- го «объекта», уносящего (или принося- щего) с собой какое-то количество дви- жения. Человек же, не привыкший мыс- лить материалистически, скорей всего сделал бы ложный вывод о том, что в данной ситуации закон сохранения количества движения попрос- ту нарушается. Мы убеждаемся в том, что правильная философская позиция играет существенную роль при рассмотрении различных физи- ческих явлений. В связи с этим поучительно привести один пример из области ядерной физики. При изучении являения p-распада атомных ядер физики заме- тили, что электроны, рождающиеся при p-распаде, не обладают опреде- ленной энергией, а вылетают от случая к случаю с разными энергиями. Казалось, что в явлении p-распада ядер нарушается закон сохранения энергии. Материалистическая философская позиция, исходящая из не- зыблемости закона сохранения энергии, привела в данном случае к предсказанию существования трудно регистрируемой новой частицы, появляющейся при p-распаде наряду с электроном. Эту частицу на- звали нейтрино. Впоследствии в результате усовершенствования тех- ники эксперимента нейтрино было обнаружено прямым эксперимен- тальным путем. Однако до сих пор принято говорить, что нейтрино была открыта «на кончике пера». ЗАДАЧИ 42.1. В однородное магнитное поле с индукцией В = 0,25 Тл, направленное верти- кально, помещен горизонтальный линейный проводник массой т = 10 г и длиной I = 20 см. Проводник подвешен на тонких длинных проволочках. На какой угол отклоняются проволочки, поддерживающие проводник, если по нему пропустить ток силой / = 2 А. 42.2. Медный проводник с площадью сечения s = 3 мм2, согнутый в виде трех сторон квадрата, может свободно поворачиваться вокруг горизонтальной оси АБ (рис. 42.6). Проводник помещен в однородное магнитное поле, индукция которого равна В = 0,2 Тл и направлена вертикально. Определить, на какой угол а откло- нится проводник, если по нему пропустить ток силой / = 2,6 А. Плотность меди р = 8,9 г/см3. 43. ХОРОШО ЛИ ВЫ ПОНИМАЕТЕ ЗАКОН ФАРАДЕЯ И ПРАВИЛО ЛЕНЦА? Автор: Дайте определение потока магнитной индукции через некоторый контур (рамку). Читатель Б: Потоком магнитной индукции (обозначается Ф) через контур площадью S называют величину, равную произведению магнитной 220
индукции В на площадь S и на косинус угла а между нормалью к контуру и вектором магнитной индукции: Ф = BS cos а. ' (43.1) Читатель А: Поток магнитной индукции через контур равен числу линий магнитной индукции, пересекающих площадь контура. Автор: Согласен с обоими заявлениями. Каковы условия применимости формулы (43.1)? Читатель Б: Я не понимаю вопроса. Автор: Это потому, что подобные вопросы Вы не привыкли ставить. Фор- мула (43.1) применима в предположении, что вектор индукции не изменяется ни по модулю, ни по направлению в пределах площади контура. Иными словами, магнитное поле в пределах контура должно быть однородным. Какова единица потока магнитной индукции в СИ? Читатель Б: Не помню. Автор: Эта единица называется вебер (Вб). Это поток сквозь площадку 1 м , пересекаемую нормальными к ней линиями магнитного поля, индукция которого 1 Тл. Следующий вопрос: изменяется ли со временем поток индукции через рамку, если она поступательно перемещается в однородном магнитном поле? Читатель Б: А какова траектория перемещения — прямая или нет? Автор: А разве это существенно? Читатель Б: Понял. Не изменяется. Автор: Правильно. А в каком случае изменяется? Читатель Б: Если рамка перемещается в неоднородном поле, или враща- ется в однородном поле. Автор: Верно. Теперь перейдем к рассмотрению явления электромагнит- ной индукции. В чем она заключается? Читатель Б: При изменении во времени потока Ф сквозь контур в послед- нем возникает электрический ток (индукционный ток). Можно считать, что этот ток возникает под действием некой электродвижущей силы — ЭДС индукции (^). Автор: Отлично. А теперь скажите, какой закон (или какие законы) описывает явление электромагнитной индукции? Читатель Б: Закон Фарадея: ЭДС пропорциональна скорости измене- ния потока во времени ЛФ/Д/ (в СИ коэффициент пропорциональности, по-моему, равен единице): ДФ * = - ~-• (43.2) Автор: Вы правильно сформулировали закон Фарадея. Однако следует помнить, что явление электромагнитной индукции описывается не только законом Фарадея, но и правилом Ленца, которое объясняет знак «—». Читатель А: Правило Ленца — индукционный ток всегда направлен так, чтобы своим полем противодействовать причине, его вызывающей. Я помню эту формулировку, до не вполне понимаю ее. 221
Автор: Закон Фарадея определяет значение ЭДС индукции, правило Ленца — на- правление этой ЭДС. Теперь о самом правиле Ленца. Рассмот- рим его с точки зрения закона сохранения энергии. Будем надевать проводящее кольцо в одном случае на обычный каран- даш, а в другом — на брусочек постоян- ного магнита. Во втором случае появляет- ся индукционный ток в кольце. Это озна- чает, что мы вынуждены были совершить некоторую дополнительную работу при надевании кольца. Но дополнительная работа может быть совершена только при наличии дополнительной силы сопротивления (против этой с.илы). Сле- довательно, индукционный ток в кольце должен быть направлен так, чтобы обеспечить возникновение указанной силы сопротивления. Читатель А: Вот теперь мне все эти вопросы стали яснее. Читатель Б: Получается, что правило Ленца тоже можно было бы открыть «на кончике пера»? Ведь здесь используется только закон сохранения энергии. Автор: Вы правы. Только замечу, что наряду с законом сохранения энер- гии используется экспериментальный факт появления индукционного тока. Иначе говоря, для «открытия» правила Ленца физик-теоретик должен знать закон сохранения энергии и закон Фарадея. Для закрепленения материала рассмотрим задачу: рамка площадью S равномерно вращается с угловой скоростью to в однородном магнитном поле с индукцией В. Как изменяется со временем ЭДС индукции? В ка- ких положениях рамки ЭДС индукции обращается в нуль? В каких положениях она максимальна? Задачу иллюстрирует рис. 43.1, на котором показано поперечное сече- ние рамки для нескольких положений. В каком положении поток сквозь рамку максимален? Читатель А: В положении 1. Автор: В каком положении рамки максимальна скорость изменения потока сквозь рамку? Читатель А (неуверенно): Тоже в положении 1... Читатель Б: По-моему, в положении 2, так как в этом положении рамка при вращении пересекает линии магнитной индукции поля под прямым углом. В положении 2 максимальна ЭДС индукции. Автор: Правильно. А в каком положении ЭДС индукции равна нулю? Читатель Б: В положении 1. Здесь рамка при вращении скользит вдоль линий магнитной индукции. Автор: Вы правы. Обратите внимание: поток через рамку и скорость изме- нения этого потока — это разные «вещи». В положении 1 поток макси- мален, а скорость его изменения равна нулю. В положении 2, напротив, скорость изменения потока максимальна, а сам поток равен нулю. Далее положим а= wf и рассмотрим общий случай, которому отвечает промежуточное положение 3. Имеем Ф = В S cos а — В S cos со/. 222
Нам требуется найти отношение ДФ/Д/ при достаточно малых значениях Д/. Запишем Дф=ВЗ[ cos(<o/4-<oA/)—coso)/] — =BS[ coso>/coso)A/— — sino)/sino>A/—cos o>/] Поскольку о>Д/<С 1, можно поло- жить coso)A/«l и sintoA/ « <оД/ (здесь угол выражается не в градусах, а в радианах). Отсюда находим <% =. — = BS со sin со/. (43.3) ' А/ Рис. 43.2 На рис. 43.2 показаны зависи- мости от времени для потока и для скорости его изменения (т. е. для ЭДС индукции). Мы получили, что в рамке, равномерно вращающейся в однородном магнитном поле, индуцируется переменная ЭДС, изме- няющаяся во времени по гармоническому закону. ЗАДАЧИ 43.1. По двум металлическим рейкам, соединенным с сопротивлением R — 10 Ом, двигают без нарушения контакта металлическую перемычку А Б со скоростью v = 1 м/с. Система расположена в однородном магнитном поле, индукция которого равна В = 1 Тл и направлена перпендикулярно скорости движения перемычки (рис. 43.3). Определить мощность Р, которую надо затратить, чтобы по пере- мычке тек постоянный ток. Длина перемычки I = 0,5 м. Определить также силу тока I, протекающего через сопротивление R. Сопротивлением металлических реек и перемычки пренебречь. 43.2. В однородном магнитном поле с индукцией В — 0,1 Тл расположен плоский проволочный виток, площадь которого S = 5 см2, а сопротивление R = 2 Ом. Плос- кость витка перпендикулярна линиям магнитной индукции поля. Виток замкнут на гальванометр. Определить заряд q, протекающий через гальванометр при по- вороте витка на угол а = 120°. 43.3. В однородное магнитное поле с магнитной индукцией В, направленной гори- зонтально, помещена конструкция, показанная на рис. 43.4. Плоскость конструкции перпендикулярна линиям магнитной индукции поля. Перемычка АБ свободно сколь- 223
зит без нарушения контакта. Определить индукцию В, если перемычка достигает максимальной скорости v = 1 м/с. Плотность вещества перемычки D = 8,8 г/см3, удельное сопротивление р = 1,75 • 10~8Ом • м. 44. ЗНАКОМЫ ЛИ ВАМ ПОНЯТИЯ САМОИНДУКЦИИ И ИНДУКТИВНОСТИ? Автор: Поговорим о явлении самоиндукции. Читатель Б: Мне кажется, именно теперь и был бы особенно уместен вопрос: почему перегорела лампочка? Автор: Что Вы имеете в виду? Читатель Б: Я читал, что когда мы выключаем свет, в электрической цепи вследствие явления самоиндукции возникает дополнительный ток. Его называют током размыкания. В результате полный ток через лам- почку в этот промежуток времени усиливается и она может перегореть. Правда, я затрудняюсь более подробно объяснить возникновение тока размыкания... Читатель А: Однако я не один раз видел, что лампочка перегорает вовсе не в момент выключения, а как раз наоборот — в момент включения. Мы щелкаем выключателем, свет на мгновение зажигается и притом довольно ярко. И в этот момент лампочка перегорает. Читатель Б: А ведь действительно, так оно и происходит. Я тоже наблюдал это. Получается какая-то неувязка с токами размыкания. Автор: Давайте разберемся во всем этом. Прежде всего замечу, что перегорание лампочки в квартире в момент включения света не имеет никакого отношения к самоиндукции. Для проявления самоиндукции нужно, чтобы в цепи была достаточно большая индуктивность (об индуктивности мы поговорим чуть позднее). В бытовой электросети индуктивность незначительна. Перегорание лампочки связано здесь с тем, что сопротивление проводника увеличивается при повышении его температуры. При включении света температура волоска лампочки повышается от комнатной до температуры 2400°С, при этом довольно заметно повышается и сопротивление нити накаливания у лампочки. Нормальным расчетным сопротивлением для лампочки является то, которое она имеет в работающем состоянии. Значит, в момент включе- ния, когда волосок нити накаливания лампочки еще не успел раска- литься, его сопротивление ниже расчетного значения. При заданной разности потенциалов напряжения сети мощность, выделяемая на во- лоске лампочки, в соответствии с формулой (40.3) будет больше рас- четной. Вот поэтому-то и возможно перегорание лампочки. Читатель Б: А как же быть с токами размыкания? Автор: Такие токи действительно существуют. Они обнаруживают себя в электрических цепях промышленного тока, имеющих значительные индуктивности. Токи размыкания действительно связаны с явлением самоиндукции. И поскольку мы разобрались с возникшим недоразуме- нием, перейдем, наконец, к этому явлению. При изменении силы тока в контуре будет, очевидно, изменяться и поток магнитного поля, созда- ваемого этим током. В результате в самом контуре будет индуциро- 224
ваться некая ЭДС. Это явление называют самоиндукцией, указанную ЭДС — ЭДС самоиндукции. Предположим, что в контуре течет по- стоянный ток. При выключении сила тока быстро уменьшается. Следо- вательно, уменьшается и поток магнитного поля через данный контур. Согласно закону Фарадея, возникает ЭДС индукции (в данном слу- чае-самоиндукции). Согласно правилу Ленца, направление этой ЭДС должно быть таким, чтобы препятствовать внешней причине, при- ведшей к уменьшению магнитного потока. Отсюда следует, что индук- ционный ток размыкания будет направлен так же, как и имевшийся ра- нее ток: поэтому результирующий ток может значительно возрасти. Очевидно, при включении возникает индукционный ток замыкания в направлении, обратном направлению основного тока (при этом резуль- тирующий ток уменьшается). В одних контурах явление самоиндукции проявляется сильнее, в других слабее. Например, самоиндукция в пря- мом проводнике проявляется значительно слабее, чем в том же провод- нике, свернутом в виде спирали (соленоида). В связи с этим вводят специальную величину, характеризующую самоиндукционные свойства контура,— его индуктивность. По закону Био—Савара—Лапласа, индукция поля пропорциональна силе тока. С другой стороны, магнит- ный поток пропорционален индукции. Отсюда видна пропорциональ- ность магнитного потока и силы тока. Запишем это в виде соотношения Ф = Ы. (44.1) Коэффициент пропорциональности L называют индуктивностью данно- го контура. За единицу индуктивности в СИ принимают индуктивность такого проводника, у которого при силе тока в нем 1 А возникает поток, равный 1 Вб. Указанная единица называется генри (Гн). Читатель А: С понятием индуктивности я знаком. Оно встречалось при рассмотрении электромагнитных колебаний в замкнутом контуре. Я имею в виду формулу для периода колебаний Т = 2л-ГьС. (44.2) Автор: Вы правы. Заметьте, что для увеличения индуктивности колеба- тельного контура провод сворачивают в виде соленоида. Иногда в лите- ратуре понятие «индуктивность» применяют в отношении самого этого соленоида (говорят: «колебательный контур состоит из емкости и ин- дуктивности») . В заключение попробуйте получить выражение для индуктивности со- леноида, если дано: сила тока I, число витков п, длина соленоида I, пло- щадь витка S. Читатель Б: Я воспользуюсь формулами (41.4) и (41.5). Индукция маг- нитного поля внутри соленоида описывается выражением В = ц0/ («//). (44.3) Поток равен Ф = BS. Автор: Тут Вы ошиблись. Надо учесть полный поток сквозь все п витков соленоида. Следовательно, Ф = пВБ. (44 4) 225
Читатель Б: Подставляя (44.3) и (44.4) в (44.1), находим nBs == = LBl/{ |ion), или окончательно L = n\QS/l. (44.5) Автор: Все правильно. 45. ЗНАЕТЕ ЛИ ВЫ, КАК ОТРАЖАЮТСЯ И ПРЕЛОМЛЯЮТСЯ ЛУЧИ СВЕТА? Автор: Сформулируйте законы отражения и преломления света. Читатель А: Закон отражения: угол падения равен углу отражения. Закон преломления: отношение синуса угла падения к синусу угла преломле- ния равно показателю преломления среды. Автор: Ваши формулировки неточны. Во-первых, Вы не указали, что лучи падающий и отраженный (или преломленный) лежат в одной плоскости с перпендикуляром, восставленным к границе отражения (или пре- ломления) из точки падения. Ведь если это не оговорить, то можно представить отражение так, как показано на рис. 45.1, что в действи- тельности не наблюдается. Во-вторых, Ваша формулировка закона пре- ломления относится к частному случаю падения луча из воздуха на границу некоторой среды. Пусть в общем случае луч падает из среды с показателем преломления п{ на границу со средой с показателем пре- ломления п2. Обозначим угол падения через о^, а угол преломления — через «2. В этом случае закон преломления записывается в виде sin а. п, ----=— (45!) sin а2 n, v ' Ваша формулировка следует отсюда при условии, что для воздуха П, = 1. Рассмотрим задачу. Монета лежит в воде на глубине Н. Будем смотреть на нее сверху по вертикали. На какой глубине мы увидим монету? Читатель А: Я знаю, что монета будет казаться приподнятой. Ответить более конкретно затрудняюсь. Автор: Проведем из точки О монеты два луча ОА и OBtB (рис. 45.2, а). Рис. 45 1 Рис. 45.2 226
Законы геометрической оптики известны человеку давно. Однако по сей день они поражают нас своей законченностью. Чтобы убедиться в этом, попрактикуйтесь в построении изображений в различных оптических системах. Обсудим законы отражения и преломления света. Луч О А не преломляется (он вертикален), луч OBtB преломляется. Предположим, что эти два расходящихся луча попадают в глаз. Глаз увидит изображение монеты в точке пересечения расходящихся лучей АО и BBt — в точке Ot. Из рисунка видно, что искомое расстояние h связано с глубиной Н соотношением h tg at = Н tg 02, отсюда h = аъ/tg о^. (45.2) Ввиду малости углов о1 и а2 можно применить приближенную формулу tg а « sin а « а (45.3) (здесь угол выражается не в градусах, а в радианах). Используя (45.3), перепишем формулу (45.2) в виде h « Н sin «2/sin al — Н/п. (45.4) Так как для воды п = 4/3, то h ~ Н. Читатель Б: А где будет изображение, если смотреть на монету не по вер- тикали, а как-то сбоку? Автор: В этом случае монета будет казаться не только приподнятой, но также и смещенной ближе к наблюдателю (рис. 45.2, б). Очевидно, что расчет в этом случае значительно усложняется. Рассмотрим задачу. Водолаз высотой h стоит на дне озера на глубине Н. Вычислить мини- 221
поверхности воды преломляется мальное расстояние от точки, где стоит водолаз, до тех точек дна, которые он может увидеть в результате полного отражения от поверхности воды. Читатель А: Такие задачи я умею решать. Обозначим искомое рас- стояние через L. Ход луча из точ- ки А в глаз водолаза показан на рис. 45.3; точка А — самая близ- кая к водолазу точка, которую он может увидеть в результате полного отражения от поверхно- сти воды. Так, например, луч, от более близкой точки Б на (см. пунктир на рис. 45.3). Угол «о является углом полного отражения. Он вычисляется по формуле sina0=l/n. (45.5) Из рисунка нетрудно видеть, что L = h tg oto + I 2 * (Я — tg «о = (2Я — h) tg Oq. Так как tg a= sin a/V 1 — sin2 a, то, используя (45.5), получаем L = (2Я - /г)/у/ n2 - 1. (45.6) Подставляя сюда п = 4/3, находим, что L = 3 (2Л7 — /г)/т/7. Автор: Совершенно верно.-А что будет видеть водолаз над своей головой? Читатель А: Он будет видеть прямо над собой светлый круг радиусом I — (И — h)/vп2 — 1 = (з/л/т) (Н — ti). За пределами этого круга он будет видеть изображения предметов дна. Читатель Б: А если Дно, на котором стоит водолаз, не горизонтальное, а наклонное? Автор: Тогда расстояние L будет, очевидно, зависеть от того, в каком направлении смотрит водолаз. Нетрудно сообразить, что это расстоя- ние минимально, когда водолаз смотрит вверх по наклонной плоскости, и максимально, когда водолаз смотрит в обратном направлении. Ре- зультат, полученный в предыдущей задаче, будет теперь применим тогда, когда водолаз смотрит в направлении, по которому высота дна не меняется (вдоль берега). Задача с наклонным дном предлагается для самостоятельного решения (см. задачу 45.2). Читатель Б: Рассмотренная задача основывалась на явлении полного отражения. Само по себе это явление представляется мне не совсем понятным. Получается, что при плавном изменении угла падения луча может наблюдаться скачкообразное изменение его дальнейшего хода. Автор: Что Вы имеете в виду? Читатель Б: Я поясню свои слова с помощью рис. 45.4. Мы постепенно увеличиваем угол падения а. При этом преломленный луч все больше отклоняется от перпендикуляра 001, угол 0 растет. И вот наступает 228
Рис. 45.4 ситуация, когда угол 0 оказывается равным 90°. Если теперь хотя бы чуть-чуть увеличить угол а, то луч ОА скачком превращается в луч ОВ. Вот это-то мне и непонятно. Автор: Но ведь Ваш рисунок ошибочен! Вот правильный рисунок (рис. 45.5). Читатель Б: Я Вас не понимаю. Автор: Все очень просто. Необходимо учитывать, что при падении луча на границу двух сред образуется два луча — преломленный и отра- женный. Существенно, что по мере увеличения угла падения а интен- сивность преломленного луча плавно уменьшается, а отраженного — плавно растет. Когда угол 0 оказывается равным 90°, интенсивность отраженного луча становится равной интенсивности падающего. Как видите, никакого скачка здесь нет. Читатель А: Можно ли изменить направление луча, поставив на его пути систему из плоскопараллельных прозрачных пластинок? Автор: А как Вы сами думаете? Читатель А: По-моему, в принципе возможно, ведь внутри пластинки луч, преломляясь, идет по иному направлению. Читатель Б: Я не согласен. После пластинок луч все равно будет парал- лелен первоначальному направлению. Автор: Докажите это на примере системы из нескольких пластинок с различными показателями преломления. Читатель Б: Я возьму три пластинки, показатели преломления которых п1( п2 и п3. Ход луча в этой системе показан на рис. 45.6. Рассматри- вая преломление луча на каждой из имеющихся границ, запишем: sin «о/sin а, = п1? sin oq/sin «2 ~ ni/no sin o^/sin 0C3 — n3/n2. s’n «з/sin а4 = l/n3. Перемножая левые и правые части этих равенств, получаем sin oto/sin а4 = 1. Таким образом, = а4, что и требовалось доказать. Рис. 45.5 229
Рис. 45.6 Автор: Совершенно верно. А теперь давайте об- судим границы применимости законов гео- метрической оптики. Читатель Б: Эти законы неприменимы, когда характерные для рассматриваемой задачи расстояния, например диаметр препятствия или отверстия, оказываются достаточно ма- лыми — порядка длины волн света. В этих случаях проявляются волновые свойства света. Автор: Такой именно ответ обычно и дают экза- менующиеся. С ним можно согласиться, хотя он и не является вполне исчерпывающим. Во всяком случае, здесь содержится правиль- ная мысль о том, что законы геометрической оптики ограничены со стороны малых рас- стояний, когда уже приходится принимать во внимание длину волны света. А существуют ли ограничения на приме- нимость законов геометрической оптики, если так можно сказать, с дру- гой стороны — со стороны больших расстояний? Читатель Б: Если расстояние больше длины волны света, то свет можно рассматривать в рамках геометрической оптики. Именно так нам гово- рили ранее. Я думаю, что со стороны больших расстояний ограничений на применимость геометрической оптики нет. Автор: Вы ошибаетесь. Представьте себе следующую картину: в космиче- ское пространство посылается луч света, причем совершенно исклю- чается возможность его рассеяния. Предположим, что в течение одной секунды Вы поворачиваете аппарат, посылающий световой луч, на 60°. Спрашивается: с какой скоростью должны перемещаться при таком повороте точки луча, находящиеся от аппарата на расстояниях больше 300 000 км? Читатель Б: Я понимаю вопрос. Такие точки должны перемещаться со скоростью, более высокой, нежели скорость света. Однако, согласно теории относительности, скорости больше скорости света невозможны только в том случае, если эти скорости движения материальных тел. Здесь же речь идет о луче. Автор: А разве световой луч не материален? Как видите, геометрическая оптика на слишком больших расстояниях оказывается несостоятель- ной. В этом случае необходимо учитывать, что световой луч — это поток частиц света (поток фотонов). Фотоны, вылетевшие из аппарата до того, как его повернули, ничего «не знают» о состоявшемся повороте и продолжают свое движение в том направлении, в каком были испу- щены. В новом направлении полетят новые фотоны. Таким образом, никакого поворота светового луча как целого мы не наблюдаем. Читатель Б: А как количественно оценить границу применимости законов геометрической оптики со стороны больших расстояний? Автор: Расстояния должны быть таковы, чтобы время, нужное свету для их прохождения, было много меньше всех характерных времен в дан- ной задаче, например много меньше времени поворота аппарата, испускающего световой луч. В этом случае луч как целое не разруша- ется, и можно пользоваться законами геометрической оптики. 230
ЗАДАЧИ 45.1. Мы смотрим по вертикали сверху на предмет, накрытый стеклянной пластин- кой, поверх которой налита вода. Толщина пластинки d} = 5 см, слоя воды d2 — 10 см. Показатель преломления стекла = 1,6, воды п2 = 1,33. На каком расстоянии h от поверхности воды будет видно изображение предмета? 45.2. Водолаз высотой h — 1,8 м стоит на глубине Н =5 м под водой на дне, пред- ставляющем собой наклонную плоскость с углом наклона 0 = 15°. Вычислить минимальное расстояние L по дну от точки, где стоит водолаз, до тех точек дна, которые водолаз может увидеть в результате полного отражения от поверхности воды. 45.3. Дана стеклянная пластинка толщиной d = 5 см с показателем преломления п =1,5. При каком угле падения а из воздуха отраженный и преломленный пластинкой лучи будут взаимно перпендикулярны? Вычислить для этого угла падения смещение х луча в результате прохождения сквозь указанную пластинку. 45.4. Дана стеклянная пластинка толщиной d с показателем преломления п. Угол падения луча из воздуха на пластинку равен углу полного отражения для стекла, из которого изготовлена пластинка. Вычислить смещение х луча в результате прохождения сквозь пластинку. 46. КАК ВЫ СТРОИТЕ ИЗОБРАЖЕНИЯ В ЗЕРКАЛАХ И ЛИНЗАХ? Автор: Нередко приходится сталкиваться с неумением экзаменующихся правильно строить изображения в плоских и кривых зеркалах, раз- личных оптических системах — линзах. Рассмотрим ряд примеров. Постройте изображение человека в плоском зеркале в случае, изоб- раженном на рис. 46.1, а. Читатель А: Мне кажется, что в этом случае изображения в зеркале не будет, так как зеркало расположено слишком высоко. Автор: Ошибаетесь. Изображение в зеркале будет. Его построение дано на рис. 46.1, б. Нетрудно убедиться, что для воссоздания изображения достаточно продолжить плоскость зеркала и симметрично показать изображение человека относительно этой плоскости. Читатель А: Но разве человек увидит свое изображение? Рис. 46.2 231
Рис 46 3 Автор: Это другой вопрос. Человек, действительно, своего изображения не увидит. Для этого зеркало расположено слишком высоко и, кроме того, неудобно наклонено. Изображение человека в данном зеркале увидят только те наблюдатели, которые будут находиться в пределах угла, образованного лучами AAt и EEt. Уместно напомнить, что в глаз наблюдателя должен войти пучок расходящихся лучей от наблюдаемо- го предмета. Глаз увидит изображение предмета в месте пересечения этих лучей или их продолжений (см. рис. 45.2 и 46.1, б). Рассмотрим пример на построение изображения в системе из двух взаимно перпендикулярных плоских зеркал (рис. 46.2, а). Читатель А: Надо отразить предмет в двух зеркальных плоскостях, в ре- зультате чего получатся два изображения (рис. 46.2, б). Автор: Вы потеряли еще одно, третье, изображение. Обратите внимание на то, что лучи, вышедшие из предмета в пределах прямого угла АОБ (рис. 46.2, в), испытывают не одно, а два отражения: сначала от одно- го, а затем от другого зеркала. На рис. 46.2, в показан ход двух таких лучей. Пересечение продолжений этих лучей определяет третье изобра- жение предмета. Рассмотрим ряд примеров с собирающей линзой. Постройте изображе- ние предмета в линзе в случае, изображенном на рис. 46.3, а. Читатель А: Это очень просто. Мое простроение показано на рис. 46.3, б. Автор: Хорошо. Теперь представьте себе, что половина линзы закрыта непроницаемым экраном (рис. 46.3, в). Что произойдет при этом с изоб- ражением? Читатель А: В этом случае изображение исчезнет. Автор: Ошибаетесь. Вы забываете, что изображение любой точки стрелки (например, ее конца) получается в результате пересечения не двух лучей, а бесконечно большого числа лучей (рис. 46.3, г). Обычно мы ограничиваемся двумя лучами потому, что для построения положения изображения достаточно рассмотреть ход всего двух лучей. Экран в данном случае задерживает часть лучей, падающих на линзу. Однако другая часть лучей проходит сквозь линзу и образует изображение предмета (рис. 46.3, д). Поскольку теперь в создании изображения участвует меньше лучей, изображение получается менее ярким. 232
Читатель Б: Из Ваших объяснений следует, что при закрывании части линзы непроницаемым экраном изменяется только яркость изображе- ния и ничего больше. Однако все, кто занимается фотографией, знают, что при диафрагмировании объектива фотоаппарата, т. е. при уменьше- ний рабочей площади линзы, наряду с уменьшением яркости изобра- жения наблюдается и другой эффект — изображение становится более четким. Чем это объясняется? Автор: Вы поставили весьма уместный вопрос. Он позволяет отметить сле- дующее обстоятельство: все наши построения делаются в предположе- нии пренебрежения дефектами оптических систем (в данном случае линз). Правда, термин «дефекты» здесь не совсем подходит, поскольку речь идет не о каких-то случайных недостатках линз, а скорее об их принципиальных свойствах. Известно, что если на линзу падают два луча, параллельных главной оптической оси линзы и отстоящих от нее иа разных расстояниях, то после преломления в линзе эти лучи пересе- кут главную оптическую ось, строго говоря, в различных точках (рис. 46.4, а), т. е. фокус линзы (точка пересечения всех лучей, парал- лельных главной оптической оси) оказывается размазанным; изобра- жение предмета будет нечетким. Эта размазанность тем больше, чем больше разница в расстояниях до главной оптической оси от различных лучей пучка. При диафрагмировании линзы сквозь нее проходит более узкий пучок лучей, вследствие чего нечеткость изображения несколько уменьшается (рис. 46.4, б). Читатель Б: Получается, что при диафрагмировании жертвуют яркостью изображения ради повышения степени его четкости? Автор: Совершенно верно. Однако следует иметь в виду, что при построе- нии изображений в линзах экзаменующийся имеет право полагать, что параллельные лучи всегда пересекаются в одной точке. Эта точка ле- жит на главной оптической оси, если пучок параллельных лучей был на- правлен вдоль указанной оси; эта точка лежит на фокальной плоскости, если пучок параллельных лучей был направлен под некоторым углом к главной оптической оси. При этом важно, чтобы экзаменующийся пони- мал, что подобное рассмотрение является приближенным и при более точном подходе требует введения поправок на дефекты оптических систем. Читатель А: А что такое фокальная плоскость линзы? 233
Рис. 46.5 Автор: Это плоскость, проведенная через фокус линзы перпендикулярно ее главной оптической оси. Рассмотрим вопрос: чем отличается изображение в плоском зеркале от изображения, полученного в собирающей линзе в примере, к которому относился рис. 46.3. Читатель А: В первом случае (в зеркале) изображение является мнимым, тогда как во втором случае оно действительное. Автор: Правильно. Укажите подробнее, в чем различие между мнимым и действительным изображениями. Читатель Б: Мнимое изображение образуется пересечением не самих лу- чей, а их продолжений, а действительное — пересечением самих лучей. Неудивительно, что мнимое изображение может находиться где-нибудь за стенкой — там, где сами лучи никак не могут оказаться. Автор: Правильно. Заметьте также, что мнимое изображение можно на- блюдать только из определенных положений. В случае же действитель- ного изображения Вы можете поместить в месте нахождений изображе- ния экран и наблюдать изображение с любой позиции. Рассмотрим пример, изображенный на рис. 46.5, а. Построить направ- ление луча АА/ после его прохождения сквозь собирающую линзу, если известен ход некоторого другого луча в этой линзе (луч BBtB2 на рис. 46.5, а). Читатель А: Но ведь неизвестно фокусное расстояние линзы! Автор: Зато дан ход одного луча до и после линзы. Читатель А: Таких построений мы никогда не делали. Читатель Б: Я думаю, что сначала следует найти фокусное расстояние линзы. Для этого можно нарисовать где-нибудь слева от линзы верти- кальную стрелку так, чтобы ее конец находился как раз на луче BBt. Обозначим этот конец точкой В (рис. 46.2, б). Далее проведем из точки В луч через центр линзы. Этот луч пройдет сквозь линзу, не преломившись, и в некоторой точке Г пересечется с лучом BtB2. Точка Г 234
является, очевидно, изображением конца нашей стрелки. Теперь оста- ется провести из конца стрелки В третий луч, параллельный главной оси линзы. Преломившись, этот луч должен пройти через уже известное нам изображение конца стрелки, т. е. точку Г. Точка пересечения ука- занного луча с главной осью и явится искомым фокусом линзы. Все эти построения даны на рис. 46.5, б. Затем, зная фокусное расстояние лин- зы, построим ход луча AAt после преломления. Для этого нарисуем еще одну стрелку, конец которой должен находиться на луче AAt (рис. 46.5, в). Используя найденное фокусное расстояние линзы, строим изображение указанной стрелки. Искомый луч будет проходить через точку At и конец изображения стрелки. Эти построения показаны на рис. 46.5, в. Автор: Ваши рассуждения справедливы. Они основаны на том, что Вы отыскиваете изображение некоторого вспомогательного предмета (стрелки). Замечу, что этим методом удобно воспользоваться в случае, например, когда нужно определить положение изображения светящей- ся точки, находящейся на главной оси линзы. В этом случае удобно вос- ставить из светящейся точки стрелку и построить изображение стрелки. Ясно, что основание изображения стрелки является искомым изобра- жением светящейся точки. Однако в приведенном выше случае этот ме- тод является слишком громоздким. Есть более простой метод построе- ния. Для получения фокусного расстояния линзы проведем через ее центр луч ГО, параллельный лучу BBt (рис. 46.5, г). Эти два луча, по- скольку они параллельны, пересекутся за линзой в фокальной плоскос- ти (сечение фокальной плоскости показано на рис. 46.5, г пунктиром). Затем проведем через центр линзы луч ВО, параллельный AAt. Исходя из того, что эти два луча тоже должны пересекаться за линзой в фо- кальной плоскости, определяем искомое направление луча AAt, после того, как он прошел сквозь линзу. Как видите, все получается значи- тельно проще. Читатель Б: Да, этот способ построения заметно проще. Автор: Попробуйте рассмотреть по этому способу аналогичную задачу, в которой вместо собирающей линзы используется рассеивающая линза (рис. 46.6, а). Читатель Б: Я проведу луч, параллельный лучу BBt, и проходящий через центр линзы (рис. 46.6, б). В отличие от предыдущей задачи теперь будут пересекаться не сами лучи, а их продолжения (заметим, что для луча, проходящего через центр, продолжение совпадает с самим лу- чом), вследствие чего фокальная плоскость, содержащая точку пересе- чения, будет теперь не справа, а слева от линзы (см. пунктир на рис. 46.6, б). Автор (перебивая): Подчеркнем, что в рассеивающих линзах изображе- ния всегда являются мнимыми. Читатель Б (продолжая): Затем я проведу через центр линзы луч, парал- лельный лучу AAt и, исходя из условия пересечения продолжений этих лучей в фокальной плоскости, построю искомый луч. Автор: Хорошо. Ответьте качественно на вопрос: где будет изображение предмета, часть которого находится перед фокусом собирающей линзы, а часть — за фокусом (предмет имеет конечную ширину)? Читатель Б: Я построю изображения нескольких точек предмета, располо- 235
женных на различных расстояниях от линзы. При этом точки, располо- женные за фокусом, дадут действительное изображение (оно будет на- ходиться справа от линзы), а точки, расположенные перед фокусом, дадут мнимое изображение (оно будет находиться слева от линзы). По мере приближения выбранных точек к фокусу изображения будут отодвигаться в бесконечность (левую или правую). Автор: Прекрасно. Таким образом, в данном случае изображение пред- мета состоит из двух кусков (слева и справа от линзы), каждый из которых начинается на некотором конечном расстоянии от линзы и рас- пространяется до бесконечности. Как видите, возможен положитель- ный ответ на вопрос: может ли предмет иметь одновременно и мнимое, и действительное изображения. Я вижу, что методика построения изображений в линзах Вами понята достаточно хорошо. Поэтому перейдем к более сложному вопросу — построению изображений в системе из двух линз. Рассмотрим задачу: имеются две собирающие линзы с общей главной оптической осью и разными фокусными расстояниями. Построить изображение вертикаль- ной стрелки в такой оптической системе (рис. 46.7, а). Фокусы одной линзы обозначены на рисунке крестиками, а фокусы другой — кружоч- ками. Читатель Б: Чтобы построить изображение стрелки в системе двух линз, надо сначала построить ее изображение в первой линзе. При этом вто- рую линзу можно не принимать во внимание. Затем надо это изобра- жение рассматривать как предмет и, не обращая внимания на первую линзу, построить его изображение во второй линзе. Автор: Здесь Вы делаете весьма характерную ошибку. Мне приходилось много раз сталкиваться с подобным неверным ответом. Давайте рассмотрим два луча, выходящих из острия стрелки, и просле- дим за их ходом через данную систему линз (рис. 46.7, б). Ход лучей после первой линзы определяется просто. Для выяснения хода лучей после второй линзы проведем вспомогательные лучи, параллельные нашим лучам и проходящие через центр второй линзы. При этом ис- пользуем метод, применявшийся в предыдущих примерах (параллель- ные лучи, проходя сквозь линзу, должны пересекаться в фокальной плоскости). В точке пересечения лучей после второй линзы и будет находиться искомое изображение острия стрелки. Все эти построения 236
Рис 46.7 показаны на рис. 46.7, б. Теперь посмотрите, к какому результату мы пришли бы, если бы воспользовались Вашим предложением. Построе- ние произведено на рис. 46.7, в. Здесь сплошными линиями показано по- строение изображения в первой линзе, а пунктирными — последующее построение изображения во второй линзе. Как видите, в этом случае получился бы совсем иной результат (неправильный результат!). Читатель Б: Но я хорошо помню, что мы как-то строили изображение именно тем способом, о котором я говорил. Автор: Возможно, что Вы и строили так. Дело в том, что Ваш способ построения в отдельных случаях оказывается справедливым, так как он приводит к результатам, которые совпадают с результатами, получен- ными по моему методу. Это можно продемонстрировать на указанном примере, если подвинуть стрелку поближе к первой линзе — за ее фо- кус. На рис. 46.8, а показано построение изображения по моему методу, а на рис. 46.8, б — по Вашему методу. Как видите, в данном слу- чае результаты совпадают. Читатель Б: А как узнать, в каких именно случаях можно пользоваться моим методом построения изображения? Автор: Для двух линз было бы нетрудно сформулировать условия приме- нимости Вашего метода. Для большего числа линз эти условия услож- няются. Поэтому нет нужды их вообще рассматривать. Пользуйтесь всегда моим методом, и все будет в порядке. В заключение я хочу за- Рис. 46.8 237
дать еще один вопрос: может ли двояковогнутая линза быть собираю- щей линзой? Читатель Б: Я знаю ответ на этот вопрос. Двояковогнутая линза в обыч- ных условиях является рассеивающей. Однако она станет собирающей линзой, если ее поместить в среду с более высоким показателем прелом- ления, нежели показатель преломления материала линзы, а двояковы- пуклая линза в подобной среде будет рассеивающей. ЗАДАЧИ 46.1. Найти построением ход луча 2 до рассеивающей линзы, если известны по- ложение линзы и ее оптической оси, ход луча / до и после линзы, ход луча 2' после преломления (рис. 46.9). 46.2. На рис. 46.10 даны положения главной оптической оси сферического зеркала, светящейся точки S и ее изображения S'. Найти построением положение вершины А зеркала и его фокуса F. Рассмотреть случаи а), б), в), г). 46.3. На рис. 46.11 показано положение предмета и его изображения. Найти по- строением положение линзы и ее фокусов. 47. ХОРОШО ЛИ ВЫ РЕШАЕТЕ ЗАДАЧИ С ЗЕРКАЛАМИ И ЛИНЗАМИ? Автор: Я хочу сделать несколько замечаний обобщающего характера, которые могут оказаться весьма полезными при решении различных задач с линзами и сферическими (вогнутыми и выпуклыми) зеркалами. Формулы, применяемые при решении задач, делятся на две группы. К п ер во й группе относятся формулы, связывающие друг с дру- гом фокусное расстояние F линзы (зеркала), расстояние от предмета до линзы (зеркала) ^расстояние от изображения до линзы (зеркала) f: Рис. 46.11 (47.1) где d, f и F рассматриваются как алгебраические величи- ны, знак которых может быть 238
разным в разных случаях. При этом возможны всего три случая, пред- ставленные на схеме: Собирающие линзы и вогнутые зеркала d>F d<F 1. d>0, F>0, f >0 2. d>0, F>0,/<0 Изображение действительное Изображение мнимое Рассеивающие линзы и выпуклые зеркала 3. d > О, F < 0, f < О Изображение мнимое Таким образом, d всегда положительно; фокусное расстояние F поло- жительно для собирающих линз и вогнутых зеркал, отрицательно для рассеивающих линз и выпуклых зеркал; расстояние f положительно для действительных изображений и отрицательно для мнимых изображе- ний. Читатель А: Если я правильно понял, то с помощью этой схемы можно получить из общей формулы (47.1) три формулы, в которые входят арифметические значения величин: 1) l/d + 1/f = 1/Л 2) \/d - 1/f = 1/F, (47.2) 3) \/d-\/f^~\/F. Автор: Да, Вы правильно поняли. Читатель А: Я как-то не обращал внимания на аналогию между линзами и соответствующими сферическими зеркалами. Автор: Ко второй группе относятся формулы, которые связывают фокусное расстояние линзы (зеркала) с ее прочими характеристиками. Для зеркал имеем простое соотношение F — ±R/2, (47.3) где R — радиус кривизны зеркала; знак плюс относится к вогнутым зеркалам (фокус положителен), знак минус — к выпуклым (фокус отрицателен). Для линз имеем |=(„ +д_), <47-4’ где п — показатель преломления материала линзы; R{ и R2 — ее радиус кривизны. Если радиус R относится к выпуклой стороне линзы, то он берется со знаком плюс, если же R относится к вогнутой стороне линзы, то со знаком минус. Нетрудно убедиться, что двояковыпуклые, плосковыпуклые и выпукло-вогнутые линзы являются собирающими линзами, так как в соответствии с формулой (47.4) они имеют положи- тельный фокус. Читатель А: А как будет выглядеть формула (47.4), если линзу поместить в среду с показателем преломления п0? 239
превратится в собирающую. Автор: В этом случае вместо (47.4) будем иметь У=("Г_ ')(^М <47,5> Если перейти от случая оптически менее плотной среды (п0 < п) к слу- чаю оптически более плотной среды (п0>п), то согласно фор- муле (47.5) знак фокуса изме- нится и поэтому собирающая линза и, наоборот, рассеивающая линза Перейдем к решению конкретных задач. Выпуклая сторона плосковы- пуклой линзы с радиусом кривизны R и показателем преломления п по- серебрена, в результате чего получилось своеобразное вогнутое зер- кало. Найти фокусное расстояние такого зеркала. Читатель А: Разрешите мне. Направим луч параллельно главной оптиче- ской оси. Отразившись от посеребренной поверхности линзы, он выйдет из линзы и при этом преломится (рис. 47.1). Если бы луч не прелом- лялся, то он пересек бы главную ось на расстоянии R/2 от зеркала в соответствии с (47.3). В результате преломления луч пересекает главную ось несколько ближе к зеркалу. Обозначим искомый фокус через F. Из рисунка видно, что (₽/2) tg «! = F tg о,. Ввиду малости углов cq и можно применить формулу (45.3), получим Р tg а2 sin а2 2F tg а, ~ sin Отсюда находим, что F = R/(2n). (47.6) Читатель Б: Я предлагаю решить эту задачу иначе. Известно, что если совместить две системы с фокусными расстояниями F{ и F2, то новая система будет иметь фокусное расстояние F, определяемое по правилу сложения оптических сил; +—. (47.7) В данном случае имеется линза с фокусным расстоянием F, = R/(n — 1) в соответствии с (47.4), где один из радиусов следует положить равным бесконечности, и вогнутое зеркало, для которого F.2 = R/2. Подставляя выражения F{ и F2 в формулу (47.7), находим 240
откуда F=/?/(n+l). (47.9) Отсюда видно, что читатель А неправильно сделал задачу [см. его ре- зультат (47.6)]. Автор (к читателю Б): Нет, ошибаетесь именно Вы. Результат (47.6) является правильным. Читатель Б: Но разве в данном случае правило (47.7) неверно? Автор: Это правило верно и в данном случае применимо. Читатель Б: Но если верно (47.7), то верно и (47.8). Автор: Вот тут-то Вы и неправы. Дело в том,, что в задаче луч про- ходит через линзу дважды (туда и обратно). Поэтому надо склады- вать светосилы зеркала и двух (!) линз и вместо выражения (47.8) следует записать откуда находим, что 1/F = (2п — 2 -ф 2)//? и, следовательно, F= /?/(2п), что полностью совпадает с результатом (47.6). Рассмотрим другую задачу. Собирающая линза увеличивает изображение предмета в четы- ре раза. Если этот предмет передвинуть на 5 см, то увеличение умень- шается в два раза. Найти фокусное расстояние линзы. Читатель А: В таких задачах я путаюсь: надо нарисовать ход лучей в первом положении, затем во втором и провести сопоставление. Автор: Я полагаю, что в данном случае вообще не надо рисовать ход лу- чей. В соответствии с формулой (47.1) запишем для заданного поло- жения l/F=l/d1-|-l/f1. Учитывая, что f1/d1 = fe1 —увеличение в пер- вом положении, получаем 1 1 । 1 + 1 . „ + 1 F ~ d, + k{dx ~ илиа‘ F ' й2 -J- 1 Аналогично для второго положения запишем d2 = F —--------. Таким й2 образом, k. — й, d’~d'~F^r- <4710) В соответствии с условием задачи d2 — dx = 5 см, kx = 4, k2 = 2. Под- ставляя эти числовые значения в (47.10), получаем, что F = 20 см. ЗАДАЧИ 47.1. Линза с фокусным расстоянием |f|=30 см дает уменьшенное в kx (Л,= 1,5) раз мнимое изображение предмета. Какая это линза — собирающая или рассеиваю- щая? Где находится предмет? Где и каким по величине (Л2) будет изображение предмета, если линзу отодвинуть на / = 20 см? 47.2. Светящаяся точка находится на оптической оси вогнутого зеркала с радиусом кривизны R = 50 см на расстоянии d = 15 см от зеркала. Где находится изобра- жение точки? Что будет с изображением, если зеркало отодвинуть от точки на I ~ 15 см? 47.3. Оптическая система состоит из рассеивающей и собирающей линз (рис. 47.2, а; крестики обозначают фокусы линз). Фокусное расстояние каждой линзы И = 241
Рис. 47.2 = 40 см. Предмет находится на двойном фокусном расстоянии перед рассеивающей линзой. Построить изображение предмета в данной системе и рассчитать его положение. 47.4. Оптическая система состоит из трех собирающих линз с фокусным расстоя- нием F — 30 см, размещенных, Как показано на рис. 47.2, б. Предмет находится на расстоянии d = 60 см от крайней линзы. Где будет изображение предмета? 47.5. Выпуклая сторона плосковыпуклой линзы с радиусом кривизны R = 60 см посеребрена, в результате чего получилось своеобразное вогнутое зеркало. Перед этим зеркалом на расстоянии d = 25 см от него помещен предмет. Найти расстоя- ние f *от зеркала до изображения предмета и увеличение k изображения, если показатель преломления материала лиизы п — 1,5. 47.6. Вогнутая сторона плосковогнутой линзы с радиусом кривизны R — 50 см посеребрена, в результате чего получилось своеобразное выпуклое зеркало. Перед этим зеркалом на расстоянии d = 10 см от него помещен предмет. Найти расстоя- ние f от зеркала до изображения и увеличение k изображения, если показатель преломления материала линзы п = 1,5. 47.7. Собирающая линза с фокусным расстоянием |f | = 20 см находится на рас- стоянии I = 50 см от рассеивающей линзы с фокусным расстоянием |f2| = 36 см. Перед собирающей линзой на расстоянии d{ = 28 см от нее расположен предмет. Определить положение изображения и увеличение системы. 47.8. На оптической оси собирающей линзы с фокусным расстоянием F = 12 см помещен точечный источник света S на расстоянии d = 36 см от линзы. За линзой на расстоянии I = 27 см расположено плоское зеркало. Определить положение изображения. 48. НЕОБХОДИМА ОСТОРОЖНОСТЬ! Автор: Мы живем в эпоху бурного развития науки и техники. На наших глазах совершается научно-техническая революция. Одной из важней- ших предпосылок этой революции являются достижения современной физики. Физика XX в. существенно воздействует на ход технического прогресса, властно вторгается во все области нашей жизни, оказывает на всех нас огромное влияние, формируя основы современного естест- венно-научного мировоззрения. Важно понимать, что физика XX в. — это не просто некий набор новых явлений, цовых формул. Это прежде всего система новых пред- ставлений, включающая в себя, в частности, и завоевания физики пред- шествующих столетий (так называемой классической физики). Читатель Б: Изучение физики в школе ограничивается рамками именно классической физики. О современной физике школьники знают очень мало. Наверное, это неправильно. Автор: Я не согласен с Вами. И не потому, что современная физика 242
для школьника чересчур сложна. А прежде всего потому, что без знания классической физики просто невозможно изучать физику совре- менную. Как говорится, все надо делать по порядку. Впрочем, изучая классическую физику, школьники в то же время знакомятся и с неко- торыми вопросами современной физики. Читатель Б: Это знакомство напоминает короткую экскурсию по новым, и притом непонятным, «экзотическим» местам. Что-то подглядели, чему- то удивились, а остального попросту не заметили. Автор: В этом Вы правы. Однако трудно придумать иной подход. Есте- ственно, что, начиная изучать физику, надо прежде всего освоить ее классические основы. Естественно также, что сегодня ограничиться этими основами нельзя; надо хотя бы немного познакомиться и с совре- менной физикой. Читатель Б: Мне кажется, что поверхностное ознакомление со сложными вопросами опасно появлением ошибочных представлений. Автор: Действительно, в вопросах современной физики экзаменующиеся допускают немало ошибок. Вот поэтому-то я и начал беседу об изуче- нии в школе вопросов современной физики. Здесь, как нигде, необхо- димо соблюдать осторожность. Я предлагаю пройтись по некоторым во- просам, на которые довольно часто можно получить от учащихся одни и те же ошибочные ответы. Подобных вопросов можно было бы привести немало. Мы остановимся лишь на некоторых. Они относятся ,к кван- тово-механическим представлениям, атомной и ядерной физике. Вопрос первый: как объясняет современная физика природу электрического сопротивления? Читатель А: Двигаясь внутри металла, электроны сталкиваются с атома- ми или ионами, образующими кристаллическую решетку. Читатель Б: Наверное, здесь все не так просто. У сверхпроводника тоже есть кристаллическая решетка. Однако почему-то нет электри- ческого сопротивления. Автор: Конечно, здесь не все так просто. Предположим, на минуту, что кристаллическая решетка идеальна — все образующие ее атомы нахо- дятся строго в узлах решетки, не совершая тепловых колебаний. Кроме того, будем полагать, что ₽ решетку не попали какие-либо посторонние примеси. Оказывается, что в таком кристалле электроны под действием внешнего электрического поля будут двигаться совершенно беспрепят- ственно; они не будут «замечать» атомы, из которых образована решет- ка. Получается, что дело не в самом существовании атомов, которые своим присутствием «мешают» электронам, а в чем-то ином. Современ- ная физика дает на этот вопрос определенный ответ: электронам внут- ри металла мешают двигаться не сами по себе атомы кристаллической решетки, а всевозможные нарушения идеальности решетки. Они могут быть связаны с беспорядочными отклонениями атомов из равновесных положений (отвечающих узлам решетки) вследствие тепловых колеба- ний, а также с беспорядочно разбросанными внутри решетки примес- ными атомами. Все дело в том, что атомы, а тем более электроны,— это не какие-то «шарики». Это объекты с необычными свойствами. Так, в частности, движение электрона по кристаллу напоминает в некотором отношении распространение по среде своеобразной волны. Идеальная 243
кристаллическая решетка уподобляется однородной среде, в которой волна не испытывает каких-либо возмущений. Читатель Б: Теперь понятно, почему переходят в сверхпроводящее состояние при очень низких температурах. Прекращается тепловое дви- жение атомов; они как бы «застывают» в узлах решетки. Такая решетка становится идеальной. Автор: Ваш вывод относительно природы сверхпроводимости неверен. Ко- нечно, при очень низких температурах тепловые колебания атомов в ре- шетке кристалла как бы «затормаживаются». И это весьма важно для сверхпроводимости. Однако даже при очень низких температурах ато- мы в решетке все же будут колебаться. Они будут колебаться даже при температуре ОК (если бы даже ее можно было достичь). При очень низких температурах изменяются свойства самих электронов: они объединяются в пары и в результате такого объединения перестают «замечать» слабо колеблющиеся атомы кристаллической решетки. Бо- лее понятного объяснения здесь дать, наверное, нельзя. Однако физиче- ская картина выглядит именно так. Читатель Б: Многое остается непонятным. Я имею в виду уровень знаний школьника. Стоило ли в таком случае задавать вопрос о природе элект- рического сопротивления? Автор: Думаю, что стоило. Ведь напрашивался вполне как будто бы очевидный ответ, основывающийся на представлении о движении обыч- ных тел, которые могут сталкиваться друг с другом. Наше обсуж- дение поставленного вопросжюказало, что здесь необходима осторож- ность. Кроме того, мы все же немного заглянули в картину происхо- дящих процессов. Но перейдем к другому вопросу. Как движутся электроны в атоме? Читатель Б: Согласно теории Бора, электроны движутся в атоме по ста- ционарным орбитам. Каждой орбите отвечает определенная энергия электрона (определенный энергетический уровень). Когда электрон скачком переходит с одной орбиты на другую, он либо испускает, либо поглощает фотон. Автор: Вы правильно изложили основы теории Бора. Однако, как Вы ду- маете — перескакивание электрона с одной орбиты на другую происхо- дит мгновенно или же требует некоторого промежутка времени? Читатель Б: Нет, конечно, не мгновенно. Всякое перемещение в простран- стве требует конечного времени. По теории относительности невозмож- но перемещение со скоростью, превышающей скорость света в вакууме, а она, хоть и велика, но конечна. Автор: А тогда скажите мне, чему равна энергия электрона в те моменты времени, когда он с одной орбиты уже ушел, а на новую орбиту еще не пришел? Читатель Б: Это действительно непонятно... Ведь энергия электрона меня- ется скачком; промежуточных значений энергий у него нет. Я над этим просто не задумывался. Автор: А надо бы задуматься. Во всяком случае Бор над этим вопросом очень задумывался. Тут опять-таки необходима осторожность. Все дело в том, что электроны в атоме вовсе не движутся по орбитам. Физики 244
давно уже отказались от этого представления, как не отвечающего дей- ствительности. Читатель А: Однако и сегодня атом изображают в виде некоторой «сол- нечной системы», где вокруг ядра движутся по орбитам электроны. Автор: Сегодня это всего лишь условный символ атома, тем более что изобразить наглядно истинный характер движения электронов в атоме довольно затруднительно. Читатель Б: Вы обещали, однако, продемонстрировать некоторые явные ошибки. Пока Ясе разговор идет лишь о неглубоком понимании физики отдельных явлений. Автор: Ну что же, попробуем продемонстрировать и явные ошибки. В учеб- нике физики говорится, что если возбудить атомы среды так, чтобы достаточно много атомов оказалось не на нижнем, а на верхнем уровне энергии, то при прохождении через такую среду световой волны с частотой, соответствующей разности энергий рассматриваемых уров- ней, световая волна будет не ослабляться, а, наоборот, усиливаться (за счет индуцированного излучения атомов среды). Читатель Б: Я помню. Так объясняется принцип действия лазера. Автор: Я не имею возражений против написанного в учебнике. Просто хотел бы спросить Вас, откуда берется световая волна, которая затем будет усиливаться? Лазер является генератором света. Он не просто усиливает, но прежде всего порождает световую волну. Итак, мой вопрос таков: откуда взялась световая волна, о которой говорилось в учебнике? Читатель А: Наверное, это свет от лампы, которую используют для воз- буждения лазерной среды. Автор: Вот Вам и явная ошибка. Между прочим, мне не раз приходилось слышать такой ответ. Читатель Б: Это, конечно, ошибка. Ведь не все типы лазеров возбужда- ются светом. Есть лазеры, возбуждаемые электрическим разрядом. Автор: К/тому же частота возбуждающего излучения всегда больше частоты излучения, генерируемого лазером. Читатель Б: На Ваш вопрос был дан неправильный ответ. Это я понимаю. Но я не понимаю, как надо правильно отвечать. Мне непонятно, с чего начинается процесс лазерной генерации. Автор: Атом может перейти с верхнего уровня на нижний не только инду- цированцо (под действием некоторого излучения), но и самопроизволь- но или, как говорят, спонтанно. Процесс лазерной генерации берет свое начало от световых волн, родившихся при спонтанных переходах атомов среды. Читатель Б: Но ведь такие волны должны распространяться в самых произвольных направлениях. Каким же образом от такого неупорядо- ченного процесса можно получить лазерный луч, имеющий определен- ное направление? Автор: Для этого используются зеркала резонатора, выделяющие в про странстве направление генерации. Лазерная генерация развивается именно от тех световых волн, которые были случайно испущены вдоль выделенного направления. Но перейдем от квантовой электроники к ядерной физике. Вы, навер- 245
ное, имеете общее представление о том, как развивается цепная реак- ция деления ядер урана. Читатель А: Нейтрон попадает в ядро и разваливает его на два осколка. При этом рождаются еще несколько нейтронов. Каждый из них «раска- лывает» по ядру, что приводит к рождению еще большего числа нейтро- нов, и т. д. Автор: Если не принимать во внимание некоторые тонкости, то все пра- вильно. Однако откуда берется самый первый нейтрон? Читатель А: Ну, мало ли откуда... Может быть, из космического излу- чения. Автор: Может быть. Однако как тогда быть с подводной лодкой, работаю- щей на ядерном горючем? Лодка лежит на дне под толстым слоем воды, надежно защищающим ее от нейтронов космического излучения. От каких нейтронов берет начало цепная реакция, развивающаяся в двигателе лодки при его включении? Читатель А: Не знаю... Автор: Дело в том, что ядра урана могут делиться не только индуци- рованно (под действием попадающих в них нейтронов), но и само- произвольно, спонтанно. Спонтанно родившиеся нейтроны и являются теми первичными нейтронами, от которых берет начало цепная реакция деления ядер урана. Читатель Б: Получается, что спонтанные, т. е. случайные, процессы игра- ют фундаментальную роль в развязывании различных «полезных» про- цессов. Они запускают в работу лазер. Они же запускают в работу и атомный двигатель. Автор: Процессы, конечно, в этих примерах различны. Но общая мысль выражена верно — в обоих случаях все начинается от случайных про- цессов. Этот вопрос оказывается весьма принципиальным. Однако обсуждение его явно выходит за рамки нашей беседы. Но вернемся к реакции деления ядер урана. Как известно, в этой реакции выделяется немалая энергия. Чем это объясняется? Читатель Б: На каждую частицу в ядре (каждый нейтрон и протон) приходится энергия связи, равная примерно 7 МэВ. Таких частиц в ядре урана 235; значит, всего получается 7 X 235 = 1645 МэВ. Итак, в ядре сосредоточена громадная энергия. Она освобождается при разрушении ядра. Автор: Вот и Вы не избежали ошибки. По-вашему выходит, что энергия связи как бы «заперта» внутри ядра, и надо сломать «замок» (разру- шить ядро), чтобы выпустить эту энергию, словно джина из бутылки. Следует, однако, сразу же заметить, что когда говорят об энергии связи, то имеют в виду не присутствующую, а как раз отсутствующую энергию. Упомянутая Вами энергия связи, равная 1645 МэВ (в дейст- вительности она даже немного больше — около 1700 МэВ), есть та энергия, которую надо затратить, чтобы развалить ядро урана на составляющие его нейтроны и протоны. Я подчеркиваю: это не та энергия, которая освобождается, а та, которую, напротив, Вы сами должны затратить. Читатель Б: А какая же энергия освобождается при делении ядра урана? Автор: Все дело в том, что энергия связи частиц в осколках, получаю- 246
щихся при делении ядра урана, больше, чем энергия связи частиц в ядре урана. Вот эта разница энергий связи и освобождается. Пусть в ядре урана А частиц и на каждую приходится энергия связи ДЕ,. Пусть каждый из осколков деления имеет по Д/2 частиц, на каждую из которых приходится энергия ДЕ2. Обозначим М — массу частицы (не делая различия между нейтроном и протоном). Энергия ядра урана Е, = АМс2 — ДДЕ,. Суммарная энергия обоих осколков Е2 = АМс2 - ААЕ2. Так как ДЕ2 > ДЕ] ,то Е, >Е2. Разность Е] — Е2 = Д (ДЕ2 — ДЕ]) и есть та энергия, которая освобождается при делении ядра урана. Читатель Б: Вообще-то я именно это имел в виду, когда отвечал на Ваш вопрос. Автор: Возможно. Однако Ваш ответ этой мысли не выражал. ОТВЕТЫ 1.1. s = 5,5 км. 1.2. v — \ s2 — (У|^)2/(^ ““ G) = 90км/ч. 1.3. у2 = 2 <у> — у,— 20 км/ч; s = (у) • (<ti + /2) = 160 км. 1.4. 1.5. 1.6. _____3^^ У]У2+ О]У3 + V2V3 30 км/ч. / == /j (у2 + Зу j)/(2c>= 0,75 ч; s = 3/,у2/2 = 9 км; (у) = ЗУ]У2/(у2 Зу|) = 12 км. <у> = 4у,у2(у2Н- у3) (у, + у2)(у2+ у3) + 4у,у2 2 м/с; s = (у2 — у2)/а = 48 м. 1.7. а./2 / а. \ Т - (1 + а,/о2) /, = 10 с; s = —-( 1 + — ) = 20 м; <и> = = 2 м/с; \ U2 / <а> = а2 = 0,5 м/с2. 1.8. у0 = У]/2/(/2 — /]) = 18 м/с; s = У]/2/2 (/2 — = 54 м. 1.9. |Ду| = 2у sin (а/2); 1) ул/2 — V2 « 0,76у; 2) и~/2 « 1 ,4у; 3) 2у. IДа| = 2 -^-sin 1) |Да| = у^/г - V2//? « 0,76у2//?; а 2 «.м/с’ .........1____________I__________I________ 0 2 4 6 t, с Рис. 2 Рис 1 247
2) |Да| = vrfi/R л W/R] 3) |Да| = Ztf/R. 1.10. a = ~\/ а2 + а„ — 5 м/с2. 1.11. ац — ax tg a = 2 м/с2; a = a.V2 « 2,8м/с2. 2.1. См. рис. 1; 1) (y> — 2 м/с; 2) <y> = 2 м/с; 0<t<2c;t = 3 c; 4 c < t < 6 c. 2.2. См. рис. 2,3. 2.3. См. рис. 4,5. 2.4. См. рис. 6,7. 4.1. k = (л2 — 1) tg а/(п2 4- I) 4.2. См. рис. 8. 4.3. См. рис. 9. При 0 t тело покоится, F = F; при t > тело скользит по поверхности, FTp = kmg. 4.4. 1) kmg\ 2) k {mg — F sin a); 3) kmg cos a; 4) k (mg cos a — F sin a). 6.1. v0£ = 11,3 м/с; x = 4 м; у = 0,8 м; T == 0,5 c; = 9,4 м/с; vB = 15,2 м/с. 6.2. 1) L = F\/ и2 + у2 + 2^ п2 cos( а, 4- ; 2) L = ty v? 4- — 2у(у2 sin a, sin o^. 6.3. = ал x / 1 F X 6.4. a = arcctg I —----——1. \ 4 Mg J 6.5. L = 2v^ sin P/(g cos2 0). ___________________________\ _________ 6.6. V = —-\/ -П 7.-7........j - - « 13,8 м/с. 0 cos a V 2 (L tg a — H 4- ft) 6.7. L = H ctg a = 6 m. 6.8. L ~ ( v^/g) sin 2a — 1л; 6 м. 248
6.9. h = H = 19,6 м; s = Зу0л/2H/g — I = 20 м. 6.10. u2sin2a(l+tgp-tga)/(gcOs|J)«40 м, где a=45° + p/2 — угол вылета камня. v = t [F cos a — k (Mg — F sin a)]/M « 38 м/с; A = k (Mg — F sin a) [F cos a — k (Mg — F sin a)] t2/(2M) « 1330 Дж. a=(FcOsa—2fezng4-feFsina)/(2zn)«2,7 м/с2; T=F(cosa4-Asin a)/2»43 H; F, = 2kmg/(cos a 4- k sin a) a — 7.1. 7.2. 7.3. 7.4. и = 8,2 Н. ‘/3g « 3,3 м/с2; T=2/3Mg « 13Н. т3 — (т2 + znj sin a g ^32 S ^21 S 7.5. 3,5 м/с2; + "»2 + тз mAm . + m (1 4- sin a) 3V ------—----------« 50,4 H; m , + /n2+ m3 m .m ,,(\ Ч- sin a) 1 -------- « 33,6 H. "11 + m2 + "b M 3 -j-M 4 -j-M 2(sin a — k cos a) — kM , a=^g--------———:——73——7;------------« 6,9 м/с2 (a — угол наклона сред- ней плоскости к горизонту; М , (Л13 + Л14) (1 + k) + М?М , (sin a 4- k — k cos a) M , + M 2 + M 3 + M 4 (Л4 3 4- Л4 4) [ Л4 , (Л 4- 1) + M2(1 - sin a + Л cos a)] ----------------------------------------------~ 10,2 H; ^12 8 ^23 8 f — gM 7.6. Л4 j 4-Л42 4-Л13 4-Л14 M 2 (1 — sin a 4- k cos a) 4- M ((1 + k) 4---~~ 1 >5 H- 4m 2m 3 — m м, + м2 + м3 + м4 ------------------------4-~ 1,96 м/с2; 4m ^n3 -j- m f (m2 -|- m 3) 5 5,88 м/с2; 5 - - 1,96 м/с2; 5 a. = g 4m jn 3 4- m । (m 2 — 3m 3) °2 4/n2/n3 4-zn ! (zn2 4-zn3) 4m 2m 3 — m , (3m 2 — zzi 3) °3 4zn2zn3 + m , (zn2 4- zn3) 4m 2m3 6 1 4m 2m 3 + m , (zzi 2 4- m 3) 5 a=g (sin a—sin 0—AicQsa—Aicos 0)/2; Л , =₽ tg [(a — ₽)/2]. 5 = v 2/(2kng) ~ 85 m. 8.3. Брусок покоится; FTp= mg sin a — F — 4,9 H и направлена вверх по наклонной 8.1. 8.2. mg} T — 2T { = 12/5mg. 7’=zzzg(sin a-j-sin P+^cQs (i~kcosa)/2; плоскости. 8.4. F , = F (2g sin a/a — 1) « 120H. 8.5. Если tg k, то брусок соскальзывает с плоскости при а= 0. Если tg a< k, то 1) при а = g (k — tg a)/(l + tg a) брусок начинает соскальзывать вниз; 2) при a= g (fg a — «)/(1 4- k tg а) брусок начинает соскальзывать вниз, а при a = g (tg a 4- k)/( 1 — k tg a) начинает подниматься вверх. 8.6. 8.7. 1) Все тела покоятся: а ,= а2 = а3— 0; F^.3k 3mg\ F^ 2,94 Н; 2) Вся система движется как единое целое: 04 = а2— а л = a;0< а k (g; 0 < a^0,98 м/с2; 3k3mg < F < 2mg (Л ! 4- Л2); 2,94 H < F 5,88 H; 3) a ( = a3 =/= a^ = a3= = k ^=0,98 м/с2; a2=F/m—g (Л j4-2fe 2); a2> k ^=0,98 м/с2; F > 2mg(k ( 4- 4- k 2); F > 5,88 H. 9.1. T JT2 = cos a2/cos = V2/3; <»1/«2 = i/cos a2/cos a, = Ч2/З. 9.2. T = 3Mg cos a; ац = 2g cos a; aT = g sin a; a = g^3 cos2a 4- 1. 9.3. (gR — v2)/(gR 4- v2)= 0,43. 249
9.4. (R + 2/f): 2(/f — R): (2/f — 5R) = 7 : 4 : 1. 9.5. Н = 5/2R = 75 см. 9.6. F л = Mg [1 — v 2/(gR)], Рв = Mg [1 + v 2/(gR)]; в точке Б сила давления FB изменяется скачком: при приближении к точке Б слева она равна Mg [cos а—у 2/(gR)j; при приближении справа она равна Mg [cos а-(-у 2/(gR)]. 9.7. v 0 = Vg/ (и — 5) = ^5gl. 9.8. Л = R [1 - g/(w2R)];F= mu2R. 9.9. h = 251/27 от положения равновесия. 9.10. T= mln2 = 3,6 Н. 9.11. I ~ 1.3R (при решении пользоваться советами § 6). 10.1. р = 81л/(8уГ2). 10.2. Т = 2лл/R/g as 1,396 ч as 1 ч 24 мин. 10.3. р = 3л/(?'2у)~ Н0кг/м3. 10.4. v x/v 2 = а/(Л + /12)/(/? + /11); т Л 2 = [ (R + л ,)/(* + h 2)]3/2. 10.5. v , = -y/yM3/R3 as 8 км/с. 11.1. А = mgH/2 = 1225 Дж. 11.2. А = mgl/2 = 19,6 Дж. 11.3. А = 2mgh = 784 Дж. 11.4. А = F (А/)2/(2А/ о) = 1 Дж. 11.5. s = уд/тА//2Д = 2 см; А = mv2/2 = 1 кДж. 11.6. А = m/2g2/8 = 57,5 Дж. 11.7. Работа силы реакции нити равна нулю за любой промежуток времени, так как сила реакции перпендикулярна перемещению (cos 90° = 0). Работа силы тя- жести за период равна нулю, так как равно нулю изменение полной энергии. 12.1. k = v 2 sin а/(у j — v 2) as 0,2, здесь sin а = 0,05. 12.2. (N) = mv [v 2/(2s) + g sin а + kg cos а]/2 as 9,4 кВт, sin а= 0,02, cos a as 1. 12.3. A = m {gH <y> Ay) as 12,3 кДж; (N) = m <y> (g-|- <y> Ay///)as61,5 кВт; Д2 = m {g + <у> Ау/Д) (<y> + Ay/2) as 64,6 кВт. 12.4. N = 2mgv sin а = 30 кВт. 12.5. у = 2y ,v 2 cos а/(у ] + у 2) ~ 75 км/ч. 12.6. <Д> = АД/А/. 1) 1,5 Вт; 2) 1 Вт; 3) 0,75 Вт. 12.7. 1) 6 Дж; 2) 10 Дж; 3) 8 Дж. 13.1. А = — /п(у0+у)(у — у0 — gt)/2 = 3900Дж. 13.2. k = sin а/(1 + cos а) as 0,27. 13.3. 1) у j/у 2== 1; 2) у!/у 2 = д/0 — ctg а i)/(l — k ctg а2)- 13.4. у = у М/{М + m) as 13,8 м/с; I = у 2пт 2/[2М (М + т) kg] х, 0,38 м. 13.5. F = mg {H/h + 1) — Vp „g; h , = 2h (Vp Jm — 1) — H (p — плотность воды). Уд / M , \2 13... V = ,/(М, + М) = 7.5км/ч; / = = 4,43м. vo / т \2 cos2 а sin а 1 3*7• li Л I » Ж I I с 2g \ М / sin а + k cos а 13.8. 1) F= М ,g {1 + 4НМ 2/[ h (М , + М 2)2]} « 89 кН; 2) F=(M1 + M2)g{l +ЯМ2/[/г(Л11 + Л12)2]}«92кН. 13.9. v = voml/(ml + т2); 1)2/3у0, 2) 72у0,3) ‘/зу0; т .v2 / т. \ к Q = (1 - т 1) 7бМу2,2)1 /4му3)1 /зМу2. 13.10. у ] = У од/(п — 1) (n + 1); у 2 = у ^2/{п2 А- «)’, Р = 1 /2 arccos (1 /п); АД = — mv ^/{п + 1); A (my) = mv ov 2п/{п + 1), здесь п = М/т. 13.11. s = I (1 + п)/(п — 3) = 9 м. 250
13.12. 1)Л, = 2/,Я;Лг = '/|Л;2)4- 7uftr®# = 4/sin2(<x/2) + к?Д'. 13.13. т 13.14. h = Н(М- т)/(М + т). 13.15. h = НМ/(Ш_ + 3m). 14.1. У1/у2=3/в^6. 14.2. Т JT 2 = (3 — 2 cos a)/cos a = 4. 14.3. v = ga/w — 0,43 м/с (a выражено в радианах). .. . , т / Л4 + m п I М т 14.4. А = у0 —--------Л/ —7-------, Т = 2л -Л/ —f------. ° Af + т V k N k 14.5. 2ял/ M/g « 0,35 с. 14.6. A//?/(24 • 60 — А/) « 6,6 км. 15.1. /^з т3 ₽3 V мЛ « 2,4. 15.2. а= g[(T^/t)2 - 1] » 17 м/с2; Т 0= 1 с; ускорение направлено вертикально вверх. 15.3. Т = 2jtV//Vg2 + a2» 2с; tg р = ‘/2; р « 27°. 15.4. (^Ia2 + g2/g — 1) а? 0,13. 16.1. Н= vt/2 = 5800 м. 16.2. Ах = Аф • i>/(2nv); 1) Ах = jw/(2n;v) = i>/(2v) = 1 м; 2) Ax=y/v = 2 м; 3) Ах - p/8v = 0,25 м. 16.3. v — 4lv/(2n — 1) = 8 м/с; п = 3. 17.1. k < tg р. 17.2. ТВЕ = Гс£= Afg/(2sin р)« 98Н; 7\в = ТCD = (m + М) g/(2 sin a) « 92Н; F = g [ M ctg p - (M + m) ctg a] /2 « 39 H. 17.3. T — mg sin a/sin (a + p), N = mg sin 0/sin (a + p). / tg a2(l —c°s a2) \ 17.4. a=g I -----д---------г-— 1 i« 27,3 м/с2; ускорение направлено вертикально у tg af —cos aj у вверх. 17.5. В положении 1: FБВ= mgcos2p/sin aa; 1,25 H, FAB = mgcos^cosfa -f- pi/^in a « 0,6 H; в положении 2: Fвв= 1,25 H, FAB=mg [cos pcos(a+p)/sin a-|-siii2p| « 1,54 H; [cos P = 1 - v2/(2gl)= 0,8] 18.1. M Б= F }а-Р2а-Р3а/2+Р5а=0-, M B=-F ta/2+F 2a/2+F 4a/2-F м/2=0-, тело покоится. 18.2. R « 1,5Н;МЛ = 2H • м. » » 18.3. Т = mg/[2 (sin a + cos a)]; R == mgУ 1 + (2+ctg a)2/2 (1 + ctg a); сила R со- ставляет с вертикалью угол p = arcctg (2 + ctg a). 251
18.4. /, = lk/{\ 4- /г). 18.5. A = Mg W/p [(^2 — 1)/2]. 18.6. а = arctg [ 2k (т -j- М)/(2М 4- m)]. 19.1. На расстоянии 3/22/? справа от центра диска. 19.2. 1) На середине высоты, опущенной из вершины с грузом 2/п; 2) центр тяжести расположен на высоте, опущенной из вершины с грузом 2/п, на расстоянии от него 7л/за/24. 19.3. На расстоянии слева от центра шара. 19.4. На линии, проходящей через вершину треугольника и центр квадрата, на расстоянии h (За — 2Л)/6 (2а — Л) вправо от центра квадрата. 19.5. 1) х0 = 2,45 см, у0 = 0,91 см (см. рис. 19.5, а); 2) х0 = 2,45 см, yQ = 1,5 см (см. рис. 19.5^6); 3) х0= 2 см, у0= 2 см (см. рис. 19.5, в). 20.1. А = mgh j/n — 300 Дж; F = mgjn = 750 Н. 20.2. АЛ = рв//[ррт(и + 1) — Рвп] = 0,5 см. 20.3. 1) Уровень ртути расположится в узком и широком сосудах соответственно на высоте . V Л Рв Л j = ; 7Г~( 1 + S(n+1)\ Ррт Уровень воды находится на высоте у / Рв \ V Л3 = е 7 Г.'< 2п + 1 - —) «'0,82 —. Sn (п 4-1) \ Ррт/ S 2) Уровень ртути находится в широком и на высоте у / рв \ у h 1 = с/"-, 1 + —) ~ 0,36 Л2 = - $("4-1)\ Ррт/ S S Уровень воды находится на высоте у / Рв \ S(n+ 1) (2 + П“П7/ 21.1. р = (Р 2р ! — Р !р 2)/(Р 2 ~ Р 1)- 21.2. р^р^Р-Р^Р-Р,). V (Рсв ~ Рв) (19/20рв — рл) у 21.3. h = тг—-------------V----°’048 Т-- 5 ______РДРсв - Рл) S 21.4. Т = 2nVM/(pgS). 26.1. Расширение. 26.2. Масса газа уменьшилась в два раза. 26.3. См. рис. 10. 26.4. См. рис. И. V 0,36-^-, Л2 = Рв \ V 2 + п — п ——) « 1,28 —. Л о Рв (Рев Рл) V / _ Л_\ _ А О Г Sn(n+1) V РртЛ ’ S‘ узком сосудах соответственно Рв \ V п —) « 0,28 тг. Ррт/ 5 V 252
26.5. См. рис. 12. 27.1. h2 = h, (р0 -f- pgh^/pn « 0,11 м; m=(p0-J-pgft|) htSM/(RT) « 1,3 • 10-5 кг; здесь и ниже Т — (t + 273) К; р = 13,8 • 103 кг/м3 — плотность ртути. 27.2. Понизится на Зсм;/п =(р0 — pgl^) l^SM/^RT,) а? 1,5 • IO- 1 кг. 27.3. \m = УМ(р, - р2)/(/?Т) ж 0,059 кг. 27.4. Увеличится на - Z‘l ripg/t^ - cos а) — A7’(PII — Pg^)] Л/~ Т1(р0 - pgftcosa) (Ро — Pg/1) Л5Л1 m = ----= 2.1 • 10-5 кг. Л* 7 ! as 0,016 м; 27.5. Капелька передвинется к шару Б на А/: V. АТ, + АТ V, АТ, + АТ2 ' S 2Т, +(АТ, - АТ2) ’ ’ S 2Т, -(ДТ2 - АТ,) ’ У, АТ! 3^Л/=-г— 2 У^Т 27.6. Каждая капелька отодвинется от шара Б на А/ = ————. о *3 1 । 27.7. Каждая капелька передвинется вправо на А/ = У,АТ/(5Т,). 27.8. Поршень Л сместится вправо на А/= ?п,//(/п, + /п2) = Зсм. P.MV 27.9. Н = h log, b - = 20 км. 2 mRT VM / Pi P2 \ 27.10. Am == —— ( —-— ) = 0,02 кг. “ \ t 12 / 28.1. m= V/pH,/100 = 739 г; роса не выпадет, так как fpH,/100 < рн2; /?т/Рн, ^=Tir=0-5«' 28.2. Масса сухого воздуха больше массы влажного на Д/n =(Л4в — Mn) fVpJ(RT) — 9,46 г, где М и/И — молярные массы воздуха и пара. Pi Л 28.3. / = -1Г-±. 100% = 30%. PiJ\ 29-1- Q^7:Q.^Q5r24_>= , , = |р|VI -рЛ21'-|р21/1 -р'УЛ-\р>У>-р'У2\-\р^ -р-Л2|- 29-2. Q, f): Qj ^ , = 1:1; Q, : QH _ 2 = р, : р2. 29.3. Л=р01/,(Т2-Т1)/Т, = 138Дж;2) 4=(p0+Mg/S) У, ( Т2-Т,)/Г, = 171 Дж. 29.4. А = (m/M) RT( \ — р2/р,) ~ 5 кДж, [(т/М) = 3 — число молей]. ' 253
29.5. Изменение внутренней энергии равно нулю. 30.1. А = Q , (1 — Т 2/Т « 1,47 кДж; Q2 = Q J 2/Т , = 350 кал « 1,47 кДж. 30-2' ’’Н1 “(теОО -ж)] 100% =60% 30.3. А = p2V2/4 =2кДж,п = /?/[М (cv + 2ср)] =10,5%. 30.4. N , = #/,/(7, -Т2) = 186,5 кВт; Q2 =tN пТ 2/(Т { -Т2) =5 МДж. 31.b Q =3 • 72/?7, = 18,7 кДж. 31.2. хQ = (т/М)ЬТ (Cv +R) « 102 Дж. 31.3. А R А7 = 603 Дж; Q — (R 4- с vm\ АТ 2,7 кДж. М \ М v / 33.1. В точке О: 1) нуль; 2) 2,55 • 1010 В/м, направлена горизонтально слева направо; 3) 2,55 • 1010В/м, направлена вертикально вниз; 4) нуль. В точке О,: 1) и 2) 6,45 • 109 В/м, направлена горизонтально слева направо; 3) 3,93 • 10 10 В/м, направлена вертикально вниз; 4) 3,27 • 10 10 В/м, направ- лена вертикально вверх. 33.2. В точках А, Б, Г: нуль; в точке В: ф = — 2 • Ю10В; в точке Д-. ф = 2 • Ю10В. 33.3. Е =2,2 • 1013 В/м, ф = 1,05 • 1010 В. d2d22 4\(q2—q^ d2 +((?4—<7з) d 1 33.4. Е=--------------—--------------=0,16 В/м. Угол между направлением на- d fo2 ( d 2 *7 1) 2 пряженности и короткой диагональю а = arctg ------------= 45°. 33.5. Напряженность в центре треугольника равна нулю; в вершине тетраэдра Е — ^6kq/a2 « 2,6 кВ/м. 33.6. На расстоянии I, = I ^.5 см от меньшего заряда. 34.1. 1)£, = 0, ф] = ф0 = 2 В, £2 = Фо/?//2 « 22 В/м, ф2 =/?Фо//2 « 1,03 В; 2) £ ! = 0; £2 = ф(/?/(е/2) а; 11 В/м; ф , = ф0 = 2 В; ф2 » 0,66В. 34.2. £( =0, Ф1 =0;£2=/г(<?1//2)»11МВ/м;ф2 = /г(<71//2-(72//?2) = -450кВ; £з =/г(<71//1 —<72/Z3) ~5,5МВ/м;ф3 = k. (^q x/l 3 — q 2//3) = —450 кВ. 34.3. ф = ф0(1 — Ri/R2) = 1 В. 34.4. AQ = q (R 2 - R ,)/(/? 2 + R ,) ® 2/3 мкКл. 35.1. t = ~\l2dm/(eE) =3 • 10~ 8с; v0 = fVe£/(2dm) =5 • 106м/с. 35.2. £ = mv 2 tg a/(eZ) = 147 В/м. 35.3. 7 = (mg 4- £<7)/cos а; WK = (mg А~ Eq) I sin2a/(2 cos a), (m, -m2) q +£(<?, -<72) 35.4. Cl — ~ _ i m, + m2 _ 2m xm 2g + £ (m 2q , + m $ 2) m , + m 2 35.5. v = V5 (mg A-Eq) l/m. 36.1. 1,83g. ___________________ 36.2. ш=л/-«------------ У tcosa m/3sin3a cos a 36.3. v = ^5gl — q2/(ml). 37.1. /4=/,£ ,£ 5/(₽ 4/?3+ R 5+ R 4/? 5) =0,2 A. 37.2. / = 1 A. 37.3. 1) I = 0, U = 2) 1 = E/r, U =^-lr = 0. 37.4. x =/?r —r 4-Л? ш) «0,160м. 254
37.5. (/ = «Г[ 1 -r/(r + /?v)] ®9,9B;/ = r/(r + /?v) • 100% = 1%. 37.6. /= ir/(г+ /? + /?,) «0.196А,/ = /?,/( г + /? + /?,) • 100% « 1,96%. 37.7. /,=[Г1-/3(г;!+/?)+^]Л1 = 1,5А;/2=[^-{/3(/з+/?)-£3)]/г2 = 2,5А; ' r2/? + r,/? + v2 ’ 2 r2R + r^ + rxr2 ’ r^ + r^ + r,^’ 1)7 = 1 A,7, = 0,/2 = 1 A, 2) 7 « 1,1 А, У, »0,57A,72 = 0,7 A; 3) 7 = 0,5 A. C = Q = Q = 38.1. 38.2. 38.3. 12с,с2 + 4д + зд 17 10C2 + 3Cj 3 23 W/7 ——,-'-v ~ 6 мкКл. «’C/?1/?2 Rj + 2R2r + 2RtR2' 38.4. R = rdmv2Q tg a/( “Уе/ — dmv^ tg a). 38.5. R2 = 8 0м. 38.6. a= arctg[ 3^Te//( v20/nd)]. 38.7. q = CU = 2 • 10~ 4 Кл. 38.8. <7о=С^1-У2)=2ОмкКл; ?=C[ 4-2/?2)+«?/?,]/(/?,+₽2) = 110 мкКл. 39.1. U= «ГТ? (3r 4-7?) « 3,75 B; 7 = 3«f/(3r + 7?) a; 0,25 A. 39.2. 2/3R. 39.3. 4/з/?. 39.4. 3/<R. 39.5. "/го/?- 39.6. 7/<R. 40.1. R = 5rrj/[3(1 — я)] ~ 60Ом; N — У2т] (1 — т))/г « 70 Вт (здесьт) = 0,5). . п И- . 40.2. Уменьшится в-------гг—.—-—=1,4 раза. «(*4-1) 40.3. г = R/8, т|, ж 89%, т)2 « 83%. 40.4. т = 800 г. 255
Рис. 15 40.5. 100 г воды превратится в пар; t а* 21 мин. 41.1. 1)В = 0,2)В= 2ц0//(л</). 41.2. 1) В = |x03//(4R); 2) В = p0//(4R); 3) В = |i0/V5/(4/?) 41.3. В = 0. 41.4. 1) В = Ц0/'Л? + ^/(2лг,г2) = = IO-4 Тл; 2) B=p0/V^+^/(2nr,r2) = = 10 - 4 Тл. 43.1. Р = B4W/R = 0,625Вт; / = Blv/R = 0,25 А. 43.2. q = BS/(2R) = 0,125мкКл. 43.3. В = y'Dpg/v « 4 • 10- 2 Тл. 45.1. ft. = (djл2 4- d2n^/^nxn^ « 10,8см. Мп 41.5. В = — --= 4 • 10- 4 Тл. л а 42.1. a=arctg[B///(zng)] = arctgl=45°. 42.2. а = arctg [B//(2pgs)] «45°. 45.2. 2H — h -\[n2 - 1 cos a cos (a — p) -, sin a = 1 /n. 45.3. a = arctg n = 56°; x = d (n2 — l)/n-\/rt2 + 1 = 2,3 cm. L = " \ + J 46.1. См. рис. 13; |£Ю| параллельно лучу 1', (CFD) — фокальная плоскость, луч 2 параллелен (СО). 46.2. См. рис. 14. 46.3. См. рис. 15. 47.1. Линза рассеивающая; d=( 1 — F = 15 см; изображение отодвинется от линзы на расстояние Af = f2//[(d + F) (d + 1 + F)] = 6 см; k2 = Af/f/ + ( 1 - fe,) f] =0,4. 47.2. Изображение мнимое, f = Rd/(2d — R)=—37,5 cm; f , = R (d-|-/)/[2 (<Z -pZ) — — R] = 150 см (изображение действительное). 47.3. f2 = 5/2F= 100 cm. 47.4. В месте расположения средней линзы. Изображение мнимое, fe = 1. 47.5. f = dR/(2nd - R) = 100 см; fe = R/(2nd - R) = 4. 47.6. f = - dR/(2nd + R) = - 6,25 см; fe = R/(2nd + R) = 0,625. F I---!!-----l\ Ad^-F' ) 47.7. f2 —----—--------— = 45 см (от рассеивающей линзы); F\F2 45 (F2 + /)(d1-F,)-d,F1 8 • 47.8. Изображение находится на расстоянии 18 см от источника.